Vous êtes sur la page 1sur 241

T H E BR IGH A M I N T E NSI V E R E V I E W

OF I N T E R N A L M E DICI N E QU E S T ION
A N D A NS W E R COM PA N ION
THE BR IGHA M INTENSIV E
R EVIEW OF INTER NAL
MEDICINE QUESTION
ANDANSW ER COMPANION

EDITED BY

Ajay K.Singh, MBBS, FRCP, MBA Joseph Loscalzo, MD, PhD


DIRECTOR, GLOBAL PROGR AMS HERSEY PROFESSOR OF THE THEORY
ASSOCIATE PROFESSOR OF MEDICINE ANDPR ACTICE OF PHYSIC
H A RVA R D M E DIC A L S C HO OL H A RVA R D M E DIC A L S C HO OL
PHYSICIAN, RENAL DIVISION CH A IR M A N, DEPA RTM ENT OF M EDICINE
DIR ECTOR, POSTGR ADUATE MEDICAL PHYSICIAN-IN-CHIEF
EDUCATION BRIGHAM AND WOMENS HOSPITAL
DEPA RTM ENT OF M EDICINE BOSTON, MASSACHUSETTS
BRIGHAM AND WOMENS HOSPITAL
BOSTON, MASSACHUSETTS

1
1
Oxford University Press is a department of the University of
Oxford. It furthers the Universitys objective of excellence in research,
scholarship, and education by publishing worldwide.
OxfordNewYork
Auckland Cape Town Dar es Salaam Hong Kong Karachi
Kuala LumpurMadridMelbourneMexico CityNairobi
New DelhiShanghaiTaipeiToronto
With offices in
ArgentinaAustriaBrazilChileCzech RepublicFranceGreece
GuatemalaHungaryItalyJapanPolandPortugalSingapore
South KoreaSwitzerlandThailandTurkeyUkraineVietnam
Oxford is a registered trademark of Oxford University Press
in the UK and certain other countries.
Published in the United States of America by
Oxford University Press
198 Madison Avenue, NewYork, NY 10016
Oxford University Press 2014
All rights reserved. No part of this publication may be reproduced, stored in
a retrieval system, or transmitted, in any form or by any means, without the prior
permission in writing of Oxford University Press, or as expressly permitted by law,
by license, or under terms agreed with the appropriate reproduction rights organization.
Inquiries concerning reproduction outside the scope of the above should be sent to the
Rights Department, Oxford University Press, at the address above.
You must not circulate this work in any other form
and you must impose this same condition on any acquirer.
Library of Congress Cataloging-in-Publication Data
The Brigham intensive review of internal medicine question and answer companion / edited by Ajay K. Singh, MBBS, FRCP (UK), MBA,
Director, Global Programs, Associate Professor of Medicine, Harvard Medical School Physician, Renal Division, Director, Postgraduate
MedicalEducation, Department of Medicine, Brigham and Womens Hospital, Boston, Massachusetts, Joseph Loscalzo, MD, PhD,
Hersey Professor Of The Theory And Practice Of Physic, Harvard Medical School, Chairman, Department Of Medicine,
Physician-In-Chief, Brigham And Womens Hospital, Boston, Massachusetts.
pages cm
ISBN 9780199358496 (alk. paper)
1. Internal medicineExaminations, questions, etc. 2. American Board of Internal MedicineExaminationsStudy guides.
I. Singh, Ajay, 1960 editor. II. Loscalzo, Joseph, editor.
RC58.B75 2014
616.0076dc23
2014019690
This material is not intended to be, and should not be considered, a substitute for medical or other professional advice.
Treatment for the conditions described in this material is highly dependent on the individual circumstances. And, while
this material is designed to offer accurate information with respect to the subject matter covered and to be current as of
the time it was written, research and knowledge about medical and health issues is constantly evolving and dose schedules
for medications are being revised continually, with new side effects recognized and accounted for regularly. Readers
must therefore always check the product information and clinical procedures with the most up-to-date published product
information and data sheets provided by the manufacturers and the most recent codes of conduct and safety regulation.
The publisher and the authors make no representations or warranties to readers, express or implied, as to the accuracy
or completeness of this material. Without limiting the foregoing, the publisher and the authors make no representations
or warranties as to the accuracy or efficacy of the drug dosages mentioned in the material. The authors and the publisher
do not accept, and expressly disclaim, any responsibility for any liability, loss or risk that may be claimed or incurred as a
consequence of the use and/or application of any of the contents of this material.

135798642
Printed in the United States of America
on acid-free paper
PR EFACE

Preparing for the American Board of Internal Medicine physicians in the department. We sincerely thank them
(ABIM) certifying or recertifying examination requires for their efforts and commitment to this project. We asked
knowledge and clinical experience that can be evaluated them to put themselves in the head of the ABIM to iden-
by successfully answering questions in a test format. In this tify the topics that might be addressed in the board exami-
Question and Answer book, our goal is to provide you with nation. We believe this book will be a valuable study tool to
500 questions across 10 subspecialties in internal medicine. gauge your knowledge as you prepare for the exaxmination.
These questions test knowledge on topics relevant to the We wish to thank Stephanie Tran and Michelle Deraney
ABIM boards. As a companion to the Brigham Intensive for supporting us in the development of this book. Thanks
Review of Internal Medicine, now in its second edition, this also to Anika Singh for helping with the final stages of the
book is focused on how you apply knowledge to answer book. Without them this book would not have been possible.
board questions successfully. The annotated answers are Our thanks also go to our families who have supported all
detailed, and they review the steps in critical thinking of our academic activities, including this important project.
required to get to the correct answer.
The authors who have contributed questions and anno- Ajay K.Singh, MBBS, FRCP, MBA
tated answers to this book are some of our most senior Joseph Loscalzo, MD, PhD

v
CONTENTS

Contributors  ix 6. Nephrology and Hypertension  125


Karandeep Singh and Ajay K. Singh
1. Infectious Diseases  1 7.Digestive Diseases and Disorders of the Pancreas
Jennifer A.Johnson, Michael S.Calderwood, and Liver  141
Sarah P.Hammond, Rebeca M.Plank, Robert S.Burakoff, Muthoka Mutinga, and
DylanB.Tierney,andSigal Yawetz Molly L.Perencevich
2. Hematology and Oncology  27 8. Cardiovascular Disease  160
Lawrence Shulman, Ann LaCasce, Wendy Y.Chen, Thomas S.Metkus Jr., Patrick OGara,
Yuksel Urun, Toni K.Choueiri, Jean M.Connors, and Donna M.Polk
Peter Enzinger, Nancy Berliner, Maureen M.Okam,
Mark M.Pomerantz, David M.Jackman, 9.Neurology  185
Brett E.Glotzbecker, Edwin P.Alyea, Galen V.Henderson
DanielJ.DeAngelo,Robert I.Handin, 10. General Internal Medicine  202
JeffreyA.Meyerhardt, and
Whitney W.Woodmansee Nikhil Wagle, Christopher Gibson, Ami Bhatt,
MollyL.Perencevich, William Martinez,
3.Rheumatology  49 Jason Ojeda, RoseKakoza, and Lindsay King
Derrick J.Todd and Jonathan S.Coblyn
4. Pulmonary and Critical Care Medicine  76 Index  223
Michael H.Cho and Christopher H.Fanta
5.Endocrinology  101
Carolyn Becker, Amir Tirosh, Ole-Petter R.Hamnvik,
Le Min, Klara Rosenquist, Anand Vaidya,
Whitney W.Woodmansee, Bindu Chamarthi,
and Matthew Kim

vii
CONTR IBUTOR S

Edwin P.Alyea III, MD Michael S.Calderwood, MD


Associate Professor of Medicine Instructor in Medicine
Harvard Medical School Harvard Medical School
Medical Oncologist Division of Infectious Diseases
Dana Farber Cancer Institute Department of Medicine
Department of Medicine Brigham and Womens Hospital
Brigham and Womens Hospital Boston, MA
Boston, MA
Bindu Chamarthi, MD
Carolyn Becker, MD Instructor in Medicine
Associate Professor of Medicine Harvard Medical School
Harvard Medical School Division of Endocrinology, Diabetes and Hypertension
Division of Endocrinology, Diabetes and Hypertension Department of Medicine
Department of Medicine Brigham and Womens Hospital
Brigham and Womens Hospital Boston, MA
Boston, MA
Wendy Y.Chen, MD, MPH
Nancy Berliner, MD Assistant Professor of Medicine
Professor of Medicine Harvard Medical School
Harvard Medical School Medical Oncology
Division of Hematology Dana Farber Cancer Institute
Department of Medicine Department of Medicine
Brigham and Womens Hospital Brigham and Womens Hospital
Boston, MA Boston, MA

Ami Bhatt, MD, PhD Michael H.Cho, MD, MPH


Instructor in Medicine Assistant Professor in Medicine
Harvard Medical School Harvard Medical School
Medical Oncology Division of Pulmonary and Critical Care Medicine
Dana Farber Cancer Institute Department of Medicine
Department of Medicine Brigham and Womens Hospital
Brigham and Womens Hospital Boston, MA
Boston, MA
Toni K.Choueiri, MD
Robert S.Burakoff, MD, MPH Associate Professor of Medicine
Associate Professor of Medicine Harvard Medical School
Harvard Medical School Medical Oncology
Division of Gastroenterology, Hepatology and Endoscopy Dana-Farber Cancer Institute
Department of Medicine Department of Medicine
Brigham and Womens Hospital Brigham and Womens Hospital
Boston, MA Boston, MA

ix
Jonathan S.Coblyn, MD Sarah P.Hammond, MD
Associate Professor of Medicine Instructor in Medicine
Harvard Medical School Harvard Medical School
Division of Rheumatology, Immunology and Allergy Division of Infectious Diseases
Department of Medicine Department of Medicine
Brigham and Womens Hospital Brigham and Womens Hospital
Boston, MA Boston, MA

Jean M.Connors, MD Ole-Petter R.Hamnvik, MBChB, BAO


Assistant Professor of Medicine Instructor in Medicine
Harvard Medical School Harvard Medical School
Division of Hematology Division of Endocrinology, Diabetes and Hypertension
Department of Medicine Department of Medicine
Brigham and Womens Hospital Brigham and Womens Hospital
Boston, MA Boston, MA
Daniel J.DeAngelo, MD
Associate Professor of Medicine Robert I.Handin, MD
Harvard Medical School Professor of Medicine
Medical Oncologist Harvard Medical School
Dana-Farber Cancer Institute Division of Hematology
Department of Medicine Department of Medicine
Brigham and Womens Hospital Brigham and Womens Hospital
Boston, MA Boston, MA

Peter Enzinger, MD Galen V.Henderson, MD


Associate Professor of Medicine Assistant Professor of Medicine
Harvard Medical School Harvard Medical School
Medical Oncologist Department Neurology
Dana-Farber Cancer Institute Brigham and Womens Hospital
Department of Medicine Boston, MA
Brigham and Womens Hospital
Boston, MA David M.Jackman, MD
Christopher H.Fanta, MD Assistant Professor of Medicine
Professor of Medicine Harvard Medical School
Harvard Medical School Medical Oncology
Division of Pulmonary and Critical Care Medicine Dana Farber Cancer Institute
Department of Medicine Department of Medicine
Brigham and Womens Hospital Brigham and Womens Hospital
Boston, MA Boston, MA

Christopher Gibson, MD Jennifer A.Johnson, MD


Instructor in Medicine Instructor in Medicine
Harvard Medical School Harvard Medical School
Medical Oncology Division of Infectious Diseases
Dana-Farber Cancer Institute Department of Medicine
Department of Medicine Brigham and Womens Hospital
Brigham and Womens Hospital Boston, MA
Boston, MA
Rose Kakoza, MD
Brett E.Glotzbecker, MD
Research Fellow in Medicine
Instructor of Medicine
Harvard Medical School
Harvard Medical School
Division of General Medicine and Primary Care
Medical Oncology
Department of Medicine
Dana-Farber Cancer Institute
Brigham and Womens Hospital
Department of Medicine
Boston, MA
Brigham and Womens Hospital
Boston, MA

x Con t r i bu tor s
Matthew Kim, MD Muthoka Mutinga, MD
Instructor in Medicine Assistant Professor of Medicine
Harvard Medical School Harvard Medical School
Division of Endocrinology, Diabetes and Hypertension Division of Gastroenterology, Hepatology and Endoscopy
Department of Medicine Department of Medicine
Brigham and Womens Hospital Brigham and Womens Hospital
Boston, MA Boston, MA

Lindsay King, MD Patrick OGara, MD


Research Fellow in Medicine Professor of Medicine
Harvard Medical School Harvard Medical School
Gastrointestinal Unit Division of Cardiovascular Medicine
Department of Medicine Department of Medicine
Massachusetts General Hospital Brigham and Womens Hospital
Boston, MA Boston, MA

Ann LaCasce, MD Jason Ojeda, MD


Assistant Professor of Medicine Department of Internal Medicine
Harvard Medical School Primary Care Physician
Medical Oncology Jefferson University Hospital
Dana Farber Cancer Institute Philadelphia, PA
Department of Medicine
Brigham and Womens Hospital Maureen M.Okam, MD, MPH
Boston, MA Instructor in Medicine
Harvard Medical School
William Martinez, MD, MS Division of Hematology
Assistant Professor of Medicine Department of Medicine
Division of General Internal Medicine and Public Health Brigham and Womens Hospital
Vanderbilt University Medical Center Boston, MA
Nashville, TN
Molly L.Perencevich, MD
Thomas S.Metkus, Jr., MD Instructor in Medicine
Fellow in Cardiovascular Medicine Harvard Medical School
Division of Cardiovascular Medicine Division of Gastroenterology, Hepatology and Endoscopy
John Hopkins Hospital Department of Medicine
Baltimore, MD Brigham and Womens Hospital
Boston, MA
Jeffrey A.Meyerhardt, MD
Associate Professor of Medicine Rebeca M.Plank, MD
Harvard Medical School Instructor in Medicine
Medical Oncology Harvard Medical School
Dana Farber Cancer Institute Division of Infectious Diseases
Department of Medicine Department of Medicine
Brigham and Womens Hospital Brigham and Womens Hospital
Boston, MA Boston, MA

Le Min, MD, PhD Donna M.Polk, MD, MPH


Instructor in Medicine Lecturer on Medicine
Harvard Medical School Harvard Medical School
Division of Endocrinology, Diabetes and Hypertension Division of Cardiovascular Medicine
Department of Medicine Department of Medicine
Brigham and Womens Hospital Brigham and Womens Hospital
Boston, MA Boston, MA

Con t r i bu tor s x i
Mark M.Pomerantz, MD Derrick J.Todd, MD
Assistant Professor of Medicine Instructor in Medicine
Harvard Medical School Harvard Medical School
Medical Oncology Division of Rheumatology, Immunology and Allergy
Dana Farber Cancer Institute Department of Medicine
Department of Medicine Brigham and Womens Hospital
Brigham and Womens Hospital Boston, MA
Boston, MA
Yuksel Urun, MD
Klara Rosenquist, MD Research Fellow
Research Fellow in Medicine Lank Center for Genitourinary Oncology
Harvard Medical School Dana-Farber Cancer Institute
Division of Endocrinology, Diabetes and Hypertension Boston, MA
Department of Medicine
Brigham and Womens Hospital Anand Vaidya, MD, MMSc
Boston, MA Instructor in Medicine
Harvard Medical School
Lawrence Shulman, MD Division of Endocrinology, Diabetes and Hypertension
Associate Professor of Medicine Department of Medicine
Harvard Medical School Brigham and Womens Hospital
Medical Oncology Boston, MA
Dana-Farber Cancer Institute
Department of Medicine Nikhil Wagle, MD
Brigham and Womens Hospital Instructor in Medicine
Boston, MA Harvard Medical School
Medical Oncology
Karandeep Singh, MD Dana Farber Cancer Institute
Clinical Fellow in Medicine Department of Medicine
Harvard Medical School Brigham and Womens Hospital
Renal Division Boston, MA
Department of Medicine
Brigham and Womens Hospital Whitney W.Woodmansee, MD
Boston, MA Assistant Professor in Medicine
Harvard Medical School
Dylan B.Tierney, MD Division of Endocrinology, Diabetes and Hypertension
Instructor in Medicine Department of Medicine
Harvard Medical School Brigham and Womens Hospital
Division of Infectious Diseases Boston, MA
Department of Medicine
Brigham and Womens Hospital Sigal Yawetz, MD
Boston, MA Assistant Professor in Medicine
Harvard Medical School
Amir Tirosh, MD, PhD Division of Infectious Diseases
Instructor in Medicine Department of Medicine
Harvard Medical School Brigham and Womens Hospital
Division of Endocrinology, Diabetes and Hypertension Boston, MA
Department of Medicine
Brigham and Womens Hospital
Boston, MA

x ii Con t r i bu tor s
1.
INFECTIOUS DISEASES

Jennifer A. Johnson, Michael S. Calderwood, Sarah P. Hammond, Rebeca M. Plank,


DylanB.Tierney, and Sigal Yawetz

1. A28-year-old woman who has lived her entire life in D. Daily changing of the ventilatory circuit
Providence, Rhode Island, presents 3days after return- E. Nasotracheal intubation rather than orotracheal
ing from a 2-week trip to Thailand complaining of fever intubation
to 102F, muscle aches, and severe retro-orbital head-
ache. She has no gastrointestinal symptoms. She trav- 3. A47-year-old woman with a history of renal trans-
eled only to the towns of Bangkok, Chiang Mai, and plantation who was recently treated for a urinary tract
Phuket. She attended a travel clinic prior to traveling infection presents with 4 days of profuse nonbloody
and was told there was no malaria in these towns, so diarrhea and 2days of nausea and fevers. She received
she did not take prophylaxis. She denied contact with her kidney transplant 10years ago from a living unre-
bodies of fresh water. Examination is unremarkable lated donor for polycystic kidney disease and has had
other than fever of 101.8F. Remarkable laboratory no episodes of rejection since. Her donor was CMV
findings include a leukocyte count of 2,200 cells/L3, IgG negative and she was CMV IgG positive prior
hematocrit of 37%, and platelets of 62,000 cells/L3. to transplant. She is a third-grade teacher and has
Chemistries are normal. Aperipheral blood smear for recently been taking care of a pet turtle for her class.
parasites is sent and is negative. On presentation she has a fever to 102.1F and has dif-
Which of the following is the most likely diagnosis fuse tenderness of the abdomen with more intense focal
in this traveler: tenderness over the right lower quadrant, the location
of her transplanted kidney.
A. Leptospirosis The least likely cause of her present illness is:
B. Malaria
C. Typhoid A. Salmonellosis
D. Hepatitis A B. C.difficileassociated diarrhea
E. Dengue C. Cytomegalovirus colitis
D. Norovirus gastroenteritis complicated by bacterial
2. A 55-year-old male smoker with severe chronic transplant pyelonephritis
obstructive pulmonary disease (COPD) is hospital-
ized in the medical intensive care unit. He now requires 4. A 25-year-old man who has a history of moderate
intubation and mechanical ventilation for hypercar- asthma and depression presents in January for a first pri-
bic respiratory failure after failing noninvasive ven- mary care clinic visit with you as his new primary care
tilation. To reduce this patients risk for developing physician. He was hospitalized in December for 2days
ventilator-associated pneumonia, you recommend: for pneumonia; all cultures and viral swabs were nega-
tive. He was treated empirically with a short course of
A. Elevation of the head of the bed to 15 degrees to levofloxacin and improved. He is feeling well today. He
prevent aspiration presents for the visit with his male partner, who is help-
B. Suctioning of subglottic secretions ing the patient manage his asthma by reminding him
C. Twenty-four hours of prophylactic systemic to use his inhalers and record his peak flow when he is
antibiotics, especially if the intubation was emergent ill. Physical examination is unremarkable; he is afebrile

1
and well appearing. You review his immunization his- agents is/are not recommended for the empiric manage-
toryhe has not received any vaccines within the past ment of acute uncomplicated cystitis:
7years; his last vaccination was the tetanus, diphtheria
(Td) vaccine at age 18. A. Trimethoprim-sulfamethoxazole 160/800 mg
All of the following vaccines would be appropriate (1double strength) twice daily, for 3days
for him based on his medical history, except: B. Ciprofloxacin 250 mg twice daily, for 3days
C. Fosfomycin 3 grams, single-dose therapy
A. Tetanus, diphtheria, pertussis (Tdap) vaccine D. Nitrofurantoin macrocrystals 100 mg twice daily,
B. Pneumococcal 13-valent conjugate (PCV13) vaccine for 5days
C. Hepatitis Avaccine E. Amoxicillin 500 mg three times daily, for 7days
D. Human papillomavirus (HPV) vaccine F. Options B and E
E. Influenza vaccine
8. A 65-year-old woman with recently diagnosed dif-
5. A 36-year old man is found to have a 17 mm reac- fuse large B cell lymphoma presents with malaise, nau-
tion to tuberculin skin testing as part of a workplace sea, and mild jaundice. She was diagnosed with diffuse
screening program. He is originally from Bangladesh large B cell lymphoma after developing a massive cer-
and immigrated to the United States 6months ago. He vical lymphadenopathy and daily fevers 1 month ago.
reports feeling well. He has no fever, cough, or weight She was profoundly anemic when she presented and
loss. Physical examination is normal. required a blood transfusion at that time. She started
The next best step in his management should be: treatment for lymphoma with rituximab, cyclophos-
phamide, doxorubicin, vincristine, and prednisone
A. Sputum for smear microscopy and mycobacterial (R-CHOP) 2 weeks ago. Basic labs are notable for AST
culture 527, ALT 495, total bilirubin 3.5, with a normal INR.
B. Initiation of isoniazid prophylaxis to prevent Her past history is notable for immigrating to the
reactivation of latent tuberculosis infection United States from rural Vietnam 15years ago. She had
C. Chest X-ray to assess for active pulmonary disease a positive PPD at the time she immigrated and was also
D. Interferon gamma release assay to confirm skin test told she was a hepatitis B chronic carrier (hepatitis B
result surface antigen positive, hepatitis B e antibody positive,
hepatitis B virus load <1000 IU/mL) at that time. She
6. A previously well 62-year-old man presents to the was started on treatment for latent tuberculosis with
hospital with increasing weakness in his lower extremi- isoniazid a month ago during her workup for fevers and
ties. An examination reveals decreased reflexes symmet- lymphadenopathy.
rically, which progressed proximally over the course of A likely cause of her abnormal liver function tests
several hours. He was diagnosed with GuillainBarr and malaise is:
syndrome and admitted to the intensive care unit for
treatment. On history, he did report several days of nau- A. Hepatitis C infection resulting from blood
sea, vomiting, and diarrhea approximately 2 months transfusion
prior. B. Progression of her lymphoma
The most likely infectious cause of his gastrointesti- C. Reactivation of hepatitis B infection
nal illness was: D. Isoniazid toxicity
E. Aor D
A. Campylobacter F. C or D
B. Giardia
C. Salmonella 9. A34-year-old female teacher presents with 1 week of
D. Cryptosporidium severe cough. Her symptoms began 2 weeks prior with
E. Escherichia coli O157:H7 a mild fever and rhinorrhea. She is admitted to hospi-
tal with suspicion of community-acquired pneumo-
7. A24-year-old woman calls your office complaining of nia. On hospital day 2, the intern raises the possibility
burning with urination, and increased urinary urgency of Bordetella pertussis, given that the patient has been
and frequency. She has no fever, no nausea or vomiting, experiencing posttussive emesis every couple of hours.
and no flank pain. She has been in a monogamous rela- Polymerase chain reaction (PCR) of a respiratory
tionship for 3years, and she had one prior episode of cys- specimen is positive for Bordetella pertussis, confirm-
titis, more than a year ago. According to the 2010 IDSA ing this diagnosis. The patient had not been placed on
and International Guidelines, which of the following droplet precautions on admission to the hospital. You

2 T h e B r i g h a m I n t e n s i v e R e v i e w o f I n t e r n a l M e d i c i n e Q u e s t i o n a n d A n s w e r C o m pa n i o n
recommend the following prophylaxis for close hospital 12. A26-year-old graduate student presents for evalu-
contacts: ation after being bitten on the right shin by her neigh-
bors playful Labrador puppy. Her past medical history
A. Amacrolide antibiotic is notable for undergoing splenectomy for treatment of
B. No prophylaxis is needed given that the patient has idiopathic thrombocytopenic purpura at age 24, which
been symptomatic for >1 week. was curative. She takes no medications and has no aller-
C. Afluoroquinolone antibiotic gies. On examination she has normal vital signs and
D. Administration of the pertussis vaccine to prevent appears well. On the right shin are two puncture marks,
secondary cases which are still bleeding slightly. There is no purulence
E. Administration of immunoglobulin for passive and no surrounding erythema of the skin.
immunization The most appropriate management is:

10. The criteria for prophylactic doxycycline 200 mg A. Sequester the puppy and treat the patient with
orally 1 against Lyme disease include all of the fol- human rabies immunoglobulin
lowing except: B. Oral trimethoprim-sulfamethoxazole
C. No therapy is necessary
A. Attached tick is identified as a deer tick, Ixodes D. Intravenous ceftriaxone
scapularis, estimated to have been attached for at E. Oral amoxicillin-clavulanate
least 36 hours on the basis of engorgement or of
certainty about the time of exposure 13. A67-year-old female is recovering following an elec-
B. There is no potential risk for anaplasma or babesia tive total hip arthroplasty. On postoperative day 5, she
coinfection. complains of worsening diarrhea and abdominal pain.
C. Prophylaxis can be started within 72 hours of tick Her white blood cell count has risen from 9,000 to
removal. 21,000 cells/L. Testing of the stool reveals a positive
D. Local rate of infection of ticks with Borrelia glutamate dehydrogenase antigen and a positive toxin
burgdorferi is 20%; it generally occurs in parts of A/B assay confirming the diagnosis of Clostridium dif-
New England, in parts of the mid-Atlantic States, ficile. The team notes that this is the third patient with
and in parts of Minnesota and Wisconsin. Clostridium difficile infection on their unit in the past 2
E. Doxycycline treatment is not contraindicated due to weeks. To help reduce further horizontal transmission,
concurrent condition, such as pregnancy. you recommend:

11. A 53-year-old woman with well-controlled non- A. Identification and treatment of asymptomatic
insulin-dependent diabetes, obesity, and chronic carriers
lower-extremity edema presented 4days previously with B. Strict adherence to standard precautions, including
erythema, swelling, and pain of the right lower leg. She hand hygiene with an alcohol hand rub
was treated empirically with dicloxacillin, but her symp- C. Use of a chlorine-containing cleaning agent to
toms worsened, and a fluctuant collection developed in address environmental contamination
a skin fold on her leg. She presented to the emergency D. Contact precautions for any patient suspected of
room, where she was found to have a low-grade fever having Clostridium difficile infection until 48 hours
but was otherwise stable. The collection was lanced, of therapy have been given
producing a small amount of purulent material, which E. Prophylactic metronidazole for all patients on the
was drained and swabbed for culture. The culture of the unit
wound grew Staphylococcus aureus, which was resistant
to penicillin and oxacillin/methicillin but otherwise 14. A 49-year-old incarcerated man is diagnosed with
susceptible to all other antibiotics that were tested. In latent tuberculosis infection. He has no evidence of
addition to excellent wound care, all of the following active disease. Options for treatment include all of the
antibiotics would be appropriate choices for outpatient following except:
single-drug treatment at this point except:
A. Isoniazid 300 mg daily for 9months
A. Linezolid B. Directly observed rifapentine 900 mg and isoniazid
B. Doxycycline 900 mg weekly for 3months
C. Trimethoprim-sulfamethoxazole C. Rifampin 600 mg daily for 4months
D. Clindamycin D. Rifampin 600 mg and pyrazinamide 1750 mg daily
E. Rifampin for 2months

1. I n f e c t i o u s D i s e a s e s 3
15. A43-year-old woman calls to state she is very con- D. HAV IgG positive, HBSAb positive, HBSAg
cerned because she found a large lump under her left negative, HBcAb negative, HCV Ab negative
arm. She has no personal or family history of malig- E. HAV IgG negative, HBSAb negative, HBSAg
nancy. She has no other complaints. On examination positive, HBcAb negative, HCV Ab negative
her vital signs are normal. She has one tender 2 3cm
mobile mass under her left arm, with overlying ery- 17. A 45-year-old woman was diagnosed with HIV
thema. She works at an urban animal shelter. At home infection (224 CD4 T cells/m3) and smear-negative,
she has two cats, a dog, and a rabbit. The rabbit lives culture-positive pulmonary tuberculosis after present-
indoors but had been sick with ileus. The dog has been ing with chronic cough. Chest computed tomography
well. One of her young cats (adopted at 5 weeks from the (CT) showed mild left lower lobe interstitial infiltrates.
animal shelter) was ill about 2months earlier, and she Rapid tuberculosis drug susceptibility testing showed
tried force feeding it antibiotics, and due to this she was no evidence of drug resistance. The patient was started
bitten and scratched repeatedly. on isoniazid, rifampin, ethambutol, and pyrazinamide.
Among cat-associated zoonoses, the most likely After 2 weeks, the patient was started on antiretroviral
pathogen in this case is: therapy with emtricitabine/tenofovir and efavirenz. Her
cough initially improved but then worsened after about
A. Bartonella henselae 1month of tuberculosis treatment. She also developed
B. Francisella tularensis progressive shortness of breath. A chest X-ray showed
C. Toxocara cati new extensive right upper lobe opacities. Sputum smear
D. Toxoplasma gondii microscopy was negative.
E. Yersinia pestis The patient should now:

16. A 42-year-old woman presents to the urgent care clinic A. Switch to an empiric regimen for treatment of
complaining of 2 weeks of progressive abdominal pain, multidrug-resistant tuberculosis.
bloating, nausea, and vomiting. Three weeks earlier she B. Initiate systemic corticosteroids to control symptoms
developed a severe reaction to poison ivy while hiking, of paradoxical tuberculosis immune reconstitution
and she was prescribed a 7-day course of high-dose pred- inflammatory syndrome.
nisone. Around the time of completion of the prednisone C. Enroll in directly observed tuberculosis treatment
she started to develop abdominal discomfort and nausea, given the high likelihood of poor adherence to therapy.
which are now significant. She has a history of hyper- D. Begin trimethoprim/sulfamethoxazole for treatment
tension and depression, for which she takes lisinopril of Pneumocystis jerovici infection.
and citalopram regularly. She immigrated to the United
States from Vietnam 10 years previously, lives with her 18. A 45-year-old man presents to the emergency room
husband and two children, and works as an office man- with 2 days of severe headache over the left eye and
ager. She previously smoked cigarettes but quit several fevers. He reports nearly constant tearing of the left eye
years ago, she does not drink alcohol, and she has never and tenderness over the left eye for 24 hours. His medi-
used recreational drugs. Family history is significant for cal history is notable for type 1 diabetes mellitus com-
a sister with diabetes mellitus and mother with hepatitis plicated by peripheral neuropathy and chronic kidney
B. On physical examination she appears uncomfortable, disease. His examination is notable for an ill-appearing
has mild tachycardia and tachypnea, and she is tender in middle-aged man with chemosis of the left eye and peri-
the right upper quadrant. Laboratory data are notable orbital and palpebral erythema extending over the left
for WBC 12,000/L with 82% neutrophils, hemato- side of the nose. Laboratory data are notable for a blood
crit 36%, platelets 140,000/L, electrolytes are normal, glucose of 527 mg/dL. Urinalysis is remarkable for
BUN 11 mg/dL, creatinine 0.9 mg/dL, AST/SGOT 524/ ketones and glucose. CT of the sinuses reveals thicken-
L, ALT/SGPT 789/L, alkaline phosphatase 247/L, ing of the left frontal, sphenoid, and ethmoid sinuses.
total bilirubin 1.9. Serologies for viral hepatidities are Endoscopic evaluation by an otolaryngologist reveals a
most likely to show: black eschar over the left middle turbinate.
In addition to treatment with insulin for diabetic
A. HAV IgG positive, HBSAb positive, HBSAg ketoacidosis, appropriate management includes:
positive, HBcAb positive, HCV Ab positive
B. HAV IgG positive, HBSAb negative, HBSAg A. Voriconazole
positive, HBcAb positive, HCV Ab negative B. Lipid formulation of amphotericin B
C. HAV IgG positive, HBSAb negative, HBSAg C. Caspofungin
negative, HBcAb positive, HCV Ab positive D. Fluconazole

4 T h e B r i g h a m I n t e n s i v e R e v i e w o f I n t e r n a l M e d i c i n e Q u e s t i o n a n d A n s w e r C o m pa n i o n
19. Antibiotic treatment is often indicated for persons chemotherapy with cytarabine and daunorubicin
with deeply penetrating animal bites. All of the follow- 11 days ago has recurrent fever to 103.1F associated
ing are acceptable antibiotic choices except: with malaise and sweats. She also endorses a sore mouth
and throat as well as mild diarrhea, which she attributes
A. Amoxicillin/clavulanate to her recent chemotherapy. Her labs are notable for
B. Ampicillin/sulbactam an absolute neutrophil count of 28. She first developed
C. Ciprofloxacin fever and neutropenia 8days ago and was treated with
D. Imipenem empiric cefepime. Her fevers resolved within 24 hours
E. Piperacillin/tazobactam and she remained afebrile until 2days ago. When fevers
redeveloped, blood cultures were drawn and empiric
20. A 25-year-old woman presents to the emergency caspofungin was added 2days ago; yeast is now growing
department with fever and back pain. The patient has in one of four blood culture bottles. Based on the infor-
been using intravenous heroin for the past few years; she mation given, possible pathogens are:
had one prior episode of soft tissue abscess after injec-
tion but no other illnesses in the past. She now com- A. Aspergillus fumigatus
plains of 2 weeks of progressive lower back pain and B. Candida albicans
fevers. On examination she is tachycardic, diaphoretic, C. Cryptococcus neoformans
febrile (102F), and ill appearing. Cardiac examination D. B or C
reveals a new systolic murmur. There is marked tender- E. A, B, or C
ness to palpation over the lumbar spine. Blood is drawn
for basic labs and blood cultures (two sets). Given his 23. A 22-year-old man presents to his primary care physi-
ill appearance, the admitting physician decides to start cians office with fever and sore throat. The patient was
empiric antibiotics for the most likely pathogens imme- feeling well until 1 week ago when he developed fever,
diately. The best empiric antibiotic regimen for this malaise, fatigue, and sore throat; he later developed diar-
patient is: rhea. He has lost 5 pounds in the past month. The patient
is a student at a local college, he does not smoke, drinks
A. Vancomycin + ceftriaxone alcohol a few times per week, and does not use any rec-
B. Vancomycin + gentamicin + rifampin reational drugs. He is sexually active with men and has
C. Nafcillin + gentamicin had two new males partner over the past few months. He
D. Vancomycin + caspofungin is up to date on all of his vaccinations. On examination
E. Ampicillin + gentamicin he is thin, febrile, and mildly tachycardic. His orophar-
ynx is erythematous without exudates; he has palpable
21. A 47-year-old male with quadriplegia secondary to small lymphadenopathy in bilateral cervical and groin
a motor vehicle accident as a young adult presents with distribution. He has a maculopapular rash over his chest
fever, fatigue, and foul-smelling urine. He has a chronic and back. Laboratory data: WBC 2,200/L, hematocrit
indwelling Foley catheter due to urinary retention, and 30%, platelets 103,000/L, BUN 20 mg/dL, creatinine
he reports multiple hospitalizations for catheter-associ- 1.0 mg/dL, AST/AGOT 66/L, ALT/SGPT 72/L,
ated urinary tract infections. On arrival to the emergency alkaline phosphatase 120/L, total bilirubin 0.9 mg/dL.
department, he is noted to be confused with a tempera- Blood cultures are drawn. Which of the following is the
ture of 102.3F, a heart rate of 116 beats per minute, and most appropriate panel of tests to order next:
a blood pressure of 75/43 mm Hg. Urinalysis finds 3+
leukocyte esterase and positive nitrite with microscopy A. Hepatitis A, B, and C serologies
revealing 100200 WBC/hpf with 3+ bacteria. After B. Urine gonorrhea and chlamydia probes, syphilis
diagnosing urosepsis, you recommend: serology
C. Rheumatic factor (RF), echocardiogram
A. Administration of broad-spectrum antibiotics D. Blood smear, LDH, bone marrow biopsy
within 4 hours E. Blood smear, EBV serologies, CMV serologies, HIV
B. Initiation of dopamine as a vasopressor antibody, HIV RNA
C. Administration of intravenous hydrocortisone
D. Administration of 30 mL/kg of crystalloid 24. A 38-year-old male presents to the emergency
E. Initiation of norepinephrine as a vasopressor department complaining of shortness of breath and
cough productive of thick sputum that has been wors-
22. A 23-year-old woman with acute myeloid leu- ening over the past 2days. He is noted to have a room
kemia (AML) who underwent standard induction air oxygen saturation of 89% and a respiratory rate of

1. I n f e c t i o u s D i s e a s e s 5
35 breaths per minute. Achest X-ray reveals consolida- 27. A 37-year-old woman with HIV (CD4 count 523/
tion in the left lower lobe of his lung. On review of his L, HIV viral load <50/L), asthma, and allergic rhi-
past medical history you learn that he has severe type nitis presents to her primary care physician office
1 diabetes mellitus complicated by nephropathy requir- complaining of worsening symptoms of allergic rhi-
ing hemodialysis for the past year. Based on clinical nitis. She has had worsening rhinorrhea, itchy and
and radiologic evidence suggesting pneumonia, you watery eyes, and dry cough over the past few weeks this
recommend: spring. Her current medications are tenofovir, emtric-
itabine, darunavir, ritonavir, loratadine, and injectable
A. Levofloxacin medroxyprogesterone. She requests an additional med-
B. Ertapenem plus vancomycin ication for control of her allergic rhinitis symptoms.
C. Piperacillin/tazobactam plus amikacin Which of the following drugs should not be pre-
D. Imipenem plus ciprofloxacin scribed because of a potentially harmful drugdrug
E. Cefepime plus levofloxacin plus vancomycin interaction:

25. A 21-year-old college student with moderate to A. Oral cetirizine


severe asthma presents to the student health center B. Inhaled albuterol
with a sore mouth for several days. Though it is bother- C. Ophthalmic nedocromil
some, it has not interfered with his eating or drinking. D. Inhaled fluticasone
He denies odynophagia. Ten days ago he was hospi- E. Oral montelukast
talized for 2 days with a severe asthma flare. He was
treated with a 2-week prednisone taper for his asthma 28. A 48-year-old man presents with 4 days of fevers,
and with a 5-day course of azithromycin for possible nightsweats, and malaise. He has a history of aortic
respiratory tract infection. His regular medications valve replacement with a bovine prosthetic valve 8years
include inhaled salmeterol and fluticasone. On exami- previously for aortic valve dysfunction due to congeni-
nation he is a relatively well-appearing young man tal bicuspid aortic valve. On examination he is febrile
with white curd-like plaques adherent to the soft and (102.4F) and tachycardic. Blood cultures are drawn.
hard palate. A transthoracic echocardiogram does not show any
Appropriate management includes any or all of the valvular vegetations or abscess. Which of the follow-
following except: ing pathogens, if found on blood culture in this patient,
would be least consistent with a diagnosis of bacterial
A. Oral nystatin swish and swallow endocarditis in this patient:
B. Education about rinsing the mouth after using a
steroid inhaler A. Haemophilus parainfluenzae
C. Consideration of HIV testing B. Eikenella corrodens
D. Oral voriconazole C. Streptococcus bovis
E. Clotrimazole troches D. Bacteroides fragilis
E. Enterococcus faecalis
26. A 26-year-old female is hospitalized in a burn
unit after suffering deep burns over 55% of her body 29. A 28-year-old woman presents to the emergency
in a house fire. She remains intubated with access department with fever, malaise, abdominal pain, nau-
via central venous catheter. On hospital day 10, the sea, and vomiting. Initial labs show WBC 1,800/L,
microbiology lab calls with the result of a blood cul- hematocrit 27%, platelets 100,000/L, AST/SGOT
ture that was sent in the setting of a fever to 102F. 170/L, ALT/SGPT 195/L, alkaline phosphatase 422,
The lab reports that Acinetobacter baumannii is and total bilirubin 2.1 mg/dL. ACT scan of the abdo-
growing in the blood, which is testing positive for men and pelvis reveals diffuse lymphadenopathy and
extended-spectrum beta-lactamases and a carbapen- hepatosplenomegaly. Further diagnostics reveal HIV
emase. Based on these results, you select the follow- antibody is positive with a CD4 count of 18/L (4%
ing antibiotic as most likely to be effective against this of total lymphocytes). Biopsy of a lymph node reveals
pathogen: numerous organisms on acid-fast staining. PCR of the
sample with numerous organisms is consistent with
A. Meropenem mycobacterium avium intracellulare (or mycobacte-
B. Colistimethate rium avium complex [MAC]). The patient is started on
C. Trimethoprim-sulfamethoxazole treatment with clarithromycin and ethambutol while
D. Ciprofloxacin awaiting the results of resistance testing from mycobac-
E. Piperacillin/tazobactam terial isolator blood cultures.

6 T h e B r i g h a m I n t e n s i v e R e v i e w o f I n t e r n a l M e d i c i n e Q u e s t i o n a n d A n s w e r C o m pa n i o n
When is the best time to start antiretroviral therapy nausea. On the morning of presentation he was found
for this patient: in his room confused. In the emergency room he was
febrile and confused, and became somnolent during
A. Immediately; start the same day as clarithromycin his care there. Head CT showed no acute processes.
and ethambutol Alumbar puncture was performed, cerebrospinal fluid
B. Afew days after initiation of clarithromycin and (CSF) examination showed glucose 22 mg/dL, protein
ethambutol but within 2 weeks 97 mg/dL, RBC 7, WBC 489 with 87% neutrophils,
C. After 2 weeks on clarithromycin and ethambutol, if 7% monocytes, and 6% lymphocytes. CSF is sent to the
no side effects Microbiology lab for gram stain and culture.
D. After completion of 6 weeks of clarithromycin Which of the following is the most appropriate
and ethambutol, to decrease risk of immune empiric antibiotic regimen to initiate while awaiting
reconstitution inflammatory syndrome (IRIS) the results of the CSF gram stain and culture:
E. After the patient is discharged from the hospital,
in the outpatient setting with documented patient A. Vancomycin, ceftriaxone, ampicillin
capacity for adherence to follow-up and medications B. Vancomycin, cefepime, acyclovir
C. Vancomycin, imipenem
30. A49-year-old man with a long-standing history of D. Ampicillin, gentamicin, acyclovir
well-controlled HIV presents to his primary care phy- E. Ampicillin, sulfamethoxazole-trimethoprim,
sician for a routine visit. He is feeling well, with no amphotericin B
symptomatic complaints. He reports 100% adherence
to his antiretroviral regimen:tenofovir, emtricitabine, 32. A 27-year-old woman presents to her primary care
and efavirenz. He works in real estate and lives with his physician complaining of fever, cough, sinus pressure,
husband and their dog. He smokes approximately one and malaise. She reports onset of symptoms 6 days ago,
pack of cigarettes per day, as he has for the past 20years. with fevers for the first 2 days with temperatures as high
He drinks six alcoholic beverages per week, and he does as 102.1F. Highest temperature in the past 4 days has
not use recreational drugs. Vital signs: heart rate 82 been 99.8F. She reports ongoing symptoms of frequent
beats per minute, blood pressure 137/86mm Hg, BMI cough productive of scant white sputum, nasal conges-
32 kg/m2, and physical examination is unremarkable. tion, mild sinus pressure, and fatigue. The patient has
Labs show CD4 count 470/L, HIV viral load is <20/L a history of childhood asthma, but she has not used
(undetectable), and CBC and chem-20 are normal. You inhalers or other medications for asthma in several
plan to continue his current antiretroviral regimen and years. She is otherwise healthy, and she takes no medi-
you discuss additional health care maintenance efforts cations. She has a history of a rash reaction to clarithro-
with him. mycin. On examination, her temperature is 99.0, heart
Given his history, which of the following is the most rate is 92 beats per minute, blood pressure 132/85 mm
important health care maintenance item to pursue dur- Hg, respiratory rate 18 breaths per minute, and oxy-
ing this visit: gen saturation is 96% on room air. She appears mildly
uncomfortable and is coughing during the examina-
A. Start sulfamethoxazole/trimethoprim for tion; she has no lesions in the oropharynx, sclerae are
prophylaxis against Pneumocystis pneumonia. clear, maxillary sinuses are mildly tender on percussion,
B. Counsel patient to decrease alcohol intake and refer no nasal discharge/drainage, tympanic membranes are
for alcohol dependence treatment. clear, and she has a few small (<1.5 cm) palpable cervical
C. Counsel patient to quit smoking and discuss lymph nodes. Her lungs are clear except for occasional
medications and supports for smoking cessation. faint expiratory wheezes.
D. Screen for toxoplasma serostatus to determine risk The most appropriate management is:
for toxoplasma reactivation in the future.
E. Refer for early colon cancer screening by A. Oral moxifloxacin, inhaled albuterol, and intranasal
colonoscopy. oxymetazoline
B. Oral amoxicillin-clavulanate, inhaled fluticasone,
31. An 84-year-old man with a history of coronary artery and saline nasal irrigation
disease, diabetes mellitus, and chronic renal insuffi- C. Oral tessalon perles, inhaled albuterol, and
ciency presents to the emergency room with progres- intranasal fluticasone
sive headache, fevers, and neck pain. The patient was D. Intramuscular ceftriaxone, oral pseudophedrine, and
in his usual state of health until 4days prior when the inhaled salmeterol
staff and his friends at his assisted living facility noted E. Intramuscular influenza vaccine, oral guaifenesin,
he started complaining of feeling ill with headache and and inhaled tiotropium

1. I n f e c t i o u s D i s e a s e s 7
33. A 29-year-old woman who is otherwise healthy multiple shallow ulcerations grouped in the right
presents for a routine prenatal visit at 14 weeks gesta- perianal area. There is no rectal discharge and no pal-
tional age. She is feeling well and has no symptomatic pable lymphadenopathy, but the ulcerations are ten-
complaints, and physical examination is consistent with der and painful even when not palpated. The type of
normal pregnancy, otherwise unremarkable. She has diagnostic test that is most likely to confirm the diag-
routine prenatal laboratories checked, which show the nosis of this active condition is:
following results:hemoglobin 11.2 g/dL, Rubella IgG
positive, HIV antibody negative, Treponemal IgG (by A. Bacterial culture of a swab of the ulcers
EIA) positive. Follow-up RPR is also positive, with a B. Viral culture of a swab of the ulcers
titer of 1:16, and FTA-ABS is also positive. The patient C. Urine NAAT probe
has never had prior syphilis testing. She reports a his- D. Blood serologic test
tory of severe allergy to penicillin with a feeling of E. PCR of a swab of the ulcers
throat closing.
The most appropriate management for this patient is: 36. A 47-year-old man presents to the emergency
department with complaint of hemoptysis. He was born
A. No treatment now due to risk of toxicity; follow and raised in upstate NewYork, currently works as an
clinically and repeat syphilis testing at 20 weeks investment banker, and is married with two children.
gestational age He has been healthy until approximately 6months prior
B. Treat with doxycycline 100 mg orally twice daily for when he developed sinusitis. He has been treated by his
a 21-day course primary care physician with courses of amoxicillin,
C. Treat with ceftriaxone 1 g IM once daily for 10 days amoxicillin-clavulanate, and moxifloxacin for episodes
D. Treat with azithromycin 2 g orally in a single dose of sinusitis over the past few months, but his symptoms
E. Allergy consultation and admission for persist. Over the past 2 weeks he developed a cough,
desensitization to penicillin in order to facilitate which was initially nonproductive, but during the past
treatment with benzathine penicillin G 2.4 million 2days he had a few episodes of hemoptysis. On physi-
units intramuscularly once weekly for 3 weeks cal examination he has lost 5 lb since his last examina-
tion 1month prior, he is thin but comfortable, and on
34. A24-year-old man with well-controlled HIV pres- sinonasal examination he has septal perforation with
ents for routine follow-up primary care visit and notes no obvious exudates or other abnormalities. Blood tests
some dysuria for the past several days. Physical exami- show white blood cell count of 4,200/L, hemoglobin
nation is unremarkable. Urinalysis shows 10 WBC, no 9.1 g/dL, and creatinine 2.2 mg/dL. Chest X-ray shows
epithelial cells, 2 RBC, no bacteria per high-powered some abnormal opacities, so chest CT is obtained,
field (HPF). Urine testing for gonorrhea is positive by which shows multiple pulmonary nodules.
NAAT probe. The patient has no known drug allergies. The test most likely to confirm the diagnosis in this
All of the following treatments are appropriate, except: case is:

A. Levofloxacin 500 mg orally once daily for 7 days plus A. Sputum smear for acid-fast bacilli (AFB)
doxycycline 100 mg orally twice daily for 7 days B. AFB smear of biopsy of a pulmonary nodule
B. Cefixime 400 mg orally single dose plus C. Serum test for antineutrophilic cytoplasmic
azithromycin 1 g orally single dose antibodies (ANCA)
C. Ceftriaxone 250 mg IM single dose plus doxycycline D. Serum test for galactomannan
100 mg orally twice daily for 7 days E. Serum interferon-gamma release assay (IGRA)
D. Cefixime 400 mg orally single dose plus doxycycline
100 mg orally twice daily for 7 days 37. A62-year-old woman with multiple sclerosis and a
E. Ceftriaxone 250 mg IM single dose plus neurogenic bladder now has a chronic indwelling uri-
azithromycin 1 g orally single dose nary catheter, after failing management with intermit-
tent use of urinary straight catheters. She presents for a
35. A 32-year-old man presents to his primary care routine primary care visit and has no current symptom-
physician complaining of painful perianal lesions. atic complaints. She is interested in discussing strate-
The patient is sexually active with multiple male gies to decrease the risk of urinary tract infections in
partners. He has had negative screening for sexually the future. In the past when she has developed urinary
transmitted diseases, including HIV, gonorrhea, and tract infections they often precipitate a worsening of
chlamydia, in the pasthis last screening tests were her multiple sclerosis, and she often requires hospital-
6 months ago. On physical examination he is over- ization, so she hopes to prevent need for hospitalization
all well appearing, but in the perianal area he has in the future.

8 T h e B r i g h a m I n t e n s i v e R e v i e w o f I n t e r n a l M e d i c i n e Q u e s t i o n a n d A n s w e r C o m pa n i o n
Which of the following strategies would be most suc- D. Disinfect emptied cat-litter box with near-boiling
cessful at achieving her goals: water before refilling
E. Wash fruits and vegetables before eating
A. Monitoring for early signs and symptoms of urinary
tract infection, with expedited early urinalysis, 40. A 74-year-old man with poorly controlled diabe-
urine culture, and empiric treatment while awaiting tes, coronary artery disease, end-stage renal disease,
culture results when symptoms develop and peripheral vascular disease presents with pain at a
B. Routine screening with urinalysis for pyuria at chronic foot ulcer site. The patient has had a nonheal-
regular intervals with early empiric treatment for ing ulcer on his left great toe for several months with
urinary tract infection if pyuria is detected, even in intermittent drainage but no other symptoms. Over the
the absence of symptoms past few days he developed increasing purulent drain-
C. Addition of gentamicin solution to the catheter age from the ulcer bed, as well as erythema, pain, and
drainage bag at regular intervals swelling of the toe, which is now tracking up the foot.
D. Chronic prophylaxis with methenamine salts to The margins of the toe ulcer have also started to turn
decrease bacteria black. He has a low-grade fever with temperature of
E. Chronic prophylaxis with ciprofloxacin to decrease 100.7F at the time of presentation. Blood cultures are
bacteria and infections sent and a swab of the discharge from the ulcer is sent
for culture as well. Which of the following antibiotic
38. A 45-year-old man with a history of prior open regimens would be appropriate initial therapy for this
reduction and internal fixation (ORIF) of a left femur patient while awaiting culture results:
fracture in the past now presents with his third episode
of cellulitis in the left leg. He was well until 1day prior A. Vancomycin alone
when he developed sudden onset of malaise, fever, nau- B. Vancomycin and ampicillin-sulbactam
sea, and erythema and pain in the left leg. He presented C. Vancomycin and ceftriaxone
to the emergency room overnight and was treated D. Vancomycin and ceftazidime
empirically with vancomycin overnight. He improves E. Linezolid and ertapenem
gradually overnight. In discussion the following day he
asks whether there are any strategies to decrease the fre- 41. A 67-year-old woman presents to the emergency
quency of his episodes of cellulitis in the future. Which room with right ear pain. She is otherwise healthy at
of the following antibiotics, when taken regularly as baseline and takes no medications. She lives in New
prophylaxis, has been shown to decrease the incidence Hampshire, spends time walking her dog in the woods
of cellulitis among patients with recurrent cellulitis: frequently, but does not remember any specific tick
bites. Approximately 10 days earlier she developed
A. Sulfamethoxazole-trimethoprim upper respiratory infection (URI) symptoms, which
B. Doxycycline improved over a few days and then resolved after 7days.
C. Clarithromycin Over the past 34days she developed right ear pain. On
D. Levofloxacin physical examination she is noted to have a right facial
E. Penicillin droop and some lesions with serous drainage in the right
external ear canal. Her mucous membranes are dry.
39. A 22-year-old woman with well-controlled HIV Which of the following is the most appropriate treat-
infection is now pregnant and presents for routine pre- ment for this patient:
natal care at 12 weeks gestational age. She is taking
her antiretroviral regimen of tenofovir, emtricitabine, A. Doxycycline 100 mg orally twice daily for 14 days
ritonavir, and atazanavir with 100% adherence and no B. Valacyclovir 1000 mg orally three times daily for
significant side effects. Her most recent CD4 count was 14days
373/L and HIV viral load was undetectable. Prior lab- C. Ciprofloxacin otic solution to the right ear 4 times
oratory studies have shown Toxoplasma IgG negative. per day for 7days
Which of the following is not a recommended strat- D. Ciprofloxacin otic solution plus amoxicillin-
egy for primary prevention of acute Toxoplasmosis dur- clavulanate 875/125 mg orally twice daily for
ing pregnancy: 10days
E. Prednisone 60 mg orally once daily for 5 days
A. Cook meat to well done
B. Avoid ingestion of pork or any pork products 42. A26-year-old man presents with dysuria, which has
C. Avoid contact with materials potentially been persistent for more than 1 week. Urinalysis shows
contaminated with cat feces 12 WBC, 1 RBC, and no bacteria HPF, respectively,

1. I n f e c t i o u s D i s e a s e s 9
and the urine culture is negative. Urine NAAT probes E. Antiretroviral therapy should be offered to all
for chlamydia and gonorrhea are negative. Serum test- HIV-positive patients regardless of CD4 count
ing for HIV 1 and 2 antibodies are negative. Which of because the benefits often outweigh the risks even at
the following organisms is least likely responsible for high CD4 counts.
the patients symptoms:
45. A55-year-old man with a history of hypertension,
A. Trichomonas vaginalis coronary artery disease, and depression presents with
B. Herpes simplex virus a soft tissue infection on the leg. He has no prior his-
C. Mycoplasma genitalium tory of soft tissue infections. His current medications
D. Haemophilus ducreyi are lisinopril, clopidogrel, aspirin, atorvastatin, and
E. Ureaplasma urealyticum duloxetine. He has no known medication allergies. On
examination he has a fever, temperature is 101.2F, but
43. A 51-year-old woman with rheumatoid arthritis vital signs are otherwise normal. There is an area of ery-
initiated treatment with infliximab after having nega- thema on the lower left leg, originating from a punctate
tive screening tests for tuberculosis and HIV. Five wound where the patient states he sustained a spider
weeks after initiation of infliximab therapy she devel- bite. The area is warm, swollen, tender, and there is a
oped cough and shortness of breath, which persisted central 4cm area of fluctuance. Incision and drainage
despite empiric treatment with azithromycin by her pri- of the abscess is performed at the bedside and puru-
mary care physician. She is evaluated in the emergency lence is drained and irrigated. Aculture of the drained
department, where she is found to have a temperature fluid grows methicillin-resistant Staphylococcus aureus
of 100.6F, respiratory rate of 38 breathes per minute, (MRSA). The patient is treated with vancomycin while
and oxygen saturation of 85% while breathing ambient an inpatient and is then discharged with oral linezolid
air. Achest X-ray shows bilateral interstitial infiltrates. for another 10days. After 7days he returns to the emer-
Serum beta-glucan was >500 pg/mL. The diagnosis in gency room complaining of fever and malaise. His
this case is confirmed by toluidine blue staining of cysts temperature is 103.7F, heart rate 116 beats per minute,
from a bronchoalveolar lavage specimen. blood pressure 180/92 mm Hg, sat 98% on room air.
Which of the following treatments is most appropri- On examination he is somewhat agitated and unable
ate for this patient: to sit still. Lungs are clear to auscultation bilaterally,
there are no murmurs on cardiac exam, abdomen is soft
A. Ivermectin and nontender, and the lower left leg prior incision and
B. Trimethoprim-sulfamethoxazole drainage (I&D) site is healing well with no residual ery-
C. Trimethoprim-sulfamethoxazole plus prednisone thema or fluctuance.
D. Liposomal amphotericin The most likely cause of this patients new
E. Liposomal amphotericin plus prednisone symptomsis:

44. A27-year-old man presents for follow-up visit after A. Drugdrug interaction
recent routine health maintenance screening. He is B. Recurrent MRSA abscess
sexually active with men, with three partners in the past C. MRSA bacteremia due to endocarditis
year. He reported 80% condom use during his last pri- D. Hospital-acquired pneumonia
mary care appointment. As a part of that appointment E. Clostridium difficile colitis
a routine screening HIV antibody test was sent and
returned positive. Additional tests revealed CD4 count 46. A 53-year-old man with a prior history of splenec-
526/L and HIV viral load (by PCR) of 12,436/L. The tomy after a splenic laceration in a motor vehicle accident
patient now presents for discussion of further HIV care. 10 years previously now presents with fever and chills.
What is the appropriate counseling to give regarding The patient spent a week on Nantucket island for a sum-
initiation of antiretroviral therapy (ART): mer vacation and was feeling well until 2 days after she
returned from vacation, when she developed fevers, with
A. Antiretroviral therapy is indicated only when an temperature as high as 103.2F, shaking chills, head-
opportunistic infection develops. ache, and neck stiffness. She presented to a local hospi-
B. Antiretroviral therapy is indicated only when the tal, where initial complete blood count showed: WBC
CD4 count falls below 250/L. 12,000/L, hemoglobin 8.2 g/dL, platelets 80,000/
C. Antiretroviral therapy is indicated only when the L, normal electrolytes, serum creatinine 2.1 mg/dL,
CD4 count falls below 350/L. and alkaline phosphatase 206/L. A blood smear
D. The risks of antiretroviral therapy outweigh the showed parasites within the red blood cells; the parasit-
benefits when the CD4 count is greater than 500/L. emia burden was assessed as 12%.

10 T h e B r i g h a m I n t e n s i v e R e v i e w o f I n t e r n a l M e d i c i n e Q u e s t i o n a n d A n s w e r C o m pa n i o n
Which of the following is the most appropriate treat- eventually discharged to a rehab facility to complete a
ment regimen for this patient at this point: course of vancomycin, gentamicin, and rifampin. After
3 weeks he returns to the emergency room with an acute
A. Quinine and clindamycin stroke, which appears embolic on MRI/MRA of the
B. Quinine and clindamycin and initiation of red cell brain. Laboratories show WBC 12,000/L; hemoglo-
exchange transfusion bin 9.8 g/dL, platelets 167,000/L, creatinine 0.9 mg/
C. Ceftriaxone and doxycycline and initiation of dL, liver function tests are normal, PTT 36.0 seconds,
intravenous immune globulin (IVIG) and INR 1.2. Blood cultures are negative at 48 hours of
D. Azithromycin and atovaquone and initiation of incubation.
plasmapheresis The most likely cause of the patients new stroke is:
E. Azithromycin and atovaquone
A. Persistent infectious vegetation on the prosthetic
47. A36-year-old man with psoriasis, for which he takes aortic valve
methotrexate, traveled to Arizona for 2 weeks for a fam- B. Toxicity from gentamicin
ily reunion and developed a fever 2days prior to return- C. Aortic valvular dysfunction due to perivalvular
ing to his home in NewYork. He was feeling well during abscess
the trip and enjoyed the first week of the reunion. He D. Hypercoaguable state due to loss of gut flora while
bunked with extended family, including small children on antibiotics
and two dogs. He ate food at the hotel and drank primar- E. Drugdrug interaction of warfarin with rifampin
ily bottled water. He went hiking in the desert on three
occasions. He swam in the hotel pool, but engaged in 49. A 53-year-old woman who is otherwise healthy
no fresh water swimming. It was dry weather through- develops fever, headache, malaise, and then cough,
out the trip and very windy at times. Two days prior to which is persistent and worsens over a couple of
returning home he developed fever, fatigue, malaise, days. The patient lives in Missouri, where she works
dry cough, and chest pain. He took acetominophen and on a dairy farm, and also tends sheep for wool as an
rested, then returned home to NewYork. After 10days additional source of income. She spends her spare
the symptoms had not significantly improved, so he time hunting deer and rabbits. She is married and is
presented to his primary care provider. Physical exami- monogamous with her husband. At the time of presen-
nation was remarkable only for low-grade fever. Chest tation she was mildly hypoxic and febrile, and rapidly
X-ray showed mild left hilar infiltrate and adenopathy. worsened, ultimately requiring mechanical ventila-
He was treated with a 5-day course of azithromycin with tion. Chest imaging showed multifocal infiltrates
no change in his symptoms. and progressive pleural effusions, as well as hilar
The best diagnostic test to send at this point lymphadenopathy. At the time of admission her labs
wouldbe: were normal with the exception of WBC 14,000/L.
However, she developed progressive renal failure and
A. Blood serologic test abnormal liver function tests over the first 2 days
B. Sputum culture after admission. Blood, urine, and sputum cultures
C. Bronchoalveolar lavage for culture are all negative repeatedly. She has been treated with
D. Biopsy of the hilar lymph node for pathology vancomycin, cefepime, and metronidazole with no
E. CT angiography of the chest improvement.
The most likely etiologic organism for her current
48. A 31-year-old man with a mechanical prosthetic condition is:
aortic valve presents with fevers. He had his aortic valve
replaced with mechanical prosthesis at age 24 due to A. Tropheryma whipplei
congenital bicuspid aortic valve, and he has been doing B. Babesia microti
well since then. His medications are warfarin and lisin- C. Borelia lonestari
opril. He has no known drug allergies. At the time of D. Francisella tularensis
admission to the hospital blood cultures are positive E. Anaplasma phagocytophilum
for MRSA. After extensive evaluation, no source for
the bacteremia is identified. He is treated for presumed 50. A 22-year-old woman who is healthy at baseline
prosthetic valve endocarditis, even in the absence of sustains minor blunt trauma to the right thigh after
suggestive findings on transesophageal echocardio- she bumps her leg on the edge of a table. Within hours
gram. His antibiotic regimen is vancomycin, gentami- after the bump she develops fever and severe right leg
cin, and rifampin. After 3days the blood cultures begin pain such that she is barely able to walk. She presents
to be free of bacteria, he clinically improves, and is to a local emergency room, where she is febrile and

1. I n f e c t i o u s D i s e a s e s 11
hypotensive. Her right thigh appears dusky, and she The most likely gram stain result in this case would be:
complains of pain tracking down to the foot and up
to the lower abdomen. She is taken immediately to the A. Gram-positive rods that are aerobic
operating room, where operative exploration reveals B. Gram-negative rods that are anaerobic
necrotizing myositis of the muscles of the thigh with- C. Gram-positive cocci in pairs and chains, with
out gas formation, with necrotizing soft tissue infection beta-hemolysis
tracking down the leg and up to the groin. Some of the D. Gram-positive cocci in pairs and chains, with
debrided tissue is sent for expedited gram stain to aid alpha-hemolysis
with choice of antibiotics. E. Gram-positive cocci in clusters, with beta-hemolysis

1 2 T h e B r i g h a m I n t e n s i v e R e v i e w o f I n t e r n a l M e d i c i n e Q u e s t i o n a n d A n s w e r C o m pa n i o n
CH A PT ER1 A NSW ER S 3. ANSWER:C. CMV colitis

1. ANSWER:E. Dengue Infection risk after solid organ transplantation depends on


two key factors:when the transplant occurred and whether
Leptospirosis can present in many different ways, including the patient has recently been treated for organ rejection.
headache, muscle aches, and fever, but it is almost always Transplant recipients who underwent transplantation less
associated with freshwater exposure, such as swimming than a month ago are typically at highest risk for nosocomial
or white-water rafting. Although all travelers returning infections and infections that result directly from surgery (e.g.,
with fever should be evaluated for malaria (even in cases wound infection). Those transplanted 16months before or
where they had reported taking prophylaxis), the normal those recently treated for rejection are at risk for opportunis-
hematocrit and lack of other laboratory abnormalities in tic infections (such as CMV disease, PCP pneumonia, etc.).
this young woman are reassuring. Typhoid can also pres- Those transplanted more than 6months ago and not treated
ent with only headache and fever, and in fact although it for rejection recently (as in this case) are at highest risk for
is cause by Salmonella spp. it can often cause little to no community-acquired pathogens, including Salmonella and
gastrointestinal symptoms. The marked thrombocytope- norovirus. In addition, this history gives several risk favors for
nia would be unusual for this diagnosis. Hepatitis Ais also these three pathogens, including recent antibiotics exposure
a risk for travelers, but the lack of gastrointestinal symp- (increased risk for C.difficile), turtle contact (a potential car-
toms and lack of liver enzyme elevation argue against it. rier of Salmonella), and classroom exposure to young children
Dengue is now the second most common cause of fever in (a potential way to be exposed to norovirus). CMV colitis is
returning travelers (10.4%), after malaria, and dengue was unlikely here given that the patient is many years past trans-
the most frequently identified cause of systemic febrile ill- plant and has not had recent rejection.
ness among travelers returning from Southeast Asia (32%)
[Freedman. N Engl J Med. 2006;354(2):119]. Diagnosis is
generally clinical, although acute and convalescent sera can
be sent. Treatment is supportive. 4. ANSWER:B. Pneumococcal 13-valent conjugate
(PCV13) vaccine

This patient is a 25-year-old men who have sex with men


2. ANSWER:B. Suctioning of subglottic secretions (MSM) with chronic lung disease, but otherwise with no
known history of immunocompromising condition. Since
There are a number of strategies that can help reduce the risk his last tetanus vaccination was a Td at age 18, he has not
of ventilator-associated pneumonia in an intubated patient. had a Tdap vaccination since reaching adulthood. Tdap is
These usually focus around preventing aspiration, reducing recommended for all adults (age 19 and older) who have
colonization of the airway and digestive tract, and mini- not previously received Tdap or for whom vaccine status is
mizing contamination of the ventilatory circuit. To prevent unknown, and it can be administered regardless of inter-
aspiration, the head of the bead should be maintained in val since the most recent tetanus vaccination. Thus, Tdap
a semirecumbent position (elevated 3045 degrees) rather (option A) would be appropriate for this patient, even
than in a fully recumbent position. One should also use a though his last tetanus vaccination was given fewer than
cuffed endotracheal tube with cuff pressure set at 20cm 10 years previously. Hepatitis A vaccination (option C) is
H2O with subglottic secretion drainage. To reduce coloni- indicated for all MSM, so this would be appropriate for this
zation of the airway and digestive tract, orotracheal intu- patient. HPV vaccination (option D) is recommended for
bation is preferred to nasotracheal intubation to decrease all MSM through age 26 years, so this would be appropriate
the risk of sinusitis, acid-reducing medications such a his- for this patient. Influenza vaccination (option E) is recom-
tamine receptor 2 (H2)blocking agents and proton pump mended annually for all adults, so this would be appropriate
inhibitors should be avoided except in patients at high risk for this patient, and it would be especially important given
for ulcers/gastritis, and regular oral care with an antiseptic his history of asthma and pneumonia. Pnuemococcal vac-
solution (e.g., chlorhexidine) should be performed. Routine cination with the pneumococcal polysaccharide (PPSV23)
use of oral or intravenous antibiotics for prophylaxis is not vaccination would be appropriate for this patient given his
recommended. Finally, to minimize contamination of the chronic lung disease; however, the pneumococcal conju-
ventilatory circuit, the tubing should only be changed when gate 13-valent vaccination (PCV13) is recommended only
visibly soiled or malfunctioning. for adults with immunocompromising conditions. Thus,

1. I n f e c t i o u s D i s e a s e s 13
the pneumococcal conjugate 13-valent vaccine (option B) There is no single best agent that is superior to all others
would not be appropriate for this patient. Please see the for the empiric management of uncomplicated cystitis. The
CDC Guidelines for Adult Vaccination schedule (http:// choice between agent should be individualized and based
www.cdc.gov/mmwr/preview/mmwrhtml/su6201a3.htm) on the patients history, including allergies, local resistance
for more information and details. patterns, drug availability, and cost. Recommended first-line
agents that are available in the United States include nitro-
5. ANSWER:C. Chest X-ray to assess for active furantoin macrocrystals, TMP/SXT, and fosfomycin.
pulmonary disease TMP/SXT is effective and inexpensive, but allergic reac-
tions are more common, and in some US geographic areas,
The patient should undergo chest X-ray to evaluate for resistance rate among community-acquired E. coli isolates
active pulmonary tuberculosis. Radiologic disease can be exceeds 20%. Nitrofurantoin is more expensive but effective.
present in the absence of symptoms. There is no indication Fosfomycin is not as effective and is also expensive. However,
for an interferon gamma release; the positive skin test is they both exert little collateral damage. This term refers to
sufficient to diagnose latent infection. Sputum evalua- the ecological adverse effects of antibiotic agents, including
tion should only be pursued if the patient has concerning their propensity to select drug-resistant organisms and pro-
symptoms or signs of active disease on chest imaging. mote colonization and infection with multidrug-resistant
bacteria. A3-day regimen of fluoroquinolones such as cipro-
floxacin is highly efficacious in the management of uncom-
plicated cystitis. However, fluoroquinolones are no longer
6. ANSWER:A. Campylobacter recommended as first-line empiric agents due to their pro-
pensity for collateral damage. Beta lactam antibiotics tend
Campylobacter infection can result in Guillain-Barr syn- to have inferior efficacy and more adverse effects compared
drome (GBS) several weeks after diarrhea. Approximately to the recommended agents. They are, therefore, considered
1/1,000 reported Campylobacter illnesses leads to GBS and second-line agents for uncomplicated cystitis and are recom-
up to 40% of GBS in the United States may be triggered mended when other agents cannot be used.
by Campylobacter. Acute Campylobacter gastrointestinal
illness can present with diarrhea (possibly bloody), cramp-
ing, abdominal pain, and fever, sometimes with nausea and
vomiting, and can last from 2 to 10days. It is not usually 8. ANSWER:F. C orD
spread from one person to another, but most cases are asso-
ciated with contact with raw or undercooked poultry. As Rituximab is a well-described cause of hepatitis B reacti-
few as 100 organisms can cause illness. Illness can also result vation (redevelopment of high levels of circulating virus,
from contact with stool of an ill pet dog or cat. Antibiotics transaminitis, and reversion from hepatitis Be antibody
are indicated in severe cases. positive to hepatitis Be antigen positive) and can cause a
symptomatic hepatitis flare. Isoniazid can also cause sig-
nificantly elevated transaminases. In this case, either is
a possible cause of her transaminitis. Her lymphoma is
7. ANSWER:F. Options B andE unlikely to progress after she just started treatment and
transaminitis would be an atypical manifestation of pro-
Recommended gression. There is very low risk of developing hepatitis C
infection as a consequence of blood transfusion in the
Empiric Nitrofurantoin macrocrystals 100 mg orally United States (<1 in 2 million transfusions) and hepati-
twice daily 5 days (minimal resistance/collateral tis C infection does not typically cause an acute illness as
damage) or described here.
Empiric TMP/SMX DS orally twice daily 3 days if
E.colis TMP/SMX resistance rates are <20% or
Empiric Fosfomycin 3 g single dose or
Pivmecillinam 400 mg twice daily 5 days (somewhat 9. ANSWER:A. Amacrolide antibiotic
lower efficacy)
A full treatment course of a macrolide antibiotic is recom-
Not recommended mended as prophylaxis of close contacts within 21days
of exposure. The clinical incubation of Bordetella pertus-
Fluoroquinolones are no longer recommended sis is 521 days, and patients are considered infectious
-lactams (amp or amox alone have lower efficacy) until 3 weeks after symptom onset. Fluoroquinolones

14 T h e B r i g h a m I n t e n s i v e R e v i e w o f I n t e r n a l M e d i c i n e Q u e s t i o n a n d A n s w e r C o m pa n i o n
have not been shown to be effective as prophylaxis. In 12. ANSWER:E. Oral amoxicillin-clavulanate
addition, vaccination after exposure and administration
of immunoglobulin do not appear to prevent secondary Splenectomy results in increased vulnerability to over-
cases. whelming infection and sepsis due to certain organisms,
including Streptococcus pneumoniae, Haemophilus influ-
enzae, Neisseria meningitidis, and Capnocytophaga cani-
morsus. C. canimorsus is an anaerobic gram-negative rod
10. ANSWER:B. There is no potential risk for that is part of canine and feline oral flora. When inocu-
anaplasma or babesia coinfection. lated into humans, C. canimorsus can cause severe infec-
tion in certain hosts, including those without a spleen.
In Massachusetts, the deer tick Ixodes scapularis can carry Given this risk, typically asplenic patients who sustain
B. burgdorferi (Lyme), A. phagocytophilum (HGA), and a dog or cat bite in which the skin is broken are treated
Babesia microti (red blood cell parasite causing Babesiosis). with a short course of preventative oral antibiotics.
Asplenia and other immune suppression increases risk of First-generation cephalosporins (such as cephalexin) and
serious Babesiosis. Doxycycline has been shown to be some- trimethoprim-sulfamethoxazole are not typically active;
what protective against Lyme disease but not against other beta lactam-beta lactamase inhibitors and third-generation
tick-borne illnesses, and those who experience a tick bite cephalosporins are usually active. In this case the patient
should still be vigilant for signs and symptoms of illness is well clinically so has no indication for intravenous anti-
and seek medical attention if symptoms develop [Wormser. biotics (ceftriaxone); thus, amoxicillin-clavulanate is the
Clin Infect Dis. 2006;43:1089134]. most appropriate choice. Rabies is unlikely in this case
since the puppy is a domestic pet.

11. ANSWER:E. Rifampin 13. ANSWER:C. Use of a chlorine-containing


cleaning agent to address environmental
This patient initially had a skin/soft tissue infection contamination
that appeared to be nonpurulent, so management with a
B-lactam antibiotic (dicloxacillin, in this case) was appropri- Standard environmental cleaning detergents are not spo-
ate. However, she then developed a purulent complication ricidal. To adequately clean the room of a patient with
and was found to have an MRSA skin/soft tissue infection Clostridium difficile infection, it is necessary to use a
(SSI) that was appropriately drained, which is the mainstay chlorine-containing cleaning agent (1,0005,000 ppm
of treatment. It is appropriate to give postdrainage antibi- available chlorine) or other sporicidal agent. Even in out-
otics as well, especially in this case, which is complicated break settings, routine identification of asymptomatic car-
by obesity and lymphedema. There are multiple oral anti- riers is not recommended and treatment is not effective in
biotics that are active against MRSA. All of the following reducing horizontal transmission. Strict adherence to con-
are available in oral formulations with good oral bioavail- tact precautions, including gowning and gloving on entry
ability and are recommended for the treatment of MRSA to a patients room and hand hygiene with soap and water,
SSI when the organism is susceptible:linezolid (option A), are key measures in reducing horizontal transmission of
doxycycline (option B), trimethoprim-sulfamethoxazole Clostridium difficile. The spore form of Clostridium diffi-
(option C), and clindamycin (option D). Rifampin (option cile is resistant to killing by alcohol. Contact precautions
E) is an oral antibiotic with good oral bioavailability and should continue at least for the duration of the diarrhea,
activity against MRSA. However, rifampin should not be with many hospitals continuing contact precautions for the
used as monotherapy in the treatment of MRSA infec- duration of the inpatient admission. There is no evidence at
tions due to a low barrier to resistance. For treatment of this time to support the use of prophylactic antibiotics to
MRSA, the role of rifampin is primarily as a second agent prevent horizontal transmission.
that is often added in cases of infection involving pros-
thetic material (e.g., prosthetic heart valves, prosthetic
joints, and other orthopedic hardware). Thus, rifampin
should not be used in the treatment of this SSI, and cer- 14. ANSWER:D. Rifampin 600 mg and pyrazinamide
tainly not as monotherapy. Additional information about 1750 mg daily for 2months
treatment of skin and soft tissue infections can be found in
the Infectious Diseases Society of America (IDSA) guide- Daily isoniazid for 9months is the most common treatment
lines at http://www.idsociety.org/uploadedFiles/IDSA/ regimen for latent tuberculosis infection. The combination
Guidelines-Patient_Care/PDF_Library/Skin%20and%20 of directly observed rifapentine and isoniazid is equally
Soft%20Tissue.pdf. effective to daily isoniazid and may be best implemented in

1. I n f e c t i o u s D i s e a s e s 15
congregate settings that can administer directly observed hepatitis B since birth, which has now become apparent
therapy. Daily rifampin for 4months is an accepted alter- after increased replication of the virus in the setting of
native regimen, although efficacy has yet to be conclusively prednisone course and then increased immune response
proven. The 2-month regimen of rifampin and pyrazin- to the virus after completion of prednisone. This could be
amide is associated with unacceptably high rates of hepatitis called a hepatitis B flare. Her laboratory tests show liver
and is no longer recommended by the Centers for Disease inflammation consistent with this. She does not report any
Control and Prevention as a possible treatment for latent food exposures or recent travel that might predispose her
tuberculosis infection. to hepatitis Ainfection (which is transmitted by fecal-oral
route and is an acute infection). She does not report any
exposure to hepatitis C, which may also be transmitted
from mother to child (her mother does not have a his-
15. ANSWER:A. Bartonella henselae tory of hepatitis C) but most commonly is transmitted by
needle-sharing in injection drug use, and recently increas-
Systemic bartonellosis can sometimes present with a pri- ingly reported as acquired by receptive anal intercourse.
mary lesion that develops 310days following inoculation So the object is to find the serologies that are consistent
from a bite or scratch from an infected cat (usually kitten). with flare of chronic hepatitis B infection; the hepatitis
Tender lymphadenopathy may develop proximally after Aand hepatitis C serologies are irrelevant. Patients with
110 weeks with overlying erythema that can suppurate and active hepatitis B infection will produce surface antigen
may last weeks. Rare complications include neuroretinitis (HBSAg) and will have positive core antibody (HBcAb)
(stellate macular exudates macular star), encephalopathy/ due to ongoing exposure to the virus. But patients with
transverse myelitis, or endocarditis. Differential includes active chronic hepatitis B usually do not have positive
mycobacteria, tularemia, plague, toxoplasmosis, histoplas- surface antibody (HBSAb) because this is a protective
mosis, sporotrichosis, Kikuchis, lupus, and malignancy. In antibody that prevents chronic infection and is typically
immune-suppressed individuals disseminated illness can found only in patients that have received the hepatitis B
present as bacillary angiomatosis. vaccine or have been exposed to hepatitis B virus in the
past and cleared the infection. Thus, the correct answer
is option B (HBSAb negative, HBSAg positive, HBcAb
positive). See Table 1.1 for anticipated results with hepati-
16. ANSWER:B. HAV IgG positive, HBSAb tis B serologies depending on patient status.
negative,HBSAg positive, HBcAb positive, HCV
Abnegative

Of the viral hepatidities, this patient is most likely to 17. ANSWER:B. Initiate systemic corticosteroids to
have chronic hepatitis B, which would have been acquired control symptoms of paradoxical tuberculosis immune
by vertical transmission from her mother. Hepatitis B reconstitution inflammatory syndrome (IRIS).
remains a major public health problem in Vietnam and
in much of Southeast Asia, and the hepatitis B vaccine The clinical syndrome is classic for paradoxical tuberculosis
and immunoglobulin are not universally available to immune reconstitution inflammatory syndrome, which is
newborns exposed to the virus, so vertical transmission characterized by new or worsening findings of tuberculo-
continues. This patient has likely had subclinical chronic sis disease within 23months of initiation of antiretroviral

Table1.1 ANTICIPATED R ESULTS WITH HEPATITIS B SEROLOGIES DEPENDING


ON PATIENT STATUS

SEROLOGIC HEPB ACUTE R ECOVER ED CHRONIC INACTIVE


TEST VACCINATION HEPATITIS B HEPATITIS B ACTIVE CARR IER

HBsAb + +
HBcAb (total) + + + +
HBeAb + +
HbsAg + + +
HBeAg + +
HBV DNA + High (>10 ) 5
Detectable

16 T h e B r i g h a m I n t e n s i v e R e v i e w o f I n t e r n a l M e d i c i n e Q u e s t i o n a n d A n s w e r C o m pa n i o n
therapy. Manifestations can be seen within lung parenchyma, appearing, it is appropriate to start empiric antibiotics as
the central nervous system, and at serosal surfaces (pleural soon as blood cultures have been drawn in order to pre-
effusions, ascites, or pericardial effusions). Symptoms are vent further complications of infection. The most common
controlled with systemic corticosteroids. Multidrug-resistant pathogens in this type of presentation are Staphylococcus
tuberculosis is unlikely given the negative drug susceptibility aureus and the viridans Streptococci (oral flora)empiric
test. The rapid onset of symptoms is not consistent with loss treatment should be targeted toward treating these organ-
of control of tuberculosis infection due to poor adherence. isms. Enterococci are a less frequent cause of endocarditis,
Pneumocystis jerovici is in the differential diagnosis but less though still important to considerempiric treatment
likely given the CD4 T cell count above 200 cells/L3. for Staph and Strep will also include empiric treatment
for Enterococci. Coagulase-negative Staphylococci are most
common in patients with indwelling central venous cath-
eters, prosthetic heart valves, or other implanted cardiac
18. ANSWER:B. Lipid formulation of amphotericinB devices. Candidal endocarditis is most common among
injection drug users but still less common than the bacterial
The case patients presentation is consistent with a severe pathogens. Option A:vancomycin provides empiric treat-
sinus infection with an angioinvasive mold resulting in a ment for Staphylococcus aureus (including MRSA) and the
rapidly progressive infection with an intranasal eschar. The penicillin-resistant Enterococci, and ceftriaxone provides
most common infection syndromes in this clinical con- empiric treatment for viridans Streptococci, so this is the cor-
text include mucormycosis and aspergillosis. Appropriate rect answer. Option B might be considered for prosthetic
empiric antifungal therapy in suspected cases of fun- valve endocarditis with MRSA, but this patient does not
gal sinusitis includes an antifungal agent active against have a prosthetic valve, rifampin is not indicated, and use
Mucorales and Aspergillus species such as amphotericin B of rifampin during active bacteremia may lead to the devel-
or a lipid formulation of amphotericin (which may be less opment of rifampin resistance; hence, this should not be
nephrotoxic in this individual with chronic kidney disease). used empirically or early in therapy. Option C:nafcillin is
Voriconazole and caspofungin are active against Aspergillus excellent therapy for methicillin-susceptible Staphylococcus
species but do not have activity against Mucorales when aureus, but the possibility of MRSA has not been ruled out
given alone. Fluconazole has no activity in Aspergillus spe- in this patient, so he should receive vancomycin until cul-
cies or Mucorales. tures are documented, and aminoglycosides (gentamicin)
are no longer indicated for Staphylococcus aureus bactere-
mia due to risk of nephrotoxicity and minimal evidence
of benefit. Option D: caspofungin (or micafungin, both
19. ANSWER:C. Ciprofloxacin echinocandins) is excellent therapy for Candida infec-
tions, but Candidal endocarditis is much less common
Treatment for animal bites should cover Pasteurella species than bacterial endocarditis, even in injection drug users,
and anaerobic organisms, as well as Capnocytophaga canimor- so would not be chosen for empiric treatment. Option
sus. Aquinolone such as ciprofloxacin could be used if given E:ampicillin and gentamicin might be used for treatment
together with metronidazole or clindamycin for anaerobic of penicillin-susceptible Enterococcal endocarditis, but it
coverage, but ciprofloxacin alone is not adequate empiric would not be appropriate for empiric therapy in the absence
treatment. Capnocytophaga canimorsus is a gram-negative of culture data.
rod that is part of normal oral flora of canines and other ani-
mals, including cats and rabbits. Those who are asplenic and
those with liver disease or other immune suppression are at
increased risk for severe disease and can develop shock, pur- 21. ANSWER:C. Administration of 30 mL/kg
puric lesions, and disseminated intravascular coagulopathy ofcrystalloid
(DIC). Capnocytophaga canimorsus can take up to 14days
to culture, so in suspected cases treat empirically with The Surviving Sepsis Campaign recommends that the
amoxicillin/clavulanate or imipenem for critical illness, or following be completed within 3 hours: measurement
piperacillin-tazobactam would also be acceptable. of lactate, obtaining blood cultures prior to antibiotics,
administration of broad-spectrum antibiotics, and admin-
istration of 30 mL/kg of crystalloid for hypotension or
lactate 4 mmol/L. There is good evidence that early
20. ANSWER:A. Vancomycin + ceftriaxone antibiotics (within 1 hour of recognizing sepsis) improves
patient outcomes. Norephinephrine is the first-choice vaso-
This patient most likely has bacterial endocarditis and pressor to maintain a mean arterial pressure 65; however,
vertebral osteomyelitis. Since this patient is febrile and ill vasopressors should be reserved for hypotension that is

1. I n f e c t i o u s D i s e a s e s 17
nonresponsive to fluid resuscitation. Dopamine is not rec- and rash and lymphadenopathy but would be unlikely to
ommended except in highly select circumstances. Likewise, present with diarrhea, weight loss, and pancytopenia. An
intravenous hydrocortisone is not recommended if hemo- echocardiogram might be an appropriate test if the blood
dynamic stability is restored with fluid resuscitation and cultures are positive, but there are no other clues specific
vasopressors. to endocarditis here aside from the fever. A blood smear
and LDH may be appropriate tests at this point given the
patients pancytopenia of uncertain etiology, but a bone
marrow biopsy should not be pursued until further nonin-
22. ANSWER:E. B orC vasive diagnostics have been performed.

Neutropenic patients with fever that persists or recurs after


47days of empiric antibiotics are at high risk for fungal
infection. These patients should be treated with empiric 24. ANSWER:C. Cefepime plus levofloxacin plus
antifungal therapy, as the patient in this case was based on vancomycin
Infectious Diseases Society of America guidelines. In this
case the patient grew yeast from blood cultures. Aspergillus While this patient is presenting from the community, he
fumigatus is a mold; thus, it is not a potential cause of this meets criteria for a health careassociated pneumonia
infection. Only Candida albicans or Cryptococcus neofor- (HCAP). These criteria include hospitalization for 2days
mans, both of which grow as yeast in blood cultures, could within 90days of infection, residence in a chronic care facil-
cause this infection. Without further microbiologic infor- ity, attending a hemodialysis clinic, and receipt of intra-
mation the exact pathogen causing the infection cannot be venous antibiotic therapy, chemotherapy, or wound care
determined. within 30days of infection. Based on the patients underly-
ing diagnosis of diabetes and receipt of hemodialysis, he is
at increased risk for multidrug-resistant pathogens, includ-
ing Pseudomonas aeruginosa and methicillin-resistant
23. ANSWER:E. Blood smear, EBV serologies, Staphylococcus aureus (MRSA). While awaiting culture
CMVserologies, HIV antibody, HIV RNA results, it is recommended to start combination therapy
with the following: (1) an antipseudomonal cephalo-
This patient has a mono-like illness, characterized by sporin (cefepime, ceftazidime) or an antipseudomonal
fever, sore throat, malaise, fatigue, rash, and lymphade- carbapenem (imipenem, meropenem) or a beta-lactam/
nopathy. Mild hepatitis and pancytopenia may also be a beta-lactamase inhibitor (piperacillin-tazobactam); plus
part of the syndrome. The most common causes of this syn- (2) an antipseudomonal fluoroquinolone (ciprofloxacin,
drome in young adults are viral pathogens, including EBV levofloxacin) or an aminoglycoside (amikacin, gentamicin,
and CMV for those who were not already infected during tobramycin); plus (3)a drug that is active against MRSA
childhood. Serologies for both EBV and CMV are helpful (vancomycin, linezolid). These antibiotics can be adjusted
diagnostics here. Acute retroviral syndrome, or acute HIV, based on clinical improvement and culture results at 48 to
may also present as a mono-like illness, often with concur- 72 hours.
rent diarrhea and weight loss just as this patient had. The
incidence of new HIV infections in the United States is
highest among MSM at present, so this patient has a doc-
umented risk factor as well. Acute HIV is the most likely 25. ANSWER:D. Oral voriconazole
diagnosis for this patient. In acute HIV the HIV antibody
is often still negative, prior to seroconversion, but the HIV The patient in this case has clinical evidence of oral candidi-
RNA (or viral load) should be positive and is usually very asis (thrush). He recently received treatment with systemic
high. It is important to check both the HIV antibody steroids and antibiotics and is also on a chronic inhaled ste-
and HIV RNA (viral load) when acute HIV is suspected. roid, all of which raise his potential risk for thrush. In order
Although hepatitis serologies may be helpful at some point to minimize the possibility that the thrush resulted from
if the liver function tests continue to be abnormal, viral the use of an inhaled steroid, reviewing proper oral hygiene
hepatitis would not explain the full constellation of this after use could be helpful. Despite these risk factors, it is
patients symptoms or lab abnormalities, and the transami- also reasonable to consider HIV testing; the CDC cur-
nases are not high enough to be consistent with acute viral rently recommends testing for all patients in the health care
hepatitis, so these would not be the appropriate next tests setting. Appropriate treatment for uncomplicated thrush
to order. It is appropriate to consider the possibility of new includes oral nystatin, clotrimazole troches, or oral flucon-
sexually transmitted infections (STIs) given his new sexual azole. Voriconazole is a broader spectrum antifungal agent
partners, and secondary syphilis can also present with fever and would not be indicated in this clinical context.

18 T h e B r i g h a m I n t e n s i v e R e v i e w o f I n t e r n a l M e d i c i n e Q u e s t i o n a n d A n s w e r C o m pa n i o n
26. ANSWER:A. Colistimethate are unpredictable and thus are difficult to monitor and
treat. Peripherally administered corticosteroids, includ-
Acinetobacter baumannii is an aerobic gram-negative coc- ing inhaled, intranasal, injected (joint injections for treat-
cobacillus that can cause hospital outbreaks with high ment of pain), and even ophthalmic, should be avoided in
rates of mortality. Outbreaks are typically associated with patients taking ritonavir. If these medications are neces-
colonized respiratory support equipment, irrigation solu- sary, then an HIV specialist should be consulted. In addi-
tions, and intravenous solutions. The most common infec- tion to the aforementioned list of drugdrug interactions,
tions include ventilator-associated pneumonia, infection there are other common drugdrug interactions with
of surgical wounds and burns, and bacteremia. Over 50% antiretroviral medications. Both atazanavir and rilpiv-
of Acinetobacter baumannii isolates in US hospitals are irine require stomach acid for absorption to therapeutic
multidrug resistant. Colistimethate (colistin) is virtually levels, so coadministration of acid-blockers (H2 block-
always active in vitro. The presence of a beta-lactamase ers and especially proton pump inhibitors) can lead to
means that both cefepime and piperacillin/tazobac- decreased blood levels, and therefore decreased efficacy, of
tam, the two most active beta-lactam antibiotics against these agents. Efavirenz is both a substrate and inducer of
Acinetobacter baumannii, are likely to be ineffective. The isoforms of the cytochrome P450 system, but interactions
presence of a carbapenemase means that imipenem/cilas- with efavirenz are generally less common and less severe,
tatin, meropenem, and doripenem are all unlikely to be though the interaction with methadone is unpredictable
effective. Finally, most multidrug-resistant Acinetobacter and can be problematic. Inhaled albuterol, ophthalmic
baumannii isolates are resistant to fluoroquinolones and antihistamines, oral cetirizine, and oral montelukast
trimethoprim-sulfamethoxazole. Even if the isolate tests would not be expected to cause a problematic drug inter-
susceptible to a fluoroquinolone, resistance often develops action for this patient.
during use.

28. ANSWER:D. Bacteroides fragilis


27. ANSWER:D. Inhaled fluticasone
Bacteroides fragilis is an anaerobic gram-negative bacteria
There are many possible problematic drugdrug inter- that colonizes the intestines and is cultured from the blood
actions with antiretroviral medications and commonly in the setting of bowel perforation. It is not a common
prescribed drugs. Physicians should review medication cause of endocarditis. The common pathogens in endocar-
lists for any possible drugdrug interactions prior to ditis include Staphylococcus aureus, the viridans Streptococci,
prescribing new medications to patients who are cur- Streptococcus bovis, the HACEK organisms (Haemophilus
rently taking antiretroviral medications. Computer soft- parainfluenzae, Haemophilus aphrophilus, Haemophilus
ware applications that predict drug interactions from a paraphrophilus, Actinobacillus, Cardiobacterium hominis,
physician-entered medication list are particularly helpful Eikenella corrodens, Kingella kingae), and Enterococci (com-
in this setting. The most problematic antiretroviral agent munity acquired, without another obvious source). Positive
in terms of risk of drug interactions is ritonavir. Ritonavir blood cultures from at least two separate cultures with any
is the most potent cytochrome P450 3A4 inhibitor of of those organisms is considered a major criteria for the
all currently available medications. The other protease diagnosis of endocarditis by the modified Dukes criteria,
inhibitors, such as atazanavir and darunavir, and the or highly positive antibody titer to Coxiella burnetti (since
pharmacologic booster agent cobicistat (coformulated in this organism is exceedingly difficult to culture). The other
a one-pill-once-daily antiretroviral regimen) also inhibit possible major criteria for diagnosis of endocarditis is an
CYP 3A4. These medications interact with many com- echocardiogram with findings consistent with infective
monly prescribed drugs, including warfarin, the statins, endocarditis (mobile intracardiac mass, abscess, new dehis-
combination estrogen/progesterone oral contraceptives, cence of prosthetic valve, or new valvular regurgitation).
clarithromycin, rifampin, amiodarone, benzodiazepines, Adiagnosis of endocarditis by Dukes criteria requires ful-
carbamazepine, phenytoin, sildenafil, tadalafil, meperi- fillment of either two major criteria (as listed previously), or
dine, methadone, and peripherally administered cor- one major and three minor criteria, or five minor criteria.
ticosteroids. Peripherally administered corticosteroids This patient meets two minor criteria for the diagnosis of
(inhaled fluticasone, in this case) may reach systemic lev- endocarditis at the time of presentation:fever and predis-
els in patients who are taking ritonavir, which can lead posing cardiac lesion (the prosthetic valve). Other minor
to corticosteroid excess (features of Cushing syndrome) criteria are as follows: other predisposing risk (injection
and then later lead to corticosteroid insufficiency when drug use), evidence of emboli (arterial emboli, pulmonary
the medication is withdrawn. The systemic effects of infarcts, Janeway lesions, conjunctival hemorrhage), immu-
peripherally administered corticosteroids with ritonavir nologic complications (glomerulonephritis, Oslers nodes),

1. I n f e c t i o u s D i s e a s e s 19
or positive blood cultures that do not meet the major cri- quickly, and maintained on antiretroviral therapy with-
teria or serologic evidence of an infection with an organ- out interruptions are estimated to have a life expectancy
ism consistent with infective endocarditis not satisfying that approaches that of the general population. Although
a major criterion (e.g., Bartonella henselae). The diagnosis this patient is unlikely to develop AIDS-related complica-
of infective endocarditis is possible if only one major tions, such as opportunistic infections and AIDS-related
and one minor criteria are fulfilled, or if three minor cri- malignancies (e.g., Kaposis sarcoma), there are other risks
teria are fulfilled. Review the Infectious Diseases Society with prolonged HIV infection. Prolonged HIV infection
of America (IDSA) guidelines for the diagnosis and man- is associated with increased risk of cardiovascular disease
agement of infective endocarditis (http://www.idsociety. and non-AIDS-related malignancies, such as lung cancer.
org/uploadedFiles/IDSA/Guidelines-Patient_Care/PDF_ Health care maintenance, including addressing modifiable
Library/Endocarditis%20Management.pdf) for further risk factors and pursuing age-appropriate cancer screening,
information. is extremely important in these patients. For this patient
the most important modifiable risk factor for cardiovascu-
lar disease and malignancy is his smoking. Among all of
the health care maintenance items that could be addressed
29. ANSWER:B. Afew days after initiation of with the patient during this visit, smoking cessation (if
clarithromycin and ethambutol, but within 2 weeks, this can be achieved) is the most likely to lead to a reduc-
tion in morbidity and mortality. It is important to address
antiretroviral therapy (ART) should be started within 2 smoking cessation with him, through counseling (use
weeks of diagnosis of most opportunistic infections, with of motivational interviewing is especially helpful here),
the possible exception of cryptococcal meningitis (contro- and discussion of nicotine replacement therapy (nicotine
versial data on this; no clear guideline as of yet). There is patches, etc.) and other medications to aid in smoking ces-
a risk of immune reconstitution inflammatory syndrome sation (varenicline, bupropion). This patient does not meet
(IRIS) in patients who start ART with low CD4 count, criteria to start prophylaxis for Pneumocystis pneumonia
high viral load, and active opportunistic infection. IRIS since his CD4 count is well above 200, and he is not at
may be less pronounced if the opportunistic infection is risk for reactivation of latent toxoplasmosis since his CD4
treated first. However, IRIS may be treated with cortico- count is greater than 100. Adiscussion about alcohol use
steroids, if needed. Patients have increased mortality from to screen for warning signs of abuse or dependence may be
AIDS-related causes if ART is not started early enough. appropriate at this visit, but his current reported number
It is appropriate to start treatment for opportunistic of alcoholic beverages per week is below the threshold of
infections first to ensure tolerance of those medications concern for men, so this is less important to address than
for a couple of days prior to ART and to start control of smoking cessation. The patient should be referred for colo-
those infections, but then ART should be started soon noscopy screening at age 50, but in the absence of symp-
afterward, within 2 weeks of the diagnosis of opportu- toms or a personal or family history of polyps or colon
nistic infection. There is no reason to wait until hospital cancer, there is no indication to refer for early colonoscopy
discharge to start ART. ART may be started during hos- screening.
pitalization and transitioned to the outpatient setting in
the same way that other new medications (e.g., statins,
beta-blockers, and clopidogrel after new cardiac event) are
started during hospitalization and transitioned to the out- 31. ANSWER:A. Vancomycin, ceftriaxone,
patient setting. ampicillin

This patient is an elderly man who is living in a close com-


munity at his assisted living facility, now presenting with
30. ANSWER:C. Counsel patient to quit smoking probable bacterial meningitis. The most common patho-
and discuss medications and supports for smoking gens in this case are Streptococcus pneumoniae, Neisseria
cessation. meningitidis, and Listeria monocytogenes. The combination
of vancomycin and high-dose ceftriaxone is used empiri-
This patient has well-controlled HIV with a CD4 count cally when S.pneumoniae meningitis is a possibility because
well above the range that would be concerning for oppor- of the small but significant prevalence of penicillin-resistant
tunistic infections. If he continues to take his antiretro- and even cefotaxime-resistant S. pneumoniae. Ampicillin
virals regularly, then he should continue to have a robust is used for treatment of Listeria meningitis. The other
immune system and low risk of opportunistic infections. antibiotic options that are listed are not appropriate for
In fact, patients with HIV who are diagnosed early (before this patient. This patient has no history of neurosurgical
diagnosis of AIDS), started on antiretroviral therapy procedures, so he does not need empiric treatment with

2 0 T h e B r i g h a m I n t e n s i v e R e v i e w o f I n t e r n a l M e d i c i n e Q u e s t i o n a n d A n s w e r C o m pa n i o n
an anti-Pseudomonal cephalosporin such as cefepime or 33. ANSWER:E. Allergy consultation and admission
ceftazidime, or a carbepenem, such as imipenem or merope- for desensitization to penicillin in order to facilitate
nem. The CSF examination of this patient shows definite treatment with penicillin
predominance of neutrophils with minimal lymphocytes
and low glucose, which is most consistent with bacterial Syphilis infection during pregnancy can have serious con-
meningitis rather than viral meningitis, so acyclovir is not sequences for both the mother and fetus if left untreated.
required for initial treatment. The combination of ampi- Therefore, it is important to treat immediately with the
cillin and gentamicin may be useful for such pathogens as most aggressive, data-supported effective treatment for
ampicillin-sensitive Enterococci or Streptococcus agalactiae syphilis, which is penicillin. Pregnant women who are
(group B Streptococcus), but it does not include appropriate diagnosed with syphilis should always be treated with
empiric treatment for S. pneumoniae or Neisseria menin- penicillin, even if this requires hospitalization for desensi-
gitidis. Fungal meningitis is rare, seen primarily in patients tization to penicillin, in the case of penicillin allergy. All
who are immunocompromised. This patient does not have a other treatments for syphilis are potentially less effective,
given history of immunocompromise, so fungal meningitis and doxycycline is contraindicated during pregnancy. This
is extremely unlikely, and empiric therapy with amphoteri- patient may be presumed to have late latent syphilis, as she
cin B is not appropriate. is asymptomatic with infection of unknown duration, so
treatment with weekly IM penicillin for 3 weeks would
be appropriate. Follow-up RPR titers should be monitored
to ensure that they decrease appropriately. See the CDC
32. ANSWER:C. Oral tessalon perles, inhaled STD treatment guidelines (http://www.cdc.gov/std/treat
albuterol, and intranasal fluticasone ment/2010/STD-Treatment-2010-RR5912.pdf) for fur-
ther information.
This patient has rhinosinusitis that is improving without
antibiotic treatment already, based on cessation of fevers
4days ago and only mild residual symptoms. Acute rhino-
sinusitis (sinusitis) is an extremely common condition, 34. ANSWER: A. Levofloxacin 500 mg orally once
especially among adults age 4574 years, and the cause is daily for 7 days plus doxycycline 100 mg orally twice
viral in more than 90% of cases, with bacterial infection daily for 7 days
accounting for less than 10% of cases. Symptoms of upper
respiratory tract infections or sinusitis may include nasal Fluoroquinolone-resistant strains of Neisseria gonorrhoeae
congestion, headache, ear pain, cough, fever, and purulent are now disseminated throughout the United States. So, flu-
nasal discharge. The indications for antibiotic treatment oroquinolones have not been recommended for treatment
of sinusitis are when symptoms suggest bacterial infection of gonorrhea in the United States since 2007. Although
with any one or combination of three patterns: (1) per- cefixime-resistant strains are on the rise in Asian countries
sistent or not improving symptoms (10 days); (b) severe and also increasing in the United States, the proportion of
symptoms for 34 days (including temperature >39C these resistant strains remains low, so cefixime remains a
or 102C, or purulent nasal discharge), or (3) worsening recommended option for treatment of uncomplicated gon-
or double-sickening (onset of new/worsening symptoms orrhea as of the 2010 CDC guidelines (http://www.cdc.
as viral URI is starting to resolve after 56 days, indicat- gov/std/treatment/2010/STD-Treatment-2010-RR5912.
ing bacterial superinfection) over 34days. This patient pdf). Ceftriaxone IM is the preferred treatment option in
meets none of these criteria; therefore, antibiotic treat- all situations in which this is feasible, but treatment with
ment is not indicated, so answer options A, B, and D are cefixime may be considered, if necessary. Concurrent
all incorrect. Influenza vaccine, while not contraindi- treatment with azithromycin or doxycycline is important
cated in this situation since the patient is now afebrile, is because of the high frequency of coinfection with chla-
never indicated as a part of treatment of an acute illness, mydia. Most gonococci are also susceptible to azithromycin
so answer E is incorrect. This patient should be treated and doxycycline, so concurrent treatment may also decrease
with supportive care, including symptom management the rates of development of antibiotic-resistant gonorrhea.
all of the medications in option C may be used for symp-
tom management, so this is the correct answer. Please see
the IDSA Guidelines for Acute Rhinosinusitis for more
information: http://www.idsociety.org/uploadedFiles/ 35. ANSWER:B. Viral culture of a swab of the ulcers
IDSA/Guidelines-Patient_Care/PDF_Library/IDSA%20
Clinical%20Practice%20Guideline%20for%20Acute%20 The most likely diagnosis in this case is anogenital herpes
Bacterial%20R hinosinusitis%20in%20Children%20 simplex (HSV) infection. The features that are consistent
and%20Adults.pdf with this diagnosis are the grouped ulcerations, which

1. I n f e c t i o u s D i s e a s e s 21
are painful, with absence of significant lymphadenopathy 37. ANSWER:A. Monitoring for early signs and
or rectal discharge, which would be expected with some symptoms of urinary tract infection, with expedited
other conditions such as gonorrhea or lymphogranuloma early urinalysis, urine culture, and empiric treatment
venereum (LGV). Diagnostics for active HSV infection while awaiting culture results when symptoms develop
are somewhat limited, and the diagnosis is often made
clinically followed by somewhat empiric treatment. Patients with chronic indwelling urinary catheters often
Viral culture of a swab of the lesion has a high specific- have pyuria and bacteria even in the absence of other signs
ity, but the sensitivity is somewhat poor and operator or symptoms of active urinary tract infection. Routine
dependent. In certain institutions the direct fluorescent screening with urinalysis and urine culture is not recom-
antibody testing (DFA) of a scraping of the base of the mended in these patients because this will lead to overtreat-
lesion may be highly sensitive and specific, but it is also ment of asymptomatic patients, and it increases the risk for
operator dependent and therefore not consistent across antibiotic-resistant infections in the future. Early diagnosis
institutions. PCR for HSV is both sensitive and specific and treatment, based on prior available culture results while
from CSF during episodes of HSV meningitis, but it does awaiting pending acute cultures, is the safest approach
not perform well from a swab of an ulcer. Serologic test- to management to decrease the risk of overtreatment.
ing for HSV type-specific antibodies will demonstrate Antibiotic treatments into the urinary drainage bag are
infection that was acquired at some point in the past, not recommended, and chronic prophylaxis with methena-
but it is not specific to an active outbreak. Urine NAAT mine salts or empiric broad antibiotics (such as ciprofloxa-
probes are helpful for diagnosis of gonorrhea or chla- cin) without any review of culture data or prior infections
mydia, but not helpful for diagnosis of HSV. Bacterial are not recommended. Please see the IDSA guidelines on
studies would not be expected to be positive during complicated urinary tract infections (http://www.idsoci
active HSV infection, which is a viral infection. Please ety.org/uploadedFiles/IDSA/Guidelines-Patient_Care/
see the 2010 CDC STD diagnosis and treatment guide- PDF_Library/Comp%20UTI.pdf) for details.
lines (http://www.cdc.gov/std/treatment/2010/STD-
Treatment-2010-RR5912.pdf) for details.

38. ANSWER:E. Penicillin

36. ANSWER:C. Serum test for antineutrophilic Recurrent cellulitis with episodes as described for this patient
cytoplasmic antibodies (ANCA) (sudden onset of fever, systemic symptoms, and pain and
erythema of the leg) is commonly attributed to recurrent
This question stem describes a patient with an infec- Streptococcal infection, though the microbiologic diagnosis is
tion mimicker, which can be diagnostically challeng- not confirmed in most cases. Risk factors for recurrent celluli-
ing. Many of the features of this patients illness may be tis may include prior surgery or prior infections in the affected
consistent with an infectious etiologychronic sinusitis, limb, lower-extremity edema, poor hygiene status, skin ulcer-
especially in immunocompromised patients, may be due ations, and onychomycosis. Arandomized, placebo-controlled
to fungal infection. Pulmonary nodules and hemoptysis trial of chronic suppressive/prophylactic oral penicillin showed
may be due to fungal infection or tuberculosis, or non- that this was an effective strategy to decrease recurrences of
tuberculous mycobacterial infection. However, when the cellulitis; incidence of cellulitis was 37% among placebo recipi-
entirety of the patients history is reviewed, the salient ents and 22% among penicillin recipients during the follow-up
points are as follows: no obvious risk factors for tuber- period [Thomas KS etal. NEJM 2013;368:1695703]. Many
culosis or immunosuppression at baseline, chronic sinus- of the antibiotic options listed have good efficacy at treating
itis that was unresponsive to repeated treatments with Staphylococcal infections, and some have good efficacy at treat-
antibiotics, new pulmonary nodules and hemoptysis, and ing Streptococcal infections (e.g., levofloxacin), but only penicil-
blood tests showing anemia and new renal failure. Taken lin has been proven to reduce risk of recurrence of cellulitis in
together, this syndrome is most consistent with Wegners a placebo-controlled clinical trial.
granulomatosis. Most patients with active Wegners gran-
ulomatosis will have positive serum ANCA testing, espe-
cially if the disease is severe. AFB smears from sputum or
from biopsy samples would be positive if the patient had 39. ANSWER:B. Avoid ingestion of pork or any pork
tuberculosis or nontuberculous mycobacterial infection. products
Serum galactomannan would be positive if the patient
had invasive aspergillosis. Serum interferon-gamma Toxoplasma gondii is a parasite found in soil and contami-
release assay (IGRA) may be positive in patients with nated or undercooked foods (see Figure 1.1). Acute infection
tuberculosis. may be associated with malaise, fever, and lymphadenopathy,

2 2 T h e B r i g h a m I n t e n s i v e R e v i e w o f I n t e r n a l M e d i c i n e Q u e s t i o n a n d A n s w e r C o m pa n i o n
i = Infective Stage
8 i
d = Diagnostic Stage
10 d 9
3

11 d
4
Tissue
Cysts
6
i

i
7

Fecal
2 Oocysts
1
5

Figure1.1 Toxoplasma life cycle diagram. (From the Centers for Disease Control and Prevention. Available at http://dpd.cdc.gov/)

but it is often asymptomatic. Visual changes do occur when 40. ANSWER:D. Vancomycin and ceftazidime
patients develop toxoplasmic chorioretinitis, but this is
rare. Acute infection during pregnancy may result in con- This patient has poorly controlled diabetes with a new pro-
genital infection of the newborn, which may have many gressive soft-tissue infection arising from a nonhealing toe
long-term complications even if infants appear healthy at ulcer. The black areas of the toe are concerning for necrotic
birth. Early diagnosis and treatment is essential to prevent- tissue, which in this case would raise the concern for wet
ing complications. But prevention of infection may be even gangrene, or infected gangrenous tissue. This patient likely
more effective, especially since infection may be asymptom- requires surgical debridement and may require amputation of
atic. This patient does have HIV infection, but her HIV is the toe, depending on the extent of the infection. Antibiotics
well controlled and her CD4 count is well above 100, and should be started immediately while awaiting results of cul-
she does not have latent infection with Toxoplasma (her tures and operative management. It is important to treat
Toxoplasma IgG is negative). So, prophylactic medications, empirically for Staphylococcal infection, including presump-
such as sulfamethoxazole-trimethoprim, are not indicated tive treatment for MRSA, given the high incidence of this
in this case. Instead, primary behavioral prevention strat- type of infection; thus, vancomycin should be included in
egies to avoid acute infection in this uninfected patient the empiric treatment regimen. Given this patients his-
should be employed. Toxoplasma cysts may be carried in tory of diabetes, it is also important to treat empirically
cat feces, so avoiding litter boxes or soil that may be con- for gram-negative organisms, including Pseudomonas aeru-
taminated is important, and litter box sterilization is rec- ginosa. Of the listed antibiotic regimen options, only one
ommended. Fruits and vegetables may be contaminated includes empiric treatment for Pseudomonas: ceftazidime.
with cysts from the soil, so they should be washed before None of the following antibioticsampicillin-sulbactam,
ingesting. Uncooked meats may carry active infection, so all ceftriaxone, or ertapenemwould be expected to have
meats should be cooked to well done, but pork products activity against Pseudomonas; therefore, all of these regi-
do not need to be universally avoided, as long as they are men options are incorrect. Vancomycin alone is not suffi-
cooked properly [Montoya JG, Remington JS. Clin Infect cient since this does not include treatment for gram-negative
Dis. 2008;47(4):55466]. organisms.

1. I n f e c t i o u s D i s e a s e s 2 3
41. ANSWER: B. Valacyclovir 1000 mg orally thrice typically thought of as being specifically associated with
daily for 14 days advanced HIV infection, or AIDS, it is becoming more
common among patients who are immunocompromised
This patient has Ramsay Hunt syndrome type II, otherwise for other reasons. Case report series have documented an
known as herpes zoster oticus. This is a rare syndrome, which increased incidence of Pneumocystis pneumonia among
is caused by reactivation of herpes zoster virus in the genicu- patients with rheumatologic disease who are undergoing
late ganglion, which leads to facial paralysis, altered sensation treatment with infliximab. The features of the presenta-
in the ear canal, loss of moisturization of the eyes and mouth, tion that are consistent with Pneumocystis pneumonia
and can also lead to altered taste in the frontal two-thirds of include risk factor with immunosuppression; cough and
the tongue. Vesicular skin lesions typical of zoster reactivation shortness of breath, which are progressive despite antibi-
may be seen in this same nerve distribution, but they are not otics; significant hypoxia on presentation; and bilateral
always present. The concurrence of facial droop with ear canal infiltrates on chest imaging. The serum B-glucan assay
symptoms, changes in mucous membranes moisture, and skin may be positive in the setting of Pneumocystis pneumo-
lesions make this syndrome unique. This zoster reactivation nia, especially in immunocompromised patients, and also
is treated with acyclovir or valacyclovir. The addition of cor- in some other fungal infections. The toluidine blue stain
ticosteroids, such as prednisone, is of controversial benefit, can identify cysts from a bronchoalveolar lavage (BAL).
but corticosteroids alone are certainly not sufficient without Treatment is with trimethoprim-sulfamethoxazole, and
antiviral treatment. The facial droop alone might be due to corticosteroids should be added when there is severe dis-
Bells palsy, which can be caused by Lyme disease (especially ease as evidenced by a large A-a gradient, as is the case
with the epidemiologic risk factors listed for this patient) for this patient. Liposomal amphotericin would be used
and would be appropriately treated with doxycycline. Otitis to treat a fungal infection, such as aspergillosis or cryp-
externa may be treated with otic ciprofloxacin drops, and if tococcosis, but neither of these would be diagnosed by
concurrent otitis media was suspected, then additional treat- cysts evident on toluidine blue staining from a BAL.
ment with amoxicillin-clavulanate would be appropriate. Ivermectin would be used to treat Strongyloides stercora-
lis hyperinfection, which can cause respiratory failure in
42. ANSWER:D. Haemophilus ducreyi patients who are latently infected and then receive immu-
nosupression, but this infection would be diagnosed by
This patient has nongonococcal urethritis (NGU) based on visualization of the motile organisms on simple wet prep
the clinical diagnosis of urethritis and the negative testing for of the BAL.
gonorrhea. The most common cause of NGU is Chlamydia
infection, but this patient also had negative testing for
Chlamydia. Other organisms that are presumed to cause
NGU are difficult to culture and difficult to diagnose using 44. ANSWER:E. Antiretroviral therapy should be
amplification assays for the detection of DNA or rRNA, offered to all HIV-positive patients regardless of CD4
for example, mycoplasma genitalium. But Trichomonas, M. count as the benefits often outweigh the risks even at
genitalium, herpes simplex virus, and Ureaplasma urea- high CD4 counts
lyticum are all reported causes of NGU (http://www.cdc.
gov/std/treatment/2010/STD-Treatment-2010-RR5912. Current CDC guidelines recommend antiretroviral ther-
pdf). Treatment for NGU is usually empiric since the apy for all HIV-positive individuals, but with varying lev-
causative organism is often not identified. Recommended els of evidence supporting the recommendation by CD4
initial treatment is with Azithromycin 1g orally single strata, and with varying degrees of urgency depending on
dose or doxycycline 100 mg orally twice daily for 7 days. CD4 strata. For this patient, antiretroviral therapy is not
If symptoms persist beyond this treatment, then additional urgent since his CD4 count is high enough that he is not
treatment with metronidazole 2 g orally single dose (for pos- at immediate risk of opportunistic infections. However,
sible Trichomonas infection) is recommended. Haemophilus potential benefits of antiretroviral therapy at high CD4
ducreyi is the causative organism in chancroid, which is an counts include decreased risk of non-AIDS associated can-
ulcerative STI and does not typically cause urethritis. cers, decreased risk of cardiovascular disease, and decreased
risk of transmission to others. These potential benefits
should be discussed with the patient to allow the patient
to make an informed decision about when to start treat-
43. ANSWER:C. Trimethoprim-sulfamethoxazole ment. Fortunately, there are minimal risks associated with
plus prednisone antiretroviral therapy with the newer generations of anti-
retroviral agents, which have decreased toxicities compared
This patient has Pneumocystis jiroveci pneumonia, also with older drugs. So the risk/benefit analysis often finds in
still commonly known as PCP. Although this disease is favor of initiation of ART, if patients are willing to start

2 4 T h e B r i g h a m I n t e n s i v e R e v i e w o f I n t e r n a l M e d i c i n e Q u e s t i o n a n d A n s w e r C o m pa n i o n
medications and ready to commit to adherence to a medica- disease. Plasmapheresis and IVIG have not been well studied
tion regimen for anticipated lifelong treatment. in this infection and there are no guidelines for use of these
treatments in this case. Ceftriaxone and doxycycline may be
used to treat other tick-borne infections (Lyme disease and
anaplasmosis) but would not be expected to be effective treat-
45. ANSWER:A. Drugdrug interaction ment for babesiosis.

This patient most likely has developed serotonin syndrome


due to interaction of the linezolid with his baseline dulox-
etine medication for depression. The signs and symptoms 47. ANSWER:A. Blood serologic test
of serotonin syndrome include tachycardia, hyperthermia,
hypertension, agitation, and restlessness, and may also This patient most likely has primary pulmonary coccidioi-
include confusion and abdominal complaints. This con- domycosis, or valley fever. The features of his presentation
dition can be fatal if not addressed quickly, which is why that are consistent with this diagnosis are as follows:expo-
there is a black-box warning on linezolid to avoid concur- sure in an endemic area (hiking in dry windy climate in
rent administration with selective serotonin reuptake the American Southwest), somewhat immunosuppressed
inhibitors, serotonin-norepinephrine reuptake inhibitors, (due to his psoriasis and methotrexate treatment), and
and some other serotonin-active medications. There is acute-onset illness with fever, fatigue, chest pain, and cough,
no evidence of recurrent soft tissue infection on physical which all persisted despite antibiotic therapy. The chest X-ray
exam, and no reason to suspect MRSA bacteremia without may show an infiltrate, or it may be normal. Labs often also
another source, without murmurs on cardiac exam, and show new eosinophilia, but this may also be normal. This
without evidence of sepsis (blood pressure is high, rather patient has ongoing symptoms, but they are not worsening,
than low). The abdominal examination is normal and there so his disease is relatively mild and uncomplicated, despite
are no reported abdominal symptoms, so C.difficile infec- his underlying treatment with methotrexate. Serologic test-
tion is also unlikely. ing for Coccidioides (Cocci IgG) is often diagnostic in these
casesif the serology is positive, then the patient most likely
has active Coccidioides infection, since serologic reactivity
tends to wane within months of an infection. Many patients
46. ANSWER:B. Quinine and clindamycin and with this infection will have self-limited disease, often even
initiation of red cell exchange transfusion subclinical. Directed treatment, with antifungals such as
itraconazole or liposomal amphotericin, is generally only
This patient has severe babesiosis. Parasites (protozoa) seen required in severe or prolonged infection.
in the red blood cells is diagnostic of either malaria (caused
by multiple different Plasmodium sp.) or babesiosis (most
often caused by Babesia microti, though there are other infec-
tious species as well). This patient has travelled to Nantucket, 48. ANSWER:E. Drugdrug interaction of warfarin
where Babesia is commonly found as a tick-borne illness, and with rifampin
has no reported travel history to a malaria-endemic area, so it
is presumed that the infection in this case is babesiosis rather Rifampin is not a commonly prescribed antibiotic, but
than malaria. Mild babesiosis may be treated with azithro- it is used in several specific situations and it has numer-
mycin and atovaquone. Severe babesiosis is generally treated ous drugdrug interactions since it is a potent inducer of
with quinine (oral quinine or IV quinidine when necessary) cytochrome P450 3A. Patient medication lists should be
and clindamycin, though this recommendation is due, in reviewed carefully for potential interactions whenever
part, to a lack of comparison data between these regimens rifampin is initiated, and a plan for monitoring or adjust-
for severe disease. The major risk factor for severe babesio- ment in treatment should be made whenever possible. In
sis in this patient is his history of splenectomy. His age (i.e., this case, rifampin induces the metabolism of warfarin so
>50years old) is also a risk factor for severe disease. Patients that the patient has approximately half the previously avail-
with severe infection, including evidence of end-organ dys- able warfarin dose after rifampin is initiated. If the INR
function (elevated alkaline phosphatase, elevated creatinine, is not monitored carefully in order to facilitate warfarin
or respiratory compromise), severe anemia (hemoglobin < dose adjustments, then patients will quickly become sub-
10), or high-grade parasitemia (> 10%) should be evaluated therapeutic on warfarin in this setting. Even with proper
for red cell exchange transfusion, especially if the patient also monitoring and dose adjustments some patients are unable
has a history of splenectomy. This patient meets many criteria to achieve therapeutic warfarin while on rifampin and may
for red cell exchange transfusion, and it should be pursued require concurrent treatment with enoxaparin or other
immediately to prevent further complications from this medications. This patient had a recent transesophageal

1. I n f e c t i o u s D i s e a s e s 25
echocardiogram, which did not show any valvular vegeta- with abdominal pain, diarrhea, weight loss, and joint
tions or perivalvular abscess, and the blood cultures on this pains. Babesia microti is a parasite that is transmitted by
presentation are negative, so there is no evidence to sug- ticks primarily in New England and to some extent in the
gest treatment failure at this point. Gentamicin has many upper Midwest; babesiosis is characterized by fever, ane-
associated potential toxicities, but it does not cause stroke. mia, thromobocytopenia, and often respiratory distress,
Loss of gut flora while on antibiotic therapy often leads to but this patient had no anemia or parasites visualized on
decreased vitamin K production and therefore hypocoagu- initial labs. Borelia lonestari is thought to be the causative
able state, rather than hypercoaguable state. agent in Southern tick-associated rash illness (STARI),
which is a Lyme diseaselike infection described in
patients in the southeastern and south-central United
States. Anaplasma phagocytophilum causes anaplasmo-
49. ANSWER:D. Francisella tularensis sis, formerly known as human granulocytic ehrlichiosis,
which is another tick-borne illness that often presents
Tularemia is the infection caused by Francisella tularen- with fever, headache, and lab abnormalities but rarely
sis, which is a gram-negative bacteria. People may develop causes pulmonary disease.
tularemia infection after contact with infected animals
or via insect vectors such as ticks. Most of the cases in
the United States are reported in Arkansas, Missouri,
Kansas, South Dakota, Oklahoma, and California. After 50. ANSWER:C. Gram-positive cocci in pairs
exposure, patients may be asymptomatic, or they may andchains, with beta-hemolysis
develop any one of the six major clinical forms of tula-
remia: ulceroglandular tularemia, glandular tularemia, This patient has a necrotizing soft tissue infection with
oculogalndular tularemia, pharyngeal (oropharyngeal) myonecrosis after blunt trauma. This is most often caused
tularemia, typhoidal tularemia, or pneumonic tulare- by group A beta-hemolytic Streptococci. Treatment is
mia. Ulceroglandular and pneumonic tularemia are the emergent surgical debridement and aggressive anti-
two most commonly diagnosed forms of the disease. This biotics, preferably with high-dose IV penicillin and
patient has pneumonic tularemia. Primary pneumonic clindamycin to reduce toxin formation. If antibiotics are
disease occurs after inhalation of the organism from a started in the absence of gram stain results, then a more
source, such as an infected animalthis is most common broad-spectrum regimen would be appropriate until cul-
among farmers, sheep workers, landscapers, and hunters. ture data are available; broad-spectrum regimens would
Pneumonic tularemia may present similarly to pneumonic include vancomycin or linezolid, along with a carbape-
plague (caused by Yersinia pestis), except that pulmonary nem or beta-lactam/beta-lactamase inhibitor combina-
disease often consists of peribronchial infiltrates or lobar tions (such as piperacillin-tazobactam). Gram-positive
consolidations in pneumonic tularemia, whereas it is often cocci in pairs and chains with alpha-hemolysis would be
rounded and cavitated pulmonary infiltrates or nodules alpha-hemolytic Streptococci, most commonly from the
in setting of pneumonic plague. Patients may have some viridans group, which do not commonly cause necrotiz-
laboratory abnormalities, including elevated or depressed ing soft tissue infections. Gram-positive cocci in clusters
WBC and abnormal liver function tests, but laboratories would be Staphylococci, which can cause necrotizing soft
may also be normal. Patients may develop respiratory tissue infections, but more commonly with a type Inec-
failure and empyema with pneumonic tularemia, and rotizing infection, which is often polymicrobial and puru-
infection may even spread beyond the lungs, with com- lent as compared with this presentation of Streptococcal
plications such as meningitis and endocarditis. Diagnosis gangrene. Gram-negative rods may also be seen in type
is primarily by serologies and clinical suspicion, but the I polymicrobial necrotizing soft tissue infections, most
organism may be cultured (with difficulty) if cultures are commonly in the groin or originating from the gastroin-
specifically requested on cysteine-containing supportive testinal tract. Gram-positive rods would be concerning
media. Treatment is with doxycycline or ciprofloxacin for Clostridium species if they were anaerobic, rather than
for mild disease, and with streptomycin or gentamicin aerobic; but this patient did not have gas formation in the
for severe disease. Tropheryma whipplei is the etiologic myonecrosis, which is commonly seen with necrotizing
agent in Whipples disease, which commonly presents infections due to Clostridium species.

2 6 T h e B r i g h a m I n t e n s i v e R e v i e w o f I n t e r n a l M e d i c i n e Q u e s t i o n a n d A n s w e r C o m pa n i o n
2.
HEM ATOLOGY A ND ONCOLOGY

Lawrence Shulman, Ann LaCasce, Wendy Y. Chen, Yuksel Urun, Toni K. Choueiri, Jean M.
Connors, Peter Enzinger, Nancy Berliner, Maureen M. Okam, Mark M. Pomerantz ,
David M. Jackman, Brett E. Glotzbecker, Edwin P. Alyea, Daniel J. DeAngelo,
Robert I. Handin, Jeffrey A. Meyerhardt, and Whitney W. Woodmansee

1. An 80-year-old man presents with several months of also reveals a 2cm cystic lesion involving the main pan-
diffuse abdominal pain, weight loss, fatigue, and dys- creatic duct. After spontaneously passing the kidney
pnea. The patient was diagnosed with pernicious ane- stone, the patient follows up in your clinic stating that
mia 5years ago and has been receiving regular vitamin she feels entirely well. She denies jaundice, abdominal
B12 injections. His complete blood count (CBC) or back pain, weight loss, or any other symptoms. CA
today reveals a hematocrit of 24% and a mean cell vol- 19-9 is drawn and found to be normal. You refer the
ume (MCV) of 75 fL. What malignancy is most likely patient for an MRCP, which confirms a 1.9 cm muci-
responsible for his symptoms? nous lesion involving the main pancreatic duct, with
classic features of an intraductal papillary mucinous
A. Myelodysplastic syndrome neoplasm of the pancreas (IPMN). What is the proper
B. Gastric cancer next step in management?
C. Colon cancer
D. MALT lymphoma A. Follow-up CT scan in 6months
E. Pancreatic cancer B. Follow-up MRCP in 6months
C. Referral to a pancreatic surgeon for resection
2. A45-year-old man presents with back pain, jaundice, D. See the patient in 612months to assess symptoms
and weight loss of 20 pounds. Acomputed tomography but obtain no further imaging studies.
(CT) scan of the abdomen reveals a large mass in the E. This is a benign finding; no specific follow-up is
pancreatic head. CA 19-9 is elevated at 496/mL. The required.
patients family history is notable for the following:His
father was diagnosed with colon cancer at age 72. His 4. A35-year-old woman originally from the Dominican
mother is an only child and was diagnosed with breast Republic comes to the clinic for her first visit. She has
cancer at age 51. His maternal grandfather died of pan- no significant medical problems and has had two chil-
creatic cancer at age 55. The patient is most likely to have dren. After the birth of her second child 3years ago she
which of the following hereditary cancer syndromes? was told to take iron tablets twice daily. Aside from an
oral contraceptive, this is her only medication. Results
A. Lynch syndrome of her laboratory studies are shown in Table 2.1.
B. BRCA 2 mutation The most appropriate management is:
C. CDKN2a mutation
D. Li-Fraumeni syndrome A. Phlebotomy for hemochromatosis
E. CDH 1 mutation B. Continue current iron therapy and initiate workup
for chronic inflammatory process
3. A60-year-old healthy woman undergoes a CT scan of C. Switch therapy from oral iron sulfate to iron dextran
the abdomen for signs and symptoms of nephrolithia- D. Discontinue iron therapy; send ferritin and
sis. The scan confirms a small right kidney stone but hemoglobin electrophoresis

27
Table2 .1 LAB R ESULTS Table 2 .3 COMPLETE BLOOD CELL R ESULTS FOR
QUESTION 6
White blood cell count 4,600/mm3 (4,00010,000)
Hematocrit 35% (3648) Hemoglobin 5.6 g/dL
Hematocrit 17%
Mean cell volume 66 fL (8095)
Mean cell volume 123
Platelets 256,000/mm3 (150,000450,000)
White blood cell count 3,500
Iron 150g/dL (40159)
Platelets 70,000
Total iron binding capacity 275g/dL (250400)
Bilirubin 2.3 mg/dL
5. A74-year-old man with diabetes mellitus controlled
with an oral agent presents for routine follow-up. His A. Myelodysplastic syndrome
other medical problems include hypertension, for B. Vitamin B12 deficiency
which he takes an ACE inhibitor, and benign prostatic C. Sideroblastic anemia
hypertrophy. On review of his records you note that his D. Hemolytic anemia
hematocrit has been gradually declining over the past
3years. Results of his laboratory studies are shown in 7. An 18-year-old woman is referred for the evaluation
Table 2.2. of mild jaundice. She thinks she has had it intermit-
tently for years. She thinks that she has always tired
Table 2 .2 L A B R ESULTS FOR QU ESTION 5 more easily than her friends, and she has been told sev-
eral times that she is anemic. She has been treated on
White blood cell count 7,000 (4,00010,000) several occasions with iron pills, but not in the past
Hematocrit 28% (3648) 2years. On physical examination, she has scleral icterus
Mean cell volume 84 fL (8095)
and a spleen tip palpable below the left costal margin
(see Table 2.4).
Red blood cell 15 (1014.5)
distribution width Table 2 .4 L A B R ESULTS FOR QUESTION 7

Platelets 340,000/mm3 (150,000450,000) Hemoglobin 11.6 g/dL


Blood urea nitrogen 35 mg/dL (925) Hematocrit 31%
Creatinine 1.9 mg/dL (0.71.3) Mean cell volume 83 fL
Lactate dehydrogenase 230 (107231) Reticulocytes 7.0%
Platelets 220,000
The most likely etiology of his anemia is:
White blood cell count 6,500
A. Combined iron and B12 deficiency Blood smear:spherocytes, increased
B. Medication effect from the ACE inhibitor reticulocytes
C. Erythropoietin deficiency Coombs test negative
D. Anemia due to marrow replacement by metastatic
prostate cancer
The patient is most likely to respond to which of the
following:
6. A50-year-old man is seen complaining of dyspnea on
exertion. He was in a car accident 5years ago with mul- A. Corticosteroids
tiple abdominal injuries, resulting in a splenectomy and B. Intravenous iron
a resection of several feet of his terminal ileum. After a C. Splenectomy
prolonged recovery, he returned to work and his normal VD. Eculizumab
activities. For the last 46 months he has had trouble
climbing the stairs to his bedroom without stopping to 8. A 39-year-old man with sickle cell anemia is admit-
catch his breath. He is on no medications, has not lost ted for management of pneumonia. He presented with
weight, and has a well-balanced diet (see Table 2.3 for a 2-day history of a dry nonproductive cough, fever to
CBC). 101F, and was found to have a right lower lobe infiltrate.
Peripheral smear: macrocytic erythrocytes, hyper- On admission his oxygen saturation was 94% on room
segmented neutrophils, decreased platelets air, and he was not short of breath. He is placed on cefu-
The most likely diagnosis is: roxime and given intravenous hydration (see Table 2.5).

2 8 T h e B r i g h a m I n t e n s i v e R e v i e w o f I n t e r n a l M e d i c i n e Q u e s t i o n a n d A n s w e r C o m pa n i o n
Table 2 .5 L A B R ESULTS FOR QU ESTION 8 C. Admit to the hospital and arrange for emergent
RBC exchange transfusion.
CBC ON ADMISSION
D. Manage pain aggressively and discharge home with
White blood cell count 18,000/mm3 (4,00010,000) close hematology follow-up.
Hematocrit 21% (3648) E. Manage pain aggressively and discharge home on
oral antibiotics.
Platelets 247,000/mm3 (150,000450,000)

10. A 34-year-old woman presents to establish care with


One day later he complains of increasing shortness of
you. She just moved to town and needs a new primary
breath and is found to have an oxygen saturation of 86%
care provider. In the last 3 years she has lost 60 lb by
on room air. Chest radiograph reveals bilateral lower
strict calorie counting and routine exercise. She was
lobe opacities.
previously on HCTZ, metformin, and iron tablets but
The best next steps are:
stopped all medications when she checked her blood
pressure at a local pharmacy and it was 100/50 mm Hg.
A. Continue current antibiotic coverage and administer
She asks you whether she should restart her medica-
supplemental O2 .
tions again (see Table 2.7).
B. Continue current antibiotic coverage, administer
supplemental O2 , and obtain V/Q scan. Table 2 .7 R ESULTS FROM COMPLETE BLOOD
C. Continue current antibiotic coverage, administer COU NT I N YOU R OFFICE FOR QUESTION 10
supplemental O2 , and transfuse PRBC.
D. Add coverage for atypical organisms and administer White blood cell count 7,900/mm3 (4,00010,000)
supplemental O2 . Red blood cell count 6.0 (4.25.6)
E. Add coverage for atypical organisms, administer
Hematocrit 32% (3648)
supplemental O2 , and exchange transfuse.
Mean cell volume 68 fL (8098)
9. A54-year-old African American man with sickle cell Platelets 242,000/mm 3
(150,000450,000)
anemia presents to the emergency room with a 2-day his-
tory of fatigue and pain in his back and lower extremi- Red blood cell distribution 13.2% (11.514.5)
width
ties. He scores his pain at 5 out of 10. He has had episodes
of pain in the past and is well known to the emergency
department staff. His temperature is 99F, heart rate is You order a hemoglobin electrophoresis (see Table 2.8).
108 beats per minute, blood pressure is 120/70mm Hg, Table 2 .8 R ESULTS OF HEMOGLOBI N
and oxygen saturation on room air is 96% (see Table ELECT ROPHOR ESIS FOR QUESTION 10
2.6). He is started on normal saline and morphine IV.
Hb A 97% (96.598.5)
Table 2 .6 L A BOR ATORY ST U DI ES I N THE Hb A 2 2.8% (1.63.0)
EM ERGENCY ROOM FOR QU ESTION 9
Hb F 1% (02.0)

White blood cell count 14,000/mm3 (4,00010,000)


Which of the following is most likely:
Hematocrit 20% (3648)
Platelets 317,000/mm3 (150,000450,000) A. This patient has alpha thalassemia.
Reticulocyte count 0.3% (0.62.8) B. This patient has beta thalassemia.
C. This patient has sickle cell trait.
Total bilirubin 1.8 mg/dL (0.21.2) D. This patient has mild iron deficiency.
Direct bilirubin 0.3 mg/dL (0.00.3) E. This patient has anemia of inflammation.
Lactate dehydrogenase 240 U/L (135225)
11. A 47-year-old woman required emergency crani-
otomy and repair of aneurysm. On postoperative day 2
The best next step is:
she develops left calf pain and is found to have thrombus
in the popliteal vessel. Intravenous (IV) unfractionated
A. Admit to the hospital and start broad spectrum
heparin is started given recent neurosurgery. Two days
antibiotics.
later warfarin is started.
B. Admit to the hospital and transfuse packed red
On postoperative day 7 she is found to have right leg
blood cells (RBCs).
swelling and dyspnea. Evaluation reveals pulmonary

2 . H e m at o l o g y a n d O n c o l o g y 2 9
emboli and thrombus in the right common femoral left lower-extremity calf pain and is found to have popli-
vein. Platelet count is noted to be 87,000/L, down teal vein thrombus by compression US. She has no past
from 320,000/L at the time of surgery. history of thrombosis. She weighs 68kg and has normal
Next steps in her management include: renal function. The best treatment option is:

A. Immediately stop all heparin exposure, including IV A. Compression stockings


line flush. B. Aspirin 325 mg once daily
B. Start IV direct thrombin inhibitor such as C. LMWH overlapping with warfarin target INR
argatroban or bivalirudin. 1.52.0
C. Give 10 mg IV vitamin K to reverse warfarin. D. Rivaroxaban 15 mg twice daily 3 weeks, then 20
D. Test for heparin-platelet factor 4 antibodies. mg once daily
E. All of the above E. Intravenous unfractionated heparin (IV UFH)

12. A 32-year-old woman 8 weeks postpartum is seen 15. The patient in question 14 has no past history of
by her obstetrician for fevers, fatigue, and bruising. thrombosis, despite two pregnancies. Her father had a
In addition to elevated temperature, and some bruises DVT at age 43, and his father had a pulmonary embo-
and petechiae on examination, she is found to have a lism in his 60s. She has two daughters, ages 12 and 15.
hematocrit of 21%, platelet count of 23,000/L, and She should be tested for thrombophilia:
creatinine of 2.4 mg/dL. Values at time of delivery were
normal. While waiting to get more laboratory serum A. True
tests, she develops right arm weakness and confusion. B. False
She is sent directly to the emergency room and admit-
ted to the intensive care unit. Your immediate next step 16. A32-year-old woman undergoes laparoscopic resec-
should be: tion of a benign complex ovarian cyst by her gynecologist.
One week later she presents complaining of increasing
A. Request neurology consult for electroencephalogram lower back and abdominal pain. On examination she has
(EEG). a 7cm 4cm subfacial hematoma from the umbilicus to
B. Transfuse with platelets. right lower quadrant and a 6cm 3cm suprapubic ecchy-
C. Give IV fluids for dehydration. mosis. PT is 12.3 seconds (normal 11.213.4 seconds)
D. Check peripheral smear and find schistocytes; and aPTT is 72.4 seconds (23.836.6 seconds). Repeat
initiate plasmapheresis. testing reveals PT 12.8 seconds and PTT 81.5 seconds.
E. Send ADAMTS13 level and wait for results before The next test to evaluate an isolated elevated
treating. aPTTis:

13. A 39-year-old man is seen by a new primary care A. Factor XIII level
physician. Von Willebrand disease (vWD) is listed in B. Fibrinogen
his past medical history, but the patient does not know C. Mixing study
many details of this. He had an episode of gastrointes- D. Platelet aggregation studies
tinal (GI) bleeding in college requiring hospitalization E. d-dimer
and RBC transfusion when taking NSAIDS. He was
told he has vWD and to avoid aspirin and NSAIDS. 17. A 63-year-old man with type 2 diabetes mellitus and
Review of systems is negative for recent episodes of hypertension presents with recurrent painless hema-
bleeding, bruising, or epistaxis. His father has had turia. He undergoes a CT of the abdomen and pelvis
some bleeding episodes in the past. showing an enhancing 6.3 cm left kidney mass highly
What tests do you send to evaluate for vWD? suspicious of a renal cell carcinoma. There were multi-
ple retroperitoneal enlarged lymph nodes, and the lung
A. Factor VIII activity bases showed 45 pulmonary nodules, with the largest at
B. Von Willebrand factor (vWF) Ag level 1.1 cm. Patient has a good performance status. CBC and
C. Ristocetin cofactor CMP were within normal limits except for a hemoglobin
D. vWF multimer gel electrophoresis analysis of 11.5 g/dL. The next steps would include all except:
E. All of the above
A. Biopsy of the kidney mass
14. A43-year-old woman undergoes elective cholecys- B. Biopsy of the lung lesions
tectomy for gallstones. PMH is unremarkable and she is C. CT chest, brain magnetic resonance imaging (MRI),
on no medications. On postoperative day 4, she develops bone scan

3 0 T h e B r i g h a m I n t e n s i v e R e v i e w o f I n t e r n a l M e d i c i n e Q u e s t i o n a n d A n s w e r C o m pa n i o n
D. Initiating sunitinib or pazopanib C. Bone scan only
E. Proceed with cytoreductive nephrectomy D. CT scan of the abdomen/pelvis and bone scan
E. No further workup necessary
18. A72-year-old man who is an active smoker presents
with dysuria and vague abdominal/pelvic pain. His 21. A61-year-old man presents with a history of hyper-
urinalysis reveals 1020 RBC/HPF. He undergoes an tension and hyperlipidemia, and he is taking aspirin,
abdominal ultrasound that was unremarkable except hydrochlorothiazide, and simvastatin daily. He has
for potential thickening of the bladder wall on the left been undergoing PSA screening. Digital rectal exami-
side. Cystoscopy showed a 5cm bladder mass. Patient nation has been normal, but a recent PSA of 5.4 ng/mL
underwent transurehral resection of bladder tumor prompted a prostate biopsy. Biopsy demonstrated one
(TURBT) showing high-grade urothelial cancer with out of twelve cores positive for Gleason 4 + 3 prostate
focal invasion into the lamina propria. No muscle was cancer involving 15% of the core. His urologist ordered
available in the specimen. The next step would be: an endorectal coil MRI that showed a 1.6cm left-sided
lesion suspicious for tumor. There was no evidence of
A. Radical cystectomy extracapsular extension, seminal vesicle involvement,
B. Partial cystectomy (left bladder wall) or enlarged lymph nodes. Bone scan was also ordered
C. Proceed with BCG immunotherapy and showed no evidence of metastases. The patient is
D. Repeat TURBT anxious about the side effects associated with prostate
E. Chemoradiation to the bladder mass cancer treatment and asks for your guidance. Which of
the following is not a recommended option for primary
19. A 51-year-old woman presents with recurrent uri- management of this patients prostate cancer:
nary tract infections (UTIs) for the past 6months and
pelvic pain for 6 weeks. A CT pelvis showed a large A. Active surveillance
bladder mass. Cystoscopy followed by TURBT showed B. Radical prostectomy
a poorly differentiated urothelial carcinoma. Tumor C. External beam radiation therapy
invaded into the muscularis propria. Chest and abdo- D. Interstitial brachytherapy
men CT did not show distant metastases or enlarged E. None of the above (all are acceptable options)
lymph nodes. His serum creatinine is 0.75 mg/dL. Bone
scan was normal. Patient has an excellent performance 22. A66-year-old man presented with hip pain and was
status. The correct statement is: ultimately diagnosed with prostate cancer that had
metastasized to the ribs and pelvic bones. PSA at the
A. Neoadjuvant cisplatin-based chemotherapy is time of diagnosis was 180 ng/mL. He was started on
indicated based on a 15% absolute reduction in androgen deprivation therapy with leuprolide, a GnRH
death. agonist. After 6months on treatment, his pain resolved
B. Adjuvant cisplatin-based chemotherapy is indicated and his PSA has decreased to 0.8 ng/mL. He presents
based on a 15% absolute reduction in death. to the clinic to discuss side effects of his therapy. Risks
C. Neoadjuvant cisplatin-based chemotherapy is associated with androgen deprivation therapy include
indicated based on a 5% absolute reduction in death. all of the following except:
D. Adjuvant cisplatin-based chemotherapy is indicated
based on a 5% absolute reduction in death. A. Osteoporosis and bone fracture
E. Proceed with radical cystectomy. B. Prolactinoma
C. Hot flashes
20. A 59-year-old man, previously healthy and taking D. Anemia
only one 81 mg aspirin per day, was referred for pros- E. Increase in subcutaneous adipose tissue
tate biopsy after a screening showed a serum PSA of 4.8
ng/mL and unremarkable digital rectal examination 23. A64-year-old female with metastatic breast cancer
(DRE). A12-core needle biopsy was performed, reveal- to bone with metastases to ribs, pelvis, and multiple
ing two cores on the left with Gleason 3 + 3 adenocarci- vertebrae just started paclitaxel chemotherapy 1month
noma involving up to 20% of each core. All other cores ago. She complains of 12 weeks increasing mid-back
showed no evidence of disease. The patient asks you pain. She denies any weakness or bowel or bladder dif-
about imaging to complete his diagnostic workup. You ficulties. Neurologic examination is normal. The next
would advise: most appropriate step is:

A. Endorectal coil MRI only A. Obtain bone scan and CT scan to evaluate for
B. CT scan of the abdomen/pelvis only disease progression.

2 . H e m at o l o g y a n d O n c o l o g y 31
B. Prescribe NSAIDs and narcotics as needed for pain. room reveals a serum sodium of 120 mEq/L, blood urea
C. Obtain MRI of the spine. nitrogen (BUN) 17 mg/dL, and creatinine 0.8 mg/dL.
D. Recommend radiation to the spine. CT chest with IV contrast is negative for pulmonary
embolus but demonstrates a 5.5cm right hilar mass with
24. A45-year-old premenopausal women has severe hot associated mediastinal adenopathy and compression of
flashes during the day and frequent night sweats. She the superior vena cava; furthermore, there appear to
has a history of a stage II ER-negative/HER2-negative be multiple osseous and hepatic metastases. The most
breast cancer diagnosed 2 years ago treated with likely diagnosis is:
lumpectomy, radiation, and chemotherapy. She is cur-
rently disease-free. Which of the following is the best A. Breast cancer
choice for treating her hot flashes? B. Bronchial carcinoid tumor
C. Non-Hodgkins lymphoma
A. Paroxetine D. Small cell lung cancer
B. Amitryptyline E. Thymoma
C. Lorazepam
D. Soy protein 28. A70-year-old man with a history of kidney stones and
E. Vitamin E a 20-pack/year smoking history underwent a CT of the
abdomen as part of an evaluation for flank pain. CT dem-
25. A40-year-old woman with a strong family history onstrated recurrence of nephrolithiasis but also demon-
of breast cancer is considering taking tamoxifen for strated an incidental, peripheral 5mm left lower lobe lung
chemoprevention. She calls to discuss potential side nodule. Achest CT was performed and again showed the
effects of the medication. She reports she does not want small LLL nodule but did not demonstrate other nodules
to experience premature menopause, weight gain, or or mediastinal or hilar adenopathy. The most appropriate
depression. You advise her that: next step for follow-up of the lung nodule would be:

A. Tamoxifen causes none of these side effects. A. Repeat CT chest in 3months


B. She should take raloxifene instead. B. Repeat CT chest in 6months
C. She should take exemestane instead. C. Repeat CT chest in 2years
D. Tamoxifen may cause weight gain and depression, D. PET/CT now
but not premature menopause. E. Referral for CT-guided biopsy
E. She should wait until after menopause to start
tamoxifen. 29. A 58-year-old woman with minimal prior medical
history and with good performance status is found to
26. A 68-year-old man with a 40 pack/year smoking have a new 2.2 cm peripheral right lower lobe mass.
history and chronic obstructive pulmonary disease Complete staging evaluation demonstrates only the
(COPD) presented to his local emergency room after solitary lung mass, with no other sites of disease in the
falling in his bathroom rug and hitting his head and mediastinum or distantly. CT-guided biopsy confirms
right ribs. A noncontrast head CT did not show evi- a diagnosis of nonsmall cell lung cancer, adenocarci-
dence of bleed. Chest radiograph did not show any noma histology. The most appropriate next step in man-
broken ribs, but it did incidentally show a spiculated agement is:
right upper lobe lung mass, 3.6 2.4cm. Asubsequent
biopsy confirms nonsmall cell lung cancer. Which of A. Chemotherapy
the following radiographic studies is not recommended B. Cryoablation
as part of a subsequent staging evaluation: C. Repeat CT scan in 6months
D. Stereotactic radiation
A. Brain MRI with gadolinium E. Surgical resection
B. Chest CT with IV contrast
C. Positron emission tomography (PET)-CT 30. A 25-year-old woman presents with a dry cough,
D. Skeletal survey with plain films intermittent drenching night sweats, and diffuse pruri-
tis without identifiable rash. On physical examination,
27. A63-year-old woman who had smoked two packs of she appears tired. Vital signs are notable for a tem-
cigarettes daily for 30years presents with several weeks perature of 99.9F, heart rate of 110 beats per minute,
of increasing cough and dyspnea with exertion, fol- blood pressure of 100/80 mm Hg, respiratory rate of
lowed over the last few days with lethargy and swelling 18 breaths per minute, and oxygen saturation of 98%
of his face and arms. Initial evaluation in the emergency on RA. Her physical examination in notable for the

3 2 T h e B r i g h a m I n t e n s i v e R e v i e w o f I n t e r n a l M e d i c i n e Q u e s t i o n a n d A n s w e r C o m pa n i o n
absence of peripheral lymphadenopathy. Chest is clear 33. A27-year-old woman presents with a 3-month his-
to auscultation bilaterally. Cardiac examination reveals tory of amenorrhea. She reports menarche at age 12
tachycardia with a soft systolic ejection murmur. She and regular menses. She states she is healthy and denies
had no splenomegaly. Lower extremities are notable for other medical problems. She denies galactorrhea. She
linear excoriations. Laboratory studies reveal a white is not currently talking any medications except a birth
blood cell count of 14.3/L with 80% neutrophils and control pill. She had noted a 10-pound weight gain over
5% lymphocytes, 10% eospinophils, and 5% monocytes. the last 6months and occasionally feels cold. She attri-
Basic metabolic panel is normal. butes the weight gain to dietary changes and lack of
What is the next most appropriate step in her exercise. Her family history is significant for a mother
management? with primary hypothyroidism. Her examination is
unremarkable except on breast examination she had
A. Obtain a lactate dehydrogenase (LDH). galactorrhea. Aprolactin level was ordered and comes
B. Order a PET/CT scan. back elevated at 52 ng/dL (normal <22 ng/dL).
C. Refer to Dermatology for evaluation. In addition to repeating the prolactin level, what is
D. Obtain a chest X-ray. the next test(s) that should be ordered in this patient?
E. Send EBV serologies.
A. Pituitary MRI
31. A70-year-old male presents to the emergency depart- B. Serum pregnancy test
ment for evaluation of upper respiratory symptoms with C. Estradiol level
low-grade fevers. He appears clinically well. Physical D. TSH to rule out primary hypothyroidism
examination is notable for 1 cm palpable lymph nodes E. B and D
in the bilateral neck and axilla. His spleen tip is palpa-
ble just below the left costal margin. Laboratory studies 34. A35-year-old woman with history of regular men-
reveal a white blood cell count of 18,000/L with 75% ses presents with complaints of bilateral galactorrhea.
lymphocytes, with smudge cells 15% neutrophils, and 5% She notes her menses have been more irregular over the
monocytes. Chemistries are normal, including an LDH. last year and her last menstrual period was approxi-
What is the next most appropriate step? mately 6 weeks ago. She reports occasional headaches
but otherwise feels well. She does not take any medica-
A. Obtain peripheral blood flow cytometry. tion or herbal supplements. Laboratory testing reveals
B. Refer for excisional lymph node biopsy. a negative pregnancy test, TSH 2.2 mIU/L, and pro-
C. Order a PET/CT to evaluate the best node for lactin of 360 ng/mL. You suspect a prolactinoma and
biopsy. order a pituitary MRI, which reveals a 10mm 12mm
D. Perform a bone marrow aspiration and biopsy. pituitary lesion. There is no cavernous sinus invasion or
E. Treat with antibiotics and recheck a CBC in compression of the optic nerves by the pituitary tumor.
1month. You advise the patient that the initial treatment of
choice is:
32. A65-year-old woman presents complaining of a mass
in the left neck without fevers, night sweats, weight loss, A. Medical therapy with a somastatin analog
or localizing symptoms. On examination, she has nor- B. Observation only
mal vital signs and is found to have a 2.5cm left anterior C. Transsphenoidal pituitary tumor resection
cervical node, a 2cm left axillary node, and a 3cm right D. Medical therapy with a dopamine agonist
inguinal node. Her examination is otherwise normal. E. Radiation therapy to the pituitary
Laboratory studies including a CBC with differential,
basic metabolic panel, and LDH are normal. 35. A61-year-old man who has a history of a nonfunc-
What is the next most appropriate step in her tioning pituitary macroadenoma presents for routine
management? follow-up. He underwent surgical resection a number of
years ago for a large tumor and fortunately has had no
A. Refer to interventional radiology for a needle biopsy evidence of recurrence. He has hypopituitarism and is
of the most easily accessible lymph node on replacement hormone therapy for hypothyroidism,
B. Refer to otolaryngology for an FNA of the neck adrenal insufficiency, and hypogonadism. He takes levo-
mass thyroxine, hydrocortisone, and topical testosterone gel.
C. Refer to a surgeon for an excisional lymph node He returns for routine follow-up. He denies headaches
biopsy or visual changes. He complains of fatigue and inabil-
D. Obtain a PET/CT scan ity to lose weight. He has noted very dry skin but attri-
E. Send peripheral blood flow cytometry butes it to the weather. He denies tremor, palpitations,

2 . H e m at o l o g y a n d O n c o l o g y 33
insomnia, or change in bowel movements, although he lost to endocrine follow-up. He has a history of diabe-
tends toward constipation. He denies sexual dysfunc- tes mellitus, hypertension, and obstructive sleep apnea.
tion. On physical examination, he has gained 6 lb since He has been feeling poorly the last few months and
his last visit. His blood pressure is 126/82mm Hg and presents with fatigue, diffuse joint pain, and excessive
heart rate is 76 beats per minute. He does not have lid sweating. He feels his hands and feet have been swollen.
lag and his visual fields by confrontation are normal. He denies headaches or visual changes. His medications
His neurologic examination is otherwise normal. No include metformin and lisinopril. On examination,
tremor is noted. He does not look cushingoid. he has the classic course facial features of acromegaly,
His thyroid function tests show the following:TSH with enlarged tongue and spaces between his teeth.
0.1 mIU/L (normal 0.55 mIU/L), Free T4 0.6 ng/dL His blood pressure is 152/80 and pulse is 64. His visual
(normal 0.91.7 ng/dL). fields are normal on confrontation testing. He has mul-
What adjustments should be made to his levothyrox- tiple skin tags and his hands are very large and sweaty.
ine dose? The remainder of his examination is unremarkable.
Laboratory testing of his pituitary reveals normal thy-
A. No change; continue current levothyroxine dose roid, adrenal, and gonadal function. However, his
B. Decrease levothyroxine dose insulin-like growth factor I (IGF-I) and growth hor-
C. Increase levothyroxine dose mone (GH) levels are elevated at twice the upper limit
D. Discontinue levothyroxine completely of normal. Suspecting recurrence of his acromegaly, you
obtain a pituitary MRI, which does not show obvious
36. A 47-year-old woman has a long history of having significant tumor burden. An oral glucose tolerance
difficulty losing weight. She has multiple members test performed to assess GH suppression confirms his
in her extended family that are overweight but states acromegaly is not in biochemical remission.
her parents and siblings are all of normal weight. She You advise the patient that the initial treatment of
states she has always struggled to maintain her weight choice for his recurrent acromegaly is:
but in the last 2 years she has gained 45 pounds despite
exercising daily and watching her caloric intake. She A. Medical therapy with a somastatin analog
has recently been diagnosed with both type 2 diabetes B. Observation only
mellitus and hypertension. Last year she fell while rid- C. Repeat transsphenoidal pituitary tumor resection
ing her bicycle, broke her elbow, and was found to have D. Medical therapy with a dopamine agonist
osteoporosis. She reports menarche at age 12 with reg- E. Radiation therapy to the pituitary
ular menses until the last year when they have become
increasingly unpredictable. She has been distressed by 38. A 22-year-old man presents with headache and
her weight gain and recently started taking an antide- peripheral vision loss. Brain imaging reveals a large sel-
pressant. On physical examination she is obese with a lar mass with both solid and cystic components that is
BMI of 35 kg/m2 , blood pressure of 160/100 mm Hg, compressing the optic chiasm. Humphrey visual field
and heart rate of 72 beats per minute. She has a very testing demonstrated bitemporal hemianopsia. He has
round face with fine thin hair on her lateral cheeks. She no previous medical history, his BMI is 20, and he is
has supraclavicular fat accumulation and multiple pig- taking no medications. His initial pituitary functional
mented striae on her abdomen. She has several bruises evaluation revealed central hypogonadism and central
on her lower extremities. You are concerned she may hypothyroidism. His adrenal function was normal.
have Cushings syndrome. MRI imaging characteristics suggested the mass was a
Appropriate first-line screening tests for hypercorti- craniopharyngioma. He was taken to surgery to remove
solism include: the pituitary mass in attempts to decompress the optic
chiasm and restore his vision. He did well in the imme-
A. Pituitary MRI diate postoperative period but within 24 hours he began
B. Late night salivary cortisol to develop polyuria. His urine output increased to 400
C. 1 mg overnight dexamethasone suppression test mL per hour and was very dilute with a urine specific
D. 24-hour urine collection for free cortisol and gravity of less than 1.001. He complained of extreme
creatinine thirst. His serum sodium increased to 148 mEq/L, and
E. Answers B, C, and D his fasting glucose was elevated at 106 mg/dL.
What diagnosis are you suspecting in this patient?
37. A 53-year-old man with a history of acromegaly
presented to establish routine care with a primary care A. Syndrome of inappropriate antidiuretic secretion
provider. He underwent surgical resection of a pituitary (SIADH)
macroadenoma 6years prior to your visit and has been B. Diabetes mellitus, type 2

3 4 T h e B r i g h a m I n t e n s i v e R e v i e w o f I n t e r n a l M e d i c i n e Q u e s t i o n a n d A n s w e r C o m pa n i o n
C. Central diabetes insipidus fatigue and dyspnea on exertion (DOE). He notes feel-
D. Nephrogenic diabetes insipidus ing more winded during his 3-mile walks. Vital signs
E. None of the above are stable. Blood pressure is 120/60; O2 sat is 97% on
room air. No hepatosplenomegaly. Complete blood
39. A55-year-old woman was involved in a motor vehi- count is notable for white blood cell count of 4 K/L
cle accident as an unrestrained driver. Although she with 60% neutrophils, hemoglobin of 9 g/dL, and plate-
did not lose consciousness, she sustained a significant lets of 110,000/L. Labs are notable for a B12 of 700
head trauma and was brought by ambulance to the pg/mol, folate of 20 ng/ml, TSH of 1.2 mIU/mL, tes-
emergency department. She was evaluated and found tosterone of 400 ng/dL, and erythropoietin level of 90
to have a normal neurological examination, but due to U/L. Abone marrow biopsy is done and is remarkable
the mechanism of her accident, brain imaging by non- for dysplastic erythroid precursors and megakaryocytes
contrast CT scan was performed. She was incidentally with 1% blasts. Cytogenetics are normal. Based on the
found to have a large sellar lesion, estimated at approxi- IPSS-R score, he has 2 points1 for Hgb of 9 g/dL and
mately 2cm in greatest diameter. No acute intracranial 1 for normal cytogenetics. What would be the most
hemorrhage was identified. She had a laceration on her appropriate recommendation for therapy for his myelo-
forehead that was repaired and since she was otherwise dysplatic syndrome?
clinically stable she was discharged with a plan to follow
up with her primary care provider for further evaluation A. Allogeneic stem cell transplant
of this pituitary mass. B. Induction chemotherapy with an anthracycline +
In evaluating a patient with a newly discovered cytarabine
pituitary mass, what are the important initial clinical C. Revlimid
considerations? D. Erythropoiesis stimulating agent (ESA)
E. No treatment necessary
A. Evaluation for mass effects (headaches, visual loss,
cranial nerve abnormalities) 42. A 35-year-old woman presents to her PCP for her
B. Evaluation for pituitary hormonal hypersecretion annual visit. She has been feeling well. She denies fevers,
C. Evaluation for pituitary hormonal hypofunction night sweats, and weight loss. She denies GI symptoms,
D. All of the above including nausea, vomiting, abdominal pain, and early
E. None of the above satiety. She denies easy bleeding, bruising, or history
of clots. On examination, her spleen is palpable 2 fin-
40. A 22-year-old college student notes increasing gerbreaths below the costal margin. Acomplete blood
fatigue over the past few weeks that she attributes to count is notable for a white blood count of 60 K/L,
staying up late studying for final examinations. Over hemoglobin of 10 g/dL, and platelets of 600,000/L.
the past 23days she has noted several new bruises on Review of the peripheral smear is notable for a large per-
her upper and lower extremities. She does not recall centage of basophils and eosiniphils, along with myelo-
antecedent trauma leading to the bruising. This morn- cytes and metamyelocytes. JAK2 V617F mutation is
ing she developed a bloody nose lasting 20 minutes. negative.
She presents to Student Health for evaluation. Physical What is the most appropiate next step in management?
examination is notable for scattered quarter-sized
bruises on her upper arms and thighs. Lab studies are A. Start hydroxyurea
notable for a white blood cell count of 0.5 K/L, hemo- B. Check blood cultures and start empiric antibiotics
globin of 7 g/dL, and platelets of 50,000/L. Aperiph- for possible infection
eral smear is notable for 80% large atypical cells with C. Start high-dose aspirin (325 mg twice daily)
folded, bilobed, kidney-shaped nucleoli and elongated D. Send peripheral blood for cytogenetics to evaluate
bluish-red rods within the cytoplasm. What transloca- for the t(9;22) chromosome translocation
tion is the defining feature of this disease? E. Determine a leukocyte alkaline phosphatase (LAP)
score
A. t(11;14)
B. t(15;17) 43. A60-year-old woman with past medical history of
C. t(9;22) hypertension and COPD is brought to the emergency
D. t(8;14) room by her daughter. She is lethargic and is having
E. t(14;18) difficulty walking. She has noted increasing difficulty
breathing over the past few hours. Physical examina-
41. A 65-year-old man with PMH HTN presents to tion is notable for temperature of 100.7F, blood pres-
his primary care physician (PCP) with several weeks of sure of 150/70 mm Hg, respiratory rate of 24 breaths

2 . H e m at o l o g y a n d O n c o l o g y 35
per minute, and O2sat 88% on room air. She has diffi- Factor V Leiden heterozygote. He received Lovenox and
culty with finger-to-nose and rapid alternating move- then a 6-month course of warfarin. The surgeon wants
ments. She has fine crackles bilaterally. A complete your help stopping the bleeding and wants to give the
blood count is notable for a white blood cell count of patient recombinant VIIa (NovoSeven) before and dur-
110 106, hemoglobin of 8, and platelets of 75,000/L. ing the exploratory laparotomy.
Achest X-ray shows bilateral infiltrates. Which of the following is/are true?
What would be the appropriate next step in manage-
ment of this patient? A. NovoSeven is a great drug and is almost certain to
stop his bleeding.
A. Give a unit of PRBCs. B. There is objective evidence that NovoSeven will
B. Start antibiotics for possible pneumonia and follow improve hemostasis even in someone with a normal
for symptom improvement before initiating more PT and PTT, so you think it is indicated.
invasive procedures. C. While it may be worth trying NovoSeven in this
C. Start steroids and antibiotics for COPD flare. desperate situation, there is an increased risk
D. Start leukapheresis while establishing diagnosis. of developing DVT and or PE after the use of
E. Establish diagnosis and start appropriate therapy. NovoSeven.
D. Asafer alternative might be the administration
44. A32-year-old woman presents in active labor on a of the protease inhibitor Trasylol or the
Friday evening. She tells the admitting obstetrician who anti-fibrinolytic EACA.
is covering for a group practice that she has a history of
heavy periods and mucosal bleeding and was told she 46. You are asked to see a 72-year-old woman who is
has von Willebrand disease. She uses a nasal inhaler at scheduled for an elective knee replacement in 2 days.
the start of her periods, and it reduces bleeding. Her She has no prior history of bleeding but has a PTT of
mother and sister have similar bleeding histories. You 55 seconds and a PT of 15 seconds (INR 1.2). Review
find laboratory values done prior to her pregnancy of family history reveals no known bleeding disorder
showing vWF antigen of 42%, vWF activity of 38%, in parents, grandparents, three siblings, and multiple
and Factor VIII of 49%. All of these are below the lower children and grandchildren.
limit of normal. There is a note stating that the values Which of the following is most likely:
are compatible with Type Ivon Willebrand disease.
Which of the following statements is/are true? A. The patient is a female carrier of hemophilia, and the
case should be cancelled and/rescheduled when her
A. Von Willebrand factor levels go down during factor VIII and IX levels have been measured.
pregnancy. B. The patient may have factor XI deficiency and needs
B. Placement of an epidural catheter is contraindicated a factor level measured.
in a patient with von Willebrand disease. C. The patient may have factor XII (Hageman factor)
C. The correct treatment would be infusion of deficiency and should have no untoward bleeding.
recombinant VIIa (Novo Seven). D. The patient is likely to have a lupus anticoagulant
D. von Willebrand factor levels become normal during and has no risk of bleeding but an increased risk of
pregnancy in patients with Type Idisease. thrombosis.
E. Despite normal vWF levels pregnant women with
vWD have increased risk of bleeding during delivery 47. Your patient is a 56-year-old schoolteacher who
and should be treated prophylactically with Hemate P. presented with about 2months of intermittent blood
in the toilet bowl with bowel movements. Acolonos-
45. One of your patients, a 55-year-old man, was in copy is performed that demonstrates a mass in the
an automobile accident and has severe head trauma, a mid sigmoid, and biopsy confirms adenocarcinoma.
fractured femur and pelvis, and a possible liver lacera- He undergoes laparoscopic hemicolectomy and final
tion. On admission his PT and PTT are normal as is pathology reveals a 3 cm, moderately differentiated
his platelet count. The emergency room doctors and his adenocarcinoma through the muscle later into the
surgeons are having difficulty stabilizing his Hct, which serosa with two of nine lymph nodes positive. The
is 15% after 10 units of packed red cells. You are asked next step would be:
about his past medical history and state that he has no
known bleeding diathesis and that he has tolerated sur- A. Reoperate for more complete lymph node dissection
gery, including an appendectomy and cholecystectomy B. Referral to medical oncologist for consideration of
without bleeding. You do note that he has had one epi- chemotherapy
sode of DVT/PE that was unprovoked and that he is a C. Follow-up colonoscopy in 1year

3 6 T h e B r i g h a m I n t e n s i v e R e v i e w o f I n t e r n a l M e d i c i n e Q u e s t i o n a n d A n s w e r C o m pa n i o n
D. Referral to radiation oncologist for postoperative is normal except for localized tenderness over L4, with
radiation no neurologic deficits. What is the best next step?

48. A42-year-old man newly diagnosed with a Burkitts A. Increase his scheduled pain medications and see him
lymphoma presents to the emergency room 3days after back for a follow-up in 1month
beginning chemotherapy. He complains of a decreased B. Arrange for an urgent MRI of spine to evaluate for
urine output. Laboratory data show serum potassium of possible cord compression
6.5 mg/dL, BUN of 48 mg/dL, serum creatnine 2.4 mg/ C. Arrange for radiation therapy within 24 hours for
dL, uric acid 10.9 mg/dL, and phosphorus 6.4 mg/dL, probably cord compression
with uric acid crystals on his urinalysis. He is already on D. Emergency neurosurgical evaluation for suspected
allopurinol. What treatment is least likely to be effec- spinal cord compression
tive at this point?
50. Which one of the following statements is true
A. Hydration and Lasix, monitoring urine output and regarding superior vena cava (SVC) syndrome?
weight to maintain euvolemia
B. Acute management of hyperkalemia with calcium A. Establishing a tissue diagnosis is the most important
gluconate and insulin and dextrose step in management of most patients with SVC
C. Initiation of hemodialysis if diuresis does not occur syndrome.
D. Arecombinant uricase, such as rasburicase B. SVC syndrome should be treated with IV
E. Increase in dose of allopurinol thrombolytics and stenting of the vena cava.
C. Diagnosis is usually established by chest X-ray, then
49. A 69-year-old man with prostate cancer and wide- venography if needed.
spread bone metastases complains of worsening lumbo- D. Breast cancer and germ cell tumors are the most
sacral back pain for 2 weeks. His physical examination common malignant causes of SVC syndrome.

2 . H e m at o l o g y a n d O n c o l o g y 3 7
CH A PT ER2 A NSW ER S lesions of the pancreatic ductal epithelium, which have
the potential to progress to pancreatic adenocarcinoma.
1. ANSWER:B. Gastric cancer IPMNs involving the side branches of the pancreatic
ductal system (so-called branch duct IPMNs) carry an
Pernicious anemia is an autoimmune disorder that is approximate 10%20% risk of progressing to pancreatic
characterized by autoantibodies against intrinsic factor, cancer. For patients with branch duct IPMNs who do
destruction of gastric parietal cells, B12 deficiency, and not wish to have surgery, active surveillance with serial
gastric achlorhydria. Over time, the condition progresses MRI or pancreas protocol CT is considered an acceptable
to chronic atrophic gastritis, intestinal metaplasia, dyspla- option. This patient has an IPMN of the main pancreatic
sia, and ultimately gastric adenocarcinoma. The patient duct, which carries approximately a 70% risk of progres-
described here has developed occult upper GI bleeding sion to pancreatic adenocarcinoma. For this reason, all
from gastric cancer, stemming from long-standing perni- patients diagnosed with a main duct IPMN, without sig-
cious anemia and atrophic gastritis. Myelodysplastic syn- nificant contraindications to surgery, should be referred
drome has not been associated with pernicious anemia, nor to an experienced pancreatic surgeon for consideration of
is it associated with iron deficiency. Although the patient resection.
likely has iron-deficiency anemia, pernicious anemia is
also not associated with colon cancer. MALT lymphoma
is highly linked to Helicobacter pylori infection, not perni-
cious anemia. Although some reports indicate an increased 4. ANSWER:D. Discontinue iron therapy; send
risk of pancreatic cancer associated with pernicious anemia, ferritin and hemoglobin electrophoresis
pancreatic cancer would not cause iron-deficiency anemia.
This young woman has a microcytic anemia despite 3years
of oral iron therapy. Since patients are frequently noncom-
pliant with oral iron, one must always consider that the
2. ANSWER:B. BRCA 2 mutation patient has not taken the medication. Although the best
measure of iron stores would be to check a serum ferritin,
This patient presents with a likely diagnosis of pancreatic her iron and total iron binding capacity (TIBC) show an
adenocarcinoma. In addition to his young age, his family his- iron saturation of over 50%, so it seems unlikely that she
tory raises raise concern for a possible hereditary cancer syn- remains iron deficient. A ferritin will allow you to deter-
drome. The patients maternal family history, including early mine if she is iron replete or iron overloaded. The profound
breast cancer and pancreatic cancer, is highly suggestive of microcytosis in the face of a near-normal hematocrit and
BRCA mutation. Both BRCA1 and 2 mutations are associ- presumed adequate iron stores should lead one to consider
ated with increased risk of pancreatic cancer, but BRCA 2 is a diagnosis of thalassemia minor. Indeed, an MCV this low
much more strongly associated. BRCA 2 mutation is thought is extremely rare in iron deficiency and would never occur
to be responsible for approximately 15% of familial pancre- in the absence of profound anemia. Beta-thalassemia minor
atic cancer. Although Lynch syndrome (HNPCC) is associ- can be diagnosed by hemoglobin electorphoresis, and it is
ated with a modestly increased risk of pancreatic cancer, his established by the demonstration of an elevated hemoglo-
fathers diagnosis of colon cancer at age 72 is not highly sug- bin A 2 (>4%). Given the minimal anemia, it is even more
gestive of Lynch syndrome. Li-Farumeni syndrome results likely that this represents alpha-thalassemia, which can
from a mutation in the p53 tumor suppressor gene, and it only be diagnosed by DNA studies, since it does not result
is most notably associated with early diagnosis of sarcoma, in an abnormal hemoglobin electorphoresis.
brain tumors, breast cancer, and leukemia, but not pancreatic
cancer. Mutations in CDKN2a result in the familial atypical
multiple-mole melanoma syndrome, which is associated with
markedly increased risk of melanoma and pancreatic cancer. 5. ANSWER:C. Erythropoietin deficiency
Germline mutations in CDH1 (E-cadherin) are associated
with very high risk of diffuse gastric cancer and breast cancer, The patient has a normochromic, normocytic anemia. The
but not pancreatic cancer. most likely reason for the gradual decrement in his hema-
tocrit is that he has had a gradual decline in his erythropoi-
etin level coincident with his moderate renal insufficiency.
3. ANSWER:D. Referral to a pancreatic surgeon for This can be readily assessed by checking an erythropoietin
resection level, which should be several hundred with a H/H in this
range.
The patient was incidentally found on an imaging study Although this could be a result of combined iron and B12
to have a main duct IPMN. IPMNs are premalignant deficiency, one would expect the red blood cell distribution

3 8 T h e B r i g h a m I n t e n s i v e R e v i e w o f I n t e r n a l M e d i c i n e Q u e s t i o n a n d A n s w e r C o m pa n i o n
width (RDW) to be abnormally high. Furthermore, B12 8. ANSWER:E. Add coverage for atypical organisms,
deficiency (which can be a complication of metformin) administer supplemental O2, and exchange transfuse.
should also depress the white blood cell count and the
platelet count. ACE inhibitors are not typically associated The patient presented with a clearly defined right lower
with anemia. In a patient with benign prostatic hypertro- lobar pneumonia that progressed to acute chest syn-
phy, the sudden development of prostate cancer leading to drome (ACS). ACS is an acute illness characterized by
marrow infiltration and anemia with a normal white count a constellation of fever, chest pain, shortness of breath,
and platelet count and a normal LDH would seem highly hypoxia, and a new infiltrate(s) on chest radiograph. ACS
unlikely. is a life-threatening complication of sickle cell disease often
caused by atypical organisms such as Mycoplasma pneumo-
nia. It is not uncommon for adults with sickle cell disease
to present with a pain crisis or pneumonia and deteriorate
6. ANSWER:B. Vitamin B12 deficiency in the hospital to ACS. The treatment for ACS is antibiotic
coverage for typical and atypical organisms, red blood cell
This patient has megaloblastic anemia secondary to malab- exchange transfusion to a goal HbS <30% and other sup-
sorption of vitamin B12, which is absorbed in the termi- portive care. Apulmonary embolus would not explain his
nal ileum. Because people with a good diet usually have new pulmonary infiltrates.
substantial body stores of vitamin B12, patients typically
do not develop vitamin B12 deficiency until several years
following failure of B12 absorption. Megaloblastic anemia
causes pancytopenia, since the failure of DNA synthe- 9. ANSWER:B. Admit to the hospital and transfuse
sis affects all rapidly dividing cells. Patients may also have packed red blood cells.
mouth ulcers, symptoms of malabsorption, smooth tongue,
and neurologic abnormalities. Vaso-occlusive pain crises are the hallmark of sickle cell
Sideroblastic anemia typically presents with a micro- disease, and patients may be admitted to the hospital
cytic anemia. Hemolytic anemia should not cause leukope- very frequently for pain control. However, not all pre-
nia or thrombocytopenia. Although myelodysplasia can be sentations of individuals with sickle cell disease are for
associated with macrocytosis, it rarely causes an MCV over a pain crisis. Asignificant finding in this presentation is
100110. Furthermore, although you may see psuedo-Pelger anemia accompanied by reticulocytopenia. This patient
Huet anomaly (bilobed polys) in the neutrophils in patients has transient red cell aplasia (TRCA) due to Parvovirus
with myelodysplastic syndrome, hypersegmented polys are B19. His anemia is likely to worsen because of the short-
seen only in megaloblastic anemia. ened life span of sickle red blood cells (~20days) and the
inability of his bone marrow to produce reticulocytes.
Reticulocytopenia begins about 5 days post exposure
and continues for 7 to 10days. Recovery is spontaneous
7. ANSWER:C. Splenectomy but support with RBC transfusions is necessary. The two
indications for RBC exchange transfusion are stroke/
The patient has anemia with increased reticulocytes and TIA and acute chest syndrome. He has no indication for
spherocytes are seen on the peripheral smear. The diagnos- broad-spectrum antibiotics.
tic distinction to be made is between a congenital hemolytic
anemia and an immune hemolytic anemia. The long history
of jaundice and the relatively well-compensated hemolysis
is most suggestive of hereditary spherocytosis. Hereditary 10. ANSWER:A. This patient has alpha thalassemia.
spherocytosis, if severe enough to require therapy, responds
very well to splenectomy. This patient has a microcytosis with an elevated RBC count.
The negative Coombs test makes immune hemolytic This is fairly specific for the thalassemias. Other causes of
anemia (AIHA) unlikely, although about 10% of patients microcytosis (iron deficiency and anemia of inflammation)
with AIHA may have a negative Coombs test. Steroids are are associated with an RBC count that is not elevated (i.e.,
the preferred first-line therapy for AIHA. The reticulocy- low or normal). On hemoglobin electrophoresis, the feature
tosis makes iron deficiency extremely unlikely. Although diagnostic of beta thalassemia is an elevated Hb A 2 (>4%).
hemolysis is the hallmark of paroxysmal nocturnal hemo- The hemoglobin electrophoresis in alpha thalassemia is
globinuria (PNH), patients with PNH have intravascular normal, as in this case. A diagnosis of alpha thalassemia
hemolysis secondary to surface activation of complement requires DNA analysis. Sickle cell trait shows no abnormal-
and therefore do not present with spherocytes in the periph- ities on CBC. The hemoglobin electrophoresis would show
eral blood. ~48% HbS and ~52% HbA.

2 . H e m at o l o g y a n d O n c o l o g y 3 9
11. ANSWER:E. All of the above ASA or compression stockings are inadequate treat-
ment for acute VTE. Target INR of 1.52.0 is too low for
This patient has a clinical diagnosis of heparin induced acute VTE treatment. Although IV UFH could be consid-
thrombocytopenia (HIT) with development of new clots ered, at 96 hours postop she should have adequate hemosta-
while on heparin for 5 days, and with a platelet count sis to allow oral anticoagulant therapy with a short-acting
decrease greater than 50% of initial count. Despite recent new oral anticoagulant.
neurosurgery, life-saving treatment requires doing A, B, and
C immediately. Although testing should be sent to confirm
diagnosis, waiting for results before starting treatment is con-
traindicated and can lead to poor outcome, including death. 15. ANSWER:A. True

Although she has had a provoked DVT, her father had a


DVT at a young age and she has two daughters who might
12. ANSWER:D. Check peripheral smear and find consider OCP use. Finding an inherited thrombophilia
schistocytes; initiate plasmapheresis. in this patient would not change current management or
duration of anticoagulation treatment for provoked DVT
This patient has the classic pentad of findings associ- (3months), but if present it would prompt testing of daugh-
ated with TTP. Although DIC from retained products of ters prior to OCP use.
conception could result in microangipathic changes and
thrombocytpenia, DIC is not usually associated with renal
failure or focal neurologic findings. Risk factors for TTP
include pregnancy, HIV infection, autoimmune disorders, 16. ANSWER:C. Mixing study
and others. Rapid initiation of plasmapheresis is critical,
even if confirmatory laboratory test results are not available. A mixing study should be done next to determine if the
aPTT is prolonged due to the absence of a coagulation fac-
tor in the intrinsic coagulation pathway or to the presence
of a circulating inhibitor. Amixing study is performed by
13. ANSWER:E. All of the above mixing patient plasma with normal pooled plasma in a 1:1
ratio. The normal plasma supplies enough of the coagula-
This patient gives a good history for von Willebrand dis- tion factors to correct a congenital deficiency and normal-
ease, with mucosal bleeding in the setting of impaired ize the aPTT. If an inhibitor is present, such as an acquired
platelet function due to NSAIDS that was severe enough FVIII inhibitor, lupus anticoagulant or even drugs such
to require red cell transfusion. All of the tests listed above as unfractionated heparin or bivalirudin, the addition of
are required to make a diagnosis of vWD and determine plasma will not correct the aPTT, as the factors in the nor-
the type (I, II, or extremely unlikely III). vWF antigen mal plasma will be affected. If the mixing study corrects
level measures the actual vWF protein level, but vWF also the aPTT, then individual factor levels can be performed
functions to carry FVIII and prolong its plasma half-life to determine which is deficient (XII, XI, IX, VIII). If
as well as bind to plateletshence the need to measure the aPTT does not correct, tests for inhibitors need to be
factor VIII activity and ristocetin cofactor activity, a performed.
functional surrogate assay for patient vWF platelet bind- Factor XIII activity is not measured by the aPTT.
ing. vWF multimer gel electrophoresis determines vWF Since the PT is normal, fibrinogen levels should be normal.
multimer size, which is needed in order to classify vWD Platelets have no impact on the aPTT. D-dimer results
type II subtypes. would not be of help in determining etiology of prolonged
aPTT in this postop patient with normal PT.

14. ANSWER:E. Intravenous unfractionated heparin


(IV UFH) 17. ANSWER:D. Initiating sunitinib or pazopanib

This patient is an ideal candidate for treatment of acute It is mandatory to have tissue diagnosis prior to initiating
DVT with rivaroxabanshe is young, with no significant systemic therapies. While RCC is by far the most common
health problems or medications and has limited duration of diagnosis, transitional cell cancer (TCC) of the renal pelvis
therapy for provoked VTE. This dose (15 mg twice daily for is a consideration. In that case, removal of the primary will
3 weeks followed by 20 mg once daily) is the treatment dose not be indicated and the patient would proceed with a dif-
for acute DVT or pulmonary embolism. ferent systemic therapy.

4 0 T h e B r i g h a m I n t e n s i v e R e v i e w o f I n t e r n a l M e d i c i n e Q u e s t i o n a n d A n s w e r C o m pa n i o n
18. ANSWER:D. Repeat TURBT than age 70. However, a diagnosis of localized Gleason 4
+ 3 disease in a patient with life expectancy greater than
It is very important to have muscle present on the biopsy 10years, as in the case above, warrants definitive treatment.
report to document or not muscle invasion for appropriate
staging and to guide therapy.
Superficial disease:BCG for high-risk superficial disease.
Muscle-invasive disease:neoadjuvant chemo + cystectomy. 22. ANSWER:B. Prolactinoma

Androgen deprivation therapy (ADT) is the first-line treat-


ment for advanced prostate cancer and is used as neoadjuvant
19. ANSWER:C. Neoadjuvant cisplatin-based treatment along with radiation therapy for intermediate and
chemotherapy is indicated based on a 5% absolute high-risk localized disease. After about 1 week on ADT,
reduction in death. GnRH receptors in the pituitary are down-regulated and
LH and FSH production are diminished. This results in a
In patients with good PS and good kidney function, neoad- profound reduction in testosterone production by the tes-
juvant therapy is standard. It is associated with a 5% abso- ticles and induces tumor responses in 80%90% of patients.
lute reduction in death and 15% relative reduction in death. Castrate testosterone levels are associated with several com-
mon side effects:loss of libido, osteopenia/osteoporosis, hot
flashes, decrease in lean body mass, increase in subcutaneous
adipose tissue, thinning of body hair, decrease in penile and
20. ANSWER:E. No further workup necessary testicular size, gynecomastia, anemia, and mild fatigue. More
rarely treatment may be associated with cognitive decline or
The most common sites of prostate cancer metastases are bone depression. Some have observed increased incidence of car-
and lymph nodes. For patients with low-risk disease (T1c diovascular event in those receiving ADT; others have found
[normal DRE] or T2a [tumor involving 1/2 of one lobe] no convincing association. Associations between ADT and
AND PSA <10 ng/mL and Gleason score <7), the likeli- development of colorectal cancer have also been reported,
hood of spread of disease visible by imaging is exceedingly low. but a true causal relationship has not been firmly established.
Further workup is not recommended for these patients. CT There is no known association with other cancers.
of the abdomen/pelvis and bone scan are recommended for
men with serum PSA > 20 ng/mL, or serum PSA > 10 ng/
mL with a positive DRE, or a Gleason 8 tumor, or evidence 23. ANSWER:C. Obtain MRI of the spine.
of tumor extension through the prostate capsule. Several insti-
tutions use endorectal coil MRI as part of initial workup, but In patients with breast cancer and known metastatic dis-
the clinical utility of this modality is not yet clear. ease to the bone presenting with worsening back pain, there
should be a high clinical suspicion for spinal cord compres-
sion. MRI is the diagnostic study of choice for spinal cord
compression and is superior to CT. Radiation should not be
21. ANSWER:A. Active surveillance done until metastatic disease has been documented as the
source of the pain.
Given the long natural history of prostate cancer, the fre-
quency of indolent disease in a high percentage of patients,
and significant morbidity associated with local treatment,
active surveillance is increasingly recommended by urolo- 24. ANSWER:A. Paroxetine
gists and medical oncologists. Prospective series clearly
demonstrate that active surveillance (serial PSAs and Paroxetine has been shown in placebo-controlled random-
physical examinations every 34months along with repeat ized trials to reduce the severity of hot flashes and is FDA
biopsies every 12years) can safely be used to manage cer- approved for this indication.
tain patients, avoiding (or at the very least forestalling)
life-altering therapies. However, not all patients are good
candidates for active surveillance. For patients less than
70 years old, criteria for entry into most active surveil- 25. ANSWER:A. Tamoxifen causes none of these side
lance series include early-stage disease, Gleason score 6, effects.
and serum PSA 10 ng/mL. There are data suggesting that
low-volume Gleason 3 + 4 disease can be successfully man- In placebo-controlled randomized trials of tamoxifen for
aged by active surveillance, particularly for patients older breast cancer prevention among patients without a personal

2 . H e m at o l o g y a n d O n c o l o g y 41
history of breast cancer, there was no increased risk of prema- 27. ANSWER: D. Small-cell lung cancer
ture menopause, weight gain, or depression. Both raloxifene
and exemestane only have data for postmenopausal women. The findings of a large hilar mass with superior vena
cava (SVC) syndrome, hepatic and bony metastases, and
hyponatremia in a 63-year-old woman with a significant
smoking history are most consistent with a small-cell lung
26. ANSWER:D. Skeletal survey with plain films cancer (SCLC). SCLC is the tumor type most closely
associated with smoking: about 95% of SCLC patients
In the case of lung cancer, the most common sites of will have a smoking history. SCLC tends to occur in the
involvement are the mediastinum and contralateral lung, central chest, so that SVC syndrome or tracheal compres-
liver, bone, brain, and adrenal glands. The appropriate sion can be presenting symptoms. In fact, SCLC is the
radiographic staging workup of lung cancer should include most common cause of SVC syndrome. SCLC tends to
imaging of all of those sites. be aggressive in its course, with metastatic disease found
Brain imaging is required for all patients with small at the time of initial diagnosis in 60%70% of SCLC
cell lung cancer, patients with nonsmall cell lung can- patients. SCLC can also be associated with a number of
cer of stage IB or greater, or any lung cancer patient with paraneoplastic syndromes. The most common of these
any neurologic symptom. In this case, the unexplained is hyponatremia due to the syndrome of inappropriate
fall in the setting of new lung cancer warrants dedicated ADH (SIADH).
brain imaging. MRI is preferred over CT for brain imag-
ing, though CT with contrast is a reasonable alternative.
Regardless of modality, IV contrast (gadolinium for MRI,
iodinated contrast for CT) is required for sufficient detec- 28. ANSWER:B. Repeat chest CT in 6months
tion of brain metastases. The prior noncontrast head CT is
insufficient. The Fleischner Society has published guidelines for the
A chest CT with IV contrast that is performed specifi- follow-up and management of nodules smaller than 8 mm
cally for lung cancer staging should be expanded to include detected incidentally at nonscreening CT. These guidelines
the liver and adrenal glands. The administration of IV make a distinction between low- and high-risk patients, and
contrast enhances the ability to detect hepatic metastases. nodule sizes 4 mm, >46 mm, >68 mm, and >8 mm.
Furthermore, by delineating the mediastinal vasculature High risk patients are defined as those with a history of
with IV contrast, the clinician can better assess the size and smoking or other known risk factors. In this case, a 5 mm
extent of mediastinal lymph nodes. nodule in a patient with a 20-pack/year smoking history
PET/CT or bone scan is the best modality to look for merits a follow-up CT in 612 months. For a nodule that
bone metastases in lung cancer. PET/CT is the preferred small, it would be difficult to reliably perform a CT-guided
modality. Aplain film skeletal survey is not sensitive enough biopsy, and it may be below the size threshold for FDG-
to detect occult bone metastases in lung cancer. avidity on PET/CT (see Table 2.9).

Table2 .9 GUIDELINES FOR MANAGEMENT OF SMALL PULMONARY NODULES DETECTED ON COMPUTED


TOMOGR APHY (CT) SCANS:ASTATEMENT FROM THE FLEISCHNER SOCIETY

NODULE SIZE LOW-R ISK PATIENT HIGH-R ISK PATIENT


(MM)*

4 No follow-up needed Followup CT at 12 mo; if unchanged, no further follow-up


>46 Follow-up CT at 12 mo; if unchanged, no further Initial follow-up CT at 612 mo, then at 1824 mo if no change
follow-up
>68 Initial follow-up CT at 612 mo, then at 1824 mo Initial follow-up CT at 36 mo, then at 912 and 24 mo if no
if no change change
>8 Follow-up CT at around 3, 9, and 24 mo, dynamic Same as for low-risk patient
contrast-enhanced CT, PET, and/or biopsy
NOTE:Newly detected indeterminate nodule in persons 35years of age or older.
Nonsolid (ground-glass) or partly solid nodules may require longer follow-up to exclude indolent adenocarcinoma.
*Average of length and width.
SOURCE:From Radiology 2005;237(2):395400.

4 2 T h e B r i g h a m I n t e n s i v e R e v i e w o f I n t e r n a l M e d i c i n e Q u e s t i o n a n d A n s w e r C o m pa n i o n
29. ANSWER: E. Surgical resection 31. ANSWER:A. Obtain peripheral blood flow
cytometry.
The gold standard therapy for newly diagnosed, early-stage
lung cancer is anatomic resection. In patients with good This patient presents with the classical findings of a patient
performance status and adequate lung function, the goal is with CLL with diffuse lymphadenopathy and splenomegaly
anatomic resection (i.e., resection of the mass and its asso- with a lymphocytosis and smudge cells on peripheral smear.
ciated lymphatic drainage). This is classically achieved via The diagnosis of CLL may be established using peripheral
lobectomy, though segmentectomy (resection of an ana- blood flow cytometry. Excisional lymph node biopsy and
tomic segment of a lung lobe) may be adequate for some bone marrow biopsy are not necessary given the diagno-
tumors that are small enough and in the right location. sis may be established using flow cytometry. PET/CT is
For patients who are poor operative candidates, stereo- not necessary in the evaluation of CLL. CT scans may be
tactic radiation therapy (which may also be marketed as obtained in the patients with adverse cytogenetics, such as
Axesse, CyberKnife, Gamma Knife, Novalis, Primatom, 11q and 17p deletion, but are not necessary at this time. The
Synergy, X-Knife, TomoTherapy, or Trilogy) can be con- patients symptoms are suggestive of a viral infection and
sidered. The potential use of stereotactic radiation for antibiotics are not indicated.
early-stage lung cancer may be limited by tumor size and
location. In general, tumors larger than 4cm are not gen-
erally amenable to definitive control with stereotactic
radation. Furthermore, there have been concerns about 32. ANSWER: C. Refer to a surgeon for an excisional
stereotactic radiation to central tumors, as early trials of lymph node biopsy
this modality raised concerns about collapse of adjacent
airways. The patient presents with diffuse, asymptomatic lymph-
Chemotherapy is not considered a curative modality in adenopathy, suggestive of involvement by an indolent
lung cancer, and it has not been shown to be an appropriate lymphoma. To establish the diagnosis, an adequate tissue
upfront therapy for potentially resectable early stage non biopsy is critical. Given that she has palpable peripheral
small cell lung cancer. lymphadenaopthy, referral to a surgeon for a excisional
Modalities such as cryoablation and radiofrequency lymph node biopsy is the most appropriate way to obtain
ablation continue to be investigated in lung cancers but adequate diagnostic tissue. Needle biopsies lack adequate
are not considered a standard curative treatment option in tissue to evaluate for architecture. Peripheral blood flow
early-stage lung cancer at this time. cytometry is unlikely to yield a diagnosis in the setting of
More recent data suggest that this may be achieved by a normal white blood cell count and differential. PET/CT
segmentectomy depending on a tumors size and location. scan is indicated for staging in aggressive lymphomas and
For patients who are poor operative candidates, consider Hodgkin lymphoma after a diagnosis has been established.
stereotactic radiation.

33. ANSWER:E. Answers B and Dpregnancy test


30. ANSWER: D. Obtain a chest X-ray. and TSH

This young woman presents with signs and symptoms wor- Hyperprolactinemia is a relatively common cause of amen-
risome for lymphoma, particularly Hodgkin lymphoma orrhea and is certainly part of the differential diagnosis
(HL). Although pruritus is a frequent symptom in patients when a woman presents with missed menstrual cycles. It is
with HL, intense pruritus without a rash is quite rare. On important to replicate the lab abnormality, as performance
the other hand, B symptoms (fevers, drenching night sweats, of the breast examination immediately prior to blood sam-
and unexplained weight loss) are common in this disease. In pling can cause a transient elevation in prolactin. Pregnancy
addition, she is a young adult at the median age of presenta- must be excluded as a cause of hyperprolactinemia in all
tion of HL. Finally, the leukocytosis with lymphopenia and women of childbearing potential, since prolactin levels rise
elevated eospinophils may be seen with HL. Given that she as a normal physiologic response during pregnancy. ATSH
does not have peripheral lymphadenopathy, the next most level is important, particularly in this patient with a posi-
logical step in her management is to obtain a CXR to look tive family history of thyroid disease and some nonspecific
for a mediastinal mass. LDH is rarely elevated in HL. PET/ symptoms of hypothyroidism (weight gain and cold intol-
CT scan is done as part of staging once a diagnosis has been erance) since primary hypothyroidism can cause hyperp-
established. She does not have symptoms typical for EBV rolactinemia. A pituitary MRI to assess for the presence
infection. of a pituitary adenoma is indicated after other causes of

2 . H e m at o l o g y a n d O n c o l o g y 4 3
hyperprolactinemia have been excluded. For this reason a as notable physical examination findings (round face, hir-
pituitary MRI is not the next test to be performed. sutism, supraclavicular fat accumulation, pigmented striae,
and bruising). After ruling out exogenous glucocorticoid
exposure, there are three acceptable first-line screening tests
for endogenous hypercortisolism, including late night sali-
34. ANSWER:D. Medical therapy with a dopamine vary cortisol, 1 mg overnight dexamethasone suppression
agonist test, and 24-hour urine free cortisol with creatinine. Aran-
dom isolated serum cortisol level is not appropriate for
Medical therapy with a dopamine agonist is considered the screening as cortisol levels are dynamic over time and influ-
first-line initial therapy for symptomatic prolactinomas. enced by a number of factors such as acute pain and stress.
Dopamine agonists have both antiproliferative and anti- Although cortisol levels vary minute to minute, patients
secretory effects on prolactin-producing pituitary cells and with Cushings syndrome show generally elevated levels over
typically result in both decreased prolactin secretion and a 24-hour period. The late night salivary cortisol test takes
tumor shrinkage. Cabergoline is preferred over bromocrip- advantage of the fact that cortisol is secreted in a diurnal
tine for most patients due to its increased efficacy and toler- rhythm such that nadir levels are usually observed late in the
ability. Somatostatin analogs are used in the treatment of evening in a person without hypercortisolism. Patients with
acromegaly. Transsphenoidal surgery is reserved for patients hypercortisolism lose the diurnal variation and show high
who do not tolerate or are not responsive to the dopamine cortisol levels when they should be at the nadir. Similarly,
agonists. Radiation would only be considered in very rare Cushings syndrome patients will have high 24-hour urine
cases of advanced or progressive prolactinomas that have free cortisol levels that reflect their overall high continuous
failed medical or surgical interventions. Observation alone cortisol secretion over a 24-hour period. Urinary creatinine
would not be the preferred approach in a patient with a should always be checked at the same time as the 24-hour
macroadenoma (size >10mm) as control of tumor growth urinary free cortisol to ensure adequacy of the urine collec-
would be desired. tion. Finally, the 1 mg overnight dexamethasone suppres-
sion test takes advantage of the fact that individuals with
normal hypothalamic pituitary adrenal (HPA) physiology
will suppress cortisol secretion when given the synthetic ste-
35. ANSWER:C. Increase levothyroxine dose. roid dexamethasone. Individuals with a normal HPA axis
and thus normal inhibitory feedback mechanisms should
This patient has central hypothyroidism due to his pitu- suppress their cortisol level to less than 1.8g/dL at 8:00
itary macroadenoma. Clinically, he has some symptoms a.m. the morning following ingestion of 1 mg dexametha-
suggestive of inadequately treated hypothyroidism, includ- sone at 11p.m. the night before. These three tests only con-
ing fatigue, weight gain, and dry skin. He does not have firm hypercortisolism/Cushings syndrome and do not tell
symptoms or physical examination findings suggestive of the cause of the cortisol hypersecretion. Additional testing
hyperthyroidism. His laboratory tests are classic for central with an ACTH level is required to differentiate between
hypothyroidisma low free T4 in combination with either ACTH-dependent (most commonly pituitary Cushings
an inappropriately low or normal TSH. This case illustrates disease or ectopic source) and ACTH-independent (likely
how TSH alone cannot be used reliably to assess thyroid adrenal source of cortisol overproduction) Cushings syn-
hormone replacement in patients with central hypothyroid- drome. Therefore, a pituitary MRI is not a first-line screen-
ism and/or hypothalamic/pituitary dysfunction. Clinical ing test for hypercortisolism and is only indicated once an
findings and peripheral thyroid hormone levels must be ACTH-dependent cause is suspected.
used to guide levothyroxine replacement therapy. Therefore,
in this case, despite the low TSH, the levothyroxine dose
should be increased with the goal to normalize the free T4.
37. ANSWER:A. Medical therapy with a somastatin
analog

36. ANSWER:E. B, C, and DLate night salivary Multiple treatment modalities exist for recurrent acromeg-
cortisol, 1 mg overnight dexamethasone suppression aly. In a case such as this one, medical therapy would be the
test, and 24-hour urine free cortisol with creatinine initial treatment recommendation, particularly since sig-
nificant residual/recurrent pituitary tumor is not visualized
This patient should be screened for Cushings syndrome/ on MRI. There are a number of medical treatment options,
hypercortisolism due to a number of concerning clinical but somatostatin analogs are typically the initial treatment
signs, including rapid weight gain and new diagnoses of of choice in this setting. GH-secreting pituitary tumor
diabetes mellitus, hypertension, and osteoporosis, as well cells express somatostatin receptors and treatment with

4 4 T h e B r i g h a m I n t e n s i v e R e v i e w o f I n t e r n a l M e d i c i n e Q u e s t i o n a n d A n s w e r C o m pa n i o n
somatostatin analogs acting via these receptors inhibits initial evaluation of the patient. Athorough history must
somatotrope cellular proliferation and GH secretion lead- be obtained to illicit symptoms of mass effects and pitu-
ing to a reduction in IGF-I levels. Biochemical remission, itary dysfunction (both hypersecretion and hypofunction).
defined as normalization of IGF-I and GH levels, is associ- Having a complete understanding of pituitary physiology
ated with improvement or resolution of acromegaly symp- will help guide the questions related to determining the
toms. Long-acting somatostatin analogs, such as lanreotide function of each hormonal axis. The clinician should look
and octreotide LAR, are typically considered the initial for both signs and symptoms to suggest hormonal hyperse-
medical option in uncontrolled acromegaly. The dopamine cretion (i.e., hyperprolactinemia, GH excess, or hypercor-
agonist cabergoline can be used but tends to be more effica- tisolism) or hormonal hypofunction (i.e., central adrenal
cious when the prolactin is only mildly elevated. The GH insufficiency, central hypothyroidism, central hypogonad-
antagonist, pegvisomant, is also an option for this patient. ism, GH deficiency, diabetes insipidus). Laboratory evalu-
This drug tends to be second line due to the need for fre- ation should be performed to assess pituitary hormonal
quent subcutaneous injections and its lack of inhibition of function. MRI imaging should be performed to better
tumor growth. Observation would not generally be advised characterize the structure of the mass. Taken together,
as the patient is symptomatic and biochemical remission is both laboratory and imaging studies will help guide further
associated with improved clinical outcomes and reduced evaluation and treatment of the patient presenting with a
mortality. Repeat surgery would not be optimal in this case, pituitary mass.
since a clear surgical target was not identified. Finally, radia-
tion is generally reserved for patients with very aggressive
tumors or those that fail medical therapy.
40. ANSWER:B. t(15;17)

The age distribution of patients with APL differs from


38. ANSWER:C. Central diabetes insipidus other forms of AML. APL is uncommon in the first decade
of life; its incidence increases during the second decade to
The symptoms and presentation in this case are most con- reach a plateau during early adulthood, and then remains
cerning for the development of postoperative central diabe- constant until it decreases after age 60 years. The median
tes insipidus. Diabetes insipidus is a syndrome of hypotonic age is 40years old, thus considerably lower than for other
polyuria and is due to either a deficiency of arginine vaso- myeloid leukemias.
pressin, also known as antidiuretic hormone (ADH), or an Acute promyelocytic leukemia is a medical emer-
inadequate renal response to ADH. Patients with diabetes gency defined by the presence of a reciprocal translocation
insipidus often complain of extreme thirst and will excrete between the long arms of chromosomes 15 and 17, with the
large amounts of very dilute urine. Without this hormone, creation of a fusion gene, PML-RARA, which links the
individuals cannot adequately concentrate their urine retinoic acid receptor alpha (RARA) gene on chromosome
and, if not allowed access to liquids, may develop severe, 17 with the promyelocytic leukemia (PML) gene on chro-
life-threatening hypernatremia. Central diabetes insipidus mosome 15.
is due to deficiency, either partial or complete, of ADH. A key component of therapy is the use of all-trans reti-
Central diabetes insipidus can occur after any pituitary sur- noic acid (ATRA), which promotes the terminal differen-
gery. It can be transient or permanent and is seen more fre- tiation of malignant promyelocytes to mature neutrophils.
quently in patients with larger tumors or those that tend to t(11;14)Overexpression of cyclin D1 in mantle cell
invade the pituitary stalk such as craniopharyngiomas. The lymphomas is strongly associated with the t(11;14)(q13;q32),
diagnosis is not likely SIADH, as this condition is typically a translocation between the cyclin D1 locus and the immu-
associated with hyponatremia. Type 2 diabetes mellitus noglobulin heavy chain (IgH) locus. Apprximately 50% to
would also not be a likely cause of this patients acute severe 65% of MCLs will show the t(11;14) by cytogenetics, but
polyuria and polydipsia because his BMI is normal and by fluorescence in situ hybridization (FISH) a much higher
his fasting glucose is only mildly elevated. Given the clini- fraction of cases with cyclin D1 overexpression contain
cal presentation of a pituitary lesion, his diabetes insipidus BCL-1/IgH fusion genes.
would be much more likely be central than nephrogenic. t(9;22)The vast majority of patients (90%95%)
demonstrate the t(9;22)(q34;q11.2) reciprocal transloca-
tion that results in the Ph chromosome-BCR-ABL1 fusion
gene or its product, the BCR-ABL1 fusion mRNA. The
39. ANSWER:D. All of the above BCR-ABL1 fusion gene is the target of therapies such as
dasatanib, nilotinib, and imatinib.
This patient was found to have an incidental pituitary t(8;14)Burkitts Lymphoma is associated with a trans-
lesion. All three of the answer choices are important for the location between the long arm of chromosome 8, the site of

2 . H e m at o l o g y a n d O n c o l o g y 45
the c-MYC oncogene (8q24), and one of three locations on transfusion independence in 67% of transfusion-dependent
immunoglobulin (Ig) genes. The most common transloca- patients.
tion is with the the Ig heavy chain gene on chromosome 14
(approximately 80%).
t(14;18)Approximately 85% of patients with follicu-
lar lymphoma have t(14;18), which results in the overexpres- 42. ANSWER:D. Send peripheral blood for cytogenetics
sion of B cell leukemia/lymphoma 2 (BCL2), an oncogene to evaluate for the t(9;22) chromosome translocation.
that blocks programmed cell death (apoptosis), leading to
prolonged cell survival. CML accounts for approximately 15% to 20% of leuke-
mias in adults. It has an annual incidence of 1 to 2 cases
per 100,000, with a slight male predominance. The clnicial
presentation with splenomegaly, immature myeloid forms,
41. ANSWER:D. ESA and basophilia and eosinophilia is suggestive of CML or
another myeloproliferative disorder. The lack of JAK2 and
The International Prognostic Scoring System (IPSS) is the normal Hgb makes PV less likely. Approximately 15%30%
most widely used prognostic classification system for MDS. of CML patients have platelet counts above 600,000/L.
The Revised IPSS (IPSS-R) incorporates a larger number of The diagnosis of CML is confirmed by the demonstra-
cytogenetic abnormalities, divided into five prognostic cat- tion of the Philadelphia chromosome -t(9;22)(q34;q11.2),
egories, a lower cutoff for absolute neutrophil count (<800/ the BCR-ABL1 fusion gene or the BCR-ABL1 fusion
L versus <1,800/L), and different weights for the clinical mRNA by conventional cytogenetics, fluorescence in situ
parameters to better predict outcomes in newly diagnosed hybridization (FISH) analysis, or reverse transcription
patients. Patients with a low IPSS-R (1.5 to 3)have a median polymerase chain reaction (RT-PCR).
survival of 5.3years and risk to 25% AML transformation of Hydroxyurea can be used to reduce white blood cell
10.8years. The main goals of therapy are to control symptoms counts while awaiting confirmation of a suspected diagno-
due to cytopenias, improve quality of life, and minimize the sis of CML in a patient with significant leukocytosis (e.g.,
toxicity of therapy. There is no evidence that the treatment of >80 109 white cells/L). This patients WBC is only 60
asymptomatic patients improves long-term survival. and she is asymptomatic from her disease, so waiting for a
Immediate treatment is indicated for patients with diagnosis is appropriate.
symptomatic anemia or thrombocytopenia or recurrent This patient is afebrile and has no other symtoms of
infections in the setting of neutropenia. This patient is infection. The relative basophilia is also less consistent with
developing worsening dyspnea on exertion as a result of his an infectious cause; thus, starting empiric antibiotics for a
anemia, so therapy could be considered. presumed reactive leukocytosis is not needed.
Approximately 15%25% of unselected patients with While ET is still in the differential as 50% of patients
MDS have an improvement in hemoglobin when treated with ET do not have the JAK2 mutation, starting high-dose
with recombinant human erythropoietin. The Nordic aspirin in this patient is not recommended. Higher doses of
MDS group developed a predictive model for epoetin use aspirin (900 mg/day) have been associated with increased
in MDS:patients requiring 2 U PRBCs or more per month gastrointestinal hemorrhage as compared to low-dose aspi-
and those with serum epo level >100 (especially >500 U/L) rin (75325 mg/day).
are less likely to respond. Although morphologically normal, the neutrophils in
Allogeneic transplantation is the only curative treat- CML are cytochemically abnormal. The cytochemical reac-
ment for MDS. Transplantation at the time of diagnosis tion called leukocyte (or neutrophil) alkaline phosphatase
is shown to have the greatest benefit in patients with more (LAP) when scored is low. The low LAP score is useful in
advanced IPSS risk groups: intermediate-2 and high risk. excluding a reactive leukocytosis, typically due to infec-
Since this patient has low-risk disease, strategies to reduce tion, in which the score is typically elevated or normal,
the need for tansfusions is recommended with allogeneic or Polycythemia vera, which is also associated with an
transplantation at the time of disease progression. increased LAP activity. While the LAP score may be useful
As is the case with allogeneic HSCT, high-intensity che- supportive data, it is neither sensitive nor specific for CML.
motherapy regimens are generally reserved for patients with
intermediate-2 or high-risk IPSS scores. This allows for the
avoidance of treatment-related morbidity and mortality in
most patients with a relatively good prognosis. 43. ANSWER:D. Start leukapheresis while
Lenalidomide improves anemia in some patients with establishing diagnosis.
MDS and a normal marrow karyotype, but this agent is
particularly effective in MDS with an interstitial deletion This patient has a WBC >100,000 with anemia and throm-
of chromosome 5q. In studies, it has been associated with bocytopenia. She has a likely diagnosis of leukemia with

4 6 T h e B r i g h a m I n t e n s i v e R e v i e w o f I n t e r n a l M e d i c i n e Q u e s t i o n a n d A n s w e r C o m pa n i o n
an elevated WBC count leading to hyperleukocytosis and 45. ANSWER:C. While it may be worth trying
leukostasis. NovoSeven in this desperate situation, there is an
Hyperleukocytosis occurs in 10%20% of patients with increased risk of developing DVT and or PE after the
AML and more commonly in the monocytic and myelo- use of NovoSeven.
monoctyic variants. Leukostasis results from increased
blood viscosity as a direct complication of the presence of Although NovoSeven is widely used in trauma patients
large leukemic blasts and the interaction of the blasts with with normal hemostasis, the objective evidence that it helps
the endothelium to form aggregates leading to thrombi in to stop bleeding is lacking. There is also clear evidence that
microcirculation. Patients are usually symptomatic when it increases the risk of DVT/PE. EACA can be useful in
the WBC reaches levels of >100,000. If left untreated, the organs where there is intense fibrinolysis like the uterus
1-week mortality rate is approximately 20% to 40%. or the oral cavity. It is less effective when given to patients
In the setting of her WBC and CXR findings, it is likely who are bleeding at other sites. Trasylol, a broad-spectrum
that she has pulmonary evidence of leukostasis, present- protease inhibitor, increases the risk of DVT/PE and is
ing with dyspnea, tachypnea, and hypoxemia. Respiratory of marginal benefit in patients with normal hemostasis.
acidosis and cor pulmonale may also occur. CXR can The safest alternative here is to continue administering
show diffuse bilateral infiltrates. Approximately 80% of packed red cells, check PT and PTT, and give fresh frozen
patients with leukostasis are febrile, usually as a result of plasma if PT and PTT are prolonged. Once the source of
associated inflammation. While starting empiric antibi- intra-abdominal bleeding is discovered, the patient should
otics is reasonable, waiting to see if the patient improves stabilize.
would waste valuable time during which the patient can be
leukopheresed.
Transfusions would increase blood viscosity and
increase the risk of worsening the respiratory compromise 46. ANSWER:D. The patient is likely to have a lupus
and intracranial hemorrhage. anticoagulant and has no risk of bleeding but an
Leukapheresis should be initiated while establishing a increased risk of thrombosis.
diagnosis in patients who have symptomatic leukostasis.
It is a temporizing measure; thus, a diagnosis needs to be Given the large family with lots of male offspring, it is
promptly made and definitive therapy should be started. unlikely that she is a carrier of a mutant VIII or IX gene.
Therapy such as hydroxyurea reduces the WBC count by In addition, female carriers of mutant VIII or IX genes
50% to 80% within 24 to 48 hours and should be utilized usually have only a mildly prolonged PTT and a normal
as the sole therapy only in patients with hyperleukocytosis PT. She might have factor XI deficiency, although this
without symptoms of leukostasis. While induction che- seems unlikely as the PT is also prolonged. The same is
motherapy serves to both rapidly decrease the circulating true for possible factor XII deficiency. A lupus antico-
WBC count and target the leukemia cells in the bone mar- agulant is the only entity listed that could prolong the
row, reducing the WBC count within 24 hours of initia- PT and PTT and would not cause any bleeding. It may
tion, the patient is rapidly worsening, and it will take time confer an increased risk of postoperative thrombosis, but
to make the diagnosis. since she has been asymptomatic the risk is minimal. In
addition, the standard of care is to administer warfarin
for 3 months after knee surgery, so that will minimize
any risk of DVT/PE.
44. ANSWER:D. von Willebrand factor levels
become normal during pregnancy in patients with
Type Idisease.
47. ANSWER:B. Referral to medical oncologist for
Approximately 85% of patients with von Willebrand dis- consideration of chemotherapy
ease have Type I disease where there is a decrease in the
vWF level with no abnormality in vWF function. For rea- This patient has stage III colon cancer. While proper pathol-
sons that are not well understood, during pregnancy the ogy staging recommends sampling and analysis of at least 12
vWF levels in Type IvWD patients normalize. Thus, it is lymph nodes for ideal staging, it is not going to impact the
safe to use an epidural catheter for anesthesia. After deliv- patients current management to have more lymph nodes
ery of the baby and the placenta, the vWF level returns to sampled since the two positive nodes make the patient
the prepregnancy level. Thus, about 25% of women with stage II and guidelines recommend adjuvant chemotherapy.
Type I vWD require treatment with DDAVP 23 days Afollow-up colonoscopy in 1year would be recommended
after delivery but are not at risk for excess bleeding during as part of surveillance for this patient, though it is not the
labor and delivery. next step. Radiation is rarely utilized in colon cancer since

2 . H e m at o l o g y a n d O n c o l o g y 4 7
local recurrence rates are very low; radiation is considered compression is associated with renal, prostate, and most
for stage II and III rectal cancer. commonly, breast and lung cancers. The thoracic spine
is most often frequently affected, accounting for 70% of
patients with the condition. One should suspect an epidural
metastasis if the patient complains of new pain that wors-
48. ANSWER:E. Increase in dose of allopurinol ens when the patient is lying down or with palpation of ver-
tebral bodies, which is characteristic of this condition. Late
Tumor lysis syndrome (TLS) is acute cell lysis caused by neurologic signs such as incontinence and loss of sensory
chemotherapy and radiation therapy. The release of intra- function are associated with permanent paraplegia. MRI
cellular products (e.g., uric acid, phosphates, calcium, potas- has surpassed myelography as the imaging study of choice.
sium) overwhelms the bodys homeostasis mechanism. TLS If neurologic symptoms are present, the patient should be
is more common with hematologic malignancies or cancers treated with steroids. This treatment should not be delayed
with readily growing tumors, particularly acute leukemias while awaiting diagnostic study results. Use of high-dose
and high-grade lymphomas such as Burkitts lymphoma dexamethasone (up to 100 mg) is controversial; clinical tri-
(BL). BL patients are at high risk of TLS and uric acid als have shown that is has unclear benefits and significantly
nephropathy especially during chemotherapy. Prophylactic more serious side effects at higher doses. Most patients with
allopurinol and aggressive hydration with urine alkalini- epidural and spinal cord compression need radiation treat-
zation should be administered. TLS usually presents with ment (up to 3,000 Gy) or surgery.
AKI and metabolic derangements such as hyperphospha-
temia, hyperkalemia, and hyocalcimia. Treatment includes
inpatient monitoring, vigorous fluid resuscitation, allopu-
rinol or urate oxidase (uricase) therapy to lower uric acid 50. ANSWER:A. The most important step in
levels, urinary alkalinixation, and hemodialysis. Since the management of most patients with SVC syndrome is to
patient is already on allopurinol, it is unlikely that further establish a tissue diagnosis.
adjustment of allopurinol dosing will be effective at least
acutely. The SVC syndrome is caused by the gradual compression
of the superior vena cava, leading to edema and retrograde
flow. Lung cancer is the most common malignant cause,
although lymphoma, metastatic mediastinal tumors, and
49. ANSWER:B. Schedule MRI of spine as soon as indwelling catheters also can cause superior vena cava syn-
possible to evaluate for possible cord compression. drome. Symptoms may include cough, dyspnea, dysphagia,
and swelling or discoloration of the neck, face, or upper
This patient likely has epidural spinal cord compression extremities. Often, collateral venous circulation causes
caused by a tumor compressing the dural sac. This can cause distension of the superficial veins in the chest wall. Tissue
permanent neurologic impairment even if treatment is diagnosis (i.e., sputum cytology, thoracentesis, bronchos-
delayed for only a few hours and therefore requires atten- copy, or needle aspiration) often is necessary to direct treat-
tion and treatment with some urgency. Epidural spinal cord ment decisions.

4 8 T h e B r i g h a m I n t e n s i v e R e v i e w o f I n t e r n a l M e d i c i n e Q u e s t i o n a n d A n s w e r C o m pa n i o n
3.
R HEUM ATOLOGY

Derrick J. Todd and Jonathan S. Coblyn

1. A 76-year-old woman was admitted to the hospital and alkaline phosphatase is 250 U/L (normal 40130).
for fever, cough, dyspnea, and hypoxemia. She has a his- Transaminases, bilirubins, electrolytes, renal param-
tory of primary hyperparathyroidism and has refused eters, and serum protein electrophoresis (PEP) are all
surgical intervention. Chest radiograph showed a lobar normal. Urinalysis and urine PEP are likewise normal.
infiltrate, and sputum culture grew Streptococcus pneu- Cultures of urine and blood have revealed no growth
moniae. She was diagnosed with pneumococcal pneu- of microorganisms. The patient has no risk factors
monia and treated with levofloxacin based on antibiotic for tuberculosis infection, and PPD test was negative.
sensitivities. Her clinical status improved markedly. Serologic testing for syphilis, lyme, rheumatoid factor,
On hospital day 4, just prior to discharge, she develops antinuclear antibodies, and antineutrophil cytoplas-
rapid-onset painful swelling in the right wrist, associ- mic antibodies were all negative. Chest radiographs
ated with redness and warmth. She is afebrile, normo- and computed tomography (CT) scan of the abdomen/
tensive, and appears nontoxic. The right wrist is red, pelvis have not identified any lymphadenopathy, cul-
warm, swollen, and very painful to active or passive prit masses, hemorrhages, or abscesses. Endoscopy
range of motion. Several right metacarpal-phalangeal and colonoscopy were normal, as were abdominal and
(MCP) joints are similarly involved, but the left hand transvaginal ultrasound evaluations. Vascular ultraso-
and wrist are completely normal. Blood cultures drawn nographic imaging of the extremities has shown no evi-
on the day of admission (prior to antibiotics) have shown dence of thrombosis.
no growth. Plain radiograph of the right hand/wrist is Which of the following is the most appropriate next
as shown in Figure 3.1. step in trying to identify a source of the patients fever?
What is the most likely diagnosis?
A. Repeat blood cultures and treat with empiric
A. Streptococcus pneumoniae septic arthritis ceftriaxone.
B. Acute pseudogout arthritis B. Interferon-gamma release assay to assess for prior
C. Seronegative rheumatoid arthritis of the elderly exposure to Mycobacterium tuberculosis.
D. Wrist osteoarthritis C. Question family members out of concern for elder
E. Wrist fracture abuse.
D. Liver biopsy
2. A 92-year-old woman has been in the hospital for E. Temporal artery biopsy
3 weeks with unexplained fever, malaise, and weight
loss. She reports no other localizing symptoms. 3. A 37-year-old, previously healthy woman presents
Comprehensive physical examination (including pel- to the outpatient clinic with 6months of progressively
vic and breast evaluations) has failed to reveal any achy hands, wrists, elbows, knees, ankles, and feet.
abnormalities other than generalized muscle wasting Symptoms are worse in the morning and are alleviated
and mental lassitude. She has had a battery of labora- by physical activities. Functionally, she is finding it dif-
tory and imaging studies without a specific diagnosis. ficult to perform her duties as an orthopedic hand sur-
Complete blood count shows hemoglobin 9.1 g/dL geon because of reduced manual strength and dexterity.
(with normal red blood cell indices), platelets 525 Naproxen has provided minimal relief. She reports
103 cells per mm3, and normal white blood count with fatigue but no other associated symptoms. Physical
differential. ESR is 110 mm/hr. Albumin is 2.8 g/dL, examination reveals redness, warmth, and swelling of

49
right hip shows avascular necrosis (AVN) of the femoral
head, with trace effusion. There is no pathology of the
sacroiliac joints or psoas muscle.
All of the following are risk factors for avascular
necrosis of the hip except:

A. Antiphospholipid antibody syndrome


B. Osteoarthritis of the hip
C. Sickle cell disease
D. Alcoholism
E. Gauchers disease

5. A 27-year-old woman is diagnosed with systemic


lupus erythematosus (SLE) based on a history that
includes the following: 5 months of swollen joints in
the hands, photosensitive malar rash, documented peri-
Figure 3.1 Calcium pyrophosphate dihydrate (CPPD) and carditis, lymphopenia, autoimmune hemolytic anemia,
chondrocalcinosis of the hands/wrists for patient in Question 1.
and elevated antinuclear antibody (ANA) 1:1280 dif-
fuse pattern. Further serology demonstrates an elevated
multiple proximal interphalangeal (PIP) joints, meta- anti-Smith antibody but normal other serologies and
carpal phalangeal (MCP) joints, both wrists, both complements.
ankles, and both knees. Her examination is otherwise Which of the following therapies for SLE demands
normal. Laboratory analysis reveals normal compre- regular ophthalmologic screening for potential toxicity?
hensive metabolic profile and normal complete blood
count with differential. Rheumatoid factor (RF) and A. Mycophenolate mofetil
antinuclear antibodies (ANA) are not detectable. CRP B. Methotrexate
is 12.5 mg/L (normal <5). Plain film imaging of the C. Cyclophosphamide
hands reveals swelling of the soft tissues around the D. Hydroxychloroquine
affected joints but no other abnormalities. E. Azathioprine
Which of the following would be appropriate to
assess? 6. A 46-year-old man presents to the Emergency
Department for his fifth episode of hepatic encepha-
A. Repeat the serum ANA test. lopathy as a complication of cirrhotic end-stage liver
B. Count tender points because of a likely diagnosis of disease. He also suffers from volume overload and epi-
fibromyalgia syndrome. sodes of congestive heart failure, with an ejection frac-
C. Serum anti-cyclic citrullinated peptide (CCP) tion of 30%. His family is adamant that he has never
antibody level consumed alcohol excessively. Comorbidities include
D. Serologic testing for EpsteinBarr virus (EBV) poorly controlled diabetes mellitus on insulin therapy
E. Serum uric acid and osteoporosis. Aspiration of ascites does not sug-
gest spontaneous bacterial peritonitis. He is treated
4. A 39-year-old woman presents to her primary care with lactulose and loop diuretics, and his clinical status
provider with severe right hip pain. It has come on improves greatly. On hospital day 4, the patient stum-
steadily over the past 6months, in an unrelenting fash- bles and reports some pain in the right hand and wrist.
ion. She is now barely able to walk and uses a wheelchair It is not red, warm, or swollen, but chronic bony defor-
when in public. Physical examination reveals a woman mities are present in the distal interphalangeal (DIP),
of average stature in distress when trying to lay supine proximal interphalangeal (PIP), and metacarpal pha-
on the examining table, preferring to keep her right langeal (MCP) joints. Plain film imaging is as shown in
hip slightly flexed. She has good range of motion in the Figure 3.2.
left hip. However, the right hip has severe pain with any What is the most likely diagnosis?
degree of extension or internal rotation. She can pas-
sively externally rotate only 30 degrees. Hip flexion A. Rheumatoid arthritis
is preserved. Abdominal and knee exams are normal. B. Fifth metacarpal fracture
Plain film radiography shows complete collapse of the C. Hemachromatosis
right femoral head with severe bone-on-bone osteoar- D. Primary osteoarthritis of the hands
thritis (OA). Magnetic resonance imaging (MRI) of the E. Gauchers disease

5 0 T h e B r i g h a m I n t e n s i v e R e v i e w o f I n t e r n a l M e d i c i n e Q u e s t i o n a n d A n s w e r C o m pa n i o n
particularly painful, but the swelling has made it pro-
gressively difficult for her to get around. At first, she
cut back on mountain climbing and long-distance run-
ning. She then had to stop riding her bicycle to class.
Now she has difficulty with walking more than three
city blocks, and she avoids stairs whenever possible.
She has no previous medical history and no other joint
complaints. Comprehensive review of systems is oth-
erwise unrevealing. She is up to date with immuniza-
tions and routine health prevention and maintenance.
Physical examination is notable for a swollen, boggy
left knee with a massive effusion such that she cannot
fully straighten her knee and walks with a limp. There
are no other joint abnormalities. Neurologic examina-
tion is normal. Laboratory analysis reveals normal com-
Figure 3.2 Hemochromatosis arthropathy (right hand/wrist) for prehensive metabolic profile and complete blood count
patient in Question 6. with differential. Rheumatoid factor (RF) and anticy-
clic citrullinated peptide (CCP) antibody testing is neg-
7. A 49-year-old obese male laborer with a history of ative. ESR is 66mm/hr, and CRP is 44.8 mg/L (normal
prescription drug abuse presents to his primary care <5). Joint aspiration provides 80 mL nonbloody, slightly
provider with low back pain of 4months duration. He cloudy fluid with white blood count 8,400 cells/mm3
recalls no specific antecedent injury. Since onset, he has (80% lymphocytes). Fluid is negative for crystal analy-
had some degree of constant low back pain, punctu- sis, gram stain, and culture of infectious organisms.
ated by exacerbations of pain when he twists or turns What is the most appropriate next step diagnostic
awkwardly. Two weeks ago, he was instructed to take workup or management?
some time off from work, and he has been mostly lying
around the house hoping that his back will improve. A. Serologic testing for Lyme disease
Over the past 4 months, he has been taking escalat- B. MRI of the left knee
ing doses of oxycodone (now 15 mg three times daily), C. Antinuclear antibody (ANA) testing
obtained from friends or prescribed by physicians at D. Corticosteroid injection of the left knee
nine visits to six different emergency rooms over the E. Referral to Orthopedic Surgery for arthroscopic
past 3months. Workup available from those evaluations evaluation of the left knee
includes normal blood work and normal lumbar spine
films. He reports no fever, occult weight loss, bladder or 9. A39-year-old male kindergarten teacher presents to
bowel disturbances, or intravenous drug use. Physical the emergency department (ED) with severe joint pain
examination demonstrates no abnormalities except that all over. He was in his usual state of good health until
he has reduced spine flexion and extension because of 3days prior, when he started to feel stiff in the hands
pain. He reports that the oxycodone is not cutting it. and wrists. Subsequently, he has had progressive pain,
What is the most appropriate next course of action? redness, and swelling in the hands, wrists, and elbows.
He has difficulty moving his shoulders because of pain,
A. Reassurance that this will improve and his fianc sent him to the ED because he could not
B. Provide a prescription for physical therapy and get out of bed easily in the morning. He reports no
discuss an opiate medication contract. other symptoms. He carries sickle-cell trait but has an
C. Provide a prescription for hydromorphone 2 mg otherwise unremarkable past medical and family his-
three times daily, with reassurance that this will tory. Physical examination is notable for a generally
improve. well-appearing, athletic-appearing man in significant
D. MRI of the lumbar spine with referral to distress during the physical examination because of
Interventional Radiology for any bulging or severe pain in the arms and legs. He has such redness and
herniated disc puffiness in the hands that the assessment of his joints is
E. Discharge the patient from your practice for abuse of difficult. His elbows do not extend fully because of joint
opiate-based pain medications. pain, and he cannot lift his arms or legs off the exami-
nation table because of pain in the shoulders and hips,
8. A 28-year-old Harvard University geology gradu- respectively. His knees are tender and slightly swollen.
ate student presents to Student Health Services with Ankles, feet, and toes have puffy tenderness and are dif-
8 weeks of swelling in her left knee. Her knee is not ficult to examine further because of pain. He is afebrile

3 . R h e u m at o l o g y 51
and normotensive. Heart rate is 110 beats per minute evaluation demonstrates an accentuated paradoxically
but comes down to the 90s with narcotic medications split P2. Six-minute walk test reveals 98% oxygen sat-
for pain control. Remainder of examination is nor- uration at rest, which drops to 92% with ambulation.
mal. Laboratory analysis reveals hemoglobin 9.4 g/dL, Chest radiograph is normal.
with mean corpuscular volume 71 femtoliters (normal Which of the following most likely explains the
8197), and reticulocyte count 0.1%. Platelet count and patients dyspnea?
white blood count with differential are normal, as is the
comprehensive metabolic profile. ESR is 49 and CRP A. Physical deconditioning
is 251 mg/L (normal <5). Radiographic imaging of the B. Scleroderma cardiomyopathy with congestive heart
hands and feet demonstrates soft-tissue swelling but no failure
other abnormalities. C. Pulmonary hypertension
What is the most likely diagnosis? D. Occult aspiration from undertreated GERD
E. Pulmonary embolus
A. Septic polyarthritis
B. Viral arthritis 12. A47-year-old man with a history of rheumatoid arthri-
C. Fibromyalgia syndrome tis (RA) is admitted to the hospital with fever, dyspnea,
D. Lyme disease and chest infiltrate. His RA treatment of methotrexate
E. Rheumatoid arthritis and etanercept (TNF inhibitor) is held. The diagnosis
upon admission is presumed community-acquired bac-
10. A68-year-old woman presents to her primary care terial pneumonia, and he is treated empirically with a
provider because of fatigue, malaise, weight loss, and combination of ceftriaxone and azithromycin. However,
diffuse achiness. Symptoms started insidiously about after 3 days his status has deteriorated such that he is
4 months ago. She has no other localizing symptoms. placed on a mechanical ventilator for hypoxemia. Chest
Her weight has dropped by 12 pounds since her last visit CT scan reveals necrotic-appearing pulmonary nodules.
for annual examination, 9months previously. Physical Which of the following is the most appropriate next
examination reveals a frail, fatigued-appearing woman course of action?
with bitemporal wasting. Her gait is slow and deliber-
ate. She has reduced muscle bulk and strength testing A. Intravenous methylprednisolone 100 mg three times
in the proximal arms and legs, mostly due to achiness. daily for treatment of rheumatoid lung
Examination is otherwise unrevealing. Laboratory B. Bronchoscopy with lavage for microscopic organisms
analysis shows normal comprehensive metabolic pro- C. Open lung biopsy
file, white blood count, and platelet count. Hgb is 10.0 D. PET-CT scan to identify source of primary tumor in
g/dL with normal mean corpuscular volume. ESR is the setting of pulmonary metastases
103mm/hr, and CRP is 2.4 mg/L (normal <5). E. Switch antibiotic therapy to levofloxacin 750 mg
Which of the following is the most appropriate next daily
course of action?
13. A 44-year-old male landscaper presents to his pri-
A. Antinuclear antibody testing mary care provider because of knee complaints. He has
B. Serum and urine protein electrophoresis had pain, swelling, and limited motion in the right knee
C. Temporal artery biopsy for 5months. His only other musculoskeletal complaint
D. CT scan of the chest, abdomen, and pelvis is nontraumatic swelling of the left fourth finger such
E. Prednisone 15 mg daily that his wedding ring had to be cut off. He is otherwise
healthy, except for excessive beer consumption (12 or
11. A53-year-old man with long-standing scleroderma more per day). Physical examination reveals a massively
presents with worsening dyspnea on exertion. Over the swollen right knee with near-full extension but only 60
past 2years, he has noticed increasing difficulty climb- degrees of flexion. His left fourth finger is also swollen
ing stairs and keeping up with his spouse when walk- and tender, with minimal flexion. Remainder of the joint
ing. His scleroderma predominantly affects the hands examination is unrevealing. Laboratory analysis is essen-
as sclerodactyly, and it is associated with severe gas- tially normal or negative:comprehensive metabolic pro-
troesophageal reflux disease (GERD) and symptom- file, complete blood count and differential, CRP, ESR,
atic Raynauds phenomenon. Vital signs are normal. Lyme serology, antinuclear antibody, rheumatoid factor,
Physical examination reveals sclerodactyly with acro- and anti-CCP antibody. Arthrocentesis yielded 90 mL
cyanosis and tightness of the skin on the face. He has nonbloody, cloudy, yellow fluid with 65,000 white blood
multiple matte telangiectasia on the skin and muco- cells (60% neutrophils). Fluid analysis is negative for
sal surfaces. Lungs are clear to auscultation. Cardiac polarizable crystals and microbiologic organisms.

52 T h e B r i g h a m I n t e n s i v e R e v i e w o f I n t e r n a l M e d i c i n e Q u e s t i o n a n d A n s w e r C o m pa n i o n
Which of the following would be diagnostic in this
case?

A. Detectable anti-Smith antibody


B. Aserum uric acid of 11.5 mg/dL
C. Elevated serum angiotensin converting enzyme
(ACE) level
D. Scaly erythematous plaques on the extensor surfaces
of the arms and legs
E. Presence of chondrocalcinosis on plain film
radiography of the right knee

14. A21-year-old woman visits her primary care provider


because of diffuse joint pains for the past 18 months.
They involve her fingers, wrists, elbows, shoulders, hips, Figure3.3 Dermatomyositis rash.
knees, ankles, and spine. She has noticed no redness,
warmth, or swelling of the joints, nor any muscle pain.
She reports that her shoulders and hips feel like they are for reduced strength. She is able to get her thighs off
popping out of joint with physical fitness, including the examining table against gravity, but not against
during her activities as a semiprofessional ballerina. mild resistance testing. She has similar weakness in her
She reports palpitations and dizziness when sitting shoulders. Grip strength is preserved. Lungs have some
upright quickly and states that she has almost passed mild wheezes at the apex but also bibasilar Velcro-type
out on several occasions. Comprehensive review of sys- rales. Laboratory testing reveals elevated CPK 10,200
tems is otherwise unremarkable. Physical examination U/L, CK-MB 340 ng/ml, AST 348 U/L, and ALT
is notable for a thin woman in no acute distress. She has 419 U/L. Albumin is 3.1 g/dL. She has otherwise nor-
normal seated heart rate and blood pressure. Arms and mal comprehensive megabolic profile, complete blood
legs are well proportioned without deformity, and she count with differential, and coagulation studies.
has no red, warm, swollen joints. She has more than 90 Electrocardiogram (EKG) does not demonstrate any
degrees of passive extension at the wrists and metacar- abnormalities. Chest radiograph demonstrates fibrotic
pal phalangeal (MCP) joints. Elbow and knee joints do changes at the bibasilar lung fields.
not have effusions, and she can extend them 190 degrees Which of the following tests is most likely to be diag-
voluntarily. Spine motion is excellent, as she is able to nostic for this condition?
place her palms flat on the floor without bending her
knees. Ocular, skin, cardiopulmonary, and neurologic A. Liver biopsy
examinations are normal. Recent echocardiography for B. Temporal artery biopsy
the palpitations revealed no abnormities. C. Cardiac MRI
Which of the following studies is most likely to be D. Cardiac catheterization
positive in this patient? E. Skeletal muscle biopsy

A. Tilt-table test 16. A 25-year-old man presents to the emergency


B. MRI of the left shoulder department for an episode of acute right lower quadrant
C. CT scan of the right hip abdominal pain, which started abruptly 3days prior. He
D. Serum RF and anti-CCP antibodies has had several episodes similar to this in the past, dat-
E. Lyme serology ing back to a bout of appendicitis at age 4.Associated
symptoms include fever, nausea, and anorexia. Medical
15. A 68-year-old woman presents to her primary history also includes presumed gout (lacking a
care provider because of weakness of the shoulders crystal-proven diagnosis). Past surgical history includes
and hips. She has noticed difficulty climbing stairs four separate abdominal surgeries:appendectomy, cho-
and trouble getting out of a chair for 5 weeks. She has lecystectomy, and two exploratory laparoscopies. There
pre-existing chronic dyspnea from various comorbidi- is no family history of any similar condition. Physical
ties that include coronary artery disease, hypertension, examination reveals a thin, toxic-appearing gentleman
atrial fibrillation, type 2 diabetes mellitus, and chronic who is unwilling to maneuver in bed because of abdomi-
tobacco usage complicated by emphysema. Her only nal pain. Vital signs include temperature 39.2C, blood
other complaint is a new-onset rash on her face and pressure 92/58mm Hg, heart rate 125 beats per minute,
hands (see Figure 3.3). Physical examination is notable respiratory rate 24 breaths per minute, and 99% oxygen

3 . R h e u m at o l o g y 53
saturation on room air. The abdomen is slightly dis- 18. A21-year-old female college student presents to the
tended and exquisitely tender to gentle palpation. Bowel emergency department with new-onset fever, headache,
sounds are absent. Rectal examination reveals normal dyspnea, and weight gain. She was in her usual state of
tone but no blood. Remainder of examination is unre- excellent health until 6 weeks prior, when her current
vealing. Laboratory analysis reveals normal compre- symptoms started. She has noticed puffiness around
hensive metabolic profile except for albumin 2.3 g/dL. her eyes and swelling in her ankles. Associated symp-
Creatinine is 1.0 mg/dL. Uric acid is 4.2 mg/dL. toms include cold fingers and toes with blue discolor-
Complete blood count is notable for white blood count ation. She takes no medications regularly. Vital signs
21.2 103 cells/mm3 (72% neutrophils and 13% bands), were notable for temperature 37.9C, blood pressure
hemoglobin 10.4 g/dL (MCV 88 fl), and platelets 630 192/110mm Hg, heart rate 108 beats per minute, respi-
103 cells/mm3. CRP is 310 mg/L, and ESR is 107mm/hr. ratory rate 22 breaths per minute, and oxygen saturation
Urinalysis is notable only for 4+ protein, with a spot 92% on room air. She is ill appearing with periorbital
protein/creatinine ratio of 2.1. CT scan of the abdomen and lower-extremity edema. Other notable features of
and pelvis demonstrates some free peritoneal fluid, but the physical examination include lungs with bibasi-
no free air or other evidence of bowel rupture. lar wet rales and acrocyanosis with digital ulcerations.
Which of the following studies is most likely to pro- There is no sclerodactyly. Laboratory analysis includes
vide the correct diagnosis? BUN 82 mg/dL, creatinine 5.2 mg/dL, albumin 1.9
g/dL, Hgb 8.4 g/dL with MCV 82 fl, platelet count 74
A. MEFV genetic testing 103 cells/mm3, and white blood count 2.5 103 cells
B. Endoscopy with biopsy of the duodenum to look for per mm3 with 80% neutrophils, 10% lymphocytes, 6%
flattened villi and crypts monocytes, and 3% eosinophils. Urinalysis demon-
C. Traditional angiography of the mesenteric strates 50100 RBCs with casts, +++ protein, and no
vasculature leukocytes. ESR is 110mm/hr, CRP is 12.9 mg/L (nor-
D. Colonoscopy with biopsy of the terminal ileum mal <5), antinuclear antibody is 1:5120 speckled pat-
E. Serum antineutrophil cytoplasmic antibody tern. Blood cultures are normal. EKG is normal. Chest
(ANCA) testing radiograph demonstrates fluffy vascular congestion and
trace bibasilar effusions.
17. A 22-year-old female college student presents to Which of the following additional test results would
University Health Services because of bilateral knee be most consistent with her presentation?
pain. She is an avid runner, but she has had to curtail
her normal regimen of 30 miles per week because of this A. p-ANCA positive, antimyeloperoxidase (MPO)
knee pain. It has come on rather severely over the past antibody negative, and antiproteinase-3 (PR3)
3 weeks, and localizes mostly at the anterior knee. She antibody negative
denies any knee swelling or lateral thigh discomfort. She B. Uric acid 13.5 mg/dL and bone marrow biopsy with
has not had morning stiffness or other joint pains. Sheis 75% clonal pro-myelocytes
tearful because of the severity of the knee pain and the C. Anti-double-stranded DNA antibody 522 U/mL
effect it has had on her exercise. She is otherwise healthy. (normal <5), C3 complement 52 mg/dL (normal
Physical examination reveals a thin, athletic-appearing 75175), C4 complement 8 mg/dL (normal 1440)
young woman. She has excellent range of motion in D. CPK 8600 U/mL, anti-Jo1 antibody 82 U/mL
all joints except for the knees, where she has pain with (normal <5)
flexion of the knee beyond 110 degrees. She can extend E. HLA-B27 positive, sacroiliac joint fusion on plain
fully. There are no knee effusions. She is tender along the radiography, and renal biopsy showing changes most
patellar margins, especially if the patella is compressed consistent with IgA nephropathy
downward during active knee extension. There is no
tenderness along the lateral knee on either side, nor any 19. A58-year-old man presents to the outpatient clinic
posterior tenderness. She has negative McMurray sign with 3days of an excruciatingly painful, red, warm, swol-
and no instability on varus/valgus stress maneuvers or len left great toe. He has had to use crutches and been
anterior/posterior drawer sign testing. unable to sleep because of the severity of pain. He has had
Which of the following is the most likely diagnosis? episodes similar to this in the past, and on one occasion
had an arthrocentesis that provided a crystal-proven
A. Patellofemoral syndrome diagnosis of gout. Relevant comorbidities include Stage
B. Iliotibial band syndrome IV chronic kidney disease (glomerular filtration rate
C. Tear of the medial meniscal 19 mL/min) as a complication of hypertension. He has
D. Tear of the anterior cruciate ligament (ACL) never had nephrolithiasis. Physical examination reveals
E. Popliteal (Bakers) cyst a red, warm, very tender left first metatarsal phalangeal

5 4 T h e B r i g h a m I n t e n s i v e R e v i e w o f I n t e r n a l M e d i c i n e Q u e s t i o n a n d A n s w e r C o m pa n i o n
joint. There are no other active joints. The bilateral olec- should remain supine as much as possible for 48
ranon bursae have large tophi. hours and touch base on Monday morning if his
What is the most appropriate next course of action? symptoms persist.
B. Arrange for an expedited temporal artery biopsy first
A. Check a serum uric acid (sUA) level and initiate thing Monday morning. In the meantime, inform
allopurinol if the sUA is >6 mg/dL. the patient to call back in the event of any vision loss.
B. Check a 24-hour urine uric acid, and initiate C. Prescribe prednisone 1 mg/kg. Instruct the patient
probenecid 500 mg twice daily if the patient is an to start it immediately and to call with any new
under-excretor. visual disturbances. Arrange for an expedited
C. Treat with colchicine 0.6 mg tablets:2 tablets to temporal artery biopsy first thing Monday morning.
start and 12 tablets per day for the next 5days. D. Inform the patient that he likely has viral meningitis
D. Treat with naproxen 500 mg twice daily, with food. and should return to the ED for treatment.
E. Treat with intramuscular methylprednisolone 80 E. Inform the patient that he may have a cerebral
mg 1. aneurism and refer him back the ED for magnetic
resonance angiography of the brain.
20. A50-year-old man presents to his primary care pro-
vider because of right shoulder pain of 10days duration. 22. A66-year-old obese woman with a history of chronic
He was previously healthy, recalls no antecedent injury back pain presents for an initial evaluation of left lat-
or trauma, and has never experienced shoulder pain eral hip discomfort of 5 weeks duration. She reports no
of this severity before. On physical examination, the antecedent injury, but it hurts to sleep on that side at
shoulder is not visibly red, warm, or swollen. He expe- night. Symptoms do not radiate to the groin. On physi-
riences minimal discomfort on active abduction and cal examination, she has an antalgic gait but demon-
forward flexion (against resistance) until he reaches 90 strates good active and passive range of motion in the
degrees horizontal, at which point pain limits further left hip. Passive adduction reproduces her lateral hip
motion. Passive range of motion testing achieves an pain, as does palpation over the lateral hip.
additional 30 degrees in these planes. He does not tol- Which of the following is an appropriate next step in
erate resisted internal or external rotation, nor resisted diagnosis or management of this condition?
abduction with his arm held slightly in front of his body
in internal rotation. A. Reassurance and physical therapy, focusing on
What is the most likely diagnosis? posture, core muscle tone, and hip range of motion
B. MRI of the left hip
A. Biceps tendonitis C. Methylprednisolone taper from 24 mg down to 0
B. Septic arthritis over 6days (dropping the dose by 4 mg each day)
C. Acute calcific tendonitis D. Whole-body bone scan
D. Glenohumeral osteoarthritis E. Bone mineral densitometry
E. Complete tear of the supraspinatus
23. A 59-year-old woman with long-standing destruc-
21. An 84-year-old generally healthy man calls his pri- tive rheumatoid arthritis (RA) has recently moved to
mary care provider (PCP) on a Saturday afternoon to the area and is scheduled to undergo revision total hip
report 3 weeks of unilateral headache, scalp tenderness, arthroplasty because of refractory pain and functional
and achiness in the jaw after chewing meat. He has had limitations. She is seeing you for preoperative clearance
no visual disturbances and denies any head trauma. Two evaluation. Her primary complaint is gait instability,
nights prior, he went to a local emergency department urinary incontinence, and paresthesias in the bilateral
(ED) for these same symptoms, where he was diagnosed arms. Prior records are not immediately available, but
with a new migraine headache, given acetaminophen you know that she has already had multiple previous
1000 mg three times daily, and instructed to call his orthopedic procedures, including bilateral hip and
PCP in the event that his symptoms failed to improve. knee arthroplasties, bilateral wrist fusion surgeries,
Laboratory testing from the ED revealed ESR 110mm/ and a left elbow arthroplasty. For her RA, she is taking
hr, normal cerebrospinal fluid analysis, and a head CT etanercept and methotrexate, which will be stopped by
scan that did not show any evidence of intracerebral her rheumatologist 3 weeks prior to surgery. She has no
hemorrhage. history of heart disease or identifiable risk factors other
What is the most appropriate next step in management? than the RA. Physical examination reveals general mus-
cle atrophy and multiple joint deformities. There are no
A. Reassure the patient that his symptoms are a red, warm, or swollen joints. Neurologic testing reveals
common complication of lumbar puncture. He hyperreflexivity of the bilateral upper extremities and

3 . R h e u m at o l o g y 55
positive Hoffman sign. Gait cannot be tested because of A. Antinuclear antibody
instability. B. Hepatitis C antibody
What is the most important preoperative testing to C. Anti-CCP antibody
be done in this patient? D. Monospot
E. Lyme antibody
A. Rheumatoid factor (RF) and anticyclic citrullinated
(CCP) antibody testing 25. A68-year-old man is seen by his primary care pro-
B. ESR and CRP testing vider for osteoarthritic right knee pain of 12years dura-
C. Cardiac exercise stress testing with tion. He initially had pain only when running, but this
sestamibi-enhanced nuclear imaging progressed to pain on ambulation. Currently, he reports
D. Lateral cervical spine X-ray imaging, with flexion pain when walking more than two city blocks, standing
and extension views for more than 20 minutes, and nocturnal knee pain. He
E. Lumbar spine magnetic resonance imaging (MRI) reports no knee swelling, locking, or instability. When
out with friends, he cannot keep up with them and fre-
24. A 48-year-old man with a history of alcoholism and quently has to sit because of his knee pain. Physical
cirrhosis presents to urgent care because he has been therapy and naproxen 500 mg twice daily have not ade-
tripping over his own feet. He describes an inability to quately relieved his pain. He has derived only 2 weeks
get his toes off the ground when walking. He has not of relief from three prior corticosteroid injections into
had any change in his baseline cirrhosis, and mental the right knee. Comorbidities include type 2 diabetes
status is lucid. He describes general arthralgias with mellitus, cerebrovascular disease with mild carotid ste-
some morning stiffness. Physical examination reveals nosis, hypertension, hyperlipidemia, and obesity (body
stigmata of cirrhosis with scleral icterus, jaundice, and mass index 34.2 kg/mm2). On physical examination,
abdominal ascites. Gait is altered by left-leg toe drag he has good range of motion in the bilateral hips, left
that occurs throughout stride, and he lacks strength in knee, and bilateral ankles. Both knees have hypertro-
the tibialis anterior muscle. Lower-extremity reflexes phic osteoarthritic changes, crepitus, mild effusions,
are otherwise intact. He has a painless rash on the legs full extension, and only 90 degrees of flexion. He has
as shown in Figure 3.4. He has diffuse tenderness of genu valgus deformity of both knees, and no insta-
muscles and joints, but no red, warm, or swollen joints. bility on varus/valgus stress maneuvers or anterior/
Laboratory testing reveals the following: serum sodium posterior drawer sign testing. There is minimal tender-
129 mEq/L, serum creatinine 1.4 mg/dL, normal com- ness on the left but diffuse joint-line tenderness on the
plete blood count with differential, CRP 88.1 mg/L, right. Previous right knee aspiration has yielded 25 mL
ESR 99 mm/hr, rheumatoid factor 110 U/mL (normal fluid with 288 white blood cells. Plain film radiography
<15), C3 complement 112 mg/dL (normal 75175) and shows tricompartmental osteoarthritic change of both
C4 complement 5 mg/dL (normal 1440). Urinalysis knees with significant joint space narrowing medially.
has 100200 RBC per high-power field, with some dys- What is the most appropriate next course of action?
morphia. There is +++ protein.
Which of the following serologic tests, if positive, A. Inject the right knee with triamcinolone 40 mg,
could explain the above clinical scenario? prescribe physical therapy, and advise weight loss.
B. Switch naproxen to oxycodone 5 mg three times
daily. Reassess in 1year.
C. MRI of the right knee
D. Referral to Orthopedics for arthroscopic
debridement of loose cartilage.
E. Referral to Orthopedics for consideration of right
total knee arthroplasty.

26. A28-year-old woman presents to her primary care


provider with numbness and paresthesias in the first
three fingers of both hands. Symptoms are worst when
she first awakens in the morning, and they impact her
profession as a hair stylist. Physical examination reveals
no redness, warmth, or swelling of the fingers, thumbs,
or wrists. She has preserved thenar muscle strength and
no atrophy. Forced flexion of the wrist reproduces her
Figure3.4 Lower-extremity petechiae. symptoms after 30 seconds. There is no swelling of the

5 6 T h e B r i g h a m I n t e n s i v e R e v i e w o f I n t e r n a l M e d i c i n e Q u e s t i o n a n d A n s w e r C o m pa n i o n
wrist, and range of motion is intact. She is diagnosed Laboratory analysis is normal: comprehensive meta-
with carpal tunnel syndrome (CTS). bolic profile, complete blood count and differential,
All of the following are associated with CTS except: thyroid-stimulating hormone (TSH), creatine kinase,
and C reactive protein.
A. Acromegaly What is the next most appropriate step in diagnostic
B. Scleroderma testing or management?
C. Chondrocalcinosis of the wrist
D. Rheumatoid arthritis A. Electromyogram (EMG) with nerve conduction
E. Hypothyroidism studies (NCS) of all four extremities
B. Test antinuclear antibody (ANA) and rheumatoid
27. A51-year-old morbidly obese gentleman presents to factor (RF)
his primary care provider with paresthesias and painless C. Serology testing for Lyme disease
numbness of the anterior left thigh. Symptoms are best D. Recommend combination of physical therapy, sleep
when he first awakens in the morning and progress over routine, weight loss, and psychiatric counseling
the course of a day. He has noticed this coming on over E. Initiate oxycodone 5 mg twice daily, with dose
the previous 3 weeks. Comorbidities include diabetes escalation as needed
mellitus. Physical examination reveals clearly demar-
cated absence of sensation overlying the anterior thigh. 29. A 24-year-old woman presents to her primary
There are no motor deficits. Lower-extremity reflexes care provider with discomfort in her fingers and toes.
are intact. Hip range of motion is normal. Symptoms have been present for 4 years and are dis-
Which of the following explains the most likely tinctly worse in the winter. Cold exposure causes white
diagnosis? and blue color changes, leading to numbness and stiff-
ness in the involved digits. Symptoms are alleviated by
A. Diabetic amyotrophy rewarming the fingers, but they are accompanied by
B. Avascular necrosis of the left femoral head fiery red erythema and discomfort. Medical history is
C. Entrapment neuropathy of the left lateral femoral notable only for attention-deficit/hyperactivity dis-
cutaneous nerve order, treated with dextroamphetamine. On physical
D. Irritation of the gluteus medius and gluteus minimus examination, there is no erythema, warmth, or swell-
muscles at their tendon insertion on the left greater ing in the fingers or wrists. There is no sclerodactyly or
trochanter periungual erythema. Digital nailfold capillaroscopy
E. Degenerative arthritis of the left hip reveals mild tortuosity of capillaries without micro-
aneurisms, hemorrhage, or dropout. Radial and ulnar
28. A49-year-old woman presents to a new primary care pulses are brisk, and Allens test is normal bilaterally.
provider with complaints of diffuse joint and muscle Which of the following is the most appropriate next
pains that have been present for over 20years. The pain step in diagnostic workup?
has escalated over the past 18months to the point where
she has stopped working as a cashier and is applying A. Test for antinuclear antibodies, anti-dsDNA,
for disability. She is no longer able to derive enjoyment anti-Ro, anti-La, anti-Smith, and anti-RNP
from her young grandchildren because she is always in antibodies.
pain. When asked about red, warm, or swollen joints, B. Test for antinuclear antibodies, anti-SCL70 and
she reports a severe burning pain in her arms, legs, and anticentromere antibodies.
back. She has no significant comorbidities other than C. Test for antinuclear antibodies, antiphospholipid
obesity (body mass index = 41.1 kg/m2) and chronic antibodies, and lupus anticoagulant.
back pain. Review of systems is notable for fatigue, D. Magnetic resonance angiography of the bilateral
weight gain, and poor sleep. She does not smoke or upper extremities
drink excessive alcohol, but she describes a lot of stress E. No additional workup necessary
related to the loss of her job and prolonged divorce pro-
ceedings over the past 2years. Physical examination is 30. A 46-year-old woman with systemic sclerosis
notable for an obese woman who is tearful throughout (scleroderma) presents to the emergency department
the musculoskeletal exam. She has diffuse tenderness because of headache, dyspnea, and lower-extremity
in the arms, legs, and thorax. There are no obviously edema. She has had recent complications of her sclero-
warm, swollen, or tender joints, but the assessment derma, including Raynauds crisis with gangrene of
is partly limited by obesity and her distress even with the left fourth finger, interstitial pulmonary fibrosis,
light touch. Neurologic examination reveals reduced and pericardial effusion. Vital signs include tempera-
effort at strength testing but normal when distracted. ture 36.8C, blood pressure 140/95 mm Hg in both

3 . R h e u m at o l o g y 57
arms, heart rate 72 beats per minute, respiratory rate Which of the following is the most likely diagnosis?
24 breaths per minute, and oxygen saturation 94% on
room air. Physical examination reveals an ill-appearing A. Gouty bursitis of the right prepatellar bursa
individual with scleroderma involving the hands, arms, B. Gouty arthritis of the right knee
face, trunk, legs, and feet. She has scattered cutane- C. Septic arthritis of the right knee
ous telangiectasia. She has dry gangrenous changes of D. Right knee meniscal injury from unintended
the left fourth finger distal to the distal interphalan- intraoperative injury
geal joint. Lungs have bibasilar Velcro rales. Cardiac E. Quadriceps tendon rupture
examination is unrevealing. Laboratory analysis reveals
BUN 40 mg/dL, creatinine 3.2 mg/dL, white blood 33. A 28-year-old woman presents to the emergency
count 7.2 103 cells per mm3, hemoglobin 8.9 g/dL, department because of severe pain and swelling in mul-
and platelet count 85 103 cells per mm3. Blood smear tiple joints. Two weeks ago, she was admitted with severe
demonstrates schistocytes. Protime and partial throm- gastrointestinal dysentery, from which she has recov-
boplastin time are normal. Liver function studies are ered. Ibuprofen has offered little relief. Physical exami-
normal. Urinalysis shows ++++ protein and >100 RBC nation reveals overt erythema, warmth, and swelling of
per high-power field. Chest radiograph is consistent the fingers, hands, wrists, and elbows. She has a swollen
with pulmonary edema. There is no cardiomegaly. EKG left knee. Laboratory testing reveals an ESR 88mm/hr
is unrevealing. and CRP 182 mg/L (normal <5).
Which of the following must be included in the next All of the following microorganisms are associated
steps of managing this patient? with reactive arthritis except:

A. Plasma exchange therapy A. Staphylacoccus aureaus


B. Captopril 25 mg orally three times daily, with dose B. Streptococcus pyogenes
escalation to control blood pressure C. Chlamydia trachomatis
C. Labetalol 100 mg orally twice daily, with dose D. Yersinia pestis
escalation to control blood pressure E. Salmonella enteritidis
D. Amlodipine 5 mg orally once daily, with dose
escalation to control blood pressure 34. A52-year-old obese woman presents with bilateral
E. Methylprednisolone 250 mg intravenously every 12 foot pain of 3 weeks duration. Symptoms localize to
hours the mid-foot and are worse first thing in the morning
when she steps out of bed, lasting for at least 2 hours.
31. A 68-year-old diabetic man presents to the emer- She has not noticed any redness, warmth, or swelling of
gency department because of right ankle pain and the feet, nor any other joints. Other medical conditions
altered gait. On physical examination, the ankle is include osteoporosis secondary to surgical menopause
swollen posteriorly but has full passive range of motion. at age 31. Physical examination demonstrates no red-
Squeezing the calf does not result in dorsiflexion of the ness, warmth, or swelling of the feet and no tenderness
ankle. Two weeks ago he had been admitted to the hos- on squeeze of the metatarsal phalangeal joints. She has
pital with pneumococcal pneumonia. focal tenderness to palpation at the plantar aspect of the
What was the most likely agent used to treat his calcaneus bones. Plain film radiography of the feet is
pneumonia? normal.
Which of the following is the most likely diagnosis?
A. Imipenem
B. Trimethoprim/sulfamethoxazole A. Occult stress fracture of the fifth metatarsal bone
C. Cefpodoxime B. Plantar fasciitis
D. Levofloxacin C. Acute gouty arthritis of the talonavicular joint
E. Azathioprine D. Charcot arthropathy
E. Occult injury of the anterior talofibular ligament
32. A77-year-old gentleman is recovering in the surgi-
cal intensive care unit from coronary artery bypass graft 35. A28-year-old woman has rheumatoid arthritis (RA)
surgery. On postoperative day 3, he develops severe that is well controlled on a combination of methotrex-
right anterior knee pain with focal erythema, warmth, ate and adalimumab (anti-TNF monoclonal anti-
and swelling. On physical examination, he keeps his body). She will be travelling to the Amazon rainforest
right leg fully extended and yells in pain if the knee is as part of a graduate school degree in anthropology. She
flexed actively or passively. The knee does not demon- has never been agreeable to immunizations in the past,
strate any fluid to aspirate. but she is requesting whatever the travel clinic advises.

5 8 T h e B r i g h a m I n t e n s i v e R e v i e w o f I n t e r n a l M e d i c i n e Q u e s t i o n a n d A n s w e r C o m pa n i o n
Which of the following should she not receive? of ankle plantar flexion on the right with intact ankle
dorsiflexion and hip flexion. His gait is altered in that
A. Aprescription for ciprofloxacin that she should take he shortens the length of stride during the push off
in the event of gastrointestinal dysentery phase on the right leg and cannot stand on his toes on
B. Chloroquine chemoprophylaxis the right because of weakness. Reflexes are normal at
C. Hepatitis Avaccine the bilateral patellae, diminished at the left Achilles,
D. Hepatitis B vaccine and absent at the right Achilles. Cremaster reflex is
E. Yellow fever vaccine intact. Sensory examination reveals reduced detection
of pin prick and proprioception in the posterior right
36. A 25-year-old previously healthy woman presents thigh and lateral toes of the right foot.
to her primary care provider with bilateral ankle pain Which of the following is the most appropriate next
and a rash. Symptoms started 2 weeks ago as pain, swell- step in evaluation and management of this patient?
ing, and tenderness in the bilateral ankles. Her feet are
swollen to the point that she cannot wear her regular A. Physical therapy and reassurance. Reassess in
shoes and it hurts to walk. No other joints are involved. 4weeks.
Ten days ago, she began to notice tender erythematous B. Plain film radiography of the back. If normal, then
bumps come up on her anterior shins. Physical exam- offer physical therapy and reassurance. Reassess in
ination confirms the history. She has swelling around 4weeks.
both ankles and reduced active range of motion because C. Schedule nonurgent MRI of the lumbar spine. If
of pain. Passive range of motion is preserved, however, there is a herniated disc, then offer physical therapy
including flexion/extension and inversion/eversion at and reassurance. Reassess in 4 weeks.
the ankle. The Achilles tendon is unaffected. The shins D. Schedule nonurgent MRI of the lumbar spine. If
have fewer than a dozen scattered tender erythematous there is a herniated disc, then refer to Interventional
nodules of varying size, none larger than 1 centimeter. Radiology for corticosteroid injection.
There is no overlying skin breakdown. E. Referral to Emergency Department for urgent MRI
Which of the following tests would be expected to of the lumbar spine.
show an abnormality that would aide in the diagnosis
of her condition? 38. A50-year-old woman has had chronic systemic sar-
coidosis for 4years. She has had involvement of the heart,
A. Ankle film liver, lungs, skeletal muscle, and joints. Despite aggres-
B. Chest radiograph sive immunosuppressive measures with methotrexate
C. Rheumatoid factor and anti-CCP antibody and infliximab, she has continued to require chronic
D. Serum uric acid level corticosteroid therapy with daily prednisone. The pred-
E. Serum lyme serologies nisone dose is currently 40 mg daily. It has fluctuated
between 20 and 60 mg daily with disease activity and
37. A32-year-old man presents with low back pain and has been no less than 20 mg daily for 3years. On physi-
leg numbness. Three weeks ago, he was lifting window cal examination, she is obese with Cushingoid features.
air-conditioning units when he felt a pop in his low Blood pressure is 150/95mm Hg, and body mass index
back associated with severe pain. He rested his back is 39.5 Kg/m2 . Comorbidities include steroid-induced
for 1 week and has been using a friends prescription diabetes mellitus.
for hydrocodone. With this, the pain has improved All of the following are direct side effects of chronic
slightly. Two weeks ago, however, he began to notice corticosteroid usage except:
constipation such that he has not had a bowel move-
ment in 8 days. He has also had new-onset impotence A. Increased risk of infectious illnesses
and reports increased urinary urgency with frequent B. Accelerated cataract formation
dribbling of only small volumes of urine. This is asso- C. Hyperuricemia
ciated with paresthesias down the right posterior leg D. Avascular necrosis of bone
and numbness in the perineum. Physical examination E. Proximal muscle weakness
reveals a non-toxic-appearing gentleman who has trou-
ble finding a comfortable position on the examination 39. A 74-year-old woman presents to outpatient clinic
table. Back range of motion is limited to 30 degrees of reporting 8 weeks of sudden-onset fatigue and achiness in
forward flexion because of lumbar spine pain, and there the shoulders and hips. Two weeks ago, she saw an ortho-
is associated spasm of paraspinous muscles. Neurologic pedic surgeon, who diagnosed rotator cuff tendonitis and
examination is entirely normal in the upper extremi- injected 40 mg triamcinolone into the left subacromial
ties. Assessment of lower extremities reveals weakness bursa. She currently reports that, following the injection,

3 . R h e u m at o l o g y 59
everything went away and I felt great. However, this C. Systemic lupus erythematosus
benefit only lasted 3 days and all of her symptoms have D. Acute bacterial endocarditis
returned. She has no other medical illnesses. She has E. Behcets disease
not had any red, warm, or swollen joints. She denies any
headaches, visual disturbances, scalp tenderness, or jaw 41. A32-year-old woman gives birth to a baby girl at 34
achiness after chewing. Laboratory analysis reveals ESR weeks gestation because of fetal distress. The newborn
64mm/hr and CRP 76.2 mg/L (normal <5.0). becomes cyanotic shortly after deliver. Rapid assessment
Which of the following features also typically charac- identifies a heart rate of 50 beats per minute and com-
terizes the condition most likely affecting this patient? plete heart block. With pacemaker support, the infants
status improves quickly and the cyanosis resolves. The
A. Morning stiffness newborn develops infantile jaundice starting on day 5
B. Blindness of life, and is treated with phototherapy. On day 9 of
C. Elevated rheumatoid factor life, the infant develops a patchy well-circumscribed
D. Chondrocalcinosis maculopapular rash.
E. Joint space narrowing on plain films of shoulders Which of the following conditions in the mother
and hips might explain the newborns condition?

40. A 35-year-old woman presents to urgent care for A. Sjogrens syndrome


fever, rash, and joint pain. She was in her usual state of B. Lyme disease
health until 3 weeks ago, when she experienced a severe C. Marfan syndrome
sore throat. She was seen by her primary care provider. D. Acute rheumatic fever
Testing for Strep throat by rapid testing and culture was E. Maternal methotrexate exposure during pregnancy
negative, but she was treated with empiric amoxicillin,
without improvement (completed 10days ago). She has 42. A 75-year-old gentleman presents to his primary
subsequently developed pain and swelling in the wrists care provider for an annual physical examination. He
and high daily fevers that are accompanied by a gener- has relatively few comorbidities, but he reports ongo-
alized rash. She feels generally good between attacks ing chronic daily low back pain that radiates into the
but has not had any relief of attacks with naproxen 500 buttocks. He reports morning stiffness and progressive
mg twice daily. Physical examination is notable for an pain over the course of the day. He describes difficulty
uncomfortable but non-toxic-appearing woman of in standing for more than 10 minutes, and he has to lean
average stature. Temperature is 38.9C, blood pressure on a walker or shopping cart when trying to get out and
is 110/72 mm Hg, heart rate is 110 beats per minute, about during the day. For his back pain, he has (1)taken
respiratory rate is 18 breaths per minute, and oxygen tramadol 50 mg three times daily and acetaminophen
saturation is 98% on room air. Neck is supple but she 1,000 mg three times daily, (2)engaged in regular physi-
has cervical and axillary adenopathy. She has redness, cal therapy, and (3)received multiple epidural cortico-
warmth, and swelling of the bilateral wrists and meta- steroid injections and facet joint injections over the
carpal phalangeal joints. Skin shows a salmon-pink past 12months. None of these approaches has provided
maculopapular rash involving mostly her trunk. adequate relief. Physical examination is largely unre-
Laboratory analysis is notable for a normal comprehen- vealing except for some weakness of hip flexor strength
sive metabolic profile. Complete blood count reveals and reduced flexion and extension of the lumbar spine.
white blood count 18.5 103 cells per mm3, Hgb 11.1 His gait is slow and deliberate, but steady. Plain film
g/dL, and platelet count 772 103 cells per mm3. CRP radiography and MRI from 1 year ago have revealed
is 199 mg/L (normal <5), and ESR is 92mm/hr. Blood multilevel degenerative changes with disc desiccation,
cultures obtained 5 days prior have demonstrated no nerve impingement, facet joint arthropathy, and central
growth. ANA, RF, and anti-CCP antibodies are all neg- spinal stenosis.
ative. CT scan of the abdomen, chest, and pelvis reveals Which of the following is the next most appropriate
some mild hepatosplenomegaly and diffuse adenopa- next course of action?
thy. Peripheral blood flow cytometry does not reveal
any hematologic malignancy. Excisional biopsy of an A. Reassurance and change tramadol to oxycodone 5
axillary lymph node excludes a malignancy and demon- mg three times daily
strates reactive lymphadenitis. B. Reassurance and another prescription for physical
Which of the following is the most likely diagnosis? therapy
C. Repeat MRI of the lumbar spine
A. Lyme disease D. Referral to Interventional Radiology for
B. Adult-onset Stills disease corticosteroid injection into an involved facet joint

6 0 T h e B r i g h a m I n t e n s i v e R e v i e w o f I n t e r n a l M e d i c i n e Q u e s t i o n a n d A n s w e r C o m pa n i o n
E. Referral to Spine Surgery for decompressive
laminectomy

43. A72-year-old woman presents to her primary care


physician with 3 weeks of declining health because of
fatigue, cough, and dyspnea. She has a history of atrial
fibrillation, for which she takes warfarin and atenolol.
She has no other significant medical comorbidities.
During the appointment, she coughs up scant amounts
of fresh blood. Vitals signs include temperature 37.2C,
heart rate 62 beats per minute (irregular), blood pres-
sure 162/90mm Hg, respirator rate 25 breaths per min-
ute, and oxygen saturation 91% on room air. Physical
examination shows a fatigued-appearing woman in
some moderate amount of respiratory distress, using Figure3.5 Tophaceous gout picture.
accessory muscles. Lungs have diffuse rales. She has
mild periorbital edema and pitting leg edema to the
upper shins. Urgent chest radiograph is obtained that 5.0 mg daily and azathioprine 150 mg daily. Serum uric
shows patchy infiltrates in bilateral lung fields. Initial acid is 11.4 mg/dL.
laboratory assessment includes the following:BUN 82 What is the most appropriate next course of action?
mg/dL, creatinine 4.1 mg/dL (baseline normal), white
blood count 13.2 103 cells per mm3 with normal dif- A. Initiate allopurinol 300 mg daily and treat any flares
ferential, hemoglobin 9.2 g/dL, and platelet count 622 of gouty arthritis with higher doses of prednisone as
103 cells per mm3, ESR 104 mm/hr, and CRP 301 needed.
mg/L (normal <5). Urinalysis reveals +++ protein and B. Check a 24-hour urine uric acid, and initiate
75100 red blood cells per high-power field. probenecid 500 mg twice daily if the patient is an
Which of the following could not adequately explain under-excretor.
her current presentation? C. Discuss with patients nephrologist about alternative
immunosuppressive therapies for the renal graft.
A. Antinuclear antibody 1:2560 speckled pattern, D. Increase prednisone to 15 mg daily.
anti-double-stranded DNA antibody 721 U/ml E. Initiate febuxostat 80 mg daily and treat any flares of
(normal <5), C3 complement 57 mg/dL (normal gouty arthritis with higher doses of prednisone as
75175), and C4 complement 9 mg/dL (normal needed.
1440)
B. Detectable antiglomerular basement membrane 45. A48-year-old man with rheumatoid arthritis (RA)
antibody for 4years presents to his primary care physician (PCP)
C. Detectable antineutrophil cytoplasmic antibody with 5 days of severe right knee pain and swelling.
with cytoplasmic pattern and antiproteinase-3 His RA has been well controlled for 2years on a com-
antibody positivity bination of disease-modifying antirheumatic drugs
D. Temporal artery biopsy showing granulomatous (DMARDs):methotrexate and adalimumab (an inject-
arteritis able TNF- antagonist). He has no prosthetic joints and
E. Urine toxicology positive for cocaine no other comorbidities. His joint examination reveals
no abnormalities of the upper extremities or left leg.
44. A39-year-old man presents to the outpatient clinic His right hip and ankle move well, but the right knee
having noticed the slow accumulation of chalky white is red, warm, and swollen with a modest side effusion.
material in many fingers and toes (Figure 3.5). These He has a 20-degree flexion contracture of the knee and
have occasionally gotten red, but he reports that they pain when trying to extend it fully or flex it more than
rarely become painful or swollen. He has a history of 45 degrees. He has no fevers and is non-toxic-appearing.
end-stage renal disease from Alports syndrome, and he Laboratory analysis is notable for ESR 48 and CRP 119.1
received a cadaveric renal transplant 2years ago (func- mg/L (normal <5). Plain film radiography of the right
tioning well with serum creatinine 1.2 mg/dL). Four knee reveals an effusion but no other abnormalities.
years ago, he had an episode of acute monoarthritis What is the most appropriate next step?
of the left knee, and arthrocentesis showed negatively
birefringent needle-shaped crystals with no growth in A. Magnetic resonance imaging (MRI) of the right
fluid culture. Current medications include prednisone knee

3 . R h e u m at o l o g y 61
B. Aspiration of the right knee for fluid analysis Besides stabilizing the patient for her acute critical
C. Aspiration of the right knee for symptomatic relief, illness, which of the following is also indicated as the
followed by corticosteroid injection next most appropriate step in diagnosing this patients
D. Touch base with his rheumatologist to discuss a illness?
change in the DMARD regimen,
E. Prescribe a course of oral corticosteroids tapered over A. Chest CT scan with iodinated contrast
1 week, for treatment of an RA flare. B. Pulmonary function testing with corrected DLCO
measurement
46. A33-year-old woman is brought to the emergency C. Lumbar puncture
department (ED) by ambulance. She reports fever and D. Cardiac MRI with gadolinium contrast
progressively severe dyspnea but no cough, hemoptysis, E. Flexible bronchoscopy
or chest pain. She has a 9-month history of severe sys-
temic lupus erythematosus (SLE) complicated by non- 47. A 42-year-old man presents to his outpatient pri-
hemolytic anemia, thrombocytopenia, and Class 4 lupus mary care provider with back stiffness. He has had
nephritis. Baseline laboratory tests from 3 weeks ago these symptoms for as long as he can remember.
showed creatinine 2.8 mg/dL, hemoglobin 10.2 g/dL, Initially, his stiffness was limited to the lumbar spine,
and platelet count 95 103 cells per mm3. For her SLE, but he now reports stiffness in the thoracic and cervi-
she is being treated aggressively with monthly intrave- cal spine, accompanied by pleuritic chest discomfort.
nous cyclophosphamide 750 mg/m2, prednisone 60mg He has difficulty turning his head to look in the side
daily, and hydroxychloroquine 400 mg daily. In the mirrors when driving. His symptoms are worse in the
ED, initial vital signs show temperature 37.8C, blood morning and work themselves out over the course of
pressure 90/48 mm Hg, heart rate 122 beats per min- the day. His medical history is notable only for epi-
ute, respiratory rate 28 breaths per minute, and oxygen sodic redness in the left eye accompanied by pain and
saturation 88% on room air. Within minutes of arrival, photophobia (last episode 12 months ago). Physical
she exhibits increased work of breathing and suffers examination reveals a generally healthy individual
respiratory arrest, prompting intubation. Mechanical with limited motion of the spine and reduced chest
ventilation improves oxygenation saturation (94%) excursion. Laboratory analysis reveals CRP 22.8
on 60% inhaled oxygen. Initial admission blood work mg/L (normal <5). Radiographic imaging demon-
shows BUN 42 mg/dL, creatinine 3.1 mg/dL, hemo- strates ankylosis of the sacroiliac joints. A diagnosis
globin 7.7g/dL, white blood count 9.2 103 cells per of ankylosing spondylitis is made.
mm3 (normal differential), and platelet count 88 x103 Which of the following will most likely to lead to
cells/mm3. Toxicology tests are negative for illicit symptomatic improvement by treating the underlying
drugs. Blood cultures are drawn and are pending. EKG condition?
demonstrates no abnormalities. Chest radiograph is as
shown in Figure3.6. A. HLA-B27 testing
B. Prednisone 20 mg daily
C. Methotrexate 25 mg once weekly
D. Oxycodone 5 mg three times daily
E. Adalimumab 40 mg every other week

48. A67-year-old woman with long-standing Sjogrens


syndrome (SjS) presents with painless firm swelling of
the left cheek of 4months duration. She has had years
of dry eyes and dry mouth from the SjS, leading to cor-
neal abrasions and multiple dental caries. She also has
SjS-associated interstitial lung disease, for which she
has been on chronic prednisone 510 mg daily and aza-
thioprine 2 mg/kg for 14 years. She has osteoporosis,
for which she has been on alendronate 70 mg weekly
for 10 years. Historically, her serologic workup has
included an antinuclear antibody (ANA) 1:1280 speck-
Figure3.6 SLE DAH chest radiograph. led pattern, high-titer anti-Ro and anti-La antibodies,

62 T h e B r i g h a m I n t e n s i v e R e v i e w o f I n t e r n a l M e d i c i n e Q u e s t i o n a n d A n s w e r C o m pa n i o n
rheumatoid factor (RF) at 145 U/mL (normal <15), and affected, yet there is a yellowish discoloration of the
polyclonal hypergammaglobulinemia. She has had sclera of both eyes.
normal complement levels and negative anti-dsDNA, Which of the following is the most likely diagnosis?
anti-Smith, and anti-RNP antibodies. Physical exami-
nation reveals asymmetric swelling of the left cheek A. Scleroderma
overlying the masseter muscle. It is firm, nontender, and B. Lipodermatosclerosis
has poorly defined borders. She has dry mucous mem- C. Scleredema diabetacorum
branes and lacks salivary pooling on oral inspection. D. Eosinophilic fasciitis
She has had extensive dental work. There is no exposed E. Nephrogenic systemic fibrosis
bone, but she still has multiple caries that need to be
corrected. She has palpable purpuric lesions on the legs. 50. A62-year-old male laborer presents to a new primary
Laboratory analysis reveals normal comprehensive met- care provider (PCP) with 3 years of persistent stiffness
abolic profile and complete blood count. ESR is 68mm/ and achiness in many joints, including the hands, wrists,
hr, CRP is 119 mg/L (normal <5), C3 complement is 66 elbows, shoulders, hips, and knees. His shoulders and
mg/dL (normal 75175), and C4 complement is 3 mg/ hips bother him more than anything else. At the end of
dL (normal 1440). RF is now normal for the first time a long shift, his joints feel somewhat better than at the
on record. beginning of the day. He was told by his previous PCP
Which of the following is the most likely diagnosis? that he has arthritis related to his job in construc-
tion, and he had been managed primarily with escalat-
A. Parotid lymphoma ing doses of oxycodone for symptoms. Comorbidities
B. Sarcoidosis include chronic obstructive pulmonary disease (COPD)
C. Sialolithiasis from years of cigarette smoking, type 2 diabetes mellitus,
D. Suppurative bacterial sialadenitis hypertension, hypercholesterolemia, and Hashimotos
E. Osteonecrosis of the jaw thyroiditis. Physical examination reveals beefy hands
with boggy changes in the metacarpal phalangeal (MCP)
49. A48-year-old woman from Guatemala presents to and wrist joints. His wrists are somewhat fused, with very
the emergency department of a tertiary care center in limited range of motion. He is unable to extend his elbows
the United States in search of an opinion regarding her fully because of 5-degree flexion contractures. Both knees
poor health. She has been receiving hemodialysis (HD) have modest-sized knee effusions but he can extend them
for 12years because of end-stage renal disease (ESRD) fully. Laboratory analysis reveals normal comprehensive
as a complication of type 1 diabetes mellitus (DM). metabolic profile and normal complete blood count with
Nine months ago, while living in Guatemala, she differential. Serum uric acid is 4.0 mg/dL. Rheumatoid
experienced a generalized seizure. Part of the workup factor (RF), anticyclic citrullinated (CCP) antibodies,
included a magnetic resonance angiography (MRA), and antinuclear antibodies (ANA) are not detectable.
which revealed no intracranial structural or vascular CRP is 42.0 mg/L (normal <5). Aspiration of the right
abnormalities other than moderate cerebrovascular knee obtains 45 mL nonbloody, cloudy, yellow fluid with
disease. It was concluded that her seizure was the result a white blood count 35,000 cells/mm3 (60% neutrophils).
of an otherwise asymptomatic episode of hypoglyce- No organisms are identified upon fluid gram stain or cul-
mia. She has not had any further seizures. However, ture, and compensated polarized light microscopy does
in the past 6 months, she has noticed thickening and not reveal any birefringent crystals. Plain film imaging
hardening of the skin associated with progressive of the hands shows erosions and joint space narrowing in
immobility. Symptoms started in her legs but have multiple MCP joints and wrists.
progressed to involve her arms. She does not describe What is the most likely diagnosis?
Raynauds phenomenon. On physical examination, she
has tightness and hardening of the skin of her calves, A. Rheumatoid arthritis
thighs, and forearms. There is a brawny discoloration B. Chronic gouty arthropathy
of the skin, which is tethered down on deeper tissues C. Primary osteoarthritis of the involved joints
such that she has flexion contractures of the ankles, D. Rheumatic fever
knees, fingers, and elbows. Her face and torso are not E. Polymyalgia rheumatica

3 . R h e u m at o l o g y 63
CH A PT ER3 A NSW ER S phosphatase and hypoalbuminemia may be enough to war-
rant liver biopsy, but one might expect some sort of abnor-
1. ANSWER:B. Acute pseudogout arthritis. malities in abdominal/pelvic CT imaging or ultrasound
studies if liver disease was the source of fever. In this case,
This patient has experienced an acute inflammatory the abnormalities are secondary to GCA and will likely cor-
arthritis of the right wrist and several MCP joints, as rect with corticosteroid treatment.
evidenced by the sudden onset of redness, warmth, and
swelling, as described. Osteoarthritis (Answer D) rarely
affects the wrist and is considered mostly noninflamma-
tory. Rheumatoid arthritis (Answer C) is unlikely to come 3. ANSWER:C. This patient likely has a new
on in such an abrupt manner with unilateral findings. diagnosis of rheumatoid arthritis (RA).
While occult injury should always be considered, there
is no history of fracture, and the plain radiograph does RA is the most common cause of a chronic inflammatory
not support the diagnosis of fracture (Answer E). Septic polyarthritis, defined as having symptoms >6 weeks dura-
arthritis (Answer A) is not the most likely answer for sev- tion. Her history, physical examination, and inflammatory
eral reasons. Most important, this patient would have to indices suggest an inflammatory process. When detect-
have polyarticular septic arthritis of the wrist and several able, anti-CCP antibodies (Answer C) have high specific-
MCP joints, a very unlikely scenario when considering her ity for RA (>90%), which is even better than RF. Even if
negative blood cultures, appropriate antibiotic coverage, her anti-CCP antibody level were negative, RA remains the
and general improvement in all other clinical parameters. most likely diagnosis. It is extremely rare for premenopausal
Septic arthritis is often the result of hematogenous seed- women to have gout (Answer E) unless there is some obvi-
ing of a joint from bacteremia rather than transcutaneous ous risk factor (e.g., congestive heart failure or severe renal
inoculation of a joint. Polyarticular septic arthritis would disease). While it would not be unreasonable to test for viral
indicate a high-grade bacteremia. While larger joints like causes of polyarthritis, these rarely persist for 6 months
the knees, hips, and shoulders are more likely to be affected duration:parvovirus, HIV, hepatitis B, or hepatitis C.EBV
by septic arthritis, any joint can become infected. The most (Answer D) rarely if ever causes an inflammatory polyar-
likely answer is pseudogout arthritis (Answer B) based on thritis, and it would not persist this long in an immuno-
the clinical presentation of an acute arthritis of the MCP competent host. There is no role for repeating the ANA test
and wrist joints. Supporting evidence includes the chon- within such a short timeframe (Answer A), since she has
drocalcinosis on plain film, more commonly found in no new symptoms to suggest systemic lupus erythematosus.
patients with hyperparathyroidism. Arthrocentesis of joint The inflammatory features of her presentation are inconsis-
fluid would likely reveal elevated fluid white blood cells tent with a diagnosis of fibromyalgia syndrome, which is a
(>2,000 per mm3), negative cultures, and positively bire- chronic noninflammatory and nonstructural pain amplifi-
fringent rhomboid-shaped crystals consistent with calcium cation syndrome.
pyrophosphate dihydrate (CPPD).

4. ANSWER:B. Osteoarthritis of the hip


2. ANSWER:E. Temporal artery biopsy
Avascular necrosis (AVN) can affect almost any bone,
Giant cell arteritis is not an uncommon cause for fever but the head of the femur is by far the most commonly
of unknown origin in elderly patients, accounting for involved site. AVN can lead to cortical collapse of bone,
~17% of FUOs in this demographic group [O. Mourad. with resultant accelerated osteoarthritis (OA) of the
Arch Intern Med. 2003. Mar 10;163(5):54551]. Many of adjoining joint. Thus, OA is the result (not a cause) of
the features described in this case can be found in patients AVN (Answer B). Pain is almost always the presenting
with unrecognized GCA:anemia of chronic disease, reac- complaint, but many cases can be entirely asymptomatic
tive thrombocytosis, elevated inflammatory indices, hypo- and discovered incidentally. For patients with unrelent-
albuminemia, and elevated alkaline phosphatase. Most ing pain, total joint arthroplasty should be considered.
important, infectious and malignant etiologies have been The other conditions listed all increase the risk of AVN.
largely ruled out. There is no microbiologic evidence to war- Additional risk factors include high-dose corticosteroid
rant empiric levofloxacin (Answer A) or localizing features therapy, radiation, trauma, systemic lupus erythemato-
to suggest tuberculosis infection (Answer B). Elder abuse sus (SLE), human immunodeficiency virus (HIV) infec-
(Answer C) should always be considered in elderly patients tion, decompression (Caissons) disease, transplantation,
with failure to thrive but would not explain the fevers Legg-Calve-Perthe disease, and slipped capital femoral
or elevated inflammatory indices. The elevated alkaline epiphysis (SCFE) syndrome.

6 4 T h e B r i g h a m I n t e n s i v e R e v i e w o f I n t e r n a l M e d i c i n e Q u e s t i o n a n d A n s w e r C o m pa n i o n
5. ANSWER:D. Hydroxychloroquine and recent intravenous drug abuse. The presence of these
findings should prompt consideration of further imaging
Hydroxychloroquine is a generally safe and well-tolerated to assess for inflammatory, infectious, or neoplastic origin,
agent for the treatment of SLE. Rarely, however, it can such as epidural or paraspinous abscess, compressive mass,
cause pigment to deposit in the retina, which can lead to or cauda equina syndrome. For lumbago, this patient should
permanently impaired vision. Thus, regular ophthalmo- first engage in physical therapy to improve core muscle
logic screening for potential toxicity is very important. strength, conditioning, and range of motion. Weight loss
Chloroquine, a related agent, carries a higher risk for reti- is advisable. This patient also demonstrates misuse of oxy-
nal toxicity and therefore is used less commonly. None of codone. Thus, discussions of an opiate medication contract
the other agents (Answers AC and E) causes direct visual are indicated. It is unlikely that reassurance alone (Answer
toxicity, although each of these is immunosuppressive and A) will satisfy this patient, and it will be unlikely to change
could increase the risk of ocular infections (e.g., zoster, his current pattern of opiate usage. Switching oxycodone
toxoplasmosis, etc.). It is important to bear in mind the to the more potent hydromorphone will likely only com-
late-onset complications of cyclophosphamide: sterility, pound what appears to be recurrent dependency on opiates.
lymphoma, and malignancies of the urinary tract. These MRI of the lumbar spine (Answer D) might be appropriate
might not manifest until decades after a drug exposure. if he reported any red flags or failed a concerted effort at
physical therapy. It is inappropriate to discharge this patient
from your practice (Answer E) until you can demonstrate
that the patient intentionally misconstrues facts or mis-
6. ANSWER:C. Hemachromatosis represents himself in order to obtain opiate-based medica-
tions, or if there is deviation of his prescription medications
Hemachromatosis has many clinical manifestations related to other individuals in exchange for money or favors. An
to iron accumulation in target organs: cirrhotic liver dis- opiate medication contract would spell this out clearly, such
ease, congestive heart failure, hypogonadism, diabetes that violation of the contract could provide supporting evi-
mellitus, bronzing of the skin, and accelerated degen- dence to discharge the patient from your practice.
erative arthritis. Many of these are present in this case.
Hemochromatosis-related arthropathy is one of several
causes of secondary osteoarthritis (OA), and it classically
affects the MCP joints. This patients hand radiographs 8. ANSWER:A. Serologic testing for Lyme disease
show some characteristic changes of hemochromatosis
arthropathy, including an aggressive osteoarthritis changes The patient presents with a chronic inflammatory monoar-
and hooked osteophytes of the MCP joints. While hemo- thritis. While unlikely to be suppurative septic arthritis of
chromatosis arthropathy is largely a noninflammatory pro- 8 weeks duration (e.g., from Staphylococcus aureus), chronic
cess, it is associated with chondrocalcinosis (as seen in this infectious processes must be considered. In patients living
radiograph) and episodes of pseudogout. Involvement of in certain parts of the United States like New England and
MCP and wrist joints argues against primary OA (Answer parts of the upper Midwest and Pacific Northwest, infec-
D), which very rarely affects these joints. Primary OA tends tion from the tick-borne Borrelia burgdorferi causes Lyme
to be limited to DIP and PIP joints, and the base of the disease. One late-onset manifestation of Lyme disease is
thumb. There are no clinical features of inflammation to joint infection, occurring weeks or typically months after
suggest rheumatoid arthritis (Answer A). Metacarpal frac- exposure to the culprit tick. Lyme arthritis is almost invari-
ture (Answer B) is not present in this radiograph. Gauchers ably a chronic inflammatory monoarthritis, often of the
disease (Answer E) is associated with cirrhosis, osteoporo- knee. Patients do not necessarily recall the initial tick bite
sis, and joint pain. However, this condition mostly involves or any classic bulls-eye rash of early Lyme disease. Lyme
large joints (hips and knees) and is associated with avascular arthritis is not to be confused with chronic Lyme, a nebu-
necrosis, not accelerated osteoarthritis of the hands. lous condition that encompasses a constellation of somatic
complaints:chronic pain, fatigue, impaired cognition, and
other manifestations. Long-term antibiotic therapy for
chronic Lyme is highly controversial. Still, some patients
7. ANSWER:B. Provide a prescription for physical insist that chronic antibiotics give them a positive response
therapy and discuss an opiate medication contract. (possibly psychological), even though there may or may not
be serologic evidence of prior B.burgdorferi exposure. None
This patient describes lumbagomusculoskeletal low of the other answers would be inappropriate next steps once
back pain without evidence of any red flags:fever, occult Lyme arthritis was ruled out by serologic testing. Note that
weight loss, numbness in the perineum, new-onset impo- most patients with ANA-associated connective tissue dis-
tence, bowel or bladder retention, history of malignancy, eases like systemic lupus erythematosus do not experience

3 . R h e u m at o l o g y 6 5
chronic inflammatory monoarthritis of this nature. This tissue disorder, so an antinuclear antibody is unnecessary
would be more typical of the spondyloarthropathies:pso- (Answer A). Temporal artery biopsy (Answer C) is used
riatic arthritis, ankylosing spondylitis, reactive arthritis, or to diagnose giant cell arteritis (GCA). Sometimes, GCA
the arthropathy of inflammatory bowel disease. can present as fever, malaise, and weight loss of unknown
origin. CT scan imaging (Answer D) can identify occult
abscess or solid organ malignancy. Prior to temporal artery
biopsy or CT scan imaging, however, the discrepancy in
9. ANSWER:B. Viral arthritis ESR and CRP first should be assessed with the much less
invasive and less costly SPEP/UPEP. Finally, it is inap-
This patient likely suffers from acute viral polyarthritis propriate to treat the patient empirically for PMR with
(Answer B) from parvovirus B19. In children, this virus is low-dose corticosteroids (Answer E) until one has ruled
the cause of Fifths disease, characterized by fever, rash, out other explanations for the patients complaints and
and occasional joint pains. In adults, however, inflamma- laboratory abnormalities. PMR should always be consid-
tory joint disease may be the only manifestation. The con- ered a diagnosis of exclusion.
dition is too abrupt in onset to be diagnosed as rheumatoid
arthritis (Answer E). The arthritis of Lyme disease is most
often a chronic inflammatory monoarthritis, and therefore
Answer D is not correct. Everything about his presentation 11. ANSWER:C.Pulmonary hypertension
suggests an inflammatory process, and thus fibromyalgia
syndrome (Answer C) is incorrect. Septic polyarthritis The patient has pulmonary hypertension (pHTN) as a
(Answer A) is unlikely because patients are often much complication of scleroderma. Features associated with
sicker than this patient. Septic joints are seeded hematog- pHTN include CREST-variant scleroderma and anticen-
enously, and thus septic polyarthritis indicates a very high tromere antibodies. This condition is suggested by abnor-
level of bacteremia. Another notable feature of parvoviral malities in echocardiography and pulmonary function
arthritis is the impairment of red blood cell (RBC) produc- testing (isolated low DLCO). It is confirmed by direct
tion (hence the inappropriately low reticulocyte count). In measurement of pulmonary artery pressures by right
patients with sickle cell disease or other states of high RBC heart catheterization. Treatment options include endo-
turnover, parvoviral infection can cause an aplastic cri- thelin antagonist agents (e.g., bosentan), phosphodiester-
sis. Diagnosis of parvoviral infection is based on serologic ase inhibitors (e.g., sildenafil), and prostacyclin analogs.
testing, with elevated antiviral IgM levels consistent with Scleroderma cardiomyopathy is a fibrotic complication of
recent infection. Treatment of parvoviral arthritis is sup- scleroderma, associated with anti-SCL70 antibodies and
portive: pain relief with nonsteroidal anti-inflammatories diffuse fibrotic disease.
(NSAIDs) or low-dose systemic corticosteroids until the
viral process abates.

12. ANSWER:B. Bronchoscopy with lavage for


microscopic organisms
10. ANSWER:B. Serum and urine protein
electrophoresis This patient is immune suppressed secondary to his RA
therapy. Based on his deteriorating clinical status, fur-
This patient presents with nonspecific symptoms, anemia, ther investigation is warranted in order to assess for atypi-
and an elevated ESR. While these symptoms might repre- cal infection. This patient could have pulmonary abscess,
sent polymyalgia rheumatica (PMR), the discrepantly low fungal pneumonia, or tuberculoma. Bronchoscopy with
CRP warrants consideration of other processes. CRP is a lavage (Answer B) is the least invasive test that will still
nonspecific indicator of inflammation that can be elevated yield clinically important information. Lung biopsy
because of inflammation, infection, and some malignan- (Answer C) might be necessary if bronchoscopy fails to
cies. In this case, factors other than inflammation have yield a diagnosis, but it is invasive. Switching antibiotics
led to an increased ESR; an immunoglobulin paraprotein- (Answer E) might lead to clinical improvement if this was
emia increases the ESR without affecting the CRP because simply bacterial pneumonia resistant to the initial choice
positively charged immunoglobulin promotes electrostatic of antibiotics. If incorrect, however, this would delay diag-
adhesion of negatively charged red blood cells such that nosis and treatment. PET-CT scan (Answer D) is not yet
they settle out of suspension faster. In this case, the patient indicated because malignancy has not been shown (and
has multiple myeloma, which would be detected by serum indeed infection is more likely and of greater immediate
or urine electrophoresis (SPEP/UPEP). There is nothing to concern). Finally, RA can cause necrotic pulmonary nod-
suggest systemic lupus erythematosus or other connective ules, but empiric treatment for rheumatoid lung (Answer

6 6 T h e B r i g h a m I n t e n s i v e R e v i e w o f I n t e r n a l M e d i c i n e Q u e s t i o n a n d A n s w e r C o m pa n i o n
A) is inappropriate until infectious processes have been are counterproductive and can lead to tolerance, depen-
ruled out. dency, and addiction.

13. ANSWER:D. Scaly erythematous plaques on the 15. ANSWER:E. Skeletal muscle biopsy
extensor surfaces of the arms and legs
The patient suffers from dermatomyositis (DM). This
This answer describes a patient with psoriasis, which can patients painless proximal muscle weakness and charac-
also involve the scalp, external auditory canal, supragluteal teristic rash are classic for DM. The CPK, CK-MB, AST,
cleft, and umbilicus. Nail pitting and other nail abnormali- and ALT are all of skeletal muscle origin, so testing other
ties can also be present. In patients with psoriasis, unex- targets is unlikely to yield a diagnosis (Answers AD).
plained inflammatory joint disease indicates the presence of She also likely has interstitial lung disease (ILD), which is
psoriatic arthritis. This can take on several forms:inflam- commonly associated with DM. DM is one of several idio-
matory oligoarthritis or monoarthritis typically of the pathic inflammatory myopathies (IIMs). Others include
lower extremities (as in this case), dactylitis of fingers or toes polymyositis (PM), malignancy-associated myositis, inclu-
(as in this case), a rheumatoid-pattern polyarthritis, inflam- sion body myositis (IBM), and juvenile dermatomyositis
matory spondyloarthritis of the spine, accelerated osteo- (J-DM). In patients with some forms of IIM, autoantibody
arthritis of the distal interphalangeal (DIP) joints, and testing can reveal an elevated antinuclear antibody (ANA)
inflammatory enthesitis of the tendon/bone interface. In titer. Asubset of patients with DM will have more specific
this case, septic arthritis and crystalline arthritis have been myositis-associated autoantibodies such as anti-Jo-1, and
ruled out by arthrocentesis of inflammatory fluid that lacks these patients have higher rates of DM-associated ILD.
crystals and microorganisms. Serum uric acid (Answer B), Patients with J-DM can experience bowel vasculitis and
serum ACE level (Answer C), and radiographic evidence subcutaneous calcifications (calcinosis cutis). The diag-
of chondrocalcinosis (Answer E) are associated with gout, nosis of the various IIMs is confirmed by skeletal muscle
sarcoid arthritis, and pseudogout, respectively, but are not biopsy (Answer E). Electron microscopy is advisable if
diagnostic of these conditions. Anti-Smith antibodies are IBM is being considered in order to visualize classic vacu-
a specific autoantibody for systemic lupus erythemato- olar changes in diseased muscle. Unlike the other IIMs,
sus (SLE). However, SLE is unlikely in this patient clini- however, IBM usually causes chronic distal muscle weak-
cal based on the negative ANA and monoarticular joint ness and atrophy. Because muscle involvement in any of
involvement. the IIMs can be patchy, EMG or MRI is often utilized to
identify a muscle suitable for biopsy. There is an associa-
tion between IIMs and malignancies, typically of the lung,
GI tract, and female reproductive tract. While all patients
14. ANSWER:A. Tilt-table test should have age-appropriate cancer screening, it would not
be unreasonable to perform additional imaging to assess for
The patient demonstrates evidence of a joint hypermobility occult cancer:CT scan of the chest, abdomen, and pelvis,
syndrome (HMS). Several rare genetic disorders can lead to and consideration of pelvic ultrasound. Treatment of most
HMS (e.g., Marfan syndrome and Ehler-Danlos syndrome). forms of IIM initially involves corticosteroids, often in high
However, this patient demonstrates none of the character- doses (1 mg/kg) with a long and slow taper. Methotrexate
istic features of these, and thus he most likely has benign is the most commonly used non-steroid-based immunosup-
joint HMS. Formerly termed Type 3 Ehler-Danlos syn- pressive agent. Unfortunately, there are no effective thera-
drome, a genetic basis for benign joint HMS is not as well pies for IBM, which carries a dismal prognosis.
established as in the other forms of EDS. Thus, benign joint
HMS remains primarily a clinical diagnosis, and no further
testing for genetics or other causes of joint pain is neces-
sary (Answers BE). Benign joint HMS is associated with 16. ANSWER:A. MEFV genetic testing
dysautonomia features, such as postural orthostatic tachy-
cardia syndrome (POTS). The presence of this condition The patient suffers from familial Mediterranean fever
would be supported by a positive tilt-table test (Answer E). (FMF), one of several hereditary periodic fever syndromes.
Treatment of joint complaints in patients with benign joint FMF most often results from a homozygous mutation of
HMS is challenging, but it should include primarily physi- the MEFV gene. This gene encodes the pyrin protein, which
cal therapy to improve muscle tone and fitness, while limit- is involved with leucocyte function. FMF is an autosomal
ing extreme range-of-motion exercises. Immunosuppressive recessive disorder, so a family history may or may not be
and anti-inflammatory therapies are of no value. Narcotics present. Clinical features of FMF include recurrent attacks

3 . R h e u m at o l o g y 67
characterized by fever and abdominal pain. Accompanying dermatomyositis, Sjogrens syndrome, and rheumatoid
features can include leukocytosis, thrombocytosis, rash, arthritis. The patient does not have other features to sug-
and inflammatory joint pain (hence this patients question- gest an ANCA-associated vasculitis. Further, Answer
able history of gout). Inflammatory markers are elevated A lacks the autoantibodies associated with this condition
during attacks, but relatively normal when asymptomatic. (anti-MPO and anti-PR3). Answer B would be seen in a
An important complication of FMF is secondary amyloi- patient with acute pro-myelocytic leukemia, which would
dosis, which this patient may have based on a history of not be consistent with this patients leukopenia and differ-
chronic untreated inflammation with unexplained protein- ential WBC. The patient does not have features of derma-
uria. This can be prevented (and to some degree reversed) by tomyositis (Answer D), which does not affect the kidney to
treatment with daily colchicine. The patient does not have any significant degree. Long-standing ankylosing spondyli-
features of celiac disease or Crohns disease; hence, endos- tis (Answer E) is associated with IgA nephropathy, but the
copy and colonoscopy are not relevant (Answers B and D). remainder of her clinical presentation does not suggest this
He is not experiencing recurrent mesenteric ischemia from diagnosis. Diagnosis of LN is confirmed by kidney biopsy.
vasculitis, and there is nothing specifically suggesting sys- Treatment of lupus nephritis includes high-dose corticoste-
temic vasculitis, so working up these conditions (Answers roids (1 mg/kg), slowly tapered over months. This patient
C and E) delays proper diagnosis. would likely receive intravenous cyclophosphamide as
additional therapy, but sterility is a potential complication
(more likely in women age >30).

17. ANSWER:A. Patellofemoral syndrome

This case describes a classical presentation for patellofem- 19. ANSWER:E. Treat with intramuscular
oral syndrome (PFS), a common mechanical disorder of methylprednisolone 80 mg 1.
young active adults. One would expect the pain of iliotibial
(IT) band syndrome to be more lateral. Physical examina- The patient is experiencing an attack of acute gouty arthri-
tion testing does not suggest the other options (Answers tis. While the presence of tophi would be an indication for
CE). McMurray test assesses for tear of the medial menis- antihyperuricemic therapy in the future, it is inappropriate
cus. Anterior drawer sign testing assesses for tear of the to initiate it at the time of an acute attack because sudden
ACL. Bakers cyst should be suspected in patients with pos- drops in uric acid often exacerbate gouty inflammation.
terior fullness and pain with knee flexion. Thus, Answers Aand B are not the next most appropriate
course of action. Three classes of medications are useful for
treating the inflammation of acute gouty arthritis:colchi-
cine, nonsteroidal anti-inflammatory drugs (NSAIDS),
18. ANSWER:C. Anti-double-stranded DNA and corticosteroids. The indicated course of Colchicine
antibody 522 U/mL (normal <5), C3 complement in Answer C is inappropriate for a patient with marginal
52mg/dL (normal 75175), C4 complement 8 mg/dL renal function. Very low doses of colchicine (0.3 mg every
(normal 1440) other day) might be tolerated as a prophylactic dose against
attacks, but this is unlikely to be very effective in an attack
The patient has systemic lupus erythematosus (SLE) com- of acute gout like this. Answer D is inappropriate because
plicated by lupus nephritis (LN), with associated symptoms NSAIDS are highly nephrotoxic in patients with marginal
and findings of renal failure (peripheral edema and con- renal function. Answer E is the correct answer because
gestive heart failure). This is supported by the presence of corticosteroids provide rapid relief of inflammatory pain
aggressive Raynauds phenomenon, leukopenia (and lym- in patients with acute gouty arthritis. Appropriate routes
phopenia), thrombocytopenia, hematuria, proteinuria, and of administration include intramuscular, intra-articular,
elevated ANA. SLE has myriad clinical features, and LN intravenous, and oral.
is an important one to recognize. The serologic workup of
SLE includes more than just the ANA, which is itself a non-
specific test. Patients with LN often (but not always) will
have hematuria and proteinuria, elevated anti-dsDNA anti- 20. ANSWER:C. Acute calcific tendonitis
bodies, and low C3 and C4 complement levels. Anti-Smith
antibody is a test specific for SLE. Anti-Ro and anti-La The patient describes rapid onset of rotator cuff pain the
antibodies are associated with SLE and Sjogrens syndrome. absence of antecedent symptoms or injury. Acute calcific
Anti-RNP antibody is associated with an overlap condi- tendonitis is a common condition in which basic calcium
tion sometimes termed mixed connective tissue disease phosphate crystals deposited along a tendon trigger an
(MCTD), in which there are features of SLE, scleroderma, immune response, with resultant acute inflammation and

6 8 T h e B r i g h a m I n t e n s i v e R e v i e w o f I n t e r n a l M e d i c i n e Q u e s t i o n a n d A n s w e r C o m pa n i o n
pain. It is often unheralded by any specific antecedent injury and E) are not necessary upon initial evaluation of this con-
of overuse. The physical examination does not suggest biceps dition. Physical therapy is appropriate (Answer A), and cor-
tendonitis (Answer A), since forward flexion of the shoul- ticosteroid injection can be considered. There is little role
der up to 90 degrees is unaffected. Septic arthritis (Answer for systemic corticosteroid treatment (Answer C).
B) would likely cause a toxic-appearing patient with severe
pain, especially after 10 days. Further, the examination
would be limited on active and passive range of motion.
Likewise, glenohumeral arthritis (Answer D) would limit 23. ANSWER:D. Lateral cervical spine X-ray
shoulder motion actively and passively. Finally, while the imaging, with flexion and extension views
examination suggests involvement of the supraspinatus ten-
don, complete tear is unlikely to explain acute-onset symp- This patient is demonstrating signs and symptoms of cervi-
toms in the absence of antecedent injury. If there is clinical cal myelopathy, likely as a complication of RA affecting the
uncertainty about the presence of a complete rotator cuff cervical spine. Atlanto-axial instability is a rare but serious
tear, the clinician can inject 3 mL of 2% lidocaine into the complication of RA, most often occurring in patients with
subacromial space. If this completely corrects the shoulder long-standing deforming disease. The best initial assess-
pain and functional deficit, then complete rotator cuff tear ment for this is lateral cervical spine X-ray imaging, with
is unlikely, as is septic arthritis or significant glenohumoral flexion and extension views (Answer D). MRI and/or CT
arthritis. scan imaging of the cervical spine might also be indicated.
Testing for RF and anti-CCP antibodies (Answer A) is
part of the diagnostic workup of inflammatory arthritis
and unnecessary in this case. ESR and CRP can correlate
21. ANSWER:C. Prescribe prednisone 1 mg/kg. Instruct with RA disease activity, but this does little to clarify the
the patient to start it immediately and to call with any new patients clinical presentation. It is unlikely that the patient
visual disturbances. Arrange for an expedited temporal would tolerate exercise stress testing (Answer C). RA gen-
artery biopsy first thing Monday morning. erally spares the lumbar spine, and lumbar spine disease
would not explain the patients upper-extremity symptoms.
The patient describes symptoms consistent with giant cell Hence, lumbar spine MRI (Answer E) is not appropriate.
arteritis (GCA). Adreaded complication of GCA is blind-
ness, which results from ischemic optic neuropathy from
vasculitis involving branches of the ophthalmic and reti-
nal arteries. If blindness develops, it is often irreversible. 24. ANSWER:B. Hepatitis C antibody
Thus, empiric treatment with high-dose corticosteroids is
indicated in patients with possible GCA, while arrange- The patient has chronic hepatitis C infection complicated
ments are made for expedited temporal artery (TA) biopsy by cirrhosis and cryoglobulinemic vasculitis. As in this
to confirm the diagnosis. Only Answer C addresses these case, cryoglobulinemic vasculitis can cause palpable pur-
issues fully. All other answers risk irreversible blindness. Of pura, glomerulonephritis, and mononeuritis multiplex (the
note, once empiric corticosteroids are started for suspected foot drop). Bowel vasculitis, pulmonary hemorrhage, and
GCA, a TA biopsy should be performed within 710days cerebral vasculitis are rare complications. Immunologic dis-
if possible, to avoid a false negative biopsy. Regardless, turbances in patients with hepatitis C include detectable
10%15% of patients with GCA will still have a negative rheumatoid factor (RF), normal C3 complement, and dis-
TA biopsy. In these cases, one should consider alternative proportionately low C4 complement. The patient does not
diagnoses or event contralateral TA biopsy, which will pick describe or have symptoms of rheumatoid arthritis (RA);
up another 5%10% of cases of GCA. Finally, it is impor- hence, testing for anti-CCP antibodies (Answer E) is unnec-
tant to remember that GCA rarely if ever affects individuals essary at this time. The remaining tests (Answers A, D, and
younger than age 50, and other explanations for headache E) are notoriously nonspecific, with many false-positive
and scalp tenderness should be sought in these patients. tests.

22. ANSWER:A. Reassurance and physical therapy, 25. ANSWER:E. Referral to Orthopedics for
focusing on posture, core muscle tone, and hip range of consideration of right total knee arthroplasty
motion
This patient has end-stage osteoarthritis (OA) of the right
The patient has trochanteric bursitis based on clinical eval- knee. He would benefit from consideration of right total
uation. Further diagnostic imaging studies (Answers B, D, knee arthroplasty (TKA). He has been through conservative

3 . R h e u m at o l o g y 6 9
measures, including physical therapy and corticosteroid injec- trochanteric bursa at the lateral hip. It does not impair pas-
tions, without much benefit. Thus, advising him to do this sive hip range of motion, but it is not associated with pares-
again (Answer A) will likely be of little benefit. Narcotic-based thesias or numbness.
therapies might be useful for a short-term bridge to get him
to Orthopedics, but not as a stand-alone treatment. There
is little role for MRI of the knee or arthroscopic debride-
ment in patients with advanced knee OA; these choices do 28. ANSWER:D. Recommend combination of
not change the ultimate need for TKA. Decision making physical therapy, sleep routine, weight loss, and
for timing of TKA is made on a case-by-case basis. Pain and psychiatric counseling.
functional limitation are the driving forces behind optimal
timing of referral to Orthopedics. Patients should experience The patient has fibromyalgia syndrome (FMS), a noninflam-
severe enough pain that they will derive benefit from joint matory, nonstructural chronic dysfunctional pain syndrome
replacement, but they should not be so debilitated that they of muscles, bones, and joints. Her history, examination, and
miss a window in which optimal recovery is possible. Indeed, laboratory results are not consistent with an inflamma-
patients who are wheelchair bound or similarly disabled will tory joint disorder. With more than 20years of symptoms,
not typically derive the same benefit as patients able to main- one would expect significant abnormalities if she had an
tain muscle mass through active participation in periopera- inflammatory, infectious, or neuromuscular process present.
tive physical therapy. Thus, EMG/NCS and serologic testing for connective tis-
sue diseases (Answers Aand B) are inappropriate. Likewise,
tick-borne illnesses like Lyme disease (Answer C) do not
explain her constellation of symptoms. The history and
26. ANSWER:C. Chondrocalcinosis of the wrist examination describe hyperesthesia with allodynia (tender-
ness to normally nonpainful stimulus), common features in
Carpal tunnel syndrome (CTS) is most commonly the result patients with FMS. For the most part, narcotic-based ther-
of an overuse syndrome of the fingers. Flexor tendons swell apies are inappropriate with patients with FMS. They can
slightly in response to overuse. This compresses the median lead to tolerance, dependency, dose escalation, and some-
nerve in the carpal tunnel, resulting in a median nerve neu- times frank addiction. Rather, patients should be encour-
ropathy. Severe cases can lead to thenar muscle atrophy. aged to address lifestyle issues that are often associated with
Electromyogram with nerve conduction studies can be used FMS: physical inactivity, obesity, sleep disturbances, and
to determine the severity of median nerve deficit. Other underlying depression and anxiety (Answer D). It would not
conditions that cause swelling of wrist flexor tendons (or the be inappropriate to consider pharmacotherapy for FMS if a
median nerve itself) can increase the risk of carpal tunnel patient is unable to address these issues.
syndrome: acromegaly, scleroderma, rheumatoid arthritis,
and hypothyroidism. Chondrocalcinosis commonly affects
the wrists in older individuals, but it is not associated with
CTS. It can lead to (1)acute flares of pseudogout arthritis, 29. ANSWER:E. No additional workup necessary
(2) chronic inflammatory pseudo-rheumatoid arthritis,
or (3)accelerated degenerative pseudo-osteo arthritis. This patient describes Raynauds phenomenon:cold-induced
vasospasm of the digits with triphasic color changes. Most
patients, including this one, have primary Raynauds, indi-
cating an absence of any underlying connective tissue disor-
27. ANSWER:C. Entrapment neuropathy of the left der (CTD). Patients have benign symptoms and findings,
lateral femoral cutaneous nerve and no further workup is indicated (Answer E). Secondary
Raynauds is the term used to describe Raynauds occur-
This patient is experiencing meralgia parestheticaan ring in the setting of obvious cause. These include vascular
entrapment neuropathy of the lateral femoral cutane- trauma or malformation (Answer A), micro-angiopathic
ous nerve. Risk factors include abdominal obesity and injury (e.g., antiphospholipid syndrome, chemotherapy),
tight-fitting belts/corsets. Diabetic amyotrophy (Answer A) and the various CTDs: systemic lupus erythematosus
is a lumbosacral plexopathy that occurs in diabetic patients (Answer A), scleroderma (Answer B), Sjogrens syndrome,
even with good blood sugar control. It causes pain, weak- rheumatoid arthritis, dermatomyositis, and mixed connec-
ness, and muscle atrophy of the anterior thigh. Avascular tive tissue disorders. In this case, the patient describes no
necrosis (AVN) and degenerative arthritis (osteoarthri- symptoms of these other disorders, and the examination
tis) of the hip joint usually localizes as pain in the groin does not suggest them. Thus, an exhaustive serologic and
or deep buttock. Hip range of motion is often impaired. imaging workup for benign primary Raynauds syndrome is
Trochanteric bursitis (Answer D) causes pain overlying the unnecessary and costly. Factors that can exacerbate primary

70 T h e B r i g h a m I n t e n s i v e R e v i e w o f I n t e r n a l M e d i c i n e Q u e s t i o n a n d A n s w e r C o m pa n i o n
or secondary Raynauds attacks include caffeine ingestion, flexion will cause excruciating pain because of increased
amphetamine usage, anxiety, over-the-counter cold medica- pressure within the inflamed bursa. There is nothing to
tions containing pseudoephedrine, and cocaine abuse. suggest active inflammation of the knee from gout or sepsis
(Answers B or C). Typically, patients with these conditions
will have buildup of inflammatory fluid, and the joint will
be partly flexed to alleviate buildup of intra-articular pres-
30. ANSWER:B. Captopril 25 mg orally three times sure. The examination also does not suggest meniscal injury
daily, with dose escalation to control blood pressure (reproduced with a positive McMurrays test) or quadri-
ceps tendon rupture (which prevents extension of the knee
This patient is experience a scleroderma renal crisis (SRC). against resistance).
Evidence for this is the presence of scleroderma, hyperten-
sion, renal dysfunction, and evidence of microangiopathic
hemolytic anemia (MAHA). Other causes of MAHA
include antiphospholipid antibody syndrome, dissemi- 33. ANSWER:A. Staphylacoccus aureaus
nated intravascular coagulation (DIC), hemolytic-uremic
syndrome (HUS), thrombotic thrombocytopenic purpura Reactive arthritis typically presents as abrupt onset pain
(TTP), hypertensive emergency, hemolytic anemia/elevated and swelling in multiple small joints. It is associated with
liver studies/low platelets (HELLP) syndrome of pregnancy, various infectious agents, including Streptococcus pyogenes,
certain malignancies, and graft-versus-host disease. SRC Clostridium difficile, Chlamydia trachomatis, Yersinia pes-
represents a hyper-renin state, leading to ischemic injury to tis, Salmonella spp., Campylobacter spp., and Shigella spp.
the nephron. Even modest degrees of hypertension (as in Streptococcus aureus is a common infectious agent, but it is
this case) can lead to rapid renal demise. Thus, despite rising not associated with reactive arthritis.
creatinine levels, treatment with angiotensin-converting
enzyme inhibitors (ACEi) must be rapidly initiated, with
the goal of normalizing blood pressure. Prior to the recogni-
tion of ACEi therapy for SRC, this condition carried a grim 34. ANSWER:B. Plantar fasciitis
prognosis, accounting for a sizeable fraction of mortality in
patients with scleroderma. Other antihypertensive agents This patient has a clinical presentation consistent with plan-
(Answers C and D) might actually mask the renal injury tar fasciitis. This condition causes focal pain at the origin of
and are inappropriate. Plasma exchange (Answer A) would the plantar fascia as it comes off of the plantar calcaneus.
be indicated for patients with TTP, but it is ineffective in Obesity is a risk factor. Stress fracture (Answer A) could still
SRC. Corticosteroids (Answer E) are used for treatment of be present even with normal radiographs, but this patients
glomerulonephritis. They are counterproductive and poten- examination does not suggest involvement of the fifth
tially catastrophic in patients with SRC. metatarsal bone. Acute gouty arthritis (Answer C) would
be associated with inflammatory findings of erythema,
warmth, and swelling. Charcot arthropathy (Answer D) is
a complication of sensory deprivation of the extremity, such
31. ANSWER:D. Levofloxacin as occurs in diabetic neuropathy or tabes dorsalis (syphi-
lis). The anterior talofibular ligament (ATFL) is commonly
The patient has experienced quinolone-associated rupture injured in forced inversion injury of the ankle. Pain local-
of the Achilles tendon. This is a rare but serious adverse izes laterally and not to the plantar foot. Treatment of plan-
reaction to fluoroquinolone-class antibiotics. Classic fea- tar fasciitis is first conservative, with stretching, weight loss,
tures include altered gait because of impaired plantar flex- and nonsteroidal anti-inflammatory drugs. If these fail, one
ion from Achilles tendon rupture. The Achilles tendon is could consider corticosteroid injection.
intact with a normal Thompson test; squeezing the calf
should normally cause dorsiflexion at the ankle. The other
antibiotics are not associated with this complication.
35. ANSWER:E. Yellow fever vaccine

This patient is immune suppressed because of her


32. ANSWER:A. Gouty bursitis of the right disease-modifying antirheumatic drug (DMARD) ther-
prepatellar bursa apy. The yellow fever vaccine is a live virus and there-
fore contraindicated in patients on biologic DMARDs.
The physical examination suggests an acute inflammatory Administering it to her is dangerous and could prove fatal.
bursitis of the prepatellar bursa. With this condition, knee Neither ciprofloxacin nor chloroquine is contraindicated

3 . R h e u m at o l o g y 71
by her RA treatment regimen (Answers Aand B). Indeed, emergency. Other red flags include fever, occult weight loss,
patients receiving hydroxychloroquine for treatment of RA history of malignancy, and recent intravenous drug abuse.
are protected from most forms of malaria when traveling. These findings should prompt consideration of inflamma-
The patient is well advised to receive hepatitis Aand B vac- tory, infectious, or malignant etiologies of back pain.
cinations (Answers C and D), as well as pneumococcal vac-
cine and parenteral typhoid vaccine. Live-virus vaccines are
contraindicated. In addition to yellow fever vaccine, these
include varicella vaccines (including to prevent zoster), 38. ANSWER:C. Hyperuricemia
oral polio vaccine, intranasal influenza vaccine, measles/
mumps/rubella (MMR) vaccine, and oral typhoid vaccine. Hyperuricemia is not a direct consequence of corticosteroid
usage. Obesity, diabetes mellitus, and hypertension are all
side effects of corticosteroids that are themselves risk fac-
tors for hyperuricemia. These agents impair the immune
36. ANSWER:B. Chest radiograph system (Answer A), are a cause of cataracts even in young
patients taking them (Answer B), can precipitate avascular
This patient describes classic features of Lofgrens syn- necrosis of bone (Answer D), and cause a wasting steroid
drome:ankle periarthritis and erythema nodosum. Chest myopathy that typically presents as leg weakness in the hip
radiograph will often demonstrate hilar adenopathy, and flexors (Answer E).
biopsy of one of these lymph nodes will show noncaseat-
ing granulomatous inflammation consistent with sarcoid-
osis. This variant of sarcoidosis has an excellent prognosis.
Patients will almost universally recover fully and not experi- 39. ANSWER:A. Morning stiffness
ence other organ-specific manifestations of sarcoid disease.
Associated features include oral contraceptive pill usage, The most likely diagnosis is polymyalgia rheumatica (PMR),
antecedent streptococcal pharyngitis, inflammatory bowel without features to suggest giant cell arteritis (GCA).
disease, and gastrointestinal dysentery (specifically Yersinia Morning stiffness (Answer A) is commonly reported in
species). Ankle film (Answer A) will not add to the clini- patient with PMR. Elderly patients can have incidental
cal impression that there is ankle swelling. Serologic testing findings of rheumatoid factor, chondrocalcinosis, and joint
for RA or Lyme disease (Answers C or E) is not necessary space narrowing (Answers CE), but these are not features
because the clinical assessment does not suggest inflamma- of PMR. Blindness is a feared complication of GCA, but
tory arthritis of the ankles; normal passive range of motion the patient denies other symptoms of GCA: headaches,
of the joint argues for a periarticular process and not articu- visual disturbances, scalp tenderness, and jaw achiness after
lar disease. Further, Lyme arthritis is normally monoarticu- chewing (jaw claudication).
lar. Finally, the clinical assessment is not consistent with
gouty arthritis (Answer D), a condition unlikely to affect
a premenopausal woman without any specific risk factors.
40. ANSWER:B.Adult-onset Stills disease

This patient demonstrates many criteria that support a


37. ANSWER:E. Referral to the Emergency diagnosis of adult-onset Stills disease (AOSD):pharyngi-
Department for urgent MRI of the lumbar spine tis, fever, rash, inflammatory arthritis, leukocytosis, throm-
bocytosis, hepatosplenomegaly, lymphadenopathy, and
This patient is experience cauda equina syndrome: com- elevated inflammatory indices. Most important, an exten-
pression of the cauda equina from a paraspinal source. In sive workup for other potential etiologies for her condition
this case, a herniated disc has obliterated the spinal canal, has been negative, including serologic studies. One addi-
impairing function of sacral nerves. The patient reports tional useful test is the serum ferritin level, which can some-
symptoms that could be mistaken for an S1 radiculopathy times be strikingly elevated (>10,000g/mL). The clinical
alone: loss of plantar flexion, absent Achilles reflex, and scenario is not that of Lyme infection (Answer A). Behcets
reduced posterior thigh sensation. However, he also has red disease (Answer E) is a syndrome that includes oral ulcers,
flag features of back pain to prompt evaluation for cauda genital ulcers, and uveitis. Associated features include
equina syndrome:sensory loss in the perineum, impotence, fever, pyoderma gangrenosum, vasculitis, and pathergy
urinary retention, and bowel retention. If not identified (neutrophilic pustule formation at the site of innocuous
and corrected urgently, sacral nerve deficits can become skin perforation such as a sterile needle). Acute bacterial
long-standing. The remaining options (Answers AD) endocarditis (Answer D) is excluded by the negative blood
result in delay of diagnosis and miss this neurosurgical cultures, nontoxic appearance, and time course of 3 weeks

7 2 T h e B r i g h a m I n t e n s i v e R e v i e w o f I n t e r n a l M e d i c i n e Q u e s t i o n a n d A n s w e r C o m pa n i o n
duration. Systemic lupus erythematosus (SLE, Answer C) and corticosteroid injections. His physical function is com-
could explain her clinical presentation. However, it is essen- promised, and he is experiencing weakness of hip flexor
tially ruled out by the negative ANA test, which has excel- muscles. Repeating these interventions (Answers A, B, and
lent negative predictive value. Unlike this case, SLE is more D) is unlikely to provide much relief. Repeat MRI of the
often associated with leukopenia and thrombocytopenia. lumbar spine is unlikely to reveal any new pathology, and
he does not describe any red flags of back pain that would
prompt workup for inflammatory, infectious, or neoplastic
processes. Referral to Spine Surgery is appropriate, since
41. ANSWER:A. Sjogrens syndrome decompressive laminectomy can alleviate symptoms and
improve leg weakness.
The patient likely has circulating anti-Ro (anti-SSA) anti-
bodies, which can cross the placenta and cause a neonatal
anti-Ro syndrome characterized by fetal heart block and
photosensitive rash (similar to subacute cutaneous lupus of 43. ANSWER:D. Temporal artery biopsy showing
adults). Besides Sjogrens syndrome, this antibody can be granulomatous arteritis
found in patients with systemic lupus erythematosus (SLE)
or incidentally in healthy mothers. The rash is self-limited The patient is experiencing a pulmonary-renal syn-
and only of cosmetic concern. The congenital heart block, drome:pulmonary hemorrhage from diffuse aveolitis and
however, is often irreversible. It represents failure of for- crescentic glomerulonephritis. These syndromes result from
mation of the conductive system of the heart during the small-vessel vasculitis in the lung and kidney. Inflammatory
early second trimester of pregnancy. Thus, careful monitor- indices are elevated:ESR, CRP, thrombocytosis, and ane-
ing should be performed starting around week 15 for any mia of chronic disease. Of all answers, only giant cell
pregnant woman with known anti-Ro antibody positivity, arteritis/temporal arteritis (Answer D) is not associated
including careful monitoring of fetal heart rate. Affected with small-vessel vasculitis of this nature. The serologic
patients would benefit from having high-risk obstetrics studies indicate systemic lupus erythematosus (SLE)
follow the pregnancy. Primagravida women with anti-Ro (Answer A), Goodpastures syndrome (Answer B), and
antibodies carry an approximately 5% chance of the first ANCA-associated vasculitis (Answer C). Levamisole is an
pregnancy being affected by congenital heart block. In sub- antihelminthic agent used to cut cocaine. In some indi-
sequent pregnancies, this risk is near zero for women who viduals, it can induce an ANCA-associated vasculitic condi-
had unaffected first pregnancies. However, the risk is much tion clinically indistinguishable from granulomatosis with
higher for those women in whom their first pregnancy was polyangiitis (GPA), formerly Wegeners granulomatosis.
affected by fetal heart block. Hydroxychloroquine appears
to be effective in reducing the rate of fetal heart block in
at-risk mothers. Once detected, however, fetal heart block
is not always reversible; no treatments have been shown to 44. ANSWER:C. Discuss with patients nephrologist
be highly effective for reversing conductive abnormalities. about alternative immunosuppressive therapies for the
Treatment options include high-dose betamethasone or renal graft.
intravenous immunoglobulin. None of the other answers
are associated with fetal heart block. Both Lyme disease The patient is currently taking azathioprine (AZA), an
(Answer A) and acute rheumatic fever (Answer D) can inhibitor of purine synthesis. AZA metabolites are further
cause heart block in the affected individual. Marfan syn- metabolized by xanthine oxidase (XO), which normally
drome (Answer C) can complicate pregnancies because converts purine metabolites into uric acid. While XO
of maternal aortic dissection. Methotrexate (Answer E) is inhibitors like allopurinol and febuxostat normally reduce
highly teratogenic to the growing fetus, which often does uric acid formation, they also lead to accumulation of AZA
not survive in utero exposure. Many severe birth defects are (and toxicity), unless the AZA dose is very much reduced.
described (including neural tube defects) but not congenital Thus, Answers A and E are incorrect. Answer B is incor-
heart block. rect because probenecid increases the risk of nephrolithia-
sis, a potentially devastating complication in patients with
renal transplants. Answer D does not solve the underlying
problem of hyperuricemia with tophaceous gouty arthritis.
42. ANSWER:E. Referral to Spine Surgery for This patient is young and likely to experience complications
decompressive laminectomy of the numerous tophi such as a destructive arthropathy
or skin breakdown complicated by infections like osteo-
The patient has lumbar spinal stenosis that is refractory to myelitis. Thus, the correct answer is C, with a discussion
conservative interventions of physical therapy, analgesics, about alternatives to AZA so that the patient might benefit

3 . R h e u m at o l o g y 7 3
from the addition of allopurinol or febuxostat to prevent 47. ANSWER:E. Adalimumab 40 mg every other
long-term complications of gouty tophi. Note that calci- week
neurin inhibitors like cyclosporine increase serum uric acid
levels but are not contraindicated in conjunction with XO Ankylosing spondylitis (AS) is one of several spondyloar-
inhibitors. thropathies, which also include psoriatic arthritis, reac-
tive arthritis, and inflammatory bowel diseaserelated
arthropathy (IBD arthropathy). This patient describes
years of inflammatory back pain, worse in the morning
45. ANSWER:B. Aspiration of the right knee for and improved with activity. Symptoms of AS usually start
fluidanalysis in the sacroiliac joints and slowly progress up the spine.
Thoracic spine involvement can affect chest excursion. AS
The patient may have septic arthritis of the right knee. is associated with unilateral anterior uveitis. Inflammatory
Aspiration with fluid analysis (Answer B) is the only option markers are often (but not always) elevated. The diagno-
that can address that concern properly. In patients who sis is made based on symptoms of inflammatory back pain
present with an acute inflammatory monoarthritis, septic and supportive imaging. Plain film radiographic changes
arthritis should always be high on the differential diagno- are described in this case. MRI is more sensitive and
sis. This is particularly true when a patient with RA under specific for sacroiliitis. Testing for HLA-B27 haplotype
good control presents with a single inflamed joint. Patients (Answer A) is sometimes used to stratify the likelihood of
with RA are at increased risk for septic arthritis for many AS, but it is not diagnostic. Upward of 10% of white indi-
reasons: increased vascularity of inflamed joints, immu- viduals carry HLA-B27, such that >95% of these people
nosuppressive qualities of DMARD therapies, underlying do not have AS. For years, treatment of AS patients with
structural joint disease, and frequent presence of pros- inflammatory spine pain was limited to cyclo-oxygenase
thetic joints. Septic arthritis occurs most commonly from (COX) inhibitors: nonsteroidal anti-inflammatory drugs
hematogenous seeding of the joint from bacteremia; thus, a (NSAIDs). Corticosteroids, methotrexate, and even
source must be sought for any joint infection. Joint aspira- opiate-based therapies (Answers BD) are relatively inef-
tion with steroid injection (Answer C) misses that this is a fective or not sustainable. Sulfasalazine has a role in the
potential joint infection and delays diagnosis. Likewise, the treatment of peripheral arthritis, but it is of no value in
other answers delay diagnosis of a potentially serious septic relieving spine pain. Available since the late 1990s, antago-
arthritis. nists of tumor necrosis factor alpha (TNF) are extremely
effective in relieving the symptoms of inflammatory
back pain in patients with AS and other forms of spon-
dyloarthritis. Five TNF antagonists are currently avail-
46. ANSWER:E. Flexible bronchoscopy able: adalimumab, certolizumab, etanercept, golimumab,
and infliximab.
This patient is presenting with diffuse alveolar hemor-
rhage (DAH), a potentially life-threatening complica-
tion of SLE. Diagnosis is made by flexible bronchoscopy
(Answer E), which also allows for bronchial lavage to test 48. ANSWER:A. Parotid lymphoma
for infectious microorganisms. The most common features
of DAH are dyspnea, hypoxemia, and unexplained drop in Patients with Sjogrens syndrome have a very high risk of
hemoglobin (>2 g/dL). Fever is common, but hemoptysis developing lymphoma in the mucosa-associated lymphoid
is reported in only 50% of cases. Thus, as in this case, the tissue (MALT) of the parotid and submandibular glands,
absence of hemoptysis does not exclude DAH. Missing upward of 50100 times the normal population. One eti-
DAH can have dire consequences; the mortality rate of ology may be that chronic inflammation in these tissues
SLE-related DAH approaches 50%. CT scan with iodin- leads to malignant degeneration of abnormal autoimmune
ated contrast (Answer A) risks contrast-induced nephrop- lymphocytes. Knowing this risk is important when evalu-
athy, and MRI with gadolinium contrast (Answer D) ating asymmetric glandular swelling in a patient with SjS.
risks nephrogenic systemic fibrosis. While pulmonary Laboratory features associated with the presence of these
function tests (PFTs) might show elevated DLCO in the lymphomas include vasculitic rash, dropping comple-
presence of alveolar blood (Answer B), the patient will ments, and normalization of a previously abnormal RF.
be unable to comply with PFTs while intubated. Lumbar Imaging and surgical referral for biopsy are indicated.
puncture (Answer C) would be indicated in a patient such Sarcoidosis (Answer B) can affect the lacrimal and salivary
as this whose primary complaint is delirium without iden- glands, often in association with fever (uveoparotid fever
tifiable cause, since immunosuppression raises the risk for or Heerfordts syndrome). Sialolithiasis and suppurative
infection. bacterial sialadenitis (Answers C and D) can both occur

74 T h e B r i g h a m I n t e n s i v e R e v i e w o f I n t e r n a l M e d i c i n e Q u e s t i o n a n d A n s w e r C o m pa n i o n
in patients with SjS, but these conditions are almost always 50. ANSWER:A. Rheumatoid arthritis
acute and associated with pain. Ten years of bisphospho-
nate therapy increases this patients risk for osteonecro- Rheumatoid arthritis (RA) is the most common chronic
sis of the jaw (Answer E). However, that condition is not inflammatory polyarthritis affecting either gender or any age
suggested by the history or physical examination, which group. It is characterized by redness, warmth, and swelling of
describes extra-oral swelling of the parotid gland. the joints, often accompanied by morning stiffness. Almost
any joint can be involved with the general exception of the
thoracolumbar spine and distal interphalangeal (DIP) joints.
Approximately 25% of patients with RA do not have detect-
49. ANSWER:E. Nephrogenic systemic fibrosis able RF or anti-CCP antibodies (as in this case). While these
patients often have a less severe disease course, they can still
Nephrogenic systemic fibrosis (NSF) is a complication of develop joint damage and erosions (as noted in the films in this
gadolinium exposure that occurs in patients with severely case). Inflammatory indices are often elevated, but these are
compromised renal function:acute kidney injury, stage 4 nonspecific. Several features of this case argue against chronic
or 5 chronic kidney disease, or dialysis therapy. Retained gouty arthritis (Answer B). He reports no history of episodic
gadolinium induces a fibrotic reaction in deep dermal tis- acute gouty arthritis, and crystals were not observed in the
sues and sometimes in internal organs. This case describes a joint fluid aspirate. The relatively normal uric acid does not
classic presentation of NSF, including the yellowish scleral completely exclude gout, but the value of 4.0 mg/dL makes it
plaques and joint contractures from fibrosis of periarticu- unlikely that the patient suffers from a chronic, erosive gouty
lar tissues. Scleroderma (Answer A) is highly associated arthropathy. Primary osteoarthritis (Answer C) rarely affects
with Raynauds phenomenon and often affects the face, the MCP, wrist, or elbow joints. Rheumatic fever (Answer D)
which is spared in NSF. Lipodermatosclerosis (Answer can cause arthalgias and lead to joint deformities. Patients can
B) is a gravity-dependent complication of chronic venous experience a noninflammatory joint laxity (Jaccoud arthropa-
stasis that affects the lower extremities only and usually thy), similar to that found in patients with systemic lupus ery-
does not cause joint contractures. Scleredema diabetico- thematosus. However, the patient also lacks the other clinical
rum (Answer C) is a fibrotic reaction of dermal tissue that features of rheumatic fever, including the fever, rash, pharyngi-
occurs in diabetic patients. Unlike NSF, it predominantly tis, and cardiac complications. Finally, polymyalgia rheumatic
affects the upper torso such as the shoulder and upper (PMR) is a seronegative inflammatory disorder of individuals
back. Eosinophilic fasciitis is an inflammatory disorder of >50years old (Answer E). It primarily affects the shoulders and
fascia (not dermis) caused by aberrant eosinophilic infil- hips, as in this case. However, PMR is not considered a cause
tration, leading to edema of the affected extremity. It is of erosive inflammatory polyarthritis of the extremities, as in
not associated with renal dysfunction but patients may this case. Thus, although in some ways a diagnosis of exclusion,
have an underlying myelodysplastic syndrome. seronegative RA is the most appropriate diagnosis in this case.

3 . R h e u m at o l o g y 75
4.
PULMONARY A ND CR ITICA L CAR E MEDICINE

Michael H. Cho and Christopher H. Fanta

1. Which of the following serious adverse outcomes D. The majority of persons with alpha-1 antritrypsin
has led to a black-box warning regarding the use of deficiency develop clinically significant liver disease
long-acting inhaled beta-agonist bronchodilators in at some point during their lifetime.
the treatment of asthma? E. Acharacteristic feature that distinguishes alpha-1
antitrypsin deficiency from emphysema of other
A. Atrial fibrillation and flutter causes is a preserved diffusion capacity for carbon
B. Ventricular tachycardia and fibrillation monoxide (DLCO) in alpha-1 antitrypsin deficiency.
C. Myocardial infarction and stroke
D. Respiratory failure and death 4. A 45-year-old woman with asthma reports that she
E. Angioedema and anaphylaxis has experienced chest tightness and wheezing approxi-
mately 30 minutes after ingestion of aspirin 81 mg.
2. Which of the following pulmonary function test Which of the following is an appropriate response:
results, recorded at a time when the patient is free of an
exacerbation, is typical of severe chronic obstructive A. Your symptoms are not likely to have been caused
pulmonary disease (COPD)? by aspirin at this low dose.
B. You are allergic to aspirin and should instead
A. FEV1 2.0 L (66% of predicted), FVC 4.0 L (100% of use ibuprofen or naproxen for pain relief with
predicted), FEV1/FVC 0.5 (66% of predicted) anti-inflammatory activity.
B. FEV1 1.0 L (33% of predicted), FVC 3.0 L (75% of C. Because cross-sensitization is common, you should
predicted), FEV1/FVC 0.33 (44% of predicted) avoid peanuts and tree nuts such as cashews and
C. FEV1 1.0 L (33% of predicted), FVC 1.25 L (31% of almonds.
predicted), FEV1/FVC 0.8 (107% of predicted) D. Given your history, a leukotriene modifier such
D. FEV1 3.0 L (100% of predicted), FVC 4.0 L (100% as montelukast (Singulair), zafirlukast (Accolate),
of predicted), FEV1/FVC 0.75 (100% of predicted) or zileuton (Zyflo) might be particularly helpful as
E. FEV1 2.0 L (66% of predicted), FVC 2.67 L (67% of treatment for your asthma.
predicted), FEV1/FVC 0.75 (100% of predicted) E. Given your history, a long-acting beta-agonist
bronchodilator such as formoterol (Foradil) or
3. Which of the following statements about homozy- salmeterol (Serevent) might be particularly harmful
gous alpha-1 antitrypsin deficiency is true? as treatment for your asthma.

A. The diagnosis is suggested by the presence of 5. The BODE index can be used to predict mortality in
widespread emphysema on chest computed patients with COPD. It includes scores for body mass
tomography (CT) scan in a nonsmoker. index, severity of airflow obstruction, severity of dys-
B. The diagnosis is established if the serum alpha-1 pnea with routine activities, and exercise capacity as
antitrypsin level is less than the lower limits of the measured by the 6-minute walking distance. Taking
normal range. into account the BODE score and other indices of sur-
C. All patients with alpha-1 antitrypsin deficiency vival in COPD, which of the following fails to predict
develop obstructive lung disease. lower survival in COPD?

76
A. Low FEV1
B. Obesity
C. Hospitalization for exacerbation of COPD within
the previous 6months
D. Poor exercise capacity
E. Dyspnea on routine activities of daily living

6. A45-year-old man presents with a productive cough,


fever, and left-sided chest pain of approximately 1
weeks duration. Physical examination reveals dullness
to percussion and decreased breath sounds throughout
the lower half of the left chest posteriorly. The white
blood cell count is 17,500/L with 85% polymorpho-
nuclear leukocytes and 6% band forms. The chest
X-ray shows a moderate-to-large left pleural effusion.
Sputum Gram stain shows many polys with mixed flora,
including gram-positive cocci, gram-positive rods, and
gram-negative rods. Sputum culture grows only nor-
mal oral flora after 48 hours. Thoracentesis succeeds in
withdrawing only 60 mL of serous fluid. Pleural fluid
analysis reveals a high total protein concentration (4 g/
Figure 4.1 Chest computed tomography (CT) scan showing linear
dL), white blood cells 6,500/L with 80% polys, and no opacities with traction bronchiectasis and areas of honeycombing for
organisms on Gram stain or bacterial culture. Pleural patient in Question 8.
fluid pH is 7.10; pleural fluid amylase is low. The patient
is started on azithromycin and cefotaxime in the emer- for hypertension, aspirin 81 mg, and a multivitamin. On
gency department but fails to improve. You recommend examination he has early clubbing and high-pitched inspi-
which of the following actions? ratory crackles throughout the lower lung zones posteri-
orly. Chest X-ray reveals linear and small nodular opacities
A. Discontinuation of azithromycin and cefotaxime (reticulonodular pattern) predominantly in the lower
and initiation of levofloxacin lung zones. Spirometry suggests a moderate restrictive pat-
B. Chest tube drainage of pleural space, with tern. Chest CT scan with high-resolution images reveals
thoracoscopic lysis of pleural adhesions if necessary linear opacities, most prominent in the lung periphery,
C. Pleural biopsy with traction bronchiectasis and areas of honeycombing
D. Barium swallow to evaluate for esophageal rupture at the lung bases; there are no areas with ground-glass
(Boerhaaves syndrome) opacities (Figure 4.1). Fiberoptic bronchoscopy is per-
E. Systemic corticosteroids for probable inflammatory formed with normal visualized tracheobronchial mucosa.
pleuritis related to collagen-vascular disease. Analysis of bronchoalveolar lavage fluid shows no malig-
nant cells and no organisms. Transbronchial lung biopsies
7. Thoracentesis of a moderate-sized pleural effusion reveal nonspecific inflammatory changes in the intersti-
reveals 600 mL of milky-appearing fluid with a triglyc- tium, with collagen deposition (fibrosis) and increased
eride concentration of 145 mg/dL. Which of the follow- numbers of alveolar macrophages.
ing is the most likely to cause this effusion: Based on this clinical information you suspect a
diagnosis of:
A. Lymphangioleiomyomatosis (LAM)
B. Parapneumonic effusion A. Sarcoidosis
C. Malignant effusion B. Nonspecific interstitial pneumonia
D. Pulmonary infacrtion C. Idiopathic pulmonary fibrosis
E. Sarcoidosis D. Cryptogenic organizing pneumonia (COP)
E. Langherans cell histiocytosis
8. An 82-year-old man presents with a 6-month history of
nonproductive cough, gradually increasing exertional dys- 9. Which of the following test results would not be
pnea, and no fever, chest pain, or hemoptysis. He has no expected in the patient described in Question8?
history of collagen-vascular disease. He continues to work
as an accountant and has had no unusual exposures in his A. Normal oxygen saturation at rest that falls to 82%
home or work environments. He takes hydrochlorothiazide with exertion

4 . P u l m o n a r y a n d C r i t i c a l C a r e M e d i c i n e 7 7
B. Resting hypoxemia without change with exertion reports excessive daytime sleepiness. His examination
C. Reduced diffusing capacity for carbon monoxide is remarkable for his morbid obesity (body mass index
(DLCO) [BMI] = 42 kg/m2), narrowed posterior pharyngeal
D. Reduced lung static compliance opening, and clear chest on auscultation. His hemato-
E. Reduced total lung capacity (TLC), functional crit is 52%. Chest X-ray is normal. Spirometry identifies
residual capacity (FRC), and residual volume (RV) a pattern suggesting mild restriction. Arterial blood
on full measurement of lung volumes gases reveal PO2=55mm Hg, PCO2=72mm Hg, and
pH=7.32. Continuous positive airway pressure (CPAP)
10. Which of the following treatments would you rec- fails to correct his nocturnal hypoxemia and daytime
ommend for the patient described in Question8? hypersomnolence.
Which of the following would be an expected finding
A. High-dose oral steroids in his evaluation?
B. High-dose oral steroids plus azathioprine (Imuran)
C. Rituximab (Rituxan) A. Normal ventilatory response to carbon dioxide
D. Hydroxychloroquine (Plaquenil) B. Orthodeoxia
E. None of the above C. Central apneas
D. Restless legs syndrome
11. Which of the following attributes is a risk factor for E. Prolonged sleep latency period
obstructive sleep apnea?
15. Which of the following treatments would you rec-
A. Retrognathia ommend for the patient described in Question 14?
B. Dress shirt neck circumference >15inches
C. Nasal polyps with complete nasal airflow A. Uvulopalatopharyngoplasty and jaw advancement
obstruction surgery
D. Short stature (<60inches tall in women, <65inches B. Bilevel positive airway pressure
tall in men) C. Overnight supplemental oxygen
E. Insomnia D. High-dose progesterone
E. Diuresis and periodic phlebotomy as needed
12. Moderate and severe sleep apnea syndrome is associ-
ated with increased morbidity and/or mortality due to 16. Which of the following flow-volumes loops is
which of the following? most likely caused by a large tracheal tumor (Figures
4.24.7)?
A. Aspiration
B. Laryngospasm A.
8
C. Diabetic ketoacidosis 7
D. Motor vehicle accidents 6
E. Hypoxemic respiratory failure
5
4 FEV1 = 2.58
13. Which of the following collagen-vascular diseases is
3
more often associated with airway disease than intersti- FEV3 = 3.14
2
tial lung disease?
1
0
A. Scleroderma 1 2 3 4 5 6 7 8
1
B. Polymyositis
2
C. Relapsing polychondritis
3
D. Systemic lupus erythematosus
E. Mixed connective tissue disease 4
5

14. A30-year-old man presents for evaluation of poly- 6

cythemia. He is found to have resting hypoxemia 7

(SaO2 = 87%) and signs of right heart failure, with 8

jugular venous distension and peripheral edema. He Figure 4.2 Flow-volume loop for patient in Question 16.

7 8 T h e B r i g h a m I n t e n s i v e R e v i e w o f I n t e r n a l M e d i c i n e Q u e s t i o n a n d A n s w e r C o m pa n i o n
B. D.
10 6
9
5
8
7 4
6 FEV1 = .80
5 FEV1 = 1.52 3
4 2 FEV3 = 1.34
3 FEV3 = 1.74
2 1
1
0
0
1 2 3 4 5 6
1 1 2 3 4 5 6 7 8 9 10 1
2
2
3
4 3
5
6 4
7
5
8
9 6
10
Figure 4.5 Flow-volume loop.
Figure 4.3 Flow-volume loop.
E.
C.
8
8
7
7
6
6
5
5 FEV1 = .73
FEV1 = 1.80 4
4
3
3 FEV3 = 1.70
FEV3 = 2.33 2
2
1
1
0
0 1 2 3 4 5 6 7 8
1 2 3 4 5 6 7 8 1
1
2
2
3
3
4
4
5
5
6
6
7
7
8
8
Figure 4.6 Flow-volume loop.
Figure 4.4 Flow-volume loop.
17. Which of the following tests is most likely to iden- E. Full pulmonary function tests, including
tify an etiology of bronchiectasis for which a specific measurement of lung volumes and diffusion capacity
therapy is available? for carbon monoxide.

A. Antinuclear cytoplasmic antibody (ANCA) 18. The term that best describes the pattern of abnor-
B. Serum immunoglobulin G mality on the accompanying CT scan (Figure 4.7) is:
C. Gluten autoantibodies (serum endomysial and
antitissue transglutaminase antibodies) A. Ground-glass opacity
D. Bronchial biopsy examined by electron microscopy B. Consolidation with air bronchograms

4 . P u l m o n a r y a n d C r i t i c a l C a r e M e d i c i n e 7 9
B. Fiberoptic bronchoscopy and transbronchial lung
biopsy
C. Transthoracic needle aspirate/biopsy
D. Surgical resection of the lung nodule
E. Repeat chest imaging in 3months to assess for
growth of the nodule

21. Which of the following medications used to treat


Figure 4.7 CT for patient in Question 18.
obstructive lung diseases is most likely to cause inter-
mittent hoarseness?
C. Honeycombing A. Salmeterol
D. Tree-in-bud nodules B. Fluticasone
E. Atelectasis C. Tiotropium
D. Albuterol
19. The accompanying chest X-ray (Figure 4.8) is
E. Ipratropium
most likely from a patient with which of the following
diagnoses? 22. A43-year-old woman presents with exertional dys-
pnea progressive over the past few years. She is now
breathless climbing one flight of stairs. She is a lifelong
nonsmoker without history of asthma. Her chest X-ray
shows mild hyperinflation. Spirometry and lung vol-
umes indicate severe airflow obstruction without signif-
icant improvement following bronchodilator. Alpha-1
antitrypsin level is normal. Which of the following is
the most likely explanation for her lung disease?

A. She works in an old office building with central


ventilation and without windows that can be
opened.
B. She owns four large parrots (cockatoos).
Figure 4.8 PA and lateral chest X-ray for patient in Question 19.
C. She has ulcerative colitis.
D. She has mixed connective tissue disease.
A. Alpha-1 antitrypsin deficiency E. She has a history of breast cancer treated with
B. Ankylosing spondylitis lumpectomy and postsurgical radiation therapy to
C. Bronchiectasis due to cystic fibrosis the breast.
D. Granulomatosis with polyangiitis (Wegeners
granulomatosis) 23. Which of the following statements about the treat-
E. Lymphangioleiomyomatosis (LAM) ment of an exacerbation of COPD is true?
20. A55-year-old cigarette smoker born and reared in A. An 8-week course of oral steroids is associated with
NewYork City is found to have a round right lower lobe fewer relapses than a 2-week course.
noncalcified nodule 1.8cm in diameter, new compared B. Intravenous steroids are more effective and have
to a prior chest X-ray done 1year ago. Other than his fewer gastrointestinal side effects than oral steroids.
usual smokers cough, he has been free of respira- C. Bilevel positive airway pressure ventilation is
tory symptoms. His physical examination is normal. contraindicated because of the risk of hyperinflation
Positron emission tomography (PET) scan reveals and auto-PEEP.
increased uptake of radiolabeled glucose within the D. Antibiotics are not indicated in the absence of
lung nodule but at no other sites. Spirometry indicates sinusitis or pneumonia.
mild airflow obstruction; a cardiac stress test finds no E. Sputum Gram stain and culture are not routinely
evidence for myocardial ischemia. The next step in his necessary to guide the choice of antibiotics.
management should be:
24. A 24-year-old man is found to have bilateral hilar
A. Measurement of prostate-specific antigen (PSA), and mediastinal lymphadenopathy on a routine pre-
upper gastrointestinal series, and colonoscopy. employment chest X-ray (Figure 4.9).

8 0 T h e B r i g h a m I n t e n s i v e R e v i e w o f I n t e r n a l M e d i c i n e Q u e s t i o n a n d A n s w e r C o m pa n i o n
bicarbonate 30 mEq/L. You consider as a potential
cause of his arrhythmia which of the following:

A. Sudden reversal of chronic hypoxemia (acute


hyperoxia)
B. Acute metabolic alkalemia due to unmasking of
compensatory chronic metabolic alkalosis
C. Auto-PEEP with high intrathoracic pressures and
inadequate minute ventilation
D. Acute-on-chronic respiratory acidosis due to
refractory exacerbation of COPD
E. Hypochloremic, hypokalemic metabolic alkalosis
Figure 4.9 PA and lateral chest X-ray for patient in Question 24. induced by combination of beta agonists and
corticosteroids
He has a mild dry cough, no other respiratory or sys-
temic symptoms. He was born and reared in New 26. A67-year-old man presents to the emergency depart-
England with no history of tuberculosis or exposure ment with fever and cough. Chest X-ray demonstrates a
to anyone with known tuberculosis. Physical examina- dense right lower lobe consolidation, and he is started
tion is normal, as are routine blood studies (complete on antibiotics. While awaiting admission to the medical
blood count and comprehensive metabolic profile). floor, he becomes hypotensive and unresponsive to sev-
Mediastinoscopy with right paratracheal lymph node eral liters of intravenous fluid. Acentral line is placed.
biopsy reveals noncaseating granulomas. Stains for Which of the following statements is true regarding the
organisms are negative. Which treatment strategy choice of norepinephrine versus dopamine as treatment
would you recommend at this time? for his hypotension?

A. Low-dose oral steroids (prednisone 10 mg/day, with A. Dopamine is more likely to protect renal function.
dose adjusted according to his response to treatment) B. Dopamine is more likely to cause cardiac
B. Initial high-dose steroids (prednisone 4060 arrhythmias.
mg/day), tapered and then discontinued over C. Norepinephrine results in more overall adverse
approximately 6months events.
C. Hydroxychloroquine (Plaquenil) 200 mg p.o. twice D. Norepinephrine results in higher mortality in
daily cardiogenic shock.
D. Methotrexate 15 mg p.o. once a week E. Evidence for comparison of these vasopressors is
E. No therapy limited by lack of randomized control trials.

25. A72-year-old man with COPD and severe chronic 27. A45-year-old man is admitted with severe bilateral
airflow obstruction presents to the emergency depart- pneumonia. He is intubated and placed on ARDSNet
ment with shortness of breath, productive cough, and ventilator settings, but his fraction of inspired oxygen
wheezing. His medications at home include tiotro- (FiO2) cannot be decreased below 0.9 despite optimal
pium, fluticasone-salmeterol combination, albuterol by PEEP. Paralysis for 48 hours in this setting is most
metered-dose inhaler, aspirin (for primary prevention likely to result in which of the following?
of cardiovascular events), and lisinopril for hyperten-
sion. His chest X-ray is normal; oxygen saturation on A. An increase in neuromuscular weakness
6 L/min oxygen by nasal prongs is 90%. Despite fre- B. Areduction in mortality
quent nebulized bronchodilators and systemic steroids, C. An increase in lung injury
he has worsened respiratory distress and hypoxemia D. Adecrease in sedation requirements
and is intubated and begun on mechanical ventila- E. An increase in organ failure
tion. Ventilator settings are as follows: volume-cycled
ventilation in assist-control mode at 18 breaths/min- 28. A 65-year-old man with no significant past medi-
ute, 600 mL/breath, with positive end-expiratory cal history is admitted with septic shock due to a left
pressure at 5 cm H2O, on 40% inspired oxygen. After lower lobe pneumonia. He rapidly decompensates and
approximately 30 minutes he develops new-onset ven- requires vasopressors and intubation. A chest X-ray
tricular arrhythmias. He remains afebrile and normo- following intubation demonstrates diffuse bilateral
tensive. Arterial blood gases reveal the following:PO2 airspace opacities. His oxygen saturation is 95% with a
195mm Hg, PCO2 36mm Hg, pH 7.56, and measured PaO2 of 75mm Hg on an FiO2 of 1.0 and PEEP of 7.5cm

4 . P u l m o n a r y a n d C r i t i c a l C a r e M e d i c i n e 81
H2O. He is 5 feet, 8inches tall and weighs 100kg, with a moderate respiratory distress, and has an oxygen satu-
predicted body weight of 68.4kg. Which of the follow- ration of 93% on 3 L/min supplemental oxygen by nasal
ing mandatory (assist-control) ventilator modes would cannulae. A chest X-ray demonstrates no abnormal
be best for managing this patients condition? pulmonary opacities. She receives intensive broncho-
dilator therapy and systemic steroids. Arterial blood
A. Volume targeted, 800 cc gases demonstrate pH 7.27, PCO2 66mm Hg, and PaO2
B. Volume targeted, 700 cc 74mm Hg. Which of the following is the best next step
C. Volume targeted, 650 cc in her care?
D. Volume targeted, 410 cc
E. Pressure targeted, 25cm H2O A. Increase supplemental oxygen
B. Intubation and mechanical ventilation
29. An 87-year-old woman with severe COPD remains C. Noninvasive ventilation
in respiratory distress despite noninvasive ventilation. D. Decrease supplemental oxygen
Preparations are made to proceed with rapid sequence E. Initiate antibiotics
intubation using propofol and succinylcholine. Which
of the following conditions is a potential contraindica- 33. A78-year-old man with severe Parkinsons disease
tion to these medications? was admitted 3 days ago with an aspiration pneumo-
nia and required intubation for respiratory failure. His
A. Neuroleptic malignant syndrome fevers have resolved and his oxygenation has improved.
B. Chronic corticosteroid use His current ventilator settings are volume-cycled venti-
C. Hypernatremia lation, assist-control mode, tidal volume 400 cc, PEEP
D. Hypertension 5cm H20, and FiO2 0.4 His oxygen saturation is 95%.
E. Hyperkalemia Arapid shallow breathing index (RSBI) is measured at
90. Which of the following is the next best step?
30. A 45-year-old man is admitted to the ICU with
severe pancreatitis. Over the first 24 hours he develops A. Extubation
hypotension requiring vasopressors, respiratory failure B. Spontaneous breathing trial
requiring intubation, and renal failure. Over the last C. Pressure support weaning
several hours, his urine output has decreased, his vaso- D. Decrease PEEP and FiO2
pressor requirement has increased, and his abdomen E. Synchronized intermittent mandatory ventilation
has become progressively distended. Which of the fol- (SIMV)
lowing is the best next step in his management?
34. A55-year-old woman is admitted with severe sepsis
A. Abdominal computed tomography and renal failure. Her hemodynamics improve, and she
B. Empiric antibiotics is weaned off vasopressors. Her fever and leukocytosis
C. Abdominal ultrasound also resolve. However, she remains oliguric, and her cre-
D. Measurement of bladder pressure atinine continues to rise. Which of the following state-
E. Abdominal plain film ments about renal replacement therapy in this setting is
most accurate?
31. A 39-year-old woman with depression is admitted
after being found intoxicated next to an empty bottle A. Continuous renal replacement therapy (CRRT)
of acetaminophen. Her laboratory results are notable does not have a mortality benefit compared with
for an alanine aminotransferase (ALT) of 55 U/L and intermittent hemodialysis (IHD).
an aspartate aminotransferase (AST) of 34 U/L. Serum B. High-intensity renal replacement therapy (either
and urine drug screens are pending. Which of the fol- CRRT or IHD) has been consistently demonstrated
lowing is the best next step in her management? to improve mortality.
C. CRRT improves mortality, but only in patients on
A. Toxicology screen before providing further therapy vasopressors.
B. Sodium bicarbonate D. CRRT improves mortality, but only in patients on
C. Activated charcoal mechanical ventilation.
D. N-acetylcysteine E. IHD is contraindicated in patients on vasopressors.
E. Gastric lavage
35. A45-year-old obese woman with Crohns disease
32. A76-year-old woman with severe COPD is admit- complicated by multiple f lares is admitted with fever,
ted with 2 days of dyspnea and cough. She is alert, in abdominal pain, diarrhea, and hypotension. In the

8 2 T h e B r i g h a m I n t e n s i v e R e v i e w o f I n t e r n a l M e d i c i n e Q u e s t i o n a n d A n s w e r C o m pa n i o n
emergency department, she receives 2.5 L of intrave- the following is true about the role of red blood cell
nous f luid, but her systolic blood pressure remains in transfusion?
the 70mm Hg range, and a central venous catheter is
placed. Her central venous pressure (CVP) is 6mm Hg A. Transfusion is indicated due to concomitant
with significant respiratory variation. Vasopressors coronary artery disease.
are initiated, and her mean arterial pressure is cur- B. Transfusion is indicated due to his recent
rently 55mm Hg. Laboratory values are notable for gastrointestinal bleed.
a leukocytosis and a hematocrit of 27%; her albumin C. Transfusion is associated with fewer
is 2.8 g/dL. Which of the following is the next best ventilator-dependent days.
step? D. Transfusion may be associated with an increase in
mortality.
A. Increase vasopressor dose E. Transfusion may result in hypokalemia.
B. Intravenous crystalloid drip, at a rate of 200 cc/hr
C. Intravenous administration of colloid 39. A 75-year-old man is admitted for pneumonia and
D. Red blood cell transfusion sepsis. He requires vasopressors and is intubated for
E. Intravenous crystalloid bolus respiratory failure. He is placed on midazolam and
fentanyl. He is treated with ceftriaxone and azithro-
36. An 85-year-old woman is admitted to the ICU with mycin for severe community-acquired pneumonia and
chest pain, the new onset of atrial fibrillation, and continued on his outpatient aspirin, atorvastatin, and
an exacerbation of her chronic congestive heart fail- sertraline. Over the course of the next 18 hours, he is
ure (CHF). After several days, she starts to improve. weaned off of vasopressors and his white blood cell
Heparin was initiated at admission, and she has recently count improves; however, he is noted to become increas-
started transitioning to warfarin. Starting on the fifth ingly agitated. Additional midazolam and fentanyl are
hospital day, her platelets decrease to 100,000/L from given, and he also receives haloperidol for presumed
220,000/L on admission. She is now noted to have delirium. His vital signs are notable for a new fever to
new right lower-extremity pain and edema. In addition 103F, heart rate 120 beats per minute, and blood pres-
to discontinuing heparin, which of the following is the sure 100/60mm Hg. His examination is notable for dry
next appropriate step? mucous membranes, increased muscle tone, and tremor
and hyperreflexia with clonus, particularly in his lower
A. Await results of lower-extremity ultrasound extremities. His pupils are dilated with oscillatory eye
B. Initiate argatroban movements. Which of the following is the next best step?
C. Initiate low-molecular weight heparin
D. Continue transition to warfarin A. Discontinue sertraline
E. Await results of platelet factor 4 (PF4) antibody B. Discontinue sertraline and fentanyl
C. Initiate propofol
37. An 85-year old woman is admitted to the ICU after D. Discontinue haloperidol
suffering a stroke. She is intubated for altered mental E. Add vancomycin
status and inability to protect her airway. Which of
the following interventions could increase her risk of 40. A45-year-old man has a witnessed cardiac arrest in
ventilator-associated pneumonia? a shopping mall. Bystander cardiopulmonary resuscita-
tion is initiated. An automated external defibrillator is
A. Elevation of the head of the bed to 30 degrees applied. It advises a shock, which is delivered, but there is
B. Oropharyngeal decontamination no return of spontaneous circulation. Paramedics arrive,
C. Gastrointestinal decontamination and after two doses of epinephrine and additional shocks
D. Continuous drainage of subglottic secretions given according to advanced cardiac life support (ACLS)
E. Stress ulcer prophylaxis protocol, the patient regains a pulse. When he arrives in
the emergency department, he is tachycardic but normo-
38. A 52-year-old man with coronary artery disease is tensive. He withdraws to painful stimuli but is otherwise
admitted with hematemesis. He is intubated, and an unresponsive. Laboratory data are notable for a white
esophagogastroduodenoscopy (EGD) demonstrates blood cell count of 18,000/L, lactate 8 mEq/L, potas-
a bleeding gastric ulcer. Over the next 48 hours his sium 5.8 mEq/L, bicarbonate 18 mEq/L, and creatinine
condition stabilizes, and a repeat EGD demonstrates 1.5 mg/dL. Which of the following is true?
no further bleeding. His ICU course is complicated
by delirium and pneumonia presumed due to aspira- A. Therapeutic hypothermia is contraindicated due to
tion. His hemoglobin stabilizes at 7.5 g/dL. Which of risk of infection.

4 . P u l m o n a r y a n d C r i t i c a l C a r e M e d i c i n e 83
B. Therapeutic hypothermia is contraindicated due to C. Fomepizole
hyperkalemia. D. Hemodialysis
C. Therapeutic hypothermia results in increased E. Bicarbonate
frequency of malignant arrhythmias.
D. Therapeutic hypothermia should be initiated after 44. A65-year-old-man is admitted with several days of
reassessment of neurologic status at 12 hours. progressive weakness, dyspnea, and cough. His temper-
E. Therapeutic hypothermia should be continued for ature is 99 F, blood pressure 82/58mm Hg, heart rate
24 hours. 105 beats per minute, and respiratory rate 22 breaths per
minute. His oxygen saturation is 93% on supplemental
41. A 45-year-old man is intubated for a severe asthma oxygen at 2 L/min by nasal cannulae. Laboratory values
exacerbation. His chest X-ray demonstrates only hyperin- demonstrate an elevated white blood cell count; asym-
flation. His initial ventilator settings are volume-targeted, metric bilateral lower lobe opacities are demonstrated
assist-control mode ventilation, tidal volume 450 cc on his chest X-ray. He is given intravenous fluids and
(7 cc/kg of ideal body weight), set rate 28 breaths/min, antibiotics for pneumonia. Over the course of the next
PEEP 5cm H2O, and FiO2 0.5. The patient is not trig- several hours, his hypotension worsens. Blood lactate is
gering additional breaths and appears synchronous with elevated, and his extremities are cool. Acentral venous
the ventilator, and he has a measured end-expiratory catheter is placed for initiation of vasopressors. The
pressure of 15cm H2O with a peak inspiratory pressure central venous pressure is 14cm H2O, and a blood gas
of 45cm H2O. His blood pressure is 89/55mm Hg. His drawn from this catheter reveals an oxygen saturation
arterial blood gases reveal the following:pH 7.38, PCO2 of 40%. Which of the following is the most likely inter-
40mm Hg, and PO2 120mm Hg. Which of the following pretation of his hemodynamics?
ventilator changes would you recommend?
A. Distributive shock due to sepsis
A. No change B. Distributive shock due to underlying liver disease
B. Increase PEEP C. Hypovolemic shock due to sepsis
C. Decrease respiratory rate D. Hemorrhagic shock
D. Increase respiratory rate E. Cardiogenic shock
E. Increase inspiratory to expiratory (I/E) ratio
45. A55-year-old man is scheduled to undergo an esoph-
42. A 84-year-old man is intubated for an aspiration agogastroduodenoscopy for Barretts esophagus. His
pneumonia. Which of the following interventions is vital signs, including oxygen saturation, are normal.
most likely to be associated with an increased duration He receives topical anesthesia with benzocaine, and
of mechanical ventilation? conscious sedation is achieved using midazolam and
fentanyl. Shortly after the procedure begins, his oxy-
A. Daily interruption of sedation gen saturation drops to the mid-80% range. Other vital
B. Daily spontaneous breathing trials signs have not changed. He is placed on supplemental
C. Use of midazolam instead of propofol oxygen, but his oxygen saturation does not improve.
D. Use of dexmetomidine instead of midazolam Chest examination demonstrates clear lung fields.
E. Avoidance of routine sedative medication Which of the following is the next best diagnostic step?

43. A 55-year-old man is admitted with altered mental A. Chest X-ray


status. He has a history of heavy alcohol abuse. In the B. Chest CT angiography
emergency department, his vital signs are notable for C. Arterial blood gas with co-oximetry
tachycardia to 110 beats per minute but are otherwise D. Arterial blood gas
normal. Laboratory values include the following:sodium E. Electrocardiogram
145 mEq/L, potassium 4 meq/L, chloride 105 mEq/L,
bicarbonate 15 mEq/L, BUN 25 mg/dL, glucose 150 mg/ 46. A75-year-old man with severe chronic obstructive
dL, and creatinine 1.5 mg/dL. Urinalysis demonstrates pulmonary disease is intubated for an exacerbation
no ketones but is noted to have needle-shaped crystals. of his lung disease. His initial ventilator settings are
His serum alcohol level is 10 mg/dL and his serum osmo- volume-cycled ventilation in assist-control mode, tidal
larity is 325 mOsm/kg. Which of the following is the volume 500 cc (8 cc/kg), PEEP 5cm H2O, and FiO2 0.5.
next appropriate step in his treatment? His initial peak inspiratory pressures are 18 cm H2O.
Several hours after intubation, his ventilator alarms
A. Ethanol indicate increased peak inflation pressures. His new
B. Benzodiazepines peak inspiratory pressure is 35 cm H2O. His plateau

8 4 T h e B r i g h a m I n t e n s i v e R e v i e w o f I n t e r n a l M e d i c i n e Q u e s t i o n a n d A n s w e r C o m pa n i o n
pressure is 15cm H2O. Which of the following is likely 49. A 75-year-old man is admitted with right lower
to be the most helpful? lobe opacities and septic shock. He is treated with
broad-spectrum antibiotics. His condition worsens
A. Chest X-ray over the first 24 hours. His initial infiltrate progresses
B. Needle decompression to become bilateral, and he requires intubation. His
C. Endotracheal suctioning shock worsens to the point of requiring two vasopres-
D. Chest tube placement sors despite adequate fluid resuscitation. Which of the
E. Diuresis following is the best statement about administration of
corticosteroids in this patient?
47. A27-year-old woman with epilepsy and a history of
poor adherence to therapy is admitted to hospital after A. Corticosteroids improve mortality in patients who
a tonic-clonic seizure. Shortly after admission, she has have a low response to adrenocorticotropic hormone
two more seizures. She is given lorazepam and intu- (ACTH) stimulation.
bated. She is afebrile and normotensive; head CT scan B. Corticosteroids improve mortality in patients who
and serum chemistries, including blood glucose, are have inappropriately normal serum cortisol.
all within normal limits. Which of the following state- C. Corticosteroids will likely improve his response to
ments about her management is true? vasopressors.
D. Corticosteroids if initiated for his ARDS will be of
A. Infusion with phenytoin should be the initial greater benefit if initiated after 14days.
treatment. E. High-dose corticosteroids demonstrate mortality
B. Fosphenytoin is preferred over phenytoin because of benefit in sepsis but have an unacceptable rate of
a decreased risk of hypotension. complications.
C. Diazepam has a slower onset of action than
lorazepam. 50. An 85-year-old man with hypertension, coronary
D. Lorazepam infusion followed by fosphenytoin is a artery disease, and chronic kidney disease is admitted
reasonable treatment. after an overdose of verapamil. His initial blood pres-
E. Propofol is ineffective for this condition. sure is 130/80 mm Hg with a pulse of 75 beats/min.
His mental status is normal, and his serum chemistries
48. A67-year-old patient with COPD and coronary dis- and complete blood count are unchanged from base-
ease is admitted to the hospital for pneumonia. During line. His EKG shows a prolonged PR interval. He is
the evening of his admission, his tachypnea worsens, given activated charcoal and intravenous calcium and
and he is placed on increased supplemental oxygen. is admitted to the ICU. Over the next several hours
Several hours later, he becomes hypotensive and more he develops bradycardia and hypotension. He is given
lethargic. He is found soon thereafter without a pulse. additional calcium, atropine, and glucagon with some
The cardiac arrest team is called. Which of the follow- transient improvement. However, his bradycardia and
ing statements about his resuscitation is true? hypotension continue to worsen, and he requires intu-
bation and escalating doses of vasopressors. Which of
A. Inadequate bag-and-mask ventilation is a common the following therapies should be initiated?
cause of failure to recover from a non-VT/VF arrest.
B. Arapid response team would not have reduced the A. Hemodialysis
likelihood of cardiac arrest. B. Bicarbonate infusion
C. Apulse check should be performed immediately C. Physostigmine
after administration of medications. D. Intravenous corticosteroids
D. Apulse check should be performed immediately E. High-dose insulin therapy
after defibrillation.
E. End-tidal CO2 detection may help determine the
quality of cardiopulmonary resuscitation (CPR) and
return of spontaneous circulation.

4 . P u l m o n a r y a n d C r i t i c a l C a r e M e d i c i n e 85
CH A PT ER4 A NSW ER S obstruction; 50%79% of normal indicates moderate
obstruction; 35%49% of normal indicates severe obstruc-
1. ANSWER:D. Respiratory failure and death tion; and <35% of normal indicates very severe obstruction.
The patient in example Ahas moderate airflow obstruction;
In the Salmeterol Multicenter Asthma Research Trial the patient in example B has very severe airflow obstruc-
[Chest. 2006;129:1526], approximately 26,000 patients tion The degree to which the FEV1 is reduced (in the
were randomly assigned to receive either inhaled salmeterol absence of an acute exacerbation) correlates directly with
or placebo for 6months, added to their usual care. Study disease morbidity and mortality in COPD.
outcomes included deaths and near-deaths (respiratory The pattern in Answer C suggests severe restriction;
failure requiring admission to an intensive care unit) due Answer D indicates normal spirometry; and in Answer
to asthma. Although these events were rare, they occurred E the results suggest a pattern of moderate restriction.
more commonly in the group randomized to receive salme- Restriction is best confirmed with full measurement of
terol compared to placebo. Episodes of respiratory failure lung volumes, including total lung capacity, functional
occurred 37 times in the salmeterol-treated group versus 22 residual capacity, and residual volume, as measured by the
times in the placebo-treated group; deaths due to asthma helium-dilution technique or plethysmography.
occurred 13 times in the salmeterol-treated group versus 3
times in the placebo-treated group (both differences were
statistically significant). The explanation for these adverse
outcomes remains uncertain. The possibility that this effect 3. ANSWER:A. The diagnosis is suggested by the
would be eliminated if all patients used an inhaled corti- presence of widespread emphysema on chest CT scan
costeroid at the same time that they received a long-acting in a nonsmoker.
beta-agonist bronchodilator or placebo is currently being
investigated. The SMART study of adverse outcomes related The presence of widespread (panlobular) emphysema in a
to salmeterol found no increase in serious atrial or ventricu- nonsmoker should raise the possibility of alpha-1 antitryp-
lar arrhthymias (Answers A and B), cardiovascular events sin deficiency; other suggestive findings include COPD
(heart attacks or strokes) (Answer C), or drug-induced aller- that develops in a cigarette smoker at an unusually young
gic events (angioedema or anaphylaxis) (Answer E). age (e.g., before age 50)or a strong family history of emphy-
sema. However, many patients with homozygous alpha-1
antitrypsin deficiency do not have these classic character-
istics, and some experts recommend widespread testing for
2. ANSWER:B. FEV1 1.0 L (33% of predicted), FVC alpha-1 antitrypsin deficiency among patients with COPD
3.0 L (75% of predicted), FEV1/FVC 0.33 (44% of [Am J Respir Crit Care Med. 2012;185:24659]. Testing
predicted) for alpha-1 antitrypsin begins with measurement of serum
alpha-1 antitrypsin level, a routine blood test done in the
Let us offer a simple strategy for interpreting spirometry chemistry laboratory. Patients who are homozygous for
results. After you have checked to see if the test has been prop- alpha-1 antitrypsin deficiency will have a very low blood
erly performed, with a smooth graphic display (volume-time level of the protein, on the order of 15% of normal. This
plot) and at least 6 seconds of forced expiratory time, look at finding reflects the fact that most patients with alpha-1 anti-
the FEV1/FVC ratio. If this ratio is reduced (as in examples trypsin deficiency make an alpha-1 antitrypsin protein that
Aand B), there is an obstructive pattern. Areduced ratio is is transported ineffectively out of the liver, rather than mak-
typically defined as a value below the 95% confidence inter- ing no alpha-1 antitrypsin protein (the homozygous null/
val (not given in these examples). The Global Initiative for null genotype is rare). Further testing of the patient with a
Chronic Obstructive Lung Disease (GOLD) guidelines very low serum alpha-1 antitrypsin level can be performed
attempt to simplify this definition by using a cut-off for a by protein electrophoresis or genetic analysis to determine
reduced ratio as <0.7. This cut-off is generally accurate except the specific genetic abnormality.
for all but the oldest of our patients, in whom an FEV1/ Patients with heterozygous alpha-1 antitrypsin abnor-
FVC ratio <0.7 may still be normal. (The FEV1/FVC ratio mality will have low serum alpha-1 antitrypsin levels (often
declines with age due to loss of lung elastic recoil during the on the order of approximately 50% of normal). There is
normal aging process.) If the FEV1/FVC ratio is normal or controversy as to whether heterozygous persons are at a
increased, there is no obstruction. modest increased risk for developing COPD. In any case,
Having established the presence of airflow obstruction a blood level less than the lower limits of normal does not
based on a reduced FEV1/FVC ratio, one can judge the establish homozygous deficiency (Answer B). It is estimated
severity of the obstruction by looking at the FEV1 expressed that as many as 30% of nonsmoking persons with alpha-1
as a percent of normal. By widely accepted convention, in antitrypsin deficiency will never develop obstructive lung
this context an FEV1 80%99% of normal indicates mild disease (Answer C). The percentage of adult patients with

8 6 T h e B r i g h a m I n t e n s i v e R e v i e w o f I n t e r n a l M e d i c i n e Q u e s t i o n a n d A n s w e r C o m pa n i o n
alpha-1 antitrypsin deficiency who develop clinical evi- divided by the height in meters squared (kg/m2), is an inde-
dence of liver disease (specifically, those with ZZ phenotype pendent predictor of mortality in COPD, along with lung
and other forms in which abnormal protein accumulates in function (FEV1) (Answer A), sense of dyspnea (Answer E),
the liver) is estimated to be approximately 20% and varies and exercise capacity as measured by the 6-minute walking
with age (Answer D). Emphysema due to alpha-1 antitryp- distance (Answer D). The risk of death following hospital-
sin deficiency, like emphysema due to smoking in persons ization for a COPD exacerbation is high; one recent study
with a normal alpha-1 antitrypsin level, is associated with showed a mortality of 20% at 1year and 55% at 5years of
a reduced diffusing capacity for carbon monoxide, due to follow-up [Chest. 2007; 132:174855] (Answer C).
impaired transfer of carbon monoxide across decreased
alveolar-capillary membrane surface area (Answer E).

6. ANSWER:B. Chest tube drainage of pleural


space, with thoracoscopic lysis of pleural adhesions if
4. ANSWER:D. Given your history, a leukotriene necessary.
modifier such as montelukast (Singulair), zafirlukast
(Accolate), or zileuton (Zyflo) might be particularly The clinical scenario of fever, productive cough, chest pain,
helpful as treatment for your asthma. and leukocytosis is consistent with an acute pneumonia and
parapneumonic effusion. The effusion is moderate-to-large
Aspirin-sensitive asthma (also called aspirin-intolerant and appears to be loculated, based on difficulty aspirating
asthma and, most recently, aspirin-exacerbated respiratory more than 60 mL of fluid during thoracentesis. The pleural
disease) occurs in approximately 3%5% of adults with fluid is serous without frank pus or bacteria in the pleural
asthma. It is exceedingly rare in children and does not have space; that is, there is no evidence for an empyema. In this
a familial predisposition. Although the precise biochemical context, the pleural fluid pH is helpful. The normal pleural
abnormality leading to asthmatic exacerbations after aspirin fluid pH is >7.4. Avalue of 7.2 or less indicates a compli-
ingestion is unknown, it involves some aspect of the cyclooxy- cated parapneumonic effusion, one that is likely to behave
genase pathway, and specifically cyclooxygenase 1.Thus, any like an empyema, with formation of adhesions and difficulty
cyclooxygenase 1 inhibitor, including ibuprofen, naproxen, clearing the infection without drainage of the pleural space.
and other nonsteroidal anti-inflammatory drugs (NSAIDs) Together with the rest of the clinical picture, it points to the
can precipitate an attack and need to be avoided (Answer B). need for chest tube drainage of the pleural space and possibly
Acetaminophen and specific cyclooxygenase 2 inhibitors (e.g., surgical lysis of adhesions and decortication of the pleura. It is
celecoxib) have weak cyclooxygenase 1 inhibition and are safe thought that in this setting the low pleural fluid pH is caused
to use in most patients. The dose of aspirin that triggers an by anaerobic metabolism of glucose by infection (not found
asthmatic reaction varies among aspirin-sensitive asthmatics, in the sampled fluid) and inflammation in the pleural space,
but some patients will respond to 81 mg or less (Answer A). leading to generation of carbon dioxide and lactate.
A feature of patients with aspirin-sensitive asthma is that Like managing an empyema, antibiotics alone without
they synthesize more than the normal amount of cysteinyl pleural drainage will have a low likelihood of success (Answer
leukotrienes at baseline and markedly excess amounts after A). Pleural biopsy might be considered if malignancy or tuber-
ingestion of aspirin or other cyclooxygenase 1 inhibitors. As culosis were high in the differential diagnosis, which they are
a result, it is logical to try antileukotriene therapy (either a not (Answer C). A history of recurrent vomiting or recent
leukotriene receptor antagonist or lipoxygenase inhibitor) to upper endoscopy with severe retrosternal or upper abdominal
treat their asthma (but not to make aspirin or other NSAID pain might suggest esophageal rupture with spillage of gastric
ingestion safe) (Answer D). The reaction to aspirin/NSAIDs juices into the pleural space. Ahigh pleural fluid amylase is
is not IgE-mediated and is not associated with IgE-mediated typical of this syndrome (Answer D). Alow pleural fluid pH
allergic reactions, such as food allergy to nuts (Answer C). may be seen in pleural effusions due to collagen-vascular dis-
Aspirin sensitivity is not associated with an increased risk of ease, especially in rheumatoid effusions, but the clinical his-
adverse reactions to beta-agonist bronchodilators (Answer E). tory does not support this diagnosis (Answer E).

5. ANSWER:B. Obesity 7. ANSWER:A. Lymphangioleiomyomatosis (LAM)

The BODE index [N Engl J Med. 2004; 350:100512] Chylothorax is typically milky in appearance (white to beige
includes a measure of body mass as a predictor of mortality in color) and (in the absence of cirrhosis) has a high triglycer-
in COPD. It is based on the observation that a low (<21) ide concentration (>110 mg/dL). The diagnosis of chylotho-
body mass index, calculated as the weight in kilograms rax can be confirmed by identification of chylomicrons in

4 . P u l m o n a r y a n d C r i t i c a l C a r e M e d i c i n e 8 7
the pleural fluid on lipoprotein electrophoresis. Chylothorax Neither the radiographic findings nor the transbronchial
results from disruption of the flow of chyle from the small lung biopsy results were suggestive of alternative diagnoses.
intestines through the thoracic duct into the left brachioce- Sarcoidosis (Answer A) is typically bronchocentric in loca-
phalic vein. Causes of thoracic duct obstruction or rupture tion, generally with an upper lobe predominance, and often
include the proliferation of tumor-like smooth muscle cells associated with hilar and mediastinal lymphadenopathy.
in lymphangioleiomyomatosis (LAM); tumors such as lym- Transbronchial lung biopsy will often reveal noncaseating
phoma; radiation fibrosis; and direct surgical trauma. The granulomas. Like IPF, nonspecific interstitial pneumonia
other choices aboveparapneumonic effusion (Answer (Answer B) is an idiopathic chronic inflammatory lung dis-
B), malignant effusion (Answer C), pulmonary infarction ease. It usually presents with areas of ground-class opacities
(Answer D), and sarcoidosis (Answer E)typically cause an on chest CT scan (Figure 4.10); and transbronchial lung
exudative pleural effusion with low lipid content. biopsy findings are often nonspecific, as in this case.
Cryptogenic organizing pneumonia (Answer D), also
referred to as bronchiolitis obliterans organizing pneumonia
(BOOP), mimics infectious pneumonia, with areas of con-
8. ANSWER:C. Idiopathic pulmonary fibrosis solidation on chest imaging. Transbronchial lung biopsy may
identify areas of organizing pneumonia and occasionally the
This patient presents with a typical history for chronic inter- endobronchiolar polypoid tissue of bronchiolitis obliterans.
stitial lung disease. He has progressive dyspnea, a nonproduc- Finally, Langherans cell histiocytosis (Answer E) presents with
tive cough, crackles on chest examination, and restriction on cystic lung disease and associated small lung nodules; on immu-
pulmonary function testing. His chest X-ray is confirmatory, nohistochemical staining of bronchoalveolar lavage fluid, one
with bilateral linear and nodular opacities, the radiographic may be able to identify the characteristic Langherans cells.
correlate of an interstitial process. The chest CT scan shows
a pattern that is suggestive of the pathologic process called
usual interstitial pneumonitis (UIP). In particular, there are 9. ANSWER:B. Resting hypoxemia without change
dense bands of opacity suggesting fibrosis with associated with exertion
traction bronchiectasis (bronchial walls pulled apart by the
retraction of scar formation) and honeycombing (dilated Characteristic of the interstitial lung diseases is oxygen desatu-
alveolar spaces, likewise enlarged by the retractive forces ration with exercise. The alveolar-to-arterial gradient for oxy-
from surrounding scar formation). The peripheral and basi- gen widens with exertion for two reasons. In part, there is likely
lar predominance are consistent with UIP. worsened ventilation/perfusion mismatching when minute
Potential causes of this pathologic process include ventilation and cardiac output increase. In part, diffusion
collagen-vascular diseases, pneumoconioses, and medica- impairment is made manifest when cardiac output increases,
tions such as bleomycin and nitrofurantoin. In the absence circulation time decreases, and the transit time for red blood
of an identifiable cause or association, the diagnosis is idio- cells in alveolar capillaries shortens, limiting the amount of
pathic pulmonary fibrosis (IPF). Of note, clubbing is found oxygen that can diffuse across thickened alveolar-capillary
in as many as 50% of patients with IPF. In this case example, membranes. In the patient with idiopathic pulmonary fibrosis
bronchoscopy was performed to exclude a potential infec- described in Question 8, one might find a normal oxygen satu-
tious etiology, with results that were nonspecific, consistent ration at rest but oxygen saturation characteristically decreases
with but not diagnostic of UIP. with exertion (Answer A). In IPF and other diffuse intersti-
tial lung diseases, the diffusion capacity for carbon monoxide
(DLCO) is decreased (Answer C); lungs are stiffer than nor-
mal and so exhibit reduced lung compliance (a measure of the
change in lung volume for any change in transpulmonary pres-
sure) (Answer D); and lung volumes are reduced, characteristic
of restrictive pulmonary processes (Answer E).

10. ANSWER:E. None of the above

No currently available treatment slows the progression of


idiopathic pulmonary fibrosis or prevents the occurrence
of acute deteriorations in this disease. Supplemental oxy-
Figure 4.10 Ground-class opacities on chest computed tomography gen should be given to chronically hypoxemic patients, and
(CT) scan (Answer 8). some patients may be eligible for lung transplantation or

8 8 T h e B r i g h a m I n t e n s i v e R e v i e w o f I n t e r n a l M e d i c i n e Q u e s t i o n a n d A n s w e r C o m pa n i o n
enrollment in clinical trials. (In some countries other than epidemiologic evidence indicates that OSA is also an
the United States, pirfenidone has been approved for the independent risk factor for adverse cardiovascular events.
treatment of idiopathic pulmonary fibrosis.) These include heart attacks and strokes, as well as cardiac
High-dose steroids (Answer A) are often used to treat arrhythmias and sudden death [ J Amer Coll Cardiol. 2013;
chronic idiopathic inflammatory lung disease when the doi:10.1016]. In addition, OSA is a cause of hypertension,
diagnosis is uncertain, as a therapeutic trial. However, likely as a result of repetitive sympathetic stimulation asso-
there is no role for high-dose systemic steroids when the ciated with episodic hypoxemia and sudden awakenings [ J
diagnosis of idiopathic pulmonary fibrosis has been estab- Amer Coll Cardiol. 2008;52:686717]; and evidence indi-
lished. Arandomized, controlled clinical trial of high-dose cates that treatment of OSA results in a meaningful low-
steroids, azathioprine, and N-acetylcysteine (Answer B) ering of blood pressure. OSA does not increase the risk of
versus placebo in patients with idiopathic pulmonary fibro- aspiration (Answer A), laryngospasm (Answer B), diabetic
sis, referred to as the PANTHER-IPF trial, was terminated ketoacidosis (Answer C), or hypoxemic respiratory failure
early when it was found that mortality was greater in the (Answer E).
group randomized to receive steroids, azathioprine, and
N-acetylcysteine compared with placebo [N Engl J Med.
2012; 366:196877]. Rituximab (Answer C), a monoclonal
antibody directed at the CD20 protein found on the surface 13. ANSWER:C. Relapsing polychondritis
of B cells, is used to treat granulomatosis with polyangiitis
(Wegeners granulomatosis) and may be effective in intersti- Relapsing polychondritis, the rarest of the collagen-vascular
tial lung disease associated with rheumatoid arthritis; it is diseases listed, is associated with airway disease, including
not known to be effective in the treatment of idiopathic pul- tracheomalacia and subglottic stenosis, but not with diffuse
monary fibrosis. Hydroxychloroquine (Answer D) is used to interstitial lung disease. The other choices are all associated
treat collagen-vascular disease, especially rheumatoid arthri- with interstitial lung disease. In addition, scleroderma is
tis, but it is not a therapy for chronic inflammatory lung associated with pulmonary hypertension and with esopha-
disease. geal and sometimes oropharyngeal dysfunction with an
increased risk of aspiration pneumonia. Systemic lupus
erythematosus can cause pleural effusions, diffuse alveolar
hemorrhage, and shrinking lung syndrome, possibly due
11. ANSWER:A. Retrognathia to diaphragmatic weakness. Mixed connective tissue dis-
ease may also cause pulmonary hypertension.
Persons with a narrowed posterior pharyngeal opening are
at increased risk for critical narrowing and occlusion of
the upper airway during sleep. The posterior pharynx may
be narrowed due to a large tongue, posteriorly extending 14. ANSWER:C. Central apneas
soft palate, large tonsils, or posteriorly positioned jaw and
tongue (retrognathia). Alarge neck circumference (greater The case history is a classic description of obesity-
than 16inches in women and 17inches in men) correlates hypoventilation syndrome (OHS), previously referred to
with excess fatty deposition in the upper airway (Answer as Pickwickian syndrome, defined as the combination
B). Nasal obstruction, such as with large nasal polyps of morbid obesity (BMI >30), PCO2 >45 mm Hg, and
(Answer C), can aggravate pre-existing obstructive sleep sleep-disordered breathing in the absence of other causes of
apnea, but it is not a risk factor for the presence of obstruc- alveolar hypoventilation [Anesthesiology. 2012;117:188205].
tive sleep apnea. Short stature in the absence of obesity is This morbidly obese young man has evidence for chronic
not a risk factor for sleep-disordered breathing (Answer D). hypoxemia, causing secondary polycythemia, pulmonary
Insomnia (Answer E) may be a manifestation of obstructive hypertension, and cor pulmonale. He also has daytime hyper-
sleep apnea syndrome, but it is nonspecific and not closely capnia, which, in the absence of known lung disease, is likely
associated with the presence of obstructive sleep apnea. due to central hypoventilation. In some cases of OHS, pro-
longed obstructive apneas (with associated acute hypercapnia)
without adequate compensatory hyperventilation following
these apneas can lead to daytime hypercapnia. Continuous
12. ANSWER:D. Motor vehicle accidents positive airway pressure (CPAP) may successfully treat OHS
in these instances. In other patients, central apneas persist
Persons with obstructive sleep apnea (OSA) have a 7-fold despite CPAP therapy, reflecting a primary abnormality of
increased risk of being involved in a motor vehicle accident ventilatory drive (Answer A).
as a consequence of their associated daytime hypersom- Orthodeoxia (Answer B) refers to a decrease in arterial
nolennce [Am Rev Respir Dis. 1988;138:33740]. Strong oxygenation in the upright position that improves when

4 . P u l m o n a r y a n d C r i t i c a l C a r e M e d i c i n e 8 9
lying supine; it is the objective manifestation of the symp- 8
tom of platypnea (dyspnea in the upright position that 7
lessens when supine). Intracardiac right-to-left shunts and
6
intrapulmonary arteriovenous shunts at the lung bases may
cause orthodeoxia. Morbid obesity is more likely to cause the 5
opposite positional effects, that is, orthopnea and worsened 4
hypoxemia when supine. Nothing in the history suggests FEV1

Flow liters/sec.
3
the presence of restless leg syndrome (Answer D), character-
ized by repetitive readjustment of the positioning of the legs 2
while awake and jerking movements of the legs while asleep. 1 FVC
A shortened (rather than prolonged) sleep latency period
0
(Answer E) is a nonspecific marker of sleepiness and is found
1 2 3 4 5
in many sleep-related disorders, including narcolepsy. 1

15. ANSWER:B. Bilevel positive airway pressure 4


Volume liters

Bilevel positive airway pressure with a mandatory back-up Figure 4.11 Diffuse intrathoracic airflow obstruction (Answer 16).
respiratory rate applied during sleep in patients with
obesity-hypoventilation syndrome that fails to improve
with CPAP can correct or ameliorate sleep-related hyper- exhalation. The expiratory limb of the flow-volume curve is
capnia and improve daytime gas exchange as well. The lat- displayed above the horizontal zero flow line; the inspi-
ter spillover effect may be due to a reduction in the renal ratory limb is displayed below the zero flow line; and
compensation for hypercapnia with less blunting of the together they form the flow-volume loop.
ventilatory drive by metabolic alkalosis. Daytime hyper- Characteristic of the flow-volume curves in asthma and
somnolence will also improve. Uvulopalatopharyngoplasty COPD is the scooped, concave appearance of the expiratory
and jaw advancement surgery (Answer A) is treatment curve (Answers C and D), reflecting diffuse intrathoracic air-
for obstructive sleep apnea syndrome but will not correct ways narrowing. This appearance is seen in all diffuse obstruc-
apneas when ventilatory drive is absent (central apneas tive lung diseases and is not diagnostic of any specific disease.
characterized by absence of inspiratory effort during period On the other hand, a large tracheal tumor causing upper air-
of apnea). Supplemental oxygen (Answer C) will address way narrowing results in flow at a single, fixed rate through-
the hypoxemia and its consequences, but it may worsen the out a large portion of exhalation and again during inhalation.
patients hypoventilation and does not address the underly- The resulting appearance on the flow-volume loop is distinc-
ing disease pathophysiology. Early studies suggested a role tive: a horizontal portion (referred to as a plateau pattern) of
for high-dose progesterone (Answer D) used as a respiratory the expiratory and inspiratory curves. Although not true of
stimulant in patients with obesity-hypoventilation syn- Answer E, often the rate of flow is similar on expiration and
drome, but the benefit proved to be small and the increased inspiration at the mid-portion of the curves (50% of the vital
risk of venous thrombosis unacceptably high. Symptomatic capacity), unlike in diffuse intrathoracic airflow obstruction,
treatment with diuresis and periodic phlebotomy (Answer in which inspiratory flow at 50% of the vital capacity typi-
E) will treat some of the consequences of hypoxemia and cally exceeds expiratory flow (see, for example, Figure 4.11).
right heart failure without addressing the underlying prob- Normal flow-volume curves (Answer A) or flow-volume
lem and its associated excessive daytime sleepiness. curves with a pattern suggesting restriction (Answer B) would
not be expected in the presence of a large tracheal tumor.

16. Answer: E. Figure 4.6


17. ANSWER:B. Serum immunoglobulinG
The flow-volume loop displays the results of the same
maximal forced expiratory maneuver used to generate the Many cases of bronchiectasis are idiopathic, presumably
spirogram. Instead of displaying the results as a plot of the result of a prior necrotizing infection that caused irrevers-
volume versus time as in spirometry, it displays flow (ver- ible damage to the airway walls. In other instances, however,
tical axis) versus volume (horizontal axis). It also typically an underlying predisposition can be identified. Among the
adds display of flow versus volume during completion of a potential conditions that may predispose to the development
maximal inspiration performed immediately after maximal of bronchiectasis are hypogammaglobulinemia and primary

9 0 T h e B r i g h a m I n t e n s i v e R e v i e w o f I n t e r n a l M e d i c i n e Q u e s t i o n a n d A n s w e r C o m pa n i o n
ciliary dyskinesia (also called immotile cilia syndrome). Pathologically, the nodules on CT imaging correspond
Diagnosis of the former may result in gammaglobulin replace- to an inflammatory reaction, often granulomatous, around
ment therapy, which would be expected to slow the progres- bronchioles (which run in parallel to peripheral vessels).
sion of the disease. Diagnosis of the latter (primary ciliary The nodules are the pathologic correlate of a bronchiolitis
dyskinesia) by electron microscopic examination of nasal or or peribronchiolitis, such as may be seen in sarcoidosis,
bronchial epithelial tissue and associated ciliary ultrastruc- mycobacterial infection, aspiration, and other conditions
ture (Answer D) does not lead to any therapeutic options [Radiology. 2002; 222:7712].
that would alter treatment at this time. Pulmonary vascu- Examples of ground-glass opacity (Answer A) (Figure
litides, such as granulomatosis with polyangiitis (Wegeners 4.13), consolidation with air bronchograms (Answer B) (Figure
syndrome) or eosinophilic granulomatosis with polyangiitis 4.14), honeycombing (Answer C) (Figure 4.15), and atelectasis
(Churg-Strauss syndrome), do not typically cause bronchi-
ectasis, making assay for antinuclear cytoplasmic antibodies
(ANCA) associated with vasculitis unhelpful (Answer A).
Celiac disease (Answer C) is associated with idiopathic pul-
monary hemosiderosis (called Lane-Hamilton disease) but
not bronchiectasis. The etiology of bronchiectasis cannot be
discerned by detailed characterization of its physiologic con-
sequences on pulmonary function testing (Answer E).

18. ANSWER:D. Tree-in-bud nodules

The CT image displayed in Question 18 has the following


abnormality: small lung nodules (23 mm in diameter)
grouped around peripheral blood vessels in the lung periph-
ery. The image is suggestive of some trees in early spring,
with their buds (the nodules) sprouting along the distal
branches before leaves appear (Figure 4.12).
Figure 4.13 Ground-glass opacity (Answer 18).

Figure 4.14 Consolidation with air bronchograms (Answer 18).

Figure 4.12 Tree-in-bud nodules (Answer 18). Figure 4.15 Honeycombing (Answer 18).

4 . P u l m o n a r y a n d C r i t i c a l C a r e M e d i c i n e 91
for lung cancer. Benign etiologies are possible, including
benign tumors, infections, and inflammatory reactions, but
the risk of lung cancer is high, especially given the relatively
large size of the nodule (>1cm in diameter). The evaluation
described in the case history serves to exclude obvious evi-
dence for metastatic disease (based on the absence of other
areas of increased uptake on the PET scan) and contrain-
dications to surgical resection (adequate ventilatory reserve
based on his spirometry results and low risk for myocardial
infarction based on his negative cardiac stress test). The
next step in his management should be referral to a tho-
racic surgeon for surgical excision of the lung nodule. Not
only does surgical resection provide definitive diagnosis but
also, if this nodule is indeed a lung cancer, it offers the best
chance for cure.
The argument against attempting to establish a preop-
Figure 4.16 Atelectasis (Answer 18).
erative diagnosis with fiberoptic bronchoscopy and trans-
bronchial lung biopsy (Answer B) or tranthoracic needle
(Answer E) (Figure 4.16) are shown, each with a distinctive aspirate/biopsy (Answer C) is the following. If a diagnosis
and very different appearance than tree-in-bud nodules. of lung cancer is confirmed, the patient will then be referred
for surgical resection of the nodule. If the sample returns
with nonspecific findings (no malignant cells seen), there
remains an unacceptably high chance (10% or greater) of a
19. ANSWER:A. Alpha-1 antitrypsin deficiency false-negative result [ J Thorac Oncol. 2012;7:81520]. The
likelihood of the sample providing a specific diagnosis of a
The radiographic appearance of alpha-1 antitrypsin defi- benign lesion, such as a histoplasmoma, is small, estimated
ciency is that of emphysema, with hyperinflation (as dem- at 5% or less.
onstrated by large lung volumes and flattened diaphragms) In the absence of suggestive symptoms or a known
and hyperlucency (due to a decrease in the normal vascular extrathoracic primary malignancy, the likelihood that this
markings) of the lung fields. In some cases of emphysema nodule represents a solitary pulmonary metastasis is low,
due to alpha-1 antitrypsin deficiency, a distinctive finding especially with the PET scan indicating no abnormal areas
is predominance of emphysema in the lower lung fields of increased glucose uptake outside of the chest. Workup
(rather than the typical predisposition for the upper lobes). for an alternative primary malignancy from which this
In the chest X-ray shown in Question 19, one can see blood nodule might be a solitary pulmonary metastasis (Answer
vessels in the upper halves of the lungs bilaterally but few in A) is unnecessary beyond the results of the PET scan.
the lower halves. The cause of the basilar predominance of Monitoring for growth of the nodule before proceeding
emphysema in some patients with alpha-1 antitrypsin defi- with surgical excision (Answer E) may be appropriate for
ciency is unknown. small or PET-negative nodules, but in this example the
The chest X-ray does not have the bilateral upper lobe probability of lung cancer is sufficiently high that delay in
opacities consistent with ankylosing spondylitis (Answer excision poses an unnecessary risk of growth and spread of
B); upper-lobe predominant linear markings in the orien- malignancy.
tation of bronchi, with oval cystic spaces and hyperinfla-
tion, suggestive of cystic fibrosis (Answer C); multiple lung
nodules, potentially some with cavitation, as may be seen in
granulomatosis with polyangiitis (Wegeners granulomato- 21. ANSWER:B. Fluticasone
sis) (Answer D); or hyperinflation with diffuse, thin-walled
cystic spaces typical of lymphangioleiomyomatosis. A common side effect of inhaled corticosteroids is hoarse
voice or dysphonia (a change in voice quality). The precise
pathophysiology is uncertain, but it likely is the conse-
quence of deposition of corticosteroid onto the larynx, with
20. ANSWER:D. Surgical resection of the lung consequent irritative effect, mucosal thinning, or muscular
nodule atrophy. The dysphonia tends to be intermittent and fully
reversible with cessation of medication. It cannot be pre-
A new (over the course of 1year), PET-positive lung nod- vented by rinsing the posterior pharynx or hypopharynx
ule in a middle-aged cigarette smoker is highly suspicious after each use of the medication. Sometimes this unpleasant

9 2 T h e B r i g h a m I n t e n s i v e R e v i e w o f I n t e r n a l M e d i c i n e Q u e s t i o n a n d A n s w e r C o m pa n i o n
side effect can be ameliorated by addition of a valved hold- exacerbations of COPD without need for sputum Gram
ing chamber (spacer) to the metered-dose inhaler being stain and culture has proven effective in numerous clini-
used to deliver the inhaled steroid or changing the delivery cal trials. Although many flares of COPD are triggered by
system (e.g., from a dry-powder inhaler to a metered-dose events other than bacterial infection, omission of antibiot-
inhaler with spacer). The inhaled steroid ciclesonide ics (Answer D) when there is evidence for increased cough,
(Alvesco) is released as a pro-drug that is activated by ester- sputum purulence, and/or fever results in less frequent
ases along the bronchial mucosa. It is said to have fewer oro- resolution and more frequent deterioration than treatment
pharyngeal (and possibly laryngeal) side effects as a result of with a short course of antibiotics. Oral steroids given for 2
this unique mechanism of activation. weeks result in identical outcomes (in terms of resolution
Although any inhaled medication can potentially cause of the exacerbation and risk of recurrence) compared to 8
occasional hoarseness, none of the other choices (salme- weeks (Answer A) [N Engl J Med. 1999;340:19417], and
terol, tiotropium, albuterol, or ipratropium) commonly recent evidence suggests that 5days of therapy may suffice
causes this problem. in many cases. Oral steroids are equally effective compared
with intravenous steroids in the treatment of COPD exac-
erbations with no greater incidence of gastrointestinal side
effects (Answer B) [Chest. 2007;132:17417]. Noninvasive
22. ANSWER:C. She has ulcerative colitis. mechanical ventilation, such as with bilevel positive airway
pressure, can reduce the need for intubation with mechani-
Bronchiolitis obliterans is a potential cause of severe cal ventilation in severe exacerbations of COPD (Answer
obstructive lung disease and should be considered in a C) [Chest. 2008;133:75666].
patient without other common causes of chronic airflow
obstruction, such as asthma, cigarette smoking, or genetic
predisposition to emphysema (alpha-1 antitrypsin defi-
ciency). Physical findings may include evidence for pulmo- 24. ANSWER:E. No therapy
nary hyperinflation and sometimes an inspiratory squeak.
Findings on chest CT scan may be few, sometimes with This case history strongly points to a diagnosis of stage 1
areas of mosaic oligemia or tree-in-bud nodularity. Airflow pulmonary sarcoidosis. The patient has a mild dry cough
obstruction is often poorly reversible to bronchodilators or but is otherwise asymptomatic. No treatment is necessary.
corticosteroids. The rate of spontaneous remission of his disease over the
Bronchiolitis obliterans may be immune mediated, as a next 2 years is approximately 80%. Treatment with sys-
pulmonary manifestation of ulcerative colitis, rheumatoid temic steroids (Answers A and B) may hasten resolution,
arthritis, or graft-vs.-host disease following bone marrow but at an unacceptably high cost of medication side effects.
transplantation or rejection following lung transplantation. Importantly, there is no evidence to suggest that early
Apoorly ventilated office building is not a cause of bron- treatment of sarcoidosis prevents long-term progression or
chiolitis obliterans (Answer A). Chronic exposure to par- sequelae of the disease [Eur Respir J. 2006;28:62736].
rots (Answer B) may cause hypersensitivity pneumonitis, Consequently, there is no justification for treating an
a restrictive lung disease. Mixed connective tissue disease asymptomatic or minimally symptomatic patient, whose
(Answer D) can cause interstitial lung disease with fibrosis, disease has a high rate of spontaneous remission, with a
likewise a cause of restriction on pulmonary function test- medication that entails frequent short-term and long-term
ing. The peripheral lung scarring that results from radiation adverse side effects.
therapy for localized breast cancer (Answer E) typically has Hydroxychloroquine (Answer C) has been used to treat
no impact on pulmonary function. cutaneous sarcoidosis, including erythema nodosum, but
it is not effective in most cases of pulmonary sarcoidosis.
Methotrexate (Answer D) is a potential steroid-sparing
agent in the treatment of severe pulmonary sarcoidosis.
23. ANSWER:E. Sputum Gram stain and culture
are not routinely necessary to guide the choice of
antibiotics.
25. ANSWER:B. Acute metabolic alkalemia due to
The bacterial pathogens causing exacerbations of COPD are unmasking of compensatory chronic metabolic
most often S.pneumoniae, H.influenzae, and M.catarrha- alkalosis.
lis, bacteria that can be treated with a variety of antibiotics
chosen empirically (including macrolides, cephalosporins, In patients with chronic hypercapnia, such as from COPD
amoxicillin/clavulanate, trimethoprim-sulfamethoxazole, with severe airflow obstruction, the sudden reversal of
doxycycline, and quinolones). Empiric treatment of hypercapnia with mechanical ventilation leads to a rapid

4 . P u l m o n a r y a n d C r i t i c a l C a r e M e d i c i n e 9 3
change in extracellular pH (from acidemic to alkalemic). 27. ANSWER:B. Areduction in mortality
The resulting electrolyte shifts across myocardial cell mem-
branes can result in serious, potentially life-threatening Neuromuscular blockade in severe ARDS has traditionally
cardiac arrhythmias. By a similar mechanism, seizures been used for refractory cases and with great caution due
may develop. The target arterial PCO2 during mechanical to concerns for complications, in particular, neuromuscu-
ventilation for exacerbations of COPD in patients with lar weakness related to critical illness (neuropathy, myopa-
acute-on-chronic hypercapnia should be values close to thy, or polyneuromyopathy). However, patient respiratory
baseline hypercapnia rather than a normal arterial PCO2 . If efforts and ventilator dyssynchrony may result in lung injury
lowering the arterial PCO2 to this goal involves excessively that may be mitigated by paralysis (Answer C). A recent
high peak inflation pressures or auto-PEEP, one may choose randomized, controlled trial demonstrated a reduction in
to accept persistently higher arterial PCO2 values until lung organ failure (Answer E) and an overall mortality benefit
function improves, the ventilatory strategy referred to as for 48 hours of paralysis in relatively severe ARDS (PaO2/
permissive hypercapnia. FiO2 ratio < 150 with at least 5cm H20 of PEEP) [N Engl
Reversing chronic hypoxemia with high inspired oxygen J Med. 2010;363:110716]. Despite the concern for weak-
concentrations (Answer A) may lead to worsened hypercap- ness, there was no increase in weakness noted at 28 days
nia in the spontaneously breathing patient with chronic or ICU discharge compared with placebo (Answer A).
hypercapnia, but it will have no adverse consequences in a Neuromuscular blockade requires deep sedation to avoid
patient receiving mechanical ventilation. Inadequate time anesthesia awareness and would not be expected to decrease
for exhalation during mechanical ventilation of patients sedation requirements. Indeed, sedation was similar in the
with severe expiratory airflow obstruction may lead to the two groups in this trial (Answer D).
development of high airway and transpulmonary pres-
sures at the end of exhalation (auto-PEEP) (Answer C).
Auto-PEEP can cause hypotension and increased risk of
barotrauma, but cardiac arrhythmias would not be expected 28. ANSWER:D. Volume targeted, 410cc
in the absence of hypotension. This patient does not have a
respiratory acidosis (Answer D); his PaCO2 is 36mm Hg This patient has acute respiratory distress syndrome
and arterial pH is 7.56. One cannot exclude a chronic meta- (ARDS). His PaO2/FiO2 ratio is <100, which meets crite-
bolic alkalosis (Answer E) based on the blood gas results. ria for severe ARDS. Low tidal volume ventilation results in
However, corticosteroids and inhaled beta-agonist bron- decreased mortality and is the standard of care for ARDS.
chodilators are not likely to cause a metabolic alkalosis of The goal tidal volume of 6 cc/kg is calculated based on the
this severity; and chronic metabolic alkalosis does not pre- ideal rather than the actual body weight. For a 58 male,
cipitate ventricular arrhythmias. the ideal body weight is 68.4kg; thus, 6 cc/kg would equal
410 cc. The protocol used in the ARDSNet trial started at
8 cc/kg, or 550 cc [N Engl J Med. 2000;342:13018]; thus,
tidal volumes of 650 cc, 700 cc, and 800 cc (Answers A, B,
26. ANSWER:B. Dopamine is more likely to cause and C) are too large. Mechanical ventilation breaths can be
cardiac arrhythmias. delivered either by setting a volume or by setting a pressure,
and plateau pressures are of importance in ARDS. Thus,
In a double-blind, randomized, controlled trial of initial pressure-targeted assist-control mode (pressure control) is
treatment of shock [the SOAP II study. N Engl J Med. a reasonable ventilator mode for ARDS. However, with-
2010;362:77989], treatment with dopamine versus nor- out knowing the respiratory mechanics and tidal volume
epinephrine demonstrated a significant increase in arrhyth- achieved by this setting, it is impossible to know whether
mias with dopamine. No statistically significant difference this ventilator setting is moreor even potentially less
in overall mortality was seen, but a subgroup analysis dem- injurious (Answer E).
onstrated increased mortality in patients with cardiogenic
shock treated with dopamine (Answer D). The trend in
other subgroups did not favor dopamine, and norepineph-
rine was not associated with more adverse effects (Answer 29. ANSWER:E. Hyperkalemia
C). Dopamine has been studied for its renal protective
effects; this trial did not show improvement in renal func- Succinylcholine is a neuromuscular blocking agent that is
tion, and previous trials in early renal dysfunction also dem- frequently used for intubation due to its rapid onset, brief
onstrated no benefit of dopamine (Answer A). Based on duration, and predictable response. Succinylcholine acts as
data from this trial and others, the Surviving Sepsis guide- a depolarizing agent and thus can result in hyperkalemia.
lines recommend norepinephrine as the first-line vasopres- In the setting of pre-existing hyperkalemia, particularly if
sor in septic shock. severe, this side effect can be life threatening. Succinylcholine

9 4 T h e B r i g h a m I n t e n s i v e R e v i e w o f I n t e r n a l M e d i c i n e Q u e s t i o n a n d A n s w e r C o m pa n i o n
is also contraindicated in patients with a history of malig- begin as soon as possible, ideally within 8 hours after inges-
nant hyperthermia and in cases where acetylcholine recep- tion. The suspicion for overdose is high, and one should
tors are expected to be upregulated (e.g., denervation from not wait for the results of toxicology screening (Answer A).
stroke, inherited myopathies). Neuroleptic malignant syn- Administration of activated charcoal can be helpful; but it
drome is not related to malignant hyperthermia and is not is most effective if given within 4 hours (preferably within
a contraindication to succinylcholine or propofol (Answer 1 hour) of ingestion, and the time of this patients ingestion
A). Propofol may cause hypotension, and thus hyperten- is unknown (Answer C). Gastric lavage and induced vomit-
sion is not a contraindication (Answer D). Hypernatremia ing are not recommended (Answer E). Sodium bicarbonate
and corticosteroid use are not contraindications to seda- is a therapy for other ingestions (e.g,. tricyclic and salicylate
tives and neuromuscular blockers in general (Answer B poisoning). While it may be helpful for cases of severe aci-
and C). Etomidate is an alternative sedative commonly dosis, in this case acidosis is not mentioned (Answer B).
used at the time of intubation because of its general lack of
hemodynamic side effects; it can be associated with adrenal
suppression.
32. ANSWER:C. Noninvasive ventilation

The arterial blood gases are consistent with acute-on-chronic


30. ANSWER:D. Measurement of bladder pressure respiratory acidosis. In the setting of a COPD exacerbation
with hypercapnia (PCO2 >45mm Hg), several studies have
This patient is at high risk for abdominal compartment demonstrated that noninvasive positive pressure ventilation
syndrome. Initially recognized in surgical conditions (e.g., decreases mortality and need for intubation. Her oxygen
trauma, abdominal surgery), this syndrome is increasingly saturation is adequate, and increasing supplemental oxygen
being recognized in medical settings, particularly in the is unlikely to result in significant improvement in her dys-
context of large-volume resuscitation and intra-abdominal pnea and may worsen hypercapnia (Answer A). Conversely,
pathology. Among the consequences of increased intra- decreasing supplemental oxygen is likely to result in subop-
abdominal pressure are a decrease in venous return with timal oxygen saturation and is unlikely to result in improve-
associated hypotension and extrinsic compression of the ment in the patients dyspnea (Answer D). Intubation may
lung, resulting in increased atelectasis and dead space. The eventually be needed. However, this patient is not in severe
normal intra-abdominal pressure in a nonobese, critically distress and does not have any clear contraindications to
ill patient, traditionally measured by transducing bladder noninvasive ventilation (e.g., impaired consciousness, high
pressure, is <7 mm Hg. Intra-abdominal hypertension is aspiration risk, or severe respiratory failure). Therefore, non-
defined as a sustained intra-abdominal pressure >12 mm invasive ventilation should be attempted first (Answer B).
Hg. Abdominal compartment syndrome is defined as
organ dysfunction with intra-abdominal hypertension and
is common with intra-abdominal pressures >20mm Hg.
Conservative treatment includes bowel decompression 33. ANSWER:B. Spontaneous breathing trial
and patient positioning; surgical decompression may be
required for severe cases. While imaging may be helpful in Increased duration of mechanical ventilation is associ-
identifying an alternative etiology for his abdominal disten- ated with an increased risk of complications, especially
sion, it would diagnose abdominal compartment syndrome. ventilator-associated pneumonia. Patients should be
Thus, abdominal ultrasound and portable radiographs are assessed daily for their readiness for discontinuation of
not the best choice (Answers C and E). Given his instabil- mechanical ventilation. In most patients who are not requir-
ity, transportation to a CT scanner may be risky (Answer ing high levels of support, a spontaneous breathing trial is
A). Empiric antibiotics may be considered in severe pan- a reasonable next step. A spontaneous breathing trial has
creatitis with necrosis, or if the etiology of his worsen- the patient breathe on minimal to no support for at least 30
ing is due to sepsis, but these processes are less likely than minutes. Many ICUs also include a rapid shallow breath-
intra-abdominal hypertension (Answer B). ing index (RSBI) prior to, or as a part of, the spontaneous
breathing trial. The RSBI is performed with the patient
breathing spontaneously without pressure support or PEEP
and is the ratio of respiratory rate divided by tidal volume
31. ANSWER:D. N-acetylcysteine (in liters). A RSBI <105 identifies patients more likely to
be ready to extubate. However, the decision to extubate
Acetaminophen (paracetamol) poisoning remains a major should not be based solely on the RSBI. The RSBI has
cause of acute liver failure in the United States. Treatment only moderate specificity to predict readiness to extubate,
with N-acetylcysteine is the standard of care and should can be artificially low in the setting of sedation, and does

4 . P u l m o n a r y a n d C r i t i c a l C a r e M e d i c i n e 9 5
not take into account mental status, airway secretions, and has been controversial, since a static measurement of CVP
other factors (Answer A). In several ventilator weaning is poorly predictive of fluid responsiveness. Nonetheless, the
trials [e.g., N Engl J Med. 1995;332:34550], a substantial 2.5 L of fluid that this patient received is a relatively low
number of patients were able to be extubated immediately amount, particularly in someone who is obese; and other
after a successful spontaneous breathing trial. Thus, the indicators, such as respiratory variation in CVP, may also
decision to proceed with decreasing amounts of pressure serve as indicators of volume responsiveness. Adequate
support should ideally be initiated only after the results of resuscitation is preferred over increasing vasopressors
the spontaneous breathing trial are known, and the deci- (Answer A). Fluid should be given as a bolus and not as a
sion regarding immediate extubation is made (Answer C). continuous infusion, both to treat hypotension quickly and
Weaning via SIMV results in a longer duration of mechani- to be able to gauge responsiveness (Answer B). While col-
cal ventilation than daily spontaneous breathing trials or loid is likely safe in this setting, there is no clear justification
pressure support (Answer E) [Am J Repir Crit Care Med. for its added expense (Answer C). Although she is anemic,
1994;150:896903]. Decreasing PEEP and FiO2 to levels a baseline hematocrit is not given; and in the absence of
required to adequately support oxygenation is reasonable if active cardiac ischemia or ongoing blood loss, a transfusion
the patient is unable to be extubated but is not the most would not be worth the risks (e.g., transfusion reaction or
appropriate next step in this patients care (Answer D). increased susceptibility to ARDS) (Answer D).

34. ANSWER:A. Continuous renal replacement 36. ANSWER:B. Initiate argatroban


therapy (CRRT) does not have a mortality benefit
compared with intermittent hemodialysis (IHD). This patient has a high likelihood of having heparin-induced
thrombocytopenia with thrombosis (HITT). The pretest
Acute renal failure in the setting of septic shock is a frequent probability of HITT is often given as the 4 Ts: throm-
and morbid problem. Although only a minority of patients bocytopenia, timing, thrombosis, and exclusion of other
goes on to require renal replacement therapy, the appropriate causes of thrombocytopenia. Her thrombocytopenia has
modality has been controversial. CRRT and IHD have been no other clear etiology; the timing is within 510days after
compared in several studies; overall, the evidence is most con- exposure; her platelet count has fallen by over 50%; and
sistent with CRRT being equivalent to IHD (Answer A). she has evidence suggesting a new deep venous thrombo-
While CRRT may be easier to tolerate in the setting of vaso- sis (DVT) of her right leg. Treatment is indicated for most
pressors (or mechanical ventilation), a study by Vinsonneau patients with HIT, and certainly for patients with HITT.
and colleagues [Lancet. 2006;368:37985] did not demon- Additional testing should be done, but therapy should not
strate any mortality benefit in a population where the vast be delayed (Answers Aand E). Of the available agents, only
majority required both vasopressor and mechanical ventila- argatroban is indicated (alternatives may include fondip-
tor support (Answers C and D). In this study, despite the use arinux or bivalirudin). Low-molecular weight heparin is
of vasopressors, most patients were able to be treated with also associated with a risk of HIT, albeit lower than unfrac-
IHD (Answer E). The available data on dialysis intensity sug- tionated heparin, and therefore is still contraindicated in
gest that this factor may be important. However, studies have HIT (Answer C). Warfarin rapidly lowers protein C levels
been conflicting. Harm may be associated with low-intensity and thus is associated with increased risk of thrombosis dur-
therapy, and while some studies have demonstrated improved ing initiation (Answer D). Transition to warfarin should be
mortality with high-intensity renal replacement, several oth- considered when the patients platelet count has improved
ers have not demonstrated a benefit over standard-intensity and she has been stably anticoagulated.
dialysis (Answer B).

37. ANSWER:E. Stress ulcer prophylaxis


35. ANSWER:E. Intravenous crystalloid bolus
Ventilator-associated pneumonias (VAP) (or more broadly,
Adequate fluid resuscitation is a mainstay of hemody- ventilator-associated complications) are an important cause
namic support in septic shock. A randomized controlled of morbidity and mortality in the ICU. Several practices
trial of early goal-directed therapy (EGDT) demonstrated have been shown to reduce the incidence of VAP. Many of
improved mortality with early and aggressive resuscita- these therapies have been included in ventilator bundles
tion in septic shock [N Engl J Med. 2001;345:136877]. grouped practices to reduce complications in mechanically
Adequate resuscitation in this trial was demonstrated by a ventilated patients. These therapies include elevation of the
CVP from 8 to 12 cm H2O. This specific numeric target head of the hospital bed, oropharyngeal decontamination

9 6 T h e B r i g h a m I n t e n s i v e R e v i e w o f I n t e r n a l M e d i c i n e Q u e s t i o n a n d A n s w e r C o m pa n i o n
and daily assessment of readiness to extubate (to reduce certain toxidromes are important to recognize. The sero-
the duration of mechanical ventilation) (Answers A and tonin syndrome is a life-threatening condition associated
B). Continuous drainage of subglottic secretions has been with increased serotinergic activity and is characterized
shown in several studies to decrease VAP. However, this by mental status changes, autonomic hyperactivity, and
approach requires specially designed endotracheal or tra- neuromuscular changes. In addition to agents tradition-
cheostomy tubes (Answer D). Selective decontamination of ally associated with serotonin reuptake, many other medi-
the gastrointestinal tract has also been shown to decrease cations can increase serotonin, such as metoclopramide,
VAP, though use has been limited in part due to concerns carbidopa-levodopa, meperidine, and fentanyl. The sero-
about promotion of antibiotic resistance (Answer C). tonin syndrome shares many features with neuroleptic
While stress ulcer prophylaxis is usually included as part malignant syndrome (NMS), including fever, mental status
of a ventilator bundle, acid suppression actually results in change, and increased muscle tone. However, in contrast
an increased, not decreased, risk of VAP. The mechanism is to NMS (Answer D), serotonin syndrome is characterized
thought to be bacterial overgrowth in the stomach in the by hyperreflexia, not bradyreflexia, and clonus, which can
absence of suppressive gastric acid, with risk of reflux and be inducible, spontaneous, and ocular. In addition, this
aspiration of these bacterial pathogens. patients agitation began before administration of haloperi-
dol; the onset of serotonin syndrome usually occurs within
24 hours of initiation of medication and is dose related,
while the onset of NMS begins within days to weeks.
38. ANSWER:D. Transfusion may be associated with Management of serotonin syndrome consists of discontinu-
an increase in mortality. ation of all serotonergic agents (thus, Answer A is incor-
rect), sedation with benzodiazepines, and consideration of
Anemia results in decreased oxygen delivery. However, cyproheptadine, a serotinergic antagonist. Propofol would
correction of anemia via transfusion of packed red cells not treat serotonin syndrome (Answer C). While haloperi-
is associated with risks, including hemolytic transfusion dol should probably not be continued because of anticho-
reactions, immunosuppression, transfusion-associated cir- linergic effects, its discontinuation would not address the
culatory overload (TACO), and transfusion-related acute life-threatening serotoninergic effects of the sertraline and
lung injury (TRALI). In the Transfusion Requirements fentanyl (Answer D). There is no evidence for an infectious
in Critical Care trial, over 800 critically ill patients were etiology for this patients fever, so vancomycin is not appro-
randomized to a restrictive or liberal transfusion strategy priate (Answer E).
[N Engl J Med. 1999;340:40917]. There was no benefit
to the liberal transfusion group. In a subgroup analysis of
patients less acutely ill and those <55years of age, a restric-
tive strategy (transfusion for Hgb <7 g/dL) was associated 40. ANSWER:E. Therapeutic hypothermia should be
with decreased mortality. Although patients with a pri- continued for 24 hours.
mary cardiac diagnosis appeared less likely to be enrolled in
this trial, analysis of patients with coronary disease demon- In patients who survive an out-of-hospital cardiac arrest,
strated no increased risk of complications in the restrictive anoxic-ischemic encephalopathy is a major cause of mor-
transfusion group (Answer A) and no benefit from transfu- bidity and mortality. The benefit of induced hypothermia
sions in terms of duration of mechanical ventilation in the in ventricular fibrillation and pulseless ventricular tachy-
liberal transfusion group (Answer C). The patients gastro- cardia was demonstrated in two randomized controlled
intestinal bleed has stabilized. Particularly in the absence trials, which both demonstrated a significant improve-
of active blood loss, there is no compelling reason to trans- ment in favorable neurologic outcomes [N Engl J Med.
fuse, and a randomized trial has demonstrated a benefit for 2002;346:449556 and N Engl J Med. 2002;346:55763).
a restrictive (Hgb <7 g/dL) transfusion strategy for most Risks of therapeutic hypothermia include increased risk
patients with upper gastrointestinal bleeding (Answer B) of infection, coagulopathy, hypokalemia (not hyperkale-
[N Engl J Med. 2013;368:1121]. Transfusions are associ- mia, Answer B), and bradycardia. While these risks must
ated with several electrolyte abnormalities, including hypo- be weighed against the benefits of hypothermia, there
calcemia, and hyperkalemia, not hypokalemia (Answer E). was no statistically significant increase in sepsis or lethal
arrhythmias in these two trials (Answer C). In the larger
of the two trials, development of infection was not associ-
ated with increased mortality (Answer C). Based on results
39. ANSWER:B. Discontinue sertraline and fentanyl from these trials and other observational data, hypothermia
should be initiated early (that is, within 46 hours) and not
The differential diagnosis for fever in the ICU is broad. delayed for reassessment of neurologic status (Answer D);
While infectious etiologies are the most common causes, and it should be continued for 1224 hours.

4 . P u l m o n a r y a n d C r i t i c a l C a r e M e d i c i n e 9 7
41. ANSWER:C. Decrease respiratory rate. osmolarity:expected osmolarity=2x(sodium in mEq/L) +
(glucose in mg/dL)/18 + (BUN in mg/dL)/2.8 + (etha-
Ventilator support for obstructive lung disease should nol in mg/dL)/3.7. An ingestion of a toxic alcohol should
allow for longer exhalation times to avoid hyperinflation be considered when the osmolar gap is >10. However, it is
and barotrauma (i.e., pneumothorax and pneumomedias- important to note that this gap reflects the presence only
tinum). In this case, the measured end-expiratory pressure of the parent alcohols and may be normal in late presenta-
is above the set PEEP (i.e., autoPEEP is present), and the tions. The combination of an osmolar gap and anion-gap
patients blood pressure is borderline low. Thus, decreas- metabolic acidosis should raise concern for ethylene glycol
ing the respiratory rate and allowing adequate exhalation toxicity. The presence of crystals in the urine is consistent
time should reduce autoPEEP and, as a result of decreased with calcium oxalate, which is a consequence of ethyl-
intrathoracic pressure, improve hemodynamics. Without ene glycol metabolism. The treatment for ethylene glycol
any change in ventilator settings, the patient will continue ingestion is fomepizole (Answer C). Fomepizole is an
with relative hypotension and risk of barotrauma from high inhibitor of alcohol dehydrogenase and thereby inhibits
inflation pressures (Answer A). Increasing the respiratory the metabolism of ethylene glycol to its toxic compounds.
rate or increasing the I/E ratio would worsen hyperinfla- Administration of ethanol would also help prevent eth-
tion (Answers C and E). Increasing PEEP in the setting ylene glycol metabolism, but it is associated with addi-
of obstructive lung disease with autoPEEP could improve tional side effects (Answer A). Alcohol withdrawal is a
triggering and patient-ventilator dyssynchrony, but these concern in this patient, for which benzodiazepines would
are not problems in this case (Answer B). be appropriate, but this intervention would not treat eth-
ylene glycol poisoning (Answer B). Bicarbonate would
be more appropriate therapy for a non-anion gap acidosis
(Answer E). Hemodialysis is indicated for severe inges-
42. ANSWER:C. Use of midazolam instead tions with refractory acidosis, renal failure, deterioration
ofpropofol in vital signs, or a markedly elevated level of ethylene gly-
col (Answer D).
Providing appropriate control of discomfort during
mechanical ventilation is an important goal in the intensive
care unit. While keeping patients heavily sedated may appear
to be most appropriate, excessive sedation is associated with 44. ANSWER:E. Cardiogenic shock
prolonged mechanical ventilation and other adverse out-
comes, such as greater likelihood of delirium. Randomized The central venous pressure is elevated, inconsistent with
trials have demonstrated more rapid extubation with daily hemorrhagic or hypovolemic shock (Answers C and D). The
interruption of sedation (Answer B) [N Engl J Med. 2000; central venous oxygen saturation reflects oxygen extraction
342:14717], daily spontaneous breathing trials (Answer for blood returning via the superior vena cava and approxi-
B) [Am J Repir Crit Care Med. 1994;150:896903], and a mates the mixed venous oxygen saturation. In septic shock
combination of daily interruption of sedation with spon- following adequate fluid resuscitation, this value is typically
taneous breathing trials [Lancet. 2008; 371:12634]. The elevated, reflecting impaired tissue oxygen extraction. Alow
type of sedation used may also be important. Trials have value indicates elevated tissue extraction and is consistent
demonstrated decreased duration of mechanical ventilation with impaired cardiac output. Impaired cardiac output can
with propofol versus midazolam (thus, Answer C is associ- be due to inadequate filling (e.g., hypovolemia), obstruc-
ated with an increased duration of mechanical ventilation), tion of blood flow (e.g., massive pulmonary embolus), or
as well as dexmetomidine, an alpha-2 agonist with analge- intrinsic cardiac causes (e.g., systolic dysfunction). In this
sic and sedative properties, versus midazolam (Answer D). case, the low venous oxygen saturation and elevated cen-
Avoidance of routine sedatives altogether can be achieved tral venous pressure, together with his physical examina-
in many intubated patients, and this approach has also been tion (cool extremities) are more consistent with cardiogenic
associated with decreased duration of mechanical ventila- shock. While myocardial dysfunction is not uncommon
tion (Answer E). in the setting of severe sepsis, it is also possible that his
bilateral radiographic opacities reflect pulmonary edema.
Although central venous oxygen saturation is typically high
in the setting of sepsis, it can at times be low, particularly
43. ANSWER:C. Fomepizole in settings of inadequate fluid resuscitation or concomitant
impaired cardiac output (Answer A). Ahigh central venous
This patients presentation is highly suspicious for eth- oxygen saturation can also be seen in end-stage liver disease,
ylene glycol intoxication. His osmolar gap is 15, if one although shock is usually due to superimposed infection or
includes ethanol in the calculation of the expected serum bleeding (Answer B).

9 8 T h e B r i g h a m I n t e n s i v e R e v i e w o f I n t e r n a l M e d i c i n e Q u e s t i o n a n d A n s w e r C o m pa n i o n
45. ANSWER:C. Arterial blood gas with co-oximetry COPD one anticipates a low peak pressure due to high lung
compliance. Thus, these findings (a high peak inspiratory
The most common causes of hypoxemia are ventilation- pressure with a low plateau pressure) are consistent with an
perfusion mismatch and shunt; these abnormalities can be airway problem. Increases in ventilator pressures due to air-
due to a variety of parenchymal and vascular causes such as way resistance can be due to mucous plugging, obstructed
atelectasis, aspiration, pulmonary edema, and pulmonary or kinked endotracheal tubes, or bronchospasm. Thus,
embolism. Hypoventilation is another important cause of endotracheal suctioning is a reasonable first step. A chest
hypoxemia, particularly in the setting of sedation. In this X-ray may be helpful, but it should not delay a potentially
situation, another important cause of hypoxemia to con- beneficial intervention (Answer A). Needle decompression
sider is methemoglobinemia. Methemoglobin is formed and chest tube placement would treat a pneumothorax or
when the ferrorus (Fe2+) iron in heme is oxidized to the pleural effusion (Answers B and D), and diuresis would
ferric (Fe3+) state; the ferric state is unable to bind oxygen. treat pulmonary edema (Answer E), all of which typically
Increases in methemoglobin can be due to genetic condi- would be expected to increase the plateau pressures.
tions resulting in deficiencies of cytochrome b5 reductase
or globin mutations, but they are more commonly due to
medications. The list of medications potentially causing
methemoglobinemia includes nitrates, dapsone, metoclo- 47. ANSWER:D. Lorazepam infusion followed by
pramide, and topical anesthetics, particularly benzocaine. fosphenytoin is a reasonable treatment.
Standard pulse oximeters use photodetectors to identify
oxyhemoglobin and deoxyhemoglobin; methemoglobin Status epilepticus is defined as seizures that persist or
cannot be specifically detected, and instead causes an arti- recur frequently enough so that recovery between attacks
factual change (usually decrease) in oxygen saturation with does not occur. Management of status epilepticus includes
increasing concentrations of methemoglobin, generally to assessment of the underlying etiology and supportive care of
around 85%87%. An arterial blood gas with co-oximetry respiratory and circulatory status, which may include intu-
can quantify the degree of methemoglobinemia, if present. bation. Initial management is generally through adminis-
In severe cases treatment of methemoglobinemia is with tration of benzodiazepines, most often lorazepam, although
methylene blue. An arterial blood gas alone can provide the others can be used. In a randomized, controlled trial com-
actual oxygen saturation, but without co-oximetry cannot paring several treatments, lorazepam alone was found to
diagnosis methemoglobinemia (Answer D). Achest X-ray be superior to phenytoin alone (Answer A) [N Engl J Med.
and electrocardiogram are reasonable initial tests for this 1998;339:7928]. Fosphenytoin is preferred over phenytoin
patient, but they would not be diagnostic of this patients because it is water soluble and may be infused more rapidly
condition (Answers A and E). Pulmonary embolism can and results in less local irritation. While the absence of the
certainly cause a sudden reduction in oxygen saturation, but propylene glycol carrier in fosphenytoin should in theory
chest CT angiography would not be the first test of choice lead to fewer cardiovascular side effects, several studies sug-
(Answer B). gest that the risk of hypotension does not different between
the two drugs (Answer B). Diazepam has higher lipid sol-
ubility and likely faster, not slower, onset of action than
lorazepam (Answer C). For some cases of status epilepticus,
46. ANSWER:C. Endotracheal suctioning treatment with lorazepam alone is sufficient. However, in
many cases addition of phenytoin or fosphenytoin can help
The pressure required to deliver a breath to a mechani- provide a prolonged antiseizure effect, and thus represents
cally ventilated patient can be simplified into two compo- a reasonable treatment in this case. While propofol has not
nents:airway resistance and lung compliance (or conversely, been as well-studied as benzodiazepines or phenytoin, it has
lung elastance). Problems with airways resistance and lung been used successfully to treat status epilepticus in several
compliance can increase peak inspiratory pressures, but small trials (Answer E).
only problems affecting lung compliance will increase the
plateau pressure. The plateau pressure is calculated by per-
forming a breath-hold at end-inspiration, and subtracting
PEEP from the measured pressure. Since this pressure is 48. ANSWER:E. End-tidal CO2 detection may
calculated at zero flow, it reflects the pressure required to help determine the quality of cardiopulmonary
distend the lung parenchyma by the given tidal volume, resuscitation (CPR) and return of spontaneous
independent of airway resistance. In this case, although circulation.
the plateau pressure before the event is not given, it is not
likely to be significantly increased, because the plateau pres- Carbon dioxide is detected in exhaled breath in the presence
sure must be lower than the peak pressure and because in of circulation. Thus, end-tidal carbon dioxide detection has

4 . P u l m o n a r y a n d C r i t i c a l C a r e M e d i c i n e 9 9
been advocated as a method both to assess for adequacy of Assessment of adrenal insufficiency in critically ill patients
CPR and to detect the return of spontaneous circulation. is challenging, due to wide variations in serum cortisol
Over the past several years, advanced cardiac life support levels and in free versus bound cortisol. However, in the
(ACLS) guidelines have been updated to reflect studies absence of true adrenal insufficiency, neither the absolute
highlighting the detrimental effects of inadequate and serum cortisol level nor the response to ACTH appears reli-
interrupted chest compressions, which result in decreased ably to identify patients who will benefit from corticoste-
cardiac and cerebral perfusion. They have also discouraged roids (Answer B).
emphasis on establishing an advanced airway with assisted
ventilation, which not only interrupts chest compressions
but also may lead to overventilation, increased intrathoracic
pressure, and decreased venous return. 50. ANSWER:E. High-dose insulin therapy
While hypoxemia and hypercapnia can cause cardiac
arrest, inadequate bag-and-mask ventilation is usually not Calcium-channel blocker overdose is associated with sub-
the cause of failure to recover from an arrest (Answer A). In stantial morbidity and mortality. The effects of overdose
this case, overventilation is a particular concern given the depend on the type and dose of calcium channel blocker.
patients COPD. ACLS guidelines have recognized a detri- In addition to vasodilation and decreased inotropy, dil-
mental effect on effective CPR from frequent interruptions tiazem and verapamil, but not the dihydropyridines, typi-
for pulse checks and consequently recommend immediately cally result in bradycardia. Recognition of the overdose is
resuming compressions for 2 minutes after administration important, as hemodynamic deterioration can be delayed
of drugs (such as epinephrine) or a defibrillation attempt by hours after the ingestion. In addition to supportive
(Answers C and D). Rapid response teams have not con- and symptomatic care, treatment usually includes intrave-
vincingly been shown to decrease mortality, but several nous calcium and glucagon. Whole bowel irrigation may
studies have demonstrated that they can reduce the inci- be appropriate for ingestion of extended-release prepara-
dence of in-hospital cardiac arrest (Answer B). tions. Temporary pacemaker support may be indicated
for persistent bradycardia. High-dose insulin therapy has
been increasingly advocated for cases of calcium-channel
blocker (and beta-blocker) overdose [Intensive Care Med.
49. ANSWER:C. Corticosteroids will likely improve 2007;33:201924]. The mechanisms of action are unclear,
his response to vasopressors. but calcium channel blockers may cause insulin resistance
in the myocardium that is overcome by high doses of insu-
Despite decades of study, the use of corticosteroids in sepsis lin. While this therapy is usually instituted with a concomi-
and septic shock remains controversial. Older studies using tant glucose infusion, calcium channel blocker overdose
high-dose steroids found no beneficial effect and a possible itself can cause impaired glucose metabolism, and patients
increase in mortality (Answer E). Astudy of moderate-dose are often hyperglycemic and may not require additional glu-
corticosteroids (200 mg of hydrocortisone) appeared to cose. Recently, another treatment, intravenous lipid emul-
demonstrate benefit, limited to those who were deemed sion, has been advocated for use in several types of severe
to have inadequate adrenal reserve after an ACTH stimu- overdoses, including calcium-channel blocker overdose
lation test. However, this favorable finding was not con- [ JIntensive Care Med. 2012; June24]. While this therapy
firmed in a subsequent trial (Answer A). Experts continue has not been well studied, results from animal studies and
to recommend moderate-dose steroids for septic shock case reports of severe toxicity in humans have demonstrated
with continued need for vasopressors, based on a consistent evidence of benefit. The mechanism of action is unclear,
improvement in hemodynamics and response to vasopres- but it may work by acting as a lipid sink for the toxic
sors [ JAMA. 2009;301:236275]. medication.
The use of corticosteroids in ARDS, as in septic shock, Hemodialysis is not effective for removing verapamil
is controversial. More data are available for the use of ste- because verapamil is highly protein bound (Answer A).
roids late in the course of ARDS. In an ARDSNet trial Bicarbonate infusion may be indicated in the setting of
of late rescue (after 7 days), use of steroids was associated severe academia or a widened QRS complex due to sodium
with fewer days in shock and shorter duration of mechani- channel blockade, as in tricyclic overdose (Answer B).
cal ventilation, but a higher rate of reintubation [N Engl J Physostigmine is a therapy for anticholinergic overdose
Med. 2006; 354:167184]. Mortality was increased in the (Answer D). Intravenous corticosteroids are not helpful in
subgroup who received therapy after 14days (Answer D). calcium-channel blocker overdose (Answer E).

10 0 T h e B r i g h a m I n t e n s i v e R e v i e w o f I n t e r n a l M e d i c i n e Q u e s t i o n a n d A n s w e r C o m pa n i o n
5.
ENDOCR INOLOGY

Carolyn Becker, Amir Tirosh, Ole-Petter R. Hamnvik, Le Min, Klara Rosenquist, Anand Vaidya,
Whitney W. Woodmansee, Bindu Chamarthi, and Matthew Kim

1. An 89-year-old woman is seen at your clinic for rou- 3. A 44-year-old man is referred to your clinic for the
tine physical examination. She is known to have predia- management of new-onset type 2 diabetes. The patient
betes for many years and you have recently diagnosed has fasting blood glucose in the 130150 mg/dL range
her with new-onset type 2 diabetes. She is asymptom- and a hemoglobin A1c of 7.6%. Past medical history is
atic but her fasting blood glucose was 145 mg/dL and significant for obesity (body mass index 31.2 kg/m2)
her recent hemoglobin A1c was 7.5%. Past medical his- and asthma. He is not taking any medications. Liver and
tory is significant for hypertension, hypercholesterol- kidney functions are normal. You counsel the patient
emia, osteoarthritis, and cataract. Medications include about recommended lifestyle modifications. What is
amlodipine and atorvastatin. Serum creatinine was the recommended first-line treatment for this patient?
measured at 1.3 mg/dL.
You refer the patient for diabetes education and to A. Weight loss only
a dietician for nutritional counseling. What would B. Weight loss + a DPP4 inhibitor
be your next step in diabetes management in this C. Weight loss + a GLP-1 agonist
patient? D. Weight loss + Metformin
E. Weight loss + Sulfonylurea
A. No further treatment
B. Initiate metformin. 4. A 67-year-old man has been diagnosed with type 2
C. Initiate sulfonylurea. diabetes 14years ago. He also has hypertension for the
D. Initiate basal insulin. last 7years. Your recent evaluation was significant for
E. Initiate basal and prandial insulin. urinary microalbumin of 79 mg/day (normal <30 mg/
day) and creatinine of 1.1 mg/dL, and a dilated eye
2. A 39-year-old man is admitted to the intensive examination revealed proliferative diabetic retinopa-
care unit for severe community-acquired pneumonia thy. Medications include metformin, insulin, losartan,
requiring intubation. He is otherwise healthy, and he amlodipine, and aspirin. Hemoglobin A1c measured
is not known to have diabetes. Random blood glucose today is 7.6%. The patient has no episodes of hypogly-
levels during the first day in the ICU are 183, 195, cemia. Blood pressure was measured at 137/78mm Hg.
and 182 mg/dL. Hemoglobin A1c was 5.5%. What What is the best next step recommended in the manage-
would be your next step in glucose management of this ment of this patients diabetic complications?
patient?
A. Intensify blood pressure control.
A. Glucose monitoring only B. Intensify glucose control.
B. Intravenous insulin with glucose target of 110 to C. Discontinue metformin.
140mg/dL D. Add an angiotensin-converting enzyme inhibitor.
C. Intravenous insulin with glucose target of 140 to E. Discontinue aspirin.
180mg/dL
D. Sliding scale insulin 5. A68-year-old white woman has recently been diag-
E. Subcutaneous basal insulin nosed with hyperglycemia. Her past medical history

101
is remarkable only for cigarette smoking for the last no evidence for other diabetes-related complications.
30 years. Physical examination is significant for a For the last 5years her hemoglobin A1c has been con-
body mass index of 27.1 kg/m2 and blood pressure of sistently around 6.5%. She is treated with Metformin,
146/94mm Hg. Liraglutide, and 22 units of insulin glargine. Following
You order the following laboratory tests: fasting blood repeated episodes of asymptomatic hypoglycemia
glucose: 118 mg/dL, hemoglobin A1c: 6.3%. Total cho- you have decreased the insulin dose to 18 units. She
lesterol: 205 mg/dL, low-density lipoprotein (LDL) cho- has come today to review her blood glucose readings.
lesterol: 127 mg/dL, high-density lipoprotein (HDL) Morning glucose levels are between 90 and 125 mg/
cholesterol: 46 mg/dL, triglycerides: 165 mg/dL. dL, and before lunch and dinner glucose levels range
You recognize that this patient is at high risk for car- between 125 and 165 mg/dL. She no longer has hypo-
diovascular disease. Which of the following cannot be glycemic events since the adjustment of the insulin
used as an approach to decrease her cardiovascular risk? dose. However, hemoglobin A1c has increased to 7.4%.
The patient is concerned with the recent increase in
A. Prescribing Metformin to better control fasting hemoglobin A1c and she would like to optimize her
hyperglycemia regimen to meet treatment goals.
B. Prescribing a lipid-lowering medication, with a What is the best next step in management of this
target LDL cholesterol of <100 mg/dL patients diabetes?
C. Smoking cessation
D. Prescribing the patient with an aspirin A. Increase insulin glargine to keep the hemoglobin
E. Prescribing the patient with a blood pressure A1c below 7.0%.
medication B. Prescribe the patient with a continuous
glucose-monitoring device.
6. A44-year-old man is referred to your vlinic for diabetes C. Discontinue liraglutide and start prandial insulin.
management. Following a routine screening 6 months D. Add another oral medication to keep hemoglobin
ago he was found to have a fasting glucose level of 129 A1c below 7.0%.
mg/dL and hemoglobin A1c of 6.8%. He was started on E. No further adjustment of her regimen
Metformin 1,000 mg twice daily, which was effective
for several months in controlling his blood sugars. For 8. A60-year-old woman is found to have mild primary
the last month the patients fasting blood glucose lev- hyperparathyroidism on routine chemistry screening
els are elevated again at 150 to 180 mg/dL. After meals with serum calcium 10.9 mg/dL (normal 8.610.4),
most of his readings are in the 200300 mg/dL range, albumin 4.0 g/dL, and intact PTH 76 pg/mL (normal
and his most recent hemoglobin A1c was 8.6%. The 1065). Vitamin D levels are normal. Screening bone
patient has no other medical conditions. Blood pressure density shows osteoporosis at the spine. She generally
is 125/76 mm Hg and body mass index is 23.4 kg/m2 . feels well and denies any history of kidney stones. She
The rest of physical examination is unremarkable. Blood refuses to consider parathyroid surgery at this time.
tests were significant for HDL cholesterol of 46 mg/dL What do you recommend for her?
and fasting triglycerides of 132 mg/dL. The patient has
no other family members with diabetes. A. Neck ultrasound and Sestamibi scanning
What would be your next step in the management of B. 24-hour urine to measure calcium excretion
this patient? C. Cinacalcet 30 mg orally twice daily
D. Alendronate 70 mg orally weekly
A. Initiate treatment with insulin. E. Annual calcium, creatinine, and bone mineral
B. Add an oral antidiabetes medication. density testing
C. Evaluate for possible Cushings syndrome.
D. Test for antiglutamic acid decarboxylase (GAD) 9. A 75-year-old man with a remote history of papillary
antibodies. thyroid cancer is diagnosed with metastatic prostate
E. Test for maturity-onset diabetes of the young cancer. He undergoes prostate surgery, local radiation,
(MODY) mutations. and androgen deprivation therapy. He also receives 4
mg of zoledronic acid intravenously to prevent bone
7. A 78-year-old woman is seen at your clinic for loss. Five days later he presents to the emergency room
long-standing type 2 diabetes of over 25years. She also with severe paresthesias and muscle cramping. On
has hypertension and anterior wall myocardial infarc- examination he has positive Chvostek and Trousseaus
tion diagnosed 2years ago. Recent eye examination is signs and a well-healed neck scar. Laboratory data
significant for a mild nonproliferative diabetic retinop- include TSH 0.54, alkaline phosphatase 120 U/L (nor-
athy, which has been stable for many years, and she has mal), 25OH-vitamin D 25 ng/mL, serum calcium 6.5

10 2 T h e B r i g h a m I n t e n s i v e R e v i e w o f I n t e r n a l M e d i c i n e Q u e s t i o n a n d A n s w e r C o m pa n i o n
mg/dL (normal 8.610.4), albumin 4.0 g/L, and phos- aminotransferases. Aplain X-ray of the hip and pelvis
phate 6.0 mg/dL. What is the most likely etiology of the shows patchy areas of osteolysis with multifocal scle-
hypocalcemia? rotic patches of the left hemipelvis, including near the
hip joint. Abone scan shows uptake of the radiotracer
A. Osteoblastic bone metastases in the pelvic lesion.
B. Vitamin D deficiency Which of the following is the best management strat-
C. Hypoparathyroidism egy at this time?
D. Hungry bone syndrome
E. Hypomagnesemia A. Monitor alkaline phosphatase levels with no
treatment unless it exceeds 400 IU/L.
10. A 38-year-old woman is evaluated for progres- B. Prescribe alendronate 70 mg by mouth once
sive shortness of breath, decreased exercise tolerance, weekly.
and extreme fatigue. Heart and lung examinations are C. Refer for zoledronic acid infusion, 5 mg
negative, but chest X-ray shows bilateral interstitial intravenously.
infiltrates and ground glass opacities. Routine serum D. Prescribe salmon calcitonin, 50 units subcutaneously
chemistries show serum calcium 11.8 mg/dL, albumin once daily.
4.0 g/dL, and intact PTH <10 pg/mL. Which test is
most likely to explain the hypercalcemia? 13. A54-year-old man who is previously healthy is being
worked up for weakness and generalized body aches,
A. 1,25-dihydroxy-vitamin D progressively worsening over 10months. He denies any
B. 25-hydroxy-vitamin D change in food intake or bowel habits. Other than acet-
C. PTHrP aminophen, he has not taken any medications. He does
D. Technetium 99m bone scan not smoke or consume alcohol. Physical examination
E. 24-hour urinary calcium excretion was remarkable for bony tenderness throughout. Aplain
X-ray of a particularly painful right humerus showed
11. A 50-year-old man presents with a wrist fracture severe osteopenia and metaphysial changes consistent
after a minor fall. He is generally in good health other with osteomalacia. Laboratory testing found phos-
than a history of irritable bowel syndrome and some phorus of 0.9 mg/dL (reference range 3.55.0 mg/dL),
weight loss. He takes 1,200 mg calcium + 800 IU alkaline phosphatase 234 IU/L, 25-hydroxy-vitamin D
vitamin D3 daily because his mother had osteoporo- 35 ng/mL, and a normal basic metabolic panel and
sis. Bone mineral density measurement shows a T-score complete blood count.
of2.5 at the hip and3.0 at the 1/3 distal radius of the Which of the following laboratory tests is most likely
nonfractured wrist. On examination, he has a scaly skin to be elevated?
rash on his arms and trunk. Labs reveal serum calcium
8.7 mg/dL, 25OHD 24 ng/dL, intact PTH 120 pg/mL, A. Intact PTH
and 24-hour urine calcium excretion 38 mg with cre- B. 1,25-dihydroxy-vitamin D
atinine 1,000 mg. What intervention is most likely to C. Fibroblast growth factor-23 (FGF-23)
improve his skeletal health? D. Calcitonin
E. 24-hour urine calcium
A. Doubling his calcium intake to 2,400 mg daily
B. Doubling his vitamin D intake to 1,600 IU daily 14. A 65-year-old Caucasian woman is noted to have
C. Oral alendronate 70 mg weekly osteopenia on bone mineral density screening. When
D. Parathyroidectomy you calculate her FRAX score, you find that she has a
E. Starting a gluten-free diet 22% chance of sustaining a major osteoporotic fracture
over the next 10years.
12. A 74-year-old woman is complaining of left hip According to the National Osteoporosis Foundation,
pain for the past year, which has been progressively antiresorptive therapy should be considered when the
worsening. She denies early morning stiffness, fevers, 10-year absolute risk of a major osteoporotic fracture is
chills, weight loss, or trauma. On physical examina- above what level?
tion, her left hip has intact range of motion with no
pain on active or passive range of motion. Routine A. 5%
laboratory work reveals an elevated alkaline phospha- B. 10%
tase of 205 IU/L and a 25-hydroxy-vitamin D level of C. 15%
32 ng/mL; the remainder of her laboratory panel was D. 20%
normal, including calcium, phosphate, creatinine, and E. 25%

5 . E n d o c r i n o l o g y 10 3
15. A68-year-old woman presents for follow-up after a (normal range 2201000). On examination, he has a
spontaneous vertebral fracture. Abone mineral density BMI of 34 but is normally virilized with testicular size
(BMD) scan showed osteoporosis, with a T-score of3.5 of 15 mL bilaterally. What is the next best step?
at the lumbar spine and2.6 at the femoral neck. Three
years earlier, a screening scan showed only mild osteo- A. Start testosterone replacement
penia. She underwent menopause at age 49. Over the B. Pituitary MRI
past 3years, she has been treated for hypertension, type C. Testicular ultrasound
2 diabetes mellitus, and passage of a kidney stone. Her D. Measure AM total testosterone and sex hormone
physical examination is notable for an obese woman binding globulin
with a flushed face, proximal myopathy, and tenderness E. Karyotyping
over the mid-thoracic spine.
In addition to a CBC, basic metabolic panel, and 19. A 55-year-old man with history of advanced
25-hydroxy-vitamin D level, which of the following tests melanoma was diagnosed with panhypopituita-
is most likely to explain her worsening osteoporosis? rism secondary to ipilimumab-related hypophysitis.
His biochemical testing included an undetectable
A. Serum protein electrophoresis total testosterone and very low LH and FSH levels.
B. Serum tryptase You start him on testosterone replacement with a
C. Serum intact PTH daily topical gel. Three months later, he returns for
D. Serum IGF-1 follow-up and feels greatly improved. What tests are
E. Midnight salivary cortisol recommended?

16. A 19-year-old boy with an unremarkable medical A. Total testosterone


history presents because of delayed puberty. His height B. Prostate-specific antigen
is 67.5 in., arm span 70 in., and his weight is 202 lb. On C. Hematocrit
physical examination, he has Tanner stage 1 pubic hair, D. All of the above
gynecomastia, and prepubertal-sized testes. Sense of E. No further testing needed
smell is intact. No history of testicular injury or mumps
orchitis. Labs show total testosterone 88 ng/dL (nor- 20. A 20-year-old woman presents to clinic for oli-
mal 2201,000; confirmed by repeat testing), LH 30 gomenorrhea. She went through menarche and adre-
mU/mL (normal 1.78.6), FSH 32 mIU/mL (normal narche consistent with her peers. She is currently obese
1.512.4), normal prolactin, and A1C 6.6%. What other and has struggled with her weight since her teens. She
testing would be warranted? is concerned about several years of hair thinning, male
pattern balding, and facial and chest terminal hair
A. Echocardiogram growth. She has minimal acne. She has the follow-
B. Renal ultrasound ing laboratory tests; total testosterone 42.4 ng/dL,
C. Pituitary MRI SHBG 12 nmol/L, prolactin 9.6 ng/ml, FSH 5.2
D. Ferritin, iron/TIBC mIU/mL, LH 11.8 mU/mL, HgA1c 5.2%. She also
E. Karyotyping has a normal dexamethasone suppression testing and
normal 17 OH progesterone. You diagnose her with
17. A28-year-old male presents for evaluation of infer- PCOS. She is not currently interested in pregnancy
tility. Serum LH, FSH, and testosterone levels are all and you start treatment with an estrogen-progestin
normal. Semen analysis reveals azoospermia. Which of contraceptive. After 6 months her hirsutism persists
the following gene mutations is most likely related to and she is very concerned about this. What is the next
the above findings? best treatment strategy?

A. Kiss1 gene A. Stop treatment with an oral contraceptive and start


B. KAL1 gene treatment with metformin 1,000 mg twice daily.
C. CFTR gene B. Continue treatment with the oral contraceptive and
D. FGFR1 gene reassure her that her hirsutism will improve.
E. TAC3 gene C. Continue treatment with the oral contraceptive and
encourage weight loss.
18. A48-year-old male with history of obesity and type D. Stop treatment with an oral contraceptive and start
2 diabetes mellitus presents with low serum total testos- spironolactone 50 mg twice daily.
terone. He notes decreased libido and erectile dysfunc- E. Continue treatment with the oral contraceptive and
tion. An afternoon total testosterone was 200 ng/dL add spironolactone 50 mg twice daily.

10 4 T h e B r i g h a m I n t e n s i v e R e v i e w o f I n t e r n a l M e d i c i n e Q u e s t i o n a n d A n s w e r C o m pa n i o n
21. A 28-year-old woman presents to clinic with dL, Glucose 90 mg/dL, FSH 5.7, TSH 60, Prolactin 35,
new-onset amenorrhea of 6 months duration. She had Cholestorol 276 mg/dL, HDL 47 mg/dL, LDL 169 mg/
menarche at the age of 13. She has had normal, 28-day dL. What is the best treatment option for this patient?
menstrual cycles for the majority of her life until approx-
imately 6months ago. She specifically denies acne, hir- A. Treatment with cabergoline
sutism, galactorrhea, headaches, visual changes, and B. Treatment with a statin
heat or cold intolerance. She does, however, report that C. Treatment with HRT
she has recently started running competitively and is cur- D. Treatment with levothyroxine
rently training for her third marathon. She reports that E. 1 L fluid restriction
after experiencing a stress fracture during training for
her second marathon she was disappointed by her mar- 24. A56-year-old woman presented to the emergency
athon results. She is now following a caloric restricted room with right lower quadrant abdominal pain. An
diet and training 6days a week to improve her results at abdominal CT scan was performed to evaluate for
the upcoming marathon. What is the most likely etiol- appendicitis. No abnormal bowel findings, or other
ogy of this young womens secondary amenorrhea? abnormalities to account for the pain, were identi-
fied. However, a 1.2cm adrenal mass was incidentally
A. Hyperprolactinemia detected in the medial limb of the left adrenal gland.
B. PCOS The mass was described as homogenous, round, with
C. Cushings disease smooth borders, and a lipid-rich density (6 Hounsfield
D. Hypothalamic amenorrhea units). She denied any episodic adrenergic symptoms,
E. Pregnancy weight gain, or muscle weakness. She had no his-
tory of hypertension, impaired glucose handling, or
22. A 40-year-old woman presents to clinic for new- low bone density. Which of the following would you
onset amenorrhea of 12 months duration. She has two advise?
children and had previously had regular 28-day men-
strual cycles for her reproductive life. Her mother went A. No further testing needed
through menopause at the age of 50, and she is con- B. Serum catecholamines
cerned that she is now going through early menopause. C. Dexamethasone suppression test
On further questioning she reports a 25 lb weight gain D. Corticotropin stimulation test
over the last year. She also reports new-onset fatigue E. Magnetic resonance imaging of the adrenal gland
and has found it difficult to keep up with her children.
She had no previous history of headaches but over the 25. A 45-year-old woman had a 5-year history of
last 6 months has noted several episodes of headaches. hypertension and anxiety. She was treated with lisin-
On examination her BMI is 31 kg/m2, and she has a opril and venlafaxine. In the preceding 5 weeks, she
round, plethoric face with supraclavicular fullness. She reports a new increase in anxiety, palpitations, and
has evidence of abdominal striae. She has no express- her blood pressure has been trending higher than
ible galactorrhea. Which lab test would most likely usual at ~145/80 mm Hg (typically 135140/80 mm
help to determine the cause of this womens secondary Hg). She has been under a lot of pressure at work,
amenorrhea? which has heightened her anxiety. She denies any
illicit drug use. Plasma metanephrines were ordered
A. LH to FSH ratio and revealed: plasma metanephrines <0.20 nmol/L
B. Serum prolactin (00.49) and plasma normetanephrines 1.6 nmol/L
C. 24-hour urine free cortisol (00.89). You inform her that the most likely cause of
D. TSH, FT4 her elevated normetanephrines is:
E. HgA1c
A. Pheochromocytoma
23. A 45-year-old woman presents to clinic for her B. Extra-adrenal paraganglioma
annual examination. She is overall doing well but does C. Metamphetamine use
report new-onset fatigue, constipation, and an ~7 lb D. Lisinopril use
weight gain over the last year. She attributes these to E. Venlafaxine use
work-related stress. On further questioning she reports
oligomenorrhea. Her mother went through meno- 26. A 55-year-old man had an abdominal CT scan to
pause at the age of 50 and the patient believes she may evaluate the source of abdominal pain. No acute abnor-
be perimenopausal. Labs show the following: Sodium malities to explain his abdominal were found, and his
130 mEq/L, potassium 3.5 mEq/L, creatinine 0.80 mg/ pain spontaneously resolved. However, a 3cm adrenal

5 . E n d o c r i n o l o g y 10 5
mass was noted in his left adrenal gland. The mass was 29. A 50-year-old woman with multiple sclerosis was
described as round, without calcifications, and with a treated for over 1 year with high-dose prednisone. Her
very lipid-rich density (50 Hounsfield units). He had a prednisone dose was as high as 60 mg daily, and over
normal blood pressure and denied any episodic adrener- the course of the last 1 year, as her neurologic symptoms
gic symptoms, weight gain, muscle weakness, alopecia, improved, her prednisone dose was decreased gradually
or abnormal hair growth. He denied melena or blood and ultimately stopped. Two weeks after her last predni-
in his stools, and he had recently had a normal colo- sone dose, she was scheduled for an elective orthopedic
noscopy and prostate-specific antigen. He did smoke surgical procedure. She tolerated the anesthesia induc-
one pack of cigarettes a day for more than 25years, but tion and surgical procedure without difficulty and mini-
denied cough, bloody sputum, shortness of breath, or mal blood loss. However, postoperatively in the recovery
fever. What is the most likely diagnosis? area she remained drowsy and difficult to arouse. She
complained of severe pain at the surgical site, nausea,
A. Adrenal adenoma and vomited twice. Her blood pressure was 70/48 mm
B. Adrenal myelolipoma Hg with a heart rate of 120 beats per minute, which was
C. Adrenal pheochromocytoma substantially lower than her preoperative and intraop-
D. Adrenal carcinoma erative values. In addition to intravenous hydration and
E. Lung cancer metastases to the adrenal gland other assessment of her worsening condition, which of
the following medication(s) should be initiated?
27. A 25-year-old woman with hypothyroidism pre-
sented to the emergency room complaining of weak- A. Fludrocortisone
ness. She was described as feeling dizzy and lightheaded B. Hydrocortisone
by her friends while out on a hot summer day. In the pre- C. Hydrocortisone and fludrocortisone
ceding weeks, her friends had noticed that she had lost D. Prednisone and fludrocortisone
weight and was increasingly fatigued during the day.
In the emergency room the patient appeared lethargic 30. A 48-year-old man had a history of hypertension
and weak. Her sodium was 133 mEq/L and her potas- for the past 7 years. His treatment included lisino-
sium was 5.7 mEq/L. At 2 p.m., a plasma ACTH was pril, amlodipine, and hydrochlorothiazide. Despite
550 pg/mL (1060) and a serum cortisol was 2.4 g/dL these three medications, his blood pressure remained
(2.319.4). The patient was given 250 g of ACTH for at 170/100 mm Hg. On a routine physical examina-
a stimulation test and 60 minutes later her cortisol was tion, his serum potassium was found to be 2.7 mEql/L.
4.1 g/dL. The most likely diagnosis is: Previously, while on hydrochlorothiazide, his potas-
sium had been between 3.3 and 3.7 mEql/L. He was
A. Primary adrenal insufficiency given potassium chloride supplementation, but his
B. Secondary adrenal insufficiency serum potassium remained less than 3.5 mEq/L. To
C. Ectopic ACTH syndrome further address the hypokalemia, his hydrochlorothia-
D. Cushings disease zide was stopped and instead he was treated with spi-
E. Cushings syndrome ronolactone. With the addition of spironolactone for 4
weeks, his blood pressure improved to 140/80mm Hg
28. A74-year-old man with metastatic prostate compres- and his potassium stabilized at 3.7 mEq/L. Abiochemi-
sion developed weakness of his legs and incontinence. cal workup for hyperaldosteronism revealed a serum
Imaging studies revealed metastatic foci of cancer in his aldosterone of 42 ng/dL and a plasma renin activity of
spine with evidence of spinal cord compression. He was 4.2 ng/mL per hour. You tell the patient he has:
started on dexamethasone 2 mg every 6 hours to reduce
spinal cord edema, and radiation therapy was initiated. A. Primary hyperaldosteronism
The following day he experienced transient tachycardia B. Secondary hyperaldosteronism
and lightheadedness with mild orthostatic hypoten- C. Essential hypertension
sion. Amorning cortisol was 0.5g/dL. The best inter- D. Renal tubular acidosis
pretation of this cortisol value is: E. Insufficient information to confirm a diagnosis

A. Primary adrenal insufficiency 31. A 74-year-old male with a history of hypertension


B. Secondary adrenal insufficiency due to metastatic and hypercholesterolemia is hospitalized after pre-
lesion to the pituitary senting with a 3-month history of progressive fatigue
C. Secondary adrenal insufficiency due to radiation and dyspnea on exertion. His weight on admission is
therapy 164 lb, his pulse is 44 beats per minute, and labora-
D. This cortisol value cannot be adequately interpreted. tory tests show CPK 528 U/L, TSH 65 mU/L, free

10 6 T h e B r i g h a m I n t e n s i v e R e v i e w o f I n t e r n a l M e d i c i n e Q u e s t i o n a n d A n s w e r C o m pa n i o n
T4 0.2 ng/dL, and T4 2.3 g/dL. A pharmacologic B. Start methimazole at a dose of 10 mg daily.
nuclear stress test reveals findings consistent with C. Refer the patient to a thyroid surgeon.
diffuse ischemia. Subsequent coronary angiography D. Administer a 30 mCi dose of I-131.
reveals diffuse three vessel disease that is not ame- E. Start levothyroxine at a dose of 137g daily.
nable to percutaneous stenting. A consulting cardi-
ologist has recommended that he undergo coronary 34. A 33-year-old male is noted to have palpable
artery bypass surgery. What would you recommend? enlargement of the right side of his thyroid on a routine
physical examination. A thyroid ultrasound reveals a
A. Start levothyroxine at a dose of 125g daily. solitary 3.1 cm right-sided nodule with smooth bor-
B. Administer a 60g intravenous dose of ders. Laboratory tests show TSH 0.1 mU/L and T4
levothyroxine daily. 11.5 g/dL. He reports a history of occasional symp-
C. Check antithyroid peroxidase and antithyroglobulin tomatic palpitations and weight loss of 5 lb over the
antibodies. course of 3 months despite an increase in his appetite.
D. Start levothyroxine at a dose of 12.5g daily. He is not taking any medications and has not noted
E. Defer treatment with thyroid hormone replacement any problems with dysphagia or dysphonia. What
until after he has undergone revascularization. should you do next?

32. A 28-year-old female with a 6-year history of hypo- A. Perform a fine-needle aspiration biopsy of the
thyroidism has been treated with levothyroxine at a right-sided nodule.
dose of 125 g daily. Laboratory tests checked 5 months B. Administer a 15 mCi dose of I-131.
ago right before she stopped taking cyclic oral contra- C. Refer the patient to a thyroid surgeon.
ceptives showed TSH 1.3 mU/L. Three weeks ago she D. Start methimazole at a dose of 5 mg daily.
checked a home pregnancy test that was positive. She E. Check a thyroid scan.
had lab tests checked by her obstetrician that showed
hemoglobin 10.2 g/dL, T4 12.5 g/dL, and T3 215 ng/ 35. A 55-year-old female is noted to be tachycardic
dL. She is estimated to be at 8 weeks gestation and has with a resting pulse of 104 beats per minute dur-
started taking prenatal vitamins and iron sulfate at a ing a routine physical examination. Laboratory
dose of 325 mg twice daily. What should you do? tests checked to evaluate this show TSH 0.002
mU/L with follow-up lab tests showing free T4
A. Have her continue levothyroxine at a dose of 125g 2.3 ng/dL, T4 14.7 g/dL, and T3 289 ng/dL.
daily with instructions to take it regularly with other She denies any history of anterior neck discomfort,
medications. weight loss, palpitations, anxiety, tremor, heat intoler-
B. Increase her dose of levothyroxine to 150g daily. ance, or insomnia. Physical examination reveals tachy-
C. Decrease her dose of levothyroxine to 112g daily. cardia with a regular rhythm, a slightly enlarged thyroid
D. Check a thyroid uptake and scan. without any discrete nodularity, and no evidence of
E. Check a TSH level. proptosis or ocular irritation. She does have a history
of osteopenia with a lumbar spine T-score of 2.3 iden-
33. A 76-year-old female presenting with a 1-week his- tified on a DEXA scan checked soon after the onset of
tory of a cough, fever, and audible stridor is diagnosed menopause. What should you do next?
with community-acquired pneumonia. A chest X-ray
does not show evidence of an infiltrate, but it does reveal A. Check antithyroid peroxidase and antithyroglobulin
marked rightward tracheal deviation with a visible antibodies.
mediastinal soft tissue mass. A noncontrast chest CT B. Check a thyroid uptake and scan.
scan reveals multiple bilateral thyroid nodules with an C. Start methimazole at a dose of 10 mg daily.
8.5 cm left lower pole nodule extending below the clav- D. Refer the patient to a thyroid surgeon.
icle and sternum with compression and narrowing of E. Check an ESR.
the trachea. Laboratory tests show TSH <0.001 mU/L,
free T4 2.8 ng/dL, T4 12.9 g/dL, and T3 245 ng/dL. A 36. A 62-year-old male with a history of poorly con-
thyroid uptake and scan reveals 24-hour uptake of 37% trolled type 2 diabetes is hospitalized for treatment
with tracer accumulation localized to two right-sided of acute pyelonephritis after presenting with a 5-day
thyroid nodules and the substernal left-sided thyroid history of fever, dysuria, and left-sided flank pain.
nodule. What should you do next? Blood cultures checked on admission are positive for
E. coli. Laboratory tests checked on intake reveal TSH
A. Check pulmonary function tests with flow volume 0.22 mU/L, T4 3.1 g/dL, and T3 64 ng/dL. He is
loops. treated with broad spectrum intravenous antibiotics

5 . E n d o c r i n o l o g y 10 7
with minimal improvement. An abdominal CT scan In addition to repeating the prolactin level, what is
checked 6 days after admission reveals findings consis- the next test(s) that should be ordered in this patient?
tent with a left-sided perinephric abscess that requires
management with percutaneous drainage. The patient A. Serum pregnancy test
is discharged to a rehabilitation facility 23 days after B. Pituitary MRI
admission and has lab tests checked on intake that show C. TSH to rule out primary hypothyroidism
TSH 16.4 mU/L. His current weight is 218 lb. What D. Aand C
would you recommend? E. All of the above

A. Check a thyroid uptake and scan. 39. A35-year-old woman with history of regular men-
B. Start levothyroxine at a dose of 150g daily. ses presents with complaints of bilateral galactorrhea.
C. Start levothyroxine at a dose of 50g daily and She notes her menses have been more irregular over the
liothyronine at a dose of 5g twice daily. last year and her last menstrual period was approxi-
D. Wait 6 weeks and recheck a full profile of thyroid mately 6 weeks ago. She reports occasional headaches
hormone levels. but otherwise feels well. She does not take any medica-
E. Check a pituitary MRI scan. tion or herbal supplements. Laboratory testing reveals
a negative pregnancy test, TSH 2.2 mIU/L, and pro-
37. A31-year-old female presents with a 5-week history lactin of 360 ng/mL. You suspect a prolactinoma and
of weight loss and symptomatic palpitations. Lab tests order a pituitary MRI, which reveals a 10mm 12mm
demonstrate moderate thyrotoxicosis. She is diagnosed pituitary lesion. There is no cavernous sinus invasion or
with Graves disease based on clinical findings, includ- compression of the optic nerves by the pituitary tumor.
ing goitrous enlargement of her thyroid, mild proptosis, You advise the patient that the initial treatment of
and periorbital swelling. She is effectively treated with choice is:
methimazole for 12months, and then holds treatment to
see if she may have entered a state of remission. Lab tests A. Medical therapy with a somastatin analog
checked 2months after stopping methimazole show TSH B. Observation only
< 0.001 mU/L, free T4 1.9 ng/dL, T4 12.1g/dL, and T3 C. Transsphenoidal pituitary tumor resection
243 ng/dL. After options for further management are D. Medical therapy with a dopamine agonist
reviewed, the patient elects to proceed with radioactive E. Radiation therapy to the pituitary
iodine treatment after informing you that she hopes to
get pregnant in the next year. What should you tell her? 40. A61-year-old man who has a history of a nonfunc-
tioning pituitary macroadenoma presents for routine
A. She will need to be strictly isolated from family follow-up. He underwent surgical resection a number of
members for 2 weeks. years ago for a large tumor and fortunately has had no
B. Treatment may improve her thyroid eye disease. evidence of recurrence. He has hypopituitarism and is
C. She will not have to take any more medication after on replacement hormone therapy for hypothyroidism,
treatment. adrenal insufficiency, and hypogonadism. He takes levo-
D. The treatment should work immediately. thyroxine, hydrocortisone, and topical testosterone gel.
E. She should wait until she is 6months out from He returns for routine follow-up. He denies headaches
treatment before trying to conceive. or visual changes. He complains of fatigue and inabil-
ity to lose weight. He has noted very dry skin but attri-
38. A 27-year-old woman presents with a 3-month butes it to the weather. He denies tremor, palpitations,
history of amenorrhea. She reports menarche at age insomnia, or change in bowel movements, although he
12 and regular menses. She states she is healthy and tends toward constipation. He denies sexual dysfunc-
denies other medical problems. She denies galactor- tion. On physical examination, he has gained 6 lb since
rhea. She is not currently talking any medications his last visit. His blood pressure is 126/82 and pulse
except a birth control pill. She had noted a 10 pound is 76. He does not have lid lag and his visual fields by
weight gain over the last 6 months and occasionally confrontation are normal. His neurologic examination
feels cold. She attributes the weight gain to dietary is otherwise normal. No tremor is noted. He does not
changes and lack of exercise. Her family history is sig- look cushingoid.
nificant for a mother with primary hypothyroidism. His thyroid function tests show the following:TSH
Her examination is unremarkable except on breast 0.1 mIU/L (normal 0.55 mIU/L), Free T4 0.6 ng/dL
examination she had galactorrhea. A prolactin level (normal 0.91.7 ng/dL).
was ordered and comes back elevated at 52 ng/dL What adjustments should be made to his levothyrox-
(normal <22 ng/dL). ine dose?

10 8 T h e B r i g h a m I n t e n s i v e R e v i e w o f I n t e r n a l M e d i c i n e Q u e s t i o n a n d A n s w e r C o m pa n i o n
A. No change; continue current levothyroxine dose. his examination is unremarkable. Laboratory testing
B. Decrease levothyroxine dose. of his pituitary reveals normal thyroid, adrenal, and
C. Increase levothyroxine dose. gonadal function. However, his insulin-like growth
D. Discontinue levothyroxine completely. factor I (IGF-I) and growth hormone (GH) levels are
elevated at twice the upper limit of normal. Suspecting
41. A 47-year-old woman has a long history of having recurrence of his acromegaly, you obtain a pituitary
difficulty losing weight. She has multiple members MRI, which does not show obvious significant tumor
in her extended family that are overweight but states burden. An oral glucose tolerance test performed to
her parents and siblings are all of normal weight. She assess GH suppression confirms his acromegaly is not
states she has always struggled to maintain her weight in biochemical remission.
but in the last 2years she has gained 45 pounds despite You advise the patient that the initial treatment of
exercising daily and watching her caloric intake. She choice for his recurrent acromegaly is:
has recently been diagnosed with both type 2 diabetes
mellitus and hypertension. Last year she fell while rid- A. Medical therapy with a somastatin analog
ing her bicycle, broke her elbow, and was found to have B. Observation only
osteoporosis. She reports menarche at age 12 with reg- C. Repeat transsphenoidal pituitary tumor resection
ular menses until the last year when they have become D. Medical therapy with a dopamine agonist
increasingly unpredictable. She has been distressed by E. Radiation therapy to the pituitary
her weight gain and recently started taking an antide-
pressant. On physical examination she is obese with a 43. A 22-year-old man presents with headache and
BMI of 35kg/m2 , blood pressure 160/100mm Hg, and peripheral vision loss. Brain imaging reveals a large sel-
heart rate 72 beats per minute. She has a very round face lar mass with both solid and cystic components that is
with fine thin hair on her lateral cheeks. She has supra- compressing the optic chiasm. Humphrey visual field
clavicular fat accumulation and multiple pigmented testing demonstrated bitemporal hemianopsia. He has
striae on her abdomen. She has several bruises on her no previous medical history, his BMI is 20, and he is
lower extremities. You are concerned she may have taking no medications. His initial pituitary functional
Cushings syndrome. evaluation revealed central hypogonadism and central
Appropriate first-line screening tests for hypercorti- hypothyroidism. His adrenal function was normal.
solism include: MRI imaging characteristics suggested the mass was a
craniopharyngioma. He was taken to surgery to remove
A. Arandom isolated serum cortisol level and pituitary the pituitary mass in attempts to decompress the optic
MRI chiasm and restore his vision. He did well in the imme-
B. Late-night salivary cortisol test diate postoperative period, but within 24 hours he
C. 1 mg overnight dexamethasone suppression test began to develop polyuria. His urine output increased
D. 24-hour urine collection for free cortisol and to 400 mL/hr with a urine specific gravity of less than
creatinine 1.001. He complained of extreme thirst. His serum
E. Answers B, C, and D sodium increased to 148 mEq/L and his fasting glucose
was elevated at 106 mg/dL.
42. A 53-year-old man with a history of acromegaly What diagnosis are you suspecting in this patient?
presented to establish routine care with a primary care
provider. He underwent surgical resection of a pitu- A. Syndrome of inappropriate antidiuretic secretion
itary macroadenoma 6years prior to your visit and has (SIADH)
been lost to endocrine follow-up. He has a history of B. Diabetes mellitus, type 2
diabetes mellitus, hypertension, and obstructive sleep C. Central diabetes insipidus
apnea. He has been feeling poorly the last few months D. Nephrogenic diabetes insipidus
and presents with fatigue, diffuse joint pain, and exces- E. None of the above
sive sweating. He feels his hands and feet have been
swollen. He denies headaches or visual changes. His 44. A55-year-old woman was involved in a motor vehi-
medications include metformin and lisinopril. On cle accident as an unrestrained driver. Although she
examination, he has the classic course facial features of did not lose consciousness, she sustained a significant
acromegaly: enlarged tongue and spaces between his head trauma and was brought by ambulance to the
teeth. His blood pressure is 152/80mm Hg and heart emergency department. She was evaluated and found
rate is 64 beats per minute. His visual fields are normal to have a normal neurological examination but due to
on confrontation testing. He has multiple skin tags and the mechanism of her accident, brain imaging by non-
his hands are very large and sweaty. The remainder of contrast CT scan was performed. She was incidentally

5 . E n d o c r i n o l o g y 10 9
found to have a large sellar lesion, estimated at approxi- What would you recommend as the first step in the
mately 2cm in greatest diameter. No acute intracranial treatment of this patients dysglycemia?
hemorrhage was identified. She had a laceration on her
forehead that was repaired and since she was otherwise A. Dietary/lifestyle interventions
clinically stable she was discharged with a plan to follow B. Start metformin.
up with her primary care provider for further evaluation C. Start insulin therapy.
of this pituitary mass. D. Start a DPP4 inhibitor.
In evaluating a patient with a newly discovered
pituitary mass, what are the important initial clinical 46. A 55-year-man presents to your office as a new
considerations? patient to establish care after he recently moved to the
area. His past medical history is significant for history
A. Evaluation for mass effects (headaches, visual loss, of hypertension, which was diagnosed about 2 years ago
cranial nerve abnormalities) and has been well controlled with lisinopril 10 mg once
B. Evaluation for pituitary hormonal hypersecretion daily. He also has history of dyslipidemia for which he
C. Evaluation for pituitary hormonal hypofunction has been on a statin for 23 years and his lipid levels
D. All of the above have been at goal on his current dose of simvastatin 40
E. None of the above mg daily. He reports that he has been told in the past
that he has borderline diabetes. He has never been pre-
45. A 45-year-old man with no known history of heart scribed any medications for diabetes. He feels his diet
disease but a strong family history of premature coro- is reasonably healthy and he walks 1 mile 3 times/week
nary artery disease and a history of dyslipidemia for for exercise. He denies any other significant past medi-
which he is on therapy with atorvastatin 80 mg daily cal history. He feels well overall and his review of sys-
was recently found to have an HbA1c of 6.3% on rou- tems is negative. His current medications are lisinopril
tine screening blood tests. The patient states he tries and simvastatin as above, and he has no known drug
to eat a healthy diet but admits to eating large portion allergies. He does not smoke, drinks alcohol very occa-
sizes. His wife feels that there is certainly much room sionally, and has no history of illicit drug use. He has a
for improvement as far as the patients diet. He exercises family history of diabetes in his mother and his father
on a treadmill for 2030 minutes 2 times/week. He is had CAD, dx in his 60s. On examination, the patient is
overweight and reports that his weight has been stable a well-appearing, middle-aged man. His blood pressure
for many years. The patient feels well overall and has no is 120/70 mm Hg and heart rate 72/min. His physical
specific complaints today. examination is normal. Laboratory data reveal nor-
His past medical history is otherwise unremark- mal electolytes, serum creatinine 0.8 mg/dL, fasting
able except for the history of dyslipidemia. His current glucose 140 mg/dL, normal LFTs, total cholesterol
medications include Lipitor 80 mg once daily and aspi- 170 mg/dL, HDL 45 mg/dL, triglycerides 135 mg/dL,
rin 81 mg once daily. He has no known drug allergies. LDL 90 mg/dL, and HbA1c 7.2%. A urine test for micro-
He is married and has one child. He works as a fire- albumin is negative. After a detailed discussion of treat-
fighter. He quit smoking about 7 years ago and states ment options with the patient, the decision is made to
that, even prior to that, he was never a regular smoker start metformin 500 mg orally twice daily for his dia-
and smoked off and on only. Alcohol intake is about 5 betes. Patient was also counseled about lifestyle inter-
beers per week, and he denies any illicit drug use. His ventions and referred for formal nutrition consultation
family history is notable for both of his parents having and diabetes self-management education. You advise
type 2 diabetes mellitus and CAD, diagnosed in their him to continue his current doses of lisinopril and
60s, and a sister, who was a smoker and obese, who died statin as his blood pressure and lipids are at goal. What
of a heart attack at age 49. Many of his aunts and uncles other recommendation would you give this patient with
on his fathers side have diabetes as well, and three of his regard to cardiovascular risk reduction? Specifically,
maternal uncles had coronary artery disease diagnosed the patient asks if he should take an aspirin daily. What
in their 40s. would you advise?
On examination, his blood pressure was 126/78 mm
Hg, heart rate 70 beats per minute, and BMI 27. His A. No need for taking aspirin
physical examination was otherwise unremarkable. You B. Enteric-coated baby aspirin (81 mg) once daily
obtain screening laboratory tests and the results show C. Enteric-coated aspirin 325 mg orally once daily
a fasting blood glucose level of 110 mg/dL and a repeat
HbA1c is 6.2%. In addition, you receive copies of his old 47. A 55-year-old man with type 2 diabetes mellitus,
medical records, which indicate that his A1C was 6% CAD, hypertension, dyslipidemia, and obesity was
about a year ago. recently started on basal insulin when he was in the

110 T h e B r i g h a m I n t e n s i v e R e v i e w o f I n t e r n a l M e d i c i n e Q u e s t i o n a n d A n s w e r C o m pa n i o n
hospital a month ago with an acute myocardial infarc- fasting blood glucose level at admission on the morning
tion. The patient reports that he was diagnosed with of the surgery was 170 mg/dL. You see the patient in the
diabetes 5 years ago and was treated initially with met- cardiac surgical intensive care unit (ICU) the morning
formin alone and subsequently a combination of metfor- after his surgery. The blood glucose values in the operat-
min 1,000 mg twice daily and glipizide 10 mg twice daily ing room had ranged from 180 to 245 mg/dL. His glu-
prior to his recent hospitalization. The patient states that cose level was 195 mg/dL on his arrival in the ICU and
he was compliant with his medications but admits that has been in the 180s overnight. What would you recom-
prior to being admitted with the myocardial infarction, mend as treatment for his blood sugars at this time?
he had not been following up regularly with his doctor
or monitoring his blood sugars at home. His HbA1c at A. You recommend a continuous insulin infusion
the time of his hospital admission was 9.1%. He was seen using the standard IV insulin protocol used at your
by the inpatient diabetes management service while he hospital with a target glucose range between 100 to
was in the hospital and per their recommendation, he 140 mg/dL.
was discharged home on a regimen of insulin glargine B. You recommend a continuous insulin infusion
40 units once daily, insulin aspart 10 units before each using the standard IV insulin protocol used at your
meal, and metformin 1,000 mg twice daily. Since then, hospital with a target glucose range between 140 to
the patient has been diligent about checking his blood 180 mg/dL.
sugars regularly at home and has been fully adher- C. Since his glucose values are below 200 mg/dL, you
ent with his insulin regimen. He has also made several recommend an insulin sliding scale with regular
dietary modifications and plans to start a cardiac rehab insulin subcutaneously every 6 hours if the glucose
program as soon as he is cleared by his cardiologist to level exceeds 150 mg/dL.
start exercising. On his current diabetes treatment regi- D. You decide that the patient will likely need a basal
men, his fasting and premeal blood glucoses are mostly subcutaneous insulin added to his regimen at the
in the 200s. His physician progressively titrates his insu- time of discharge from the hospital since his diabetes
lin doses up over the next several months and during this was not adequately controlled prior to the surgery
time the patient also has enrolled in a weight manage- based on his A1C values before admission. You
ment program and lost 8 lb (BMI dropped from 32 to 31 therefore recommend starting NPH insulin 10 units
kg/m2). He plans to continue with exercise and diet for subcutaneously at night.
weight loss but otherwise does not wish to pursue any
other more aggressive approaches for weight loss. He is 49. The patient is a 78-year-old woman with type 2 dia-
now on insulin glargine 80 units twice daily and aspart betes mellitus, hypertension, dyslipidemia, obesity, and
30 units AC and his HbA1c has decreased to 8.6%. What sleep apnea that is being treated with CPAP at night.
are your treatment options now? She recently immigrated to the United States from
Thailand. The patient and her family endorse dietary
A. Add a TZD. factors such as eating candy and other sugary foods since
B. Continue to increase the dose of insulin glargine. she moved to the United States that may be adversely
C. Add a GLP1 analog. impacting her blood sugars. She also reports that she
D. Switch to U-500 insulin. used to walk regularly for exercise when she was back in
Thailand, but since moving to the United States, she has
48. A 60-year-old man with type 2 diabetes is admit- been less physically active and with the cold weather,
ted to the hospital for coronary artery bypass sur- has mostly been staying indoors and sedentary over the
gery. He was diagnosed with diabetes 4 years ago. He last few months. The patient has not been checking her
was treated with diet alone for a few months and then blood sugars at home. Her son, who is a physician in
started on metformin, initially at a dose of 500 mg Thailand and has accompanied her to the visit, reports
twice daily, which was then increased to 1,000 mg twice that the patients most recent HbA1c checked about 6
daily 6 months later. About 2 years ago, glypizide 5 months ago when she was is Thailand was 7.1% and her
mg twice daily was added to his regimen and the dose A1Cs were ranging from the high 6s to the low 7s over
was increased to 10 mg twice daily a year ago. Over the the last 12 years on her diabetes treatment regimen.
past year, his HbA1c values have ranged between 8.2% Besides diabetes, her medical history is notable for her
and 8.6%, and his most recent A1C was 8.3%. Besides history of hypertension and dyslipidemia for which she
diabetes, the patient has history of hypertension and is on medications, obesity, obstructive sleep apnea for
dyslipidemia. In addition to the diabetes medications which she uses CPAP at night, mild restrictive lung dis-
above, his medications prior to the surgery included ease, and heart failure from diastolic dysfunction. She
lisinopril 10 mg once daily, metoprolol 50 mg twice a also has a history of CKD (baseline serum creatinine
day, atorvastatin 40 mg a day, and aspirin 81 mg a day. A around 1.3 mg/dL in the last couple of years), which is

5 . E n d o c r i n o l o g y 111
thought to be from a combination of hypertension, dia- twice daily and lifestyle interventions. The patient met
betes, and medication (diuretic). She had over the past with a nutritionist and implemented various dietary
year required an increase in her diuretic dose for treat- modifications as advised by the nutritionist. In addi-
ment of diastolic heart failure. Her current medications tion, she joined a gym and started exercising for 60
include metformin as above, atorvastatin 20 mg once minutes 4 times/week. After 3 months, the patient had
daily, lisinopril 40 mg once daily, furosemide 40 mg lost 5 lb and her A1C level had dropped to 7.7%. Her
once daily, and aspirin 81 mg daily. On examination, metformin dose was increased to 1,000 mg twice daily,
patient is a pleasant, overweight, elderly woman. Her and she was encouraged to continue with lifestyle modi-
blood pressure is well controlled and her physical exam- fications and weight loss efforts. After an additional 3
ination is unremarkable except for some chronic venous months, the patient returns for follow-up. She has been
stasis skin changes and nonpitting edema around the tolerating the higher dose of metformin without prob-
ankles and mildly decreased vibratory sensation in lems. She has been unable to lose any additional weight
the feet. Monofilament sensation is intact in both feet, despite continuing her efforts with diet and exercise.
Laboratory data reveal a serum creatinine of 1.47 mg/dL, At this point her AC1 level is 7.5%. She feels her weight
eGFR 34 mL/min/1.73 m2 , and HbA1c 7.7%. loss has reached a plateau. She would like to lose more
What would you recommend as treatment for this weight but cannot consider more intensive weight loss
patient at this time? measures such as enrolling in a formal weight loss pro-
gram due to time and financial constraints, and she has
A. No change in treatment no interest in surgical weight loss interventions. The
B. Increase in metformin dose to 1,000 mg twice daily decision is made to intensify her diabetes treatment
C. Increase glipizide dose and continue metformin regimen by adding a second glucose-lowering agent.
D. Increase glipizide dose and stop metformin The patient does not want to take any injections. She has
concerns about side effects of medications, specifically
50. A 50-year-old obese woman with a history of gesta- potential weight gain.
tional diabetes and family history of diabetes and coro- Which of the following would be the best option for
nary artery disease, was diagnosed with type 2 diabetes this patient given the above considerations?
during a visit with her primary care physician when she
was found to have A1C level of 8.0% on routine screen- A. No change in treatment
ing blood tests. At the time of her diabetes diagnosis, B. Add glipizide
her body mass index (BMI) was 30 kg/m2 . Treatment C. Add liraglutide
was initiated for her diabetes with metformin 500 mg D. Add sitagliptin

11 2 T h e B r i g h a m I n t e n s i v e R e v i e w o f I n t e r n a l M e d i c i n e Q u e s t i o n a n d A n s w e r C o m pa n i o n
CH A PT ER5 A NSW ER S metformin has been reaffirmed as the preferred initial agent.
There is a long-standing evidence base for both efficacy and
1. ANSWER:A. No further treatment safety of metformin, it is not associated with weight gain or
hypoglycemia, and it may reduce the risk of cardiovascular
The first-line treatment for most patients with type 2 diabetes events and cancers. While lifestyle modifications (balanced
is metformin. However, there is a concern of using metformin diet, physical activity, and weight loss when indicated) are
in patients with decreased glomerular filtration rate (GFR). integral components of diabetes management, when blood
Based on the FDA criteria, metformin is contraindicated glucose is not at goal pharmacotherapy should be initiated
in men and women with serum creatinine concentrations in addition to lifestyle modification.
1.5 and 1.4 mg/d, respectively. However, in an 89-year-old
woman a creatinine level of 1.3 mg/dL indicates a signifi-
cant decrease in GFR. Arecent statement by the American 4. ANSWER: B. Intensify glucose control.
Diabetes Association (ADA) and European Association for
the Study of Diabetes recommended not to start new patients Tight glycemic control with a target hemoglobin A1c of
on metformin if eGFR is <45 mL/min/1.73 m2 . In addition, <7.0% has been shown to effectively prevent diabetes related
recent diabetes guidelines suggest that less stringent A1C microvascular complications (i.e., nephropathy, retinopa-
goals (such as <8%) may be appropriate for patients with a thy, and neuropathy) when initiated early in the course of
limited life expectancy. This patient has reasonably controlled the disease, and to delay the deterioration of these compli-
diabetes, metformin is contraindicated (low eGFR), and the cations in patients with long-standing diabetes. Based on
potential risk for hypoglycemia associated with sulfonylurea the American Diabetes Association guidelines, patients
or insulin treatment, especially in the elderly, outweighs the with diabetes and hypertension should be treated to a sys-
limited benefit of a better glycemic control. tolic blood pressure <140mm Hg and a diastolic pressure
of <80 mm Hg. Metformin can be safely used when the
serum creatinine levels are <1.4 and <1.5 mg/dL in women
and men, respectively. The presence of retinopathy is not
2. ANSWER: C. Intravenous insulin with glucose a contraindication to aspirin therapy, as this therapy does
target of 140 to 180 mg/dL. not increase the risk of retinal hemorrhage. The patient is
already taking angiotensin receptor blockade and currently
Hyperglycemia in hospitalized patients is associated with no data are available on a benefit in the use of combination
adverse outcomes. However, interventional clinical trials therapy in patients with established diabetic nephropathy.
to control glucose levels in critically ill patients resulted
in inconsistent data. This has led to a more conservative
approach in the recommendations for glucose manage-
ment in ICU patients. The joint consensus statement of 5. ANSWER:A. Prescribing Metformin to better
the American Association of Clinical Endocrinologist control fasting hyperglycemia
(AACE) and the American Diabetes Association (ADA)
recommended the use of intravenous insulin for treatment While hyperglycemia in the nondiabetic range (prediabe-
of persistent hyperglycemia, starting at a threshold of no tes) is not associated with increased incidence of micro-
greater than 180 mg/dL, and once initiated, a glucose range vascular complications, patients are at increased risk for
of 140 to 180 mg/dL is recommended for the majority of macrovascular complications (such as coronary artery dis-
patients. While a range of 110 to 140 mg/dL may be accept- ease). However, interventions to decrease glucose levels in
able, the benefit is questionable and it is usually associated the prediabetes range had no significant effect on cardio-
with higher rates of hypoglycemia. Subcutaneous insulin vascular outcomes. Therefore, diagnosis of impaired fasting
regimens are not recommended in critically ill patients. glucose should prompt the assessment of other cardiovas-
cular risk factors, and interventions with proven benefits in
cardiovascular primary prevention such as smoking cessa-
tion, lipid lowering, blood pressure control, and aspirin.
3. ANSWER:D. Weight loss + Metformin

There are various antidiabetic medications available and


the American Diabetes Association, as well as other profes- 6. ANSWER: D. Test for antiglutamic acid
sional associations, promotes the individualization of the decarboxylase (GAD) antibodies.
treatment approach, taking into account patient prefer-
ences, cost, and potential side effects of each class of medi- Type 1 diabetes, which accounts for 5%10% of diabe-
cations. Nevertheless, in the absence of contraindications, tes cases, is a gradually progressing disease that may be

5 . E n d o c r i n o l o g y 113
diagnosed on routine screening while patients have only At least studies studies have shown significant improve-
mild hyperglycemia. Type 1 diabetes is usually being diag- ments in BMD at the lumbar spine and hip after 12years
nosed from early childhood to early adulthood, but it may of alendronate in patients with PHPT compared to the
occur at any age. Adult patients with diabetes, who are lean, placebo-treated groups.
do not have the common metabolic abnormalities usually
observed in type 2 diabetes (such as hypertension and dys-
lipidemia) and without a family history of diabetes may
very likely have type 1 diabetes. In this setting, rapid failure 9. ANSWER:C. Hypoparathyroidism
of oral medications to control blood glucose should increase
the suspicion for type 1 diabetes. Autoantibodies such as Osteoblastic bone metastases from prostate cancer can lead
anti-GAD antibodies, islet cell autoantibodies, and insulin to hypocalcemia from deposition of calcium in the bone
autoantibodies may be used do differentiate type 1 from metastases, particularly in patients with widespread disease
type 2 diabetes. Maturity Onset Diabetes of the Young along with calcium and vitamin D deficiencies. This patient
(MODY) is a monogenic form of diabetes with autosomal has a normal alkaline phosphatase and no evidence of widely
dominant pattern of inheritance. Patients with MODY are metastatic disease. His vitamin D, though on the low end,
usually diagnosed in adolescence and have a strong family should not result in this degree of hypocalcemia. Hungry
history of diabetes. In addition, in the absence of signifi- bone syndrome generally presents after surgical cure of
cant weight gain, glycemic control in patients with MODY hyperparathyroidism and would result in both hypocalce-
is unlikely to deteriorate. The patient described in this ques- mia and hypophosphatemia. Hypomagnesemia can lead to
tion has no clinical findings to suggest Cushings syndrome. functional hypoparathyroidism, but there is nothing in the
history to suggest that this patient would be at risk for low
magnesium. The most likely etiology is hypoparathyroid-
ism resulting from previous thyroid cancer surgery. Patients
7. ANSWER:E. No further adjustment of her regimen with decreased parathyroid reserve may manifest overt
hypocalcemia when given a potent antiresorptive agent
The 2012 Position Statement of the American Diabetes such as zoledronic acid or denosumab for osteoporosis or
Association (ADA) and the European Association for the cancer. In this situation, the sudden loss of calcium resorp-
Study of Diabetes (EASD) suggests that HbA1c goal as tion from bone and inadequate PTH response can lead to
high as 7.5%8.0% or even slightly higher is appropriate profound hypocalcemia and hyperphosphatemia, as seen in
for patients with a history of severe hypoglycemia, limited this case.
life expectancy, advanced complications, extensive comor-
bid conditions, and those in whom the target is difficult
to attain despite intensive self-management education,
repeated counseling, and effective doses of multiple glucose- 10. ANSWER:A. 1,25-dihydroxy-vitaminD
lowering agents, including insulin. The patient presented in
the question has a long duration of disease with a history This patient has sarcoidosis, a granulomatous disorder in
of myocardial infarction and several unaware hypoglycemic which activated macrophages can express and secrete cal-
episodes. Therefore, maintaining an HbA1c between 7.0% citriol (1,25-dihydroxy vitamin D) leading to both hyper-
and 7.5% without hypoglycemia is appropriate. calcemia and hypercalciuria. The other choices are all
reasonable to check in cases of non-PTH mediated hyper-
calcemia but would not explain the underlying pathophysi-
ology of hypercalcemia associated with granulomatous
8. ANSWER: D. Alendronate 70 mg orally weekly disease. Treatment of the hypercalcemia in sarcoidosis
would start with hydration and glucocorticoids.
This patient has osteoporosis and therefore meets criteria
for parathyroid surgery. However, since she refuses surgery,
there is no reason to proceed with imaging procedures to
localize an adenoma. Similarly, a 24-hour urine test will 11. ANSWER:E. Starting a gluten-free diet
not change management if she refuses surgery. Cinacalcet
will reduce both serum calcium and PTH levels but does It is important to recognize secondary hyperparathyroid-
not improve bone mineral density (BMD) in patients with ism due to malabsorption and to note that in some dis-
PHPT. Option E would be correct in the absence of osteo- orders, such as celiac disease, calcium malabsorption can
porosis. Given that PHPT is associated with bone loss and occur even in the presence of reasonably normal vitamin
increased fracture risk, the best answer is to treat with an D levels. The clues to the diagnosis of celiac disease are the
antiresorptive agent for osteoporosis such as alendronate. irritable bowel syndrome, weight loss, very low urinary

114 T h e B r i g h a m I n t e n s i v e R e v i e w o f I n t e r n a l M e d i c i n e Q u e s t i o n a n d A n s w e r C o m pa n i o n
calcium excretion, and dermatitis herpetiformis on physi- FGF-23 causes phosphate loss in the urine with an elevated
cal examination. Increasing Ca and D will not necessarily urine phosphate. 1,25-dihydroxy-vitamin D is generally
correct the secondary hyperparathyroidism and oral alen- low or low-normal in this disorder, as the renal tubular
dronate is unlikely to be absorbed. Parathyroidectomy is defect causing the phosphaturia also impairs synthesis of
clearly inappropriate for secondary hyperparathyroidism. 1,25-dihydroxy-vitamin D.Calcitonin and urinary calcium
Agluten-free diet, after confirming the diagnosis of celiac are not high.
disease, will improve calcium and nutrient absorption and
is the first step in improving his skeletal health.

14. ANSWER:D. Above 20%

12. ANSWER:C. Refer for zoledronic acid infusion, Though the risk of a major osteoporotic fracture (spine,
5mg intravenously. forearm, hip, or shoulder fractures) is highest in those
with osteoporosis by bone mineral density criteria (T-score
This patient has Pagets disease, a bone condition of below 2.5), most fractures occur in patients with osteo-
unknown etiology characterized by disordered bone resorp- penia as the prevalence of osteopenia in the population is
tion with a secondary increase in osteoblast activity, lead- much higher. As a result, several clinical prediction rules
ing to disorganized bone architecture. Pagets disease can have been developed to identify those patients with osteo-
be asymptomatic, diagnosed during workup of an inciden- penia at highest risk of fractures, who would presumably
tally noted increased alkaline phosphatase level. In these benefit the most from therapy. The most validated risk
asymptomatic cases, if a bone scan does not show lesions prediction tool is the FRAX calculator, which gives the
in the skull or near joints, treatment may be deferred and 10-year absolute risk of major osteoporotic fractures and of
the patient can be monitored for the development of signs hip fractures. The current recommendation in the United
and symptoms of progressive disease. However, in this States is to initiate therapy in osteopenic individuals when
symptomatic patient with lesions close to the hip joint, the risk of a major osteoporotic fracture exceeds 20%, or
treatment should be initiated. First-line treatment is a when the risk of a hip fracture exceeds 3%.
bisphosphonate, but usually at higher doses than for osteo-
porosis. Oral options are alendronate at a dose of 40 mg
by mouth once daily, or risedronate 30 mg by mouth once
daily. Intravenous options include pamidronate (different 15. ANSWER:E. Midnight salivary cortisol
dosing regimens; a commonly used regimen is 60 mg every
3months) or zoledronic acid 5 mg (redosed when there is In a woman who is nearly 20 years post menopause, rap-
sign of recurrence after 12years). Calcitonin is no longer idly progressive bone loss, with or without a fracture, war-
recommended as first-line therapy for Pagets disease. rants an aggressive search for secondary causes. The onset
of hypertension, diabetes mellitus, kidney stones, obesity,
and proximal myopathy all point to hypercortisolism
from endogenous Cushings syndrome. Though multiple
13. ANSWER:C. Fibroblast growth factor-23 myeloma, systemic mastocytosis, hyperparathyroidism, and
(FGF-23) acromegaly could all result in some of these manifestations,
the clinical picture that fits best is Cushings syndrome.
This patient has severe hypophosphatemia causing osteo- Screening tests for Cushings syndrome can include 24-hour
malacia. In the absence of a history of poor oral intake, urinary cortisol levels, 1 mg overnight dexamethasone sup-
diarrhea, or antacid use, this is most likely from urinary pression testing, or midnight salivary cortisol testing.
losses, which can be confirmed by a high (or inappropri-
ately normal) urinary phosphate. Vitamin D deficiency and
primary hyperparathyroidism are the most common causes
of a mild hypophosphatemia, but this patients vitamin 16. ANSWER:E. Karyotyping
D was normal, as was his calcium. Fanconi syndrome
(type 2, or proximal, renal tubular acidosis) is another This patient has clinical symptoms and signs of male hypo-
cause of hypophosphatemia, especially in patients with a gonadism. His biochemical tests reveal primary gonadal
paraproteinemic disorder or taking culprit medications, failure as the cause of the hypogonadism. Klinefelter syn-
but is associated with a metabolic acidosis and hypokale- drome is relatively common among young males with
mia which were not described here. Hence, this patient is primary hypogonadism. Karyotyping will identify the
most likely to have tumor-induced osteomalacia, which is chromosomal abnormality (XXY). Echocardiogram and
caused by FGF-23 secretion from a mesenchymal tumor. renal ultrasound will not uncover the etiology of primary

5 . E n d o c r i n o l o g y 115
hypogonadism. Pituitary MRI is helpful in patients with is not interested in pregnancy and is most concerned about
hypogonadotropic hypogonadism. Hemochromatosis typi- symptoms of hirsutism. While metformin and weight loss
cally causes hypogonadotropic hypogonadism, not primary (Answers A amd C) may eventually help ameliorate her
gonadal failure. symptoms, they are unlikely to help in the short term. After
6months of treatment on oral contraceptive, it is unlikely
that continued treatment at the current dose would provide
further improvement (Answer B). Spironolactone, an anti-
17. ANSWER:C. CFTR gene androgenic agent, has been shown to help with hirsutism.
However, it has teratogenic effects and should be used in
Cystic fibrosis, which is caused by the mutations in CFTR conjunction with an oral contraceptive in young women of
gene, is associated with infertility and azoospermia. reproductive age (Answer E).
Mutations in Kiss1, Kal1, FGFR1, and TAC3 genes are
related to hypogonadotropic hypogonadism with low LH,
FSH, and testosterone.
21. ANSWER:D. Hypothalamic amenorrhea

This young women is experiencing secondary amenorrhea


18. ANSWER:D. Measure a.m. total testosterone and defined as an absence of menses for more than three cycles
sex hormone binding globulin or 6months in women who had previously been menstru-
ating. The most likely etiology in this case is from hypo-
The diagnosis of male hypogonadism requires measurement thalamic amenorrhea (Answer D). This young woman fits
of early morning serum total testosterone levels. Alow value the classic description of the female athlete triad, which
should be confirmed by repeating the measurement along consists of an eating disorder, amenorrhea, and osteoporo-
with a serum sex hormone binding globulin level (SHBG). sis. Although the exact pathophysiology of this disorder is
Men with obesity and type 2 diabetes mellitus often have unknown, it is believed that amenorrhea occurs from a mis-
low SHBG resulting in low total testosterone but normal match between nutritional intake and energy expenditure.
free (biologically active) testosterone levels. Since the
reliability of free testosterone assays varies greatly, it is
best to measure total testosterone plus SHBG and then
calculate the free testosterone level using a widely available 22. ANSWER:C. 24-hour urine free cortisol
calculation tool. Prior to confirmation of hypogonadism, it
is too early to perform other tests or to begin treatment. Given this patients presentation, the most likely etiology for
her amenorrhea is cortisol excess, possibly from Cushings
disease. Therefore, a 24-hour urine free cortisol would be a
good screening test to evaluate for cortisol excess (Answer
19. ANSWER:D. All of the above C). She has a classic presentation for cortisol excess, includ-
ing progressive weight gain, fatigue, weakness, abdomi-
According to The Endocrine Society clinical guideline on nal striae, and secondary amenorrhea. Cushings disease
hypogonadism, serum total testosterone, PSA, and hema- (Pituitary ACTH-dependent Cushings syndrome) is more
tocrit should be monitored 36months after initiating tes- common in women and typically presents between 25 and
tosterone treatment. Therapy should aim to raise the serum 45years. An LH to FSH ratio (Answer 1)could be helpful
testosterone level into the mid-normal range. Check hema- in the diagnosis of PCOS, but given her clinical presenta-
tocrit at baseline, at 3 to 6months, and then annually. If tion, cortisol excess is more likely. Hyperprolactinemia
hematocrit is >54%, stop therapy until hematocrit decreases (Answer B) and hypothyroidism (Answer D) may cause
to a safe level. In men 40years of age or older, check PSA secondary amenorrhea but are less likely given her clinical
level before initiating treatment, and at 3 to 6months. presentation.

20. ANSWER:E. Continue treatment with the oral 23. ANSWER:D. Treatment with levothyroxine
contraceptive and add spironolactone 50 mg BID
This woman presents for her annual examination with
While there are several different treatment strategies for fatigue, constipation, weight gain, and oligomenorrhea.
PCOS, the best treatment strategy should be tailored to While it may be possible that the symptoms are from
the patients presenting symptoms and goals. This patient work-related stress and perimenopause, one should keep

116 T h e B r i g h a m I n t e n s i v e R e v i e w o f I n t e r n a l M e d i c i n e Q u e s t i o n a n d A n s w e r C o m pa n i o n
in mind that hypothyroidism may also present in a similar medications include norepinephrine-serotonin reuptake
manner. Her labs reflect new-onset hypothyroidism. Given inhibitors (such as venlafaxine), selective-serotonin reup-
that her oligomenorrhea, hyponatremia, hyperprolac- take inhibitors, and tricyclic antidepressants. The discon-
tinemia, and hyperlipidemia are most likely manifestations tinuation of venlafaxine is likely to result in normalization
of her hypothyroidism, it would be best to treat her hypo- of this patients values.
thyroidism first with levothyroxine (Answer D) before con-
sidering treatment for these other clinical abnormalities.

26. ANSWER:B. Adrenal myelolipoma

24. ANSWER:C. Dexamethasone suppression test The radiographic phenotype of an adrenal mass can pro-
vide valuable clues in distinguishing benign from malig-
The incidental detection of adrenal masses is increasing as nant adrenal processes. Benign adrenal masses are typically
the frequency of abdominal imaging expands. Once discov- lipid-rich and therefore have a low density as measured on
ered, it is important to assess the mass for malignant poten- non-contrast CT by X-ray attenuation (Hounsfield units
tial and hormone functionality, as either of these features [HU]). Adipose tissue tends to have a density of approxi-
can result in adverse health sequelae. The vast majority of mately20 to150 HU. Adensity of <10 HU (lipid-rich
incidental adrenal masses are benign adenomas that are entity) on noncontrast CT almost always represents a
characterized by a lipid-rich density (<10 Hounsfield units benign adrenal adenoma, except when the density if very
on CT scan), such as in the case of this patient. Despite the low (less than40 HU) in which case it almost always rep-
presence of reassuring radiographic features, a benign ade- resents a myelolipoma. Myelolipomas are benign lipid-rich
noma may still be functional; excess cortisol, aldosterone, growths that are typically nonfunctional.
sex hormones, or catecholamines can all result in long-term Some benign adrenal adenomas may have a lower lipid
health complications. content and therefore may have a more dense phenotype
Even when overt features of hypercortisolism (Cushings (>10 HU). Higher density adrenal masses increase the
syndrome) or catecholamine excess (pheochromocytoma) concern for pheochromocytomas, carcinomas, and other
are absent, the prevalence of subclinical cortisol or cate- malignant entities.
cholamine excess is sufficiently high that it is recommended
that all patients with a newly discovered adrenal mass be
assessed for these hormone excess states. The most efficient
and specific testing involves performing a 1 mg dexameth- 27. ANSWER:A. Primary adrenal insufficiency
asone suppression test (to assess for cortisol excess) and
plasma metanephrines (for catecholamine excess). Serum This patient has primary adrenal insufficiency (Addisons
catecholamines are not reliable indicators given their vari- disease) that is most likely secondary to an autoimmune
ance in the blood; therefore, measurement of intermediates process, given her history of concomitant hypothyroidism
of catecholamine metabolism (metanephrines) is preferred. at a young age. Primary adrenal insufficiency is character-
When high blood pressure, hypokalemia, or hirsutism is ized by the destruction of the entire adrenal cortex, thereby
present, a serum aldosterone and plasma renin activity, or resulting in deficiencies in cortisol, the mineralocorti-
adrenal androgen profile, should also be assessed. coid aldosterone, and adrenal sex hormones. The resulting
clinical manifestations reflect mineralocorticoid deficiency
(hypovolemia, salt-wasting, hyponatremia, hyperkalemia),
glucocorticoid deficiency (fatigue, weight loss, nausea),
25. ANSWER:E. Venlafaxine use and adrenal androgen deficiency (loss of pubic and axillary
hair). Characteristic laboratories include a very low corti-
The assessment of plasma metanephrines should take into sol and aldosterone, and a very high ACTH and plasma
account medications that can raise their levels. By the time renin activity to reflect physiologic compensation. Patients
a catecholamine-producing tumor, such as a pheochromo- may also display significant hyperpigmentation. ACTH
cytoma or paraganglioma, is capable of causing systemic stimulation results in minimal or no cortisol increase,
adrenergic symptoms, plasma metanephrine levels are typi- given the destruction of the adrenal cortex. These patients
cally >4-fold higher than the upper limit of normal. It is warrant immediate treatment with glucocorticoids, saline
not uncommon to see mild elevations in plasma normeta- expansion, and ultimately mineralocorticoids, to prevent
nephrines that may represent high sympathetic nervous hemodynamic collapse and life-threatening electrolyte
system tone or medication-induced effects. Medications abnormalities.
that inhibit the reuptake of catecholamines in a synapse Secondary adrenal insufficiency refers to a disruption at
can increase circulating plasma normetanephrines. These the level of pituitary production of ACTH. In the absence

5 . E n d o c r i n o l o g y 117
of ACTH, patients manifest glucocorticoid insufficiency; likely resulted in ACTH suppression and thereby cortisol
however, mineralocorticoid production (and therefore deficiency, the stimulation and production of endogenous
maintenance of sodium/volume homeostasis and potassium adrenal mineralocorticoids remains intact secondary to
balance) remains intact. When ACTH deficiency is pro- stimulation by the renin-angiotensin system and potassium.
longed, the adrenal glands will gradually atrophy and display Therefore, this patient should be treated for cortisol insuffi-
a diminished response to exogenous ACTH stimulation. ciency with an intravenous glucocorticoid (hydrocortisone)
until her condition stabilizes and further assessments can be
made, but she does not need treatment for mineralocorti-
coid deficiency (with agents such as fludrocortisone).
28. ANSWER:D. This cortisol value cannot be
adequately interpreted

Metastatic solid tumors have been known to metastasize 30. ANSWER:E. Insufficient information to confirm
to the hypothalamus or pituitary and result in secondary a diagnosis
adrenal insufficiency. Solid tumors can also metastasize to
bilateral adrenal glands, although this scenario does not nec- This patients progressive and resistant hypertension and
essarily result in primary adrenal insufficiency. Symptoms newly worsening hypokalemia both likely suggest that he has
of orthostatic hypotension are not pathoneumonic for underlying primary hyperaldosteronism. However, there is
adrenal insufficiency, but this entity should certainly be insufficient information at this time to confirm that diagno-
considered. In the setting of dexmethasone use, cortisol sis. The initial screening test for hyperaldosteronism involves
values are only reliable for detecting inappropriately high obtaining a serum aldosterone and plasma renin activity
cortisol states (Cushings syndrome), but not when inves- together. These should be considered in the following set-
tigating inappropriately low cortisol states (adrenal insuffi- tings: moderate-to-severe or resistant hypertension, sponta-
ciency). Dexamethasone is a synthetic glucocorticoid that is neous or diuretic-induced hypokalemia in hypertensives, and
not detected by the conventional assay for cortisol; however, when there is hypertension in the setting of an incidentally
like all glucocorticoids, it inhibits the production of ACTH discovered adrenal mass. An aldosterone-to-renin ratio (ARR)
and therefore suppresses endogenous cortisol production. that is greater than at least 2030 should be considered as a
Therefore, a low cortisol level in this patient most likely positive screen for primary hyperaldosteronism that requires
reflects the expected action of dexamethasone. Whether further confirmatory testing (for example, a salt/saline sup-
he may also have concomitant brain metastases that have pression test). It is expected that in addition to a high ARR,
resulted in secondary adrenal insufficiency by compromis- the plasma renin activity will also be very low or suppressed
ing the pituitary apparatus, or concomitant adrenal metas- in primary hyperaldosteronism, indicating a response to high
tases that have resulted in primary adrenal insufficiency, aldosterone levels. Medications that influence the ARR must
cannot be ascertained by this single testing performed in therefore be carefully evaluated. Spironolactone, a mineralo-
the setting of dexamethasone intake. corticoid inhibitor, functions to inhibit the negative feedback
of aldosterone on renin release and therefore will cause a rise
in plasma renin activity. This patient has an ARR<20 and an
unsuppressed plasma renin activity, both of which are more
29. ANSWER:B. Hydrocortisone consistent with secondary hyperaldosteronism and likely
reflect the effect of spironolactone. Therefore, to obtain a reli-
The cause of this patients worsening clinical status, hypo- able screening ARR to diagnose primary hyperaldosteronism,
tension, and tachycardia may be attributed to a combina- spironolactone should be stopped and substituted for another
tion of many factors; however, given that she underwent a medication to control blood pressure that does not interfere
surgical procedure following the recent cessation of chronic with the ARR (such as an alpha-antagonist), and the ARR
glucocorticoid use, chronic secondary adrenal insufficiency repeated after several weeks.
as contributor to her symptoms and hemodynamic status
should be considered and treated. Exogenous glucocorticoids
suppress pituitary ACTH release and thereby cause a resul-
tant decline in adrenal cortisol production. With chronic 31. ANSWER:D. Start levothyroxine at a dose
glucocorticoid therapy and chronic ACTH inhibition, of12.5g daily
endogenous ACTH and cortisol production may require
many days, weeks, or even months to normalize following This patient requires treatment with levothyroxine, but it
the cessation of glucocorticoid therapy. It is for this reason should be started at the lowest possible dose with provisions
that glucocorticoids are typically tapered gradually after pro- to gradually increase it as tolerated in light of his coronary
longed use. While exogenous prednisone use in this patient artery disease. A full replacement dose given orally or an

118 T h e B r i g h a m I n t e n s i v e R e v i e w o f I n t e r n a l M e d i c i n e Q u e s t i o n a n d A n s w e r C o m pa n i o n
adjusted dose given intravenously could exacerbate cardiac enlargement of thyroid tissue may be marginally effective at
ischemia to the point of causing an infarction. Checking best in euthyroid patients, and it would be completely inef-
antithyroid peroxidase and antithyroglobulin antibod- fective in a patient presenting with hyperthyroidism.
ies would not provide any additional information. In the
absence of any known history of thyroid surgery or exter-
nal radiation treatment to the head and neck, it can be
presumed that his severe hypothyroidism is due to autoim- 34. ANSWER:E. Check a thyroid scan.
mune thyroiditis. Deferring treatment until he has under-
gone revascularization would not be appropriate, as severe When a patient presenting with a thyroid nodule who is
hypothyroidism can increase the risk of complications not taking levothyroxine is noted to have a suppressed TSH
related to general anesthesia and the use of sedatives. level, a thyroid scan should be checked to determine if the
nodule itself is an autonomously functioning toxic ade-
noma. If a nodule is hot on scan, it does not need to be biop-
sied. If it is cold on scan with evidence of increased tracer
32. ANSWER:E. Check a TSH level. uptake in surrounding tissue consistent with Graves dis-
ease, fine-needle aspiration biopsy should be performed to
The only way to determine if her current dose of levothyrox- guide further management. Treatment of a toxic adenoma
ine is providing an adequate level of replacement is to check or Graves disease with methimazole or radioactive iodine
a TSH level. Total T4 and T3 levels measured while taking may be indicated, but it should only be considered after it
oral contraceptives or during pregnancy may be elevated has been determined whether a biopsy is necessary. Referral
due to increased production of thyroxine-binding globulin for thyroid surgery would only be indicated if a biopsy of
stimulated by increased estrogen levels. As such, they may a cold nodule revealed suspicious or malignant cytopathol-
not correlate with free thyroid hormone levels. Decreasing ogy, or if there were contraindications to treatment of a
her dose of levothyroxine would be inappropriate, as she toxic adenoma with radioactive iodine or methimazole.
may require a moderate to substantial increase in her dose
during the course of a full-term pregnancy. Increasing a
dose of levothyroxine empirically may cause iatrogenic thy-
rotoxicosis characterized by symptoms that may be difficult 35. ANSWER:B. Check a thyroid uptake and scan.
to distinguish from normal physiologic changes of preg-
nancy. Exposure to radionuclide tracer is contraindicated This patient is presenting with thyrotoxicosis without
during pregnancy, and in any event a thyroid uptake and any referable symptoms or clinical findings suggestive of
scan would be unnecessary in a patient with known hypo- a specific cause. Checking a thyroid uptake would help to
thyroidism. Doses of levothyroxine should always be sepa- distinguish between a high-uptake state caused by hyper-
rated from doses of iron sulfate by at least 4 hours to avoid thyroidism driven by increased production of thyroid hor-
interactions that can block absorption of both agents. mone and a low-uptake state caused by inflammation with
leakage of stored thyroid hormone. If a high-uptake state is
identified, a scan will help to distinguish whether hyperthy-
roidism is caused by Graves disease, a toxic adenoma, or a
33. ANSWER:C. Refer the patient to a thyroid toxic multinodular goiter. Elevated antithyroid peroxidase
surgeon. and antithyroglobulin antibodies may identify underlying
autoimmune thyroiditis, but they will not definitively deter-
A multinodular goiter that has extended substernally to the mine the proximate cause of thyrotoxicosis. Treatment with
point of causing tracheal compression should be resected methimazole or referral for thyroid surgery would only be
by an experienced thyroid surgeon, irrespective of its func- considered after confirmation of a diagnosis. Checking
tional status. The presence of audible stridor and evidence an ESR would not be informative, as subacute thyroiditis
of tracheal narrowing on radiographic images obviate the would be unlikely in the absence of localized discomfort.
need for pulmonary function testing with flow volume
loops. Treatment with methimazole might help to control
hyperthyroidism caused by autonomously functioning thy-
roid nodules, but it would not help to shrink the dominant 36. ANSWER:D. Wait 6 weeks and recheck a full
nodule to any extent. Treatment with I-131 might help to profile of thyroid hormone levels.
shrink the dominant nodule over time, but it might cause
radiation-induced thyroiditis with expansion of affected The shifts in profiles of thyroid hormone levels tracked in
tissue that could severely compromise respiratory function. this patient are characteristic of physiologic changes that
Treatment with levothyroxine to try to suppress further may occur during acute illness and subsequent recovery in

5 . E n d o c r i n o l o g y 119
patients with normal endogenous thyroid function. Initial primary hypothyroidism can cause hyperprolactinemia.
suppression of TSH, T4, and T3 levels during progression Apituitary MRI to assess for the presence of a pituitary ade-
of a nonthyroidal illness may be misinterpreted as evidence noma is indicated after other causes of hyperprolactinemia
of underlying central hypothyroidism caused by hypotha- have been excluded. For this reason a pituitary MRI is not
lamic or pituitary dysfunction. Subsequent transient eleva- the next test to be performed.
tion of the TSH level during recovery may be misinterpreted
as evidence of underlying primary hypothyroidism. In such
cases, it is usually prudent to wait until the patient has made
a full recovery before attempting to determine whether any 39. ANSWER:D. Medical therapy with a dopamine
thyroid dysfunction is present. Athyroid uptake and scan agonist
would not provide any useful information in this setting.
Starting treatment with any form of thyroid hormone at Medical therapy with a dopamine agonist is considered the
this stage would be premature, and doing so would make first-line initial therapy for symptomatic prolactinomas.
it difficult to interpret subsequent profiles of thyroid hor- Dopamine agonists have both antiproliferative and antisecre-
mone levels. Apituitary MRI scan would only be informa- tory effects on prolactin producing pituitary cells and typi-
tive if serial measurement of thyroid hormone levels showed cally result in both decreased prolactin secretion and tumor
persistent low TSH levels and low free T4 levels consistent shrinkage. Cabergoline is preferred over bromocriptine for
with possible central hypothyroidism. most patients due to its increased efficacy and tolerability.
Somatostatin analogs are used in the treatment of acromeg-
aly. Transsphenoidal surgery is reserved for patients who do
not tolerate or are not responsive to the dopamine agonists.
37. ANSWER:E. She should wait until she is 6months Radiation would only be considered in very rare cases of
out from treatment before trying to conceive. advanced or progressive prolactinomas that have failed medi-
cal or surgical interventions. Observation alone would not
Patients treated with radioactive iodine need to take some be the preferred approach in a patient with a macroadenoma
precautions to avoid exposing family members to excreted (size >10mm) as control of tumor growth would be desired.
I-131 for the first 24days after treatment, but strict pro-
longed isolation is not necessary. Radioactive iodine treat-
ment may exacerbate thyroid eye disease. Pretreatment with 40. ANSWER:C. Increase levothyroxine dose.
glucocorticoids may help to minimize this risk in patients
with moderate to severe changes. Most patients with Graves This patient has central hypothyroidism due to his pitu-
disease who are treated with radioactive iodine will progress itary macroadenoma. Clinically, he has some symptoms
to develop postablative hypothyroidism that will require suggestive of inadequately treated hypothyroidism, includ-
treatment with levothyroxine replacement. It may take up to ing fatigue, weight gain, and dry skin. He does not have
26months to see the full effects of treatment after admin- symptoms or physical examination findings suggestive of
istration of a dose of radioactive iodine. Pregnancy should hyperthyroidism. His laboratory tests are classic for central
be delayed until at least 6 months after radioactive iodine hypothyroidisma low free T4 in combination with either
treatment to minimize the risk of fetal exposure. an inappropriately low or normal TSH. This case illustrates
how TSH alone cannot be used reliably to assess thyroid
hormone replacement in patients with central hypothyroid-
ism and/or hypothalamic/pituitary dysfunction. Clinical
38. ANSWER:D. Aand CPregnancy test and TSH findings and peripheral thyroid hormone levels must be
used to guide levothyroxine replacement therapy. Therefore,
Hyperprolactinemia is a relatively common cause of amen- in this case, despite the low TSH, the levothyroxine dose
orrhea and is certainly part of the differential diagnosis should be increased with the goal to normalize the free T4.
when a woman presents with missed menstrual cycles. It is
important to replicate the lab abnormality, as performance
of the breast examination immediately prior blood sampling
can cause a transient elevation in prolactin. Pregnancy must 41. ANSWER:E. Answers B, C, and DLate-night
be excluded as a cause of hyperprolactinemia in all women salivary cortisol, 1 mg overnight dexamethasone
of childbearing potential, since prolactin levels rise as a nor- suppression test, and 24-hour urine free cortisol with
mal physiologic response during pregnancy. ATSH level is creatinine
important, particularly in this patient with a positive family
history of thyroid disease and some nonspecific symptoms This patient should be screened for Cushings syndrome/
of hypothyroidism (weight gain and cold intolerance) since hypercortisolism due to a number of concerning clinical signs,

1 2 0 T h e B r i g h a m I n t e n s i v e R e v i e w o f I n t e r n a l M e d i c i n e Q u e s t i o n a n d A n s w e r C o m pa n i o n
including rapid weight gain and new diagnoses of diabetes somatostatin analogs acting via these receptors inhibits
mellitus, hypertension, and osteoporosis as well as notable somatotrope cellular proliferation and GH secretion, lead-
physical examination findings (round face, hirsutism, supra- ing to a reduction in IGF-I levels. Biochemical remission,
clavicular fat accumulation, pigmented striae and bruising). defined as normalization of IGF-I and GH levels, is associ-
After ruling out exogenous glucocorticoid exposure, there ated with improvement or resolution of acromegaly symp-
are three acceptable first-line screening tests for endogenous toms. Long-acting somatostatin analogs, such as lanreotide
hypercortisolism, including late-night salivary cortisol, 1 and octreotide LAR, are typically considered the initial
mg overnight dexamethasone suppression test, and 24-hour medical option in uncontrolled acromegaly. The dopamine
urine free cortisol with creatinine. Arandom isolated serum agonist cabergoline can be used but tends to be more effica-
cortisol level is not appropriate for screening as cortisol levels cious when the prolactin is only mildly elevated. The GH
are dynamic over time and influenced by a number of factors antagonist, pegvisomant, is also an option for this patient.
such as acute pain and stress. Although cortisol levels vary This drug tends to be second line due to the need for fre-
minute to minute, patients with Cushings syndrome show quent subcutaneous injections and its lack of inhibition of
generally elevated levels over a 24-hour period. The late-night tumor growth. Observation would not generally be advised
salivary cortisol test takes advantage of the fact that cortisol as the patient is symptomatic and biochemical remission is
is secreted in a diurnal rhythm such that nadir levels are usu- associated with improved clinical outcomes and reduced
ally observed late in the evening in a person without hyper- mortality. Repeat surgery would not be optimal in this case,
cortisolism. Patients with hypercortisolism lose the diurnal since a clear surgical target was not identified. Finally, radia-
variation and show high cortisol levels when they should be tion is generally reserved for patients with very aggressive
at the nadir. Similarly, Cushings syndrome patients will have tumors or those that fail medical therapy.
high 24-hour urine free cortisol levels that reflect their over-
all high continuous cortisol secretion over a 24-hour period.
Urinary creatinine should always be checked at the same time
as the 24-hour urinary free cortisol to ensure adequacy of the 43. ANSWER:C. Central diabetes insipidus
urine collection. Finally, the 1 mg overnight dexamethasone
suppression test takes advantage of the fact that individuals The symptoms and presentation in this case are most con-
with normal hypothalamic pituitary adrenal (HPA) physiol- cerning for the development of postoperative central diabe-
ogy will suppress cortisol secretion when given the synthetic tes insipidus. Diabetes insipidus is a syndrome of hypotonic
steroid dexamethasone. Individuals with a normal HPA axis polyuria and is due to either a deficiency of arginine vaso-
and thus normal inhibitory feedback mechanisms should sup- pressin, also known as antidiuretic hormone (ADH), or an
press their cortisol level to less than 1.8g/dL at 8:00 a.m. the inadequate renal response to ADH. Patients with diabetes
day following ingestion of 1 mg dexamethasone at 11p.m. the insipidus often complain of extreme thirst and will excrete
day before. These three tests only confirm hypercortisolism/ large amounts of very dilute urine. Without this hormone,
Cushings syndrome and do not tell the cause of the cortisol individuals cannot adequately concentrate their urine
hypersecretion. Additional testing with an ACTH level is and, if not allowed access to liquids, may develop severe,
required to differentiate between ACTH-dependent (most life-threatening hypernatremia. Central diabetes insipidus
commonly pituitary Cushings disease or ectopic source) and is due to deficiency, either partial or complete, of ADH.
ACTH-independent (likely adrenal source of cortisol over- Central diabetes insipidus can occur after any pituitary sur-
production) Cushings syndrome. Therefore, a pituitary MRI gery. It can be transient or permanent and is seen more fre-
is not a first-line screening test for hypercortisolism and is quently in patients with larger tumors or those that tend to
only indicated once an ACTH-dependent cause is suspected. invade the pituitary stalk such as craniopharyngiomas. The
diagnosis is not likely SIADH as this condition is typically
associated with hyponatremia. Type 2 diabetes mellitus
would also not be a likely cause of this patients acute severe
42. ANSWER:A. Medical therapy with a somastatin polyuria and polydipsia as his BMI is normal and his fasting
analog glucose is only mildly elevated. Given the clinical presen-
tation of a pituitary lesion, his diabetes insipidus would be
Multiple treatment modalities exist for recurrent acromeg- much more likely to be central than nephrogenic.
aly. In a case such as this one, medical therapy would be the
initial treatment recommendation, particularly since sig-
nificant residual/recurrent pituitary tumor is not visualized
on MRI. There are a number of medical treatment options, 44. ANSWER:D. All of the above
but somatostatin analogs are typically the initial treatment
of choice in this setting. GH-secreting pituitary tumor This patient was found to have an incidental pituitary
cells express somatostatin receptors, and treatment with lesion. All three of the answer choices are important for the

5 . E n d o c r i n o l o g y 1 21
initial evaluation of the patient. Athorough history must issued by the ADA regarding standards of medical care in
be obtained to illicit symptoms of mass effects and pitu- diabetes as well as a consensus statement by the American
itary dysfunction (both hypersecretion and hypofunction). College of Endocrinology (ACE) and the American
Having a complete understanding of pituitary physiology Association of Clinical Endocrinologists (AACE) rec-
will help guide the questions related to determining the ommend lifestyle intervention as the preferred treatment
function of each hormonal axis. The clinician should look option of prediabetes, as it has been shown to be safe and
for both signs and symptoms to suggest hormonal hyperse- highly effective. Starting metformin therapy would be a
cretion (i.e., hyperprolactinemia, GH excess, or hypercor- reasonable option to consider in this patient with other
tisolism) or hormonal hypofunction (i.e., central adrenal cardiac risk factors besides prediabetes. However, based on
insufficiency, central hypothyroidism, central hypogonad- current guidelines and available evidence, initiation/inten-
ism, GH deficiency, diabetes insipidus). Laboratory evalu- sification of lifestyle interventions, which in this patient
ation should be performed to assess pituitary hormonal would be directed toward dietary modifications, increased
function. MRI imaging should be performed to better exercise, and weight loss efforts, would be the first and most
characterize the structure of the mass. Taken together, important step in the treatment strategy for this patient.
both laboratory and imaging studies will help guide further
evaluation and treatment of the patient presenting with a
pituitary mass.
46. ANSWER:B. Enteric-coated baby aspirin (81 mg)
once daily

45. ANSWER:A. Dietary/lifestyle interventions Aspirin therapy is a well-established standard of care for
secondary prevention in individuals with known cardio-
Prediabetes is a condition in which normal glucose homeo- vascular disease. The role of aspirin for primary prevention
stasis is compromised. It is characterized by impaired fast- has, however, been debated. In patients with diabetes, the
ing glucose defined as fasting plasma glucose of 100125 current recommendation is for aspirin 75162 mg daily in
mg/dL [5.66.9 mmol/L], and/or impaired glucose toler- patients who are at increased cardiovascular risk and this
ance (IGT) defines as plasma glucose of 140199 mg/dL includes men age >50 of age or women age >60 of age with
[7.811.0 mmol/L] 2 hours after a 75 g glucose load, and/ at least one other cardiovascular risk factor (hypertension,
or HbA1c levels of 5.7%6.4%. Prediabetes confers a 3- to smoking, obesity, dyslipidemia, albuminuria, family history
7-fold increase in the risk of developing overt type 2 dia- of cardiovascular disease). In this patient, with multiple
betes compared with individuals with normal glucose val- cardiac risk factors and no history of allergy or any known
ues. Evidence from numerous studies also suggests that the contraindication for aspirin, daily low-dose aspirin therapy
chronic complications of type 2 diabetes start to develop would be indicated.
during the prediabetic state. Therefore, to minimize the
burden of complications associated with hyperglycemia,
early intervention, before overt diabetes develops, is pru-
dent. However, there are currently no approved pharmaco- 47. ANSWER:D. Switch to U-500 insulin.
therapies for prediabetes. Evidence from prevention studies
support that early intervention with lifestyle modification Severe insulin resistance (defined as the need for 200 units
or pharmacotherapy may slow down the progression to of insulin per day to achieve glycemic control) is commonly
diabetes by delaying the underlying pathophysiology of the seen with obesity and can complicate diabetes management.
disease. In the Diabetes Prevention Program (DPP), which The management of patients with diabetes who have severe
enrolled 3,234 individuals with impaired fasting glucose or insulin resistance is difficult, and at times frustrating, and
IGT, intensive lifestyle modification, aiming to achieve at requires a multifaceted approach.
least a 7% weight loss and 150 minutes of physical activity For obese patients, weight loss is the best treatment
per week, reduced the incidence of type 2 diabetes by 58% option, but weight loss can be a challenging task for patients
compared with placebo after 2.8 years of follow-up. Patients to achieve and maintain. Medications that decrease insu-
randomized to treatment with metformin in the DPP had a lin needs like metformin, thiazolidinediones (TZDs), and
31% reduction in the incidence of type 2 diabetes after 2.8 GLP agonists might help, but many patients still need high
years of follow-up. There is also evidence from randomized doses of insulin. In addition, treatment with TZDs is gen-
clinical trials suggesting potential benefit of other oral hypo- erally associated with weight gain, particularly when com-
glycemic agents, namely, acarbose, and thiazolidinediones bined with insulin. Bariatric surgery is highly effective for
in preventing the progression from prediabetes to overt dia- obese patients with severe insulin resistance.
betes, but these agents are either less effective or have safety Delivering an appropriate insulin volume to these
and tolerability issues. The most recent position statement patients can be difficult and inconvenient and may be

1 2 2 T h e B r i g h a m I n t e n s i v e R e v i e w o f I n t e r n a l M e d i c i n e Q u e s t i o n a n d A n s w e r C o m pa n i o n
best accomplished with U-500 regular insulin rather than 180 mg/dL would be the appropriate/recommended target
with standard U-100 insulin. Improved control may occur for this patient. The patient will require to be transitioned
because of better compliance with dosing (fewer total daily to a subcutaneous insulin regimen before the intravenous
injections) or better insulin action and absorption. U-500 insulin infusion is discontinued, and it is very likely that
kinetics are similar to premixed or NPH insulin. he will require at least a dose of basal insulin added to his
oral hypoglycemic medications on discharge. However, in
the immediate postoperative setting, where there are many
variables that impact blood glucose levels, including hyper-
48. ANSWER:B. You recommend a continuous metabolic neurohormonal stress responses in the setting of
insulin infusion using the standard IV insulin protocol critical illness/surgery, the patients nutritional status (e.g.,
used at your hospital with a target glucose range taking nothing or very little by mouth), potential require-
between 140 and 180 mg/dL. ment for pressors and so on, a continuous intravenous infu-
sion of insulin yields better blood glucose control and is the
The current standards of care and guidelines, including the optimum and recommended method for management of
American Diabetes Association guidelines for the medical hyperglycemia in this setting.
care of diabetes patients published in 2012, recommend
that in critically ill patients in the hospital, insulin therapy
be initiated for the treatment of persistent hyperglycemia
starting at a threshold of no greater than 180 mg/dL, and 49. ANSWER:D. Increase glipizide dose and stop
once insulin therapy is started, a glucose range of 140180 metformin.
mg/dL is recommended for the majority of these patients.
The optimal target range is a topic that has been debated. According to the current prescribing recommendations for
Van den Berghe etal. conducted the first prospective ran- metformin, impaired renal function, defined as serum cre-
domized trial comparing tight BG control (target 80110 atinine >1.5 mg/dL in men and 1.4 mg/dL in women, is a
mg/dL) with intensive insulin therapy to conventional BG contraindication for metformin use. The reason for this is
control (180200 mg/dL) in critically ill surgical patients. the concern for increased risk of lactic acidosis with metfor-
This study, which included over 1,500 patients, 63% of min use in the setting of renal insufficiency.
whom had undergone cardiac surgery before ICU admis- Biguanide therapy in type 2 diabetes can lead to lactic
sion, found that tight BG control resulted in a significant acidosis. Phenformin, a previously approved biguanide, was
reduction in mortality (10.6% with intensive treatment vs. removed from the market because of a high incidence of lac-
20.2% with conventional treatment, p=0.005) in patients tic acidosis as a side effect of this medication. The reported
who required 5 days of ICU care with multiorgan fail- incidence of lactic acidosis in metformin users has been very
ure and sepsis. In addition, cardiac surgical mortality was low. However, despite its rarity, lactic acidosis related to
reduced in those patients requiring 5 days of ICU care metformin remains a concern because if it does occur, it can
with other conditions. After this study, for a while, it was be a serious and potentially fatal complication. When used
widely accepted that tight BG control (80110 mg/dL) is according to current prescribing recommendations, the risk
better than conventional control in surgery or in the ICU. of metformin-induced lactic acidosis is close to zero and most
However, since then, several randomized trials failed to cases have occurred in patients with shock or tissue hypoxia
show the benefit of tight BG control with intensive insu- or in the presence of other predisposing conditions. These
lin therapy and a meta-analysis of 29 randomized studies predisposing factors, which are therefore also considered
focusing on the benefits and risks of tight glucose control contraindications for metformin use, include impaired renal
(very tight:110 mg/dL or moderately tight:<150 mg/dL) function (serum creatinine concentration above 1.4 mg/dL
in critically ill adult patients concluded that tight glucose in women and 1.5 mg/dL in men), concurrent liver disease,
control was not associated with significantly reduced hos- heart failure, past history of lactic acidosis, and decreased
pital mortality but was associated with an increased risk tissue perfusion or hemodynamic instability due to infec-
of hypoglycemia. In addition, a recent large prospective tion or other causes. Although patients with one or more
randomized multicenter trial (the NICE-SUGAR study) of these predisposing factors are at increased risk, the exact
demonstrated that intensive BG control with a target of incidence of lactic acidosis in these patients is not known. In
81108 mg/dL increased mortality among adults in the addition, use of serum creatinine alone may not be accurate,
ICU compared with conventional BG control with a target particularly in elderly or other patients with reduced muscle
of 180 mg/dL or less. Therefore, while more strict glycemic mass. These patients have diminished creatinine production
control (110140 mg/dL) may be appropriate in selected and, as a result, a given serum creatinine can represent a sub-
patients if it can be achieved without significant hypogly- stantially lower glomerular filtration rate (GFR) than see in
cemia, based on the currently available evidence and guide- patients with normal muscle mass. Therefore, rather than
lines, maintaining blood glucose target between 140 and using serum creatnine alone, the GFR should be estimated

5. E n docr i nology 123


in patients with stable serum creatinine concentration. The above options for this patient. DPP4 inhibitors are weight
precise serum creatinine limits and eGFR threshold for the neutral. Glipizide, a sulfolyurea, is less ideal for this patient
safe use of metformin remain uncertain. An estimated GFR as it has the potential for causing weight gain. Liraglutide, a
less than 60 mL/min/1.73m2 would be the approximate glucagon-like peptide-1 (GLP1) analog, would have been a
equivalent of the above serum creatinine cut-offs (1.4 mg/dL good choice for this patient as it may help with weight loss,
in women and 1.5 mg/dL in men), which, per the current but liraglutide and all other currently available GLP1 ana-
prescribing recommendations, are the cut-offs for renal logs are administered as subcutaneous injections, which the
function below which metformin use is contraindicated. patient is unwilling to do.
In clinical practice, clinicians frequently, however, choose Incretin-based therapies, which include the GLP1
to continue metformin use in patients with renal function analogs and DPP4 inhibitors, affect glucose control
parameters outside of these cut-offs. Some suggest reducing through several mechanisms, including enhancement of
metformin dose to half when the eGFR is between 30 and glucose-dependent insulin secretion, slowed gastric empty-
50 mL/min/1.73m2 . However, such use would be outside ing, regulation of postprandial glucagon, and reduction of
of the currently specified prescribing recommendations/ food intake. These agents do not cause hypoglycemia, in the
contraindications for metformin use and carry an unknown absence of therapies that otherwise cause hyperoglycemia.
risk. In this elderly patient who has CKD and a history of DPP4 is a ubiquitous enzyme expressed on the surface of
diastolic heart failure requiring diuresis, with serum creati- most cell types and deactivates a variety of bioactive pep-
nine levels that seem to have trended up and eGFR close to tides, including glucose-dependent insulinotropic polypep-
30, metformin should be discontinued. tide (GIP) and GLP1. DPP4 inhibitors, unlike the GLP1
analogs, can be administered orally. Sitagliptin, saxagliptin,
linagliptin, and alogliptin are the DPP4 inhibitors available
for the treatment of type 2 diabetes in the United States
50. ANSWER:D. Add sitagliptin. and many other countries. Vildagliptin is available in sev-
eral countries but not in the United States. Sitagliptin is
This patients A1C of 7.5% is still higher than goal (<7%) approved for the treatment of type 2 diabetes as mono-
despite her maximum efforts with lifestyle interventions therapy or as a second or third agent in those who are not
and metformin and therefore adding a second agent would adequately controlled. The usual dose of sitagliptin is 100
be indicated. mg once daily, with reduction to 50 mg for moderate to
Of the choices provided, given this patients weight severe renal insufficiency (glomerular filtration rate [GFR]
concerns and her refusal of injections, sitagliptin, a dipep- 30 to 50 mL/min/1.73m2) and 25 mg for severe renal insuf-
tidyl peptidase 4 (DPP4) inhibitor, would be the best of the ficiency (<30 mL/min/1.73m2).

1 2 4 T h e B r i g h a m I n t e n s i v e R e v i e w o f I n t e r n a l M e d i c i n e Q u e s t i o n a n d A n s w e r C o m pa n i o n
6.
NEPHROLOGY A ND HY PERTENSION

Karandeep Singh and Ajay K. Singh

1. A55-year-old man with a history of chronic kidney 3. For the patient described in Question 2, what is the
disease stage 3 secondary to hypertensive nephropathy appropriate next step in management?
presents with ongoing hypertension and mild bilat-
eral lower-extremity swelling. His blood pressure was A. Nitroprusside drip intravenously
previously controlled with hydrochlorothiazide. Labs B. Labetalol drip intravenously
reveal an estimated glomerular filtration rate (GFR) of C. Intravenous antihypertensives followed by emergent
30 mL/min/1.73m2 . Complete blood count, transami- plasmapheresis
nases, albumin, and coagulation studies are all normal. D. Intravenous antihypertensives followed by emergent
Urinalysis is normal. Arecent echocardiogram was nor- dialysis
mal. What is the next step in managing the patients E. Intravenous antihypertensives, followed by emergent
hypertension? plasmapheresis and dialysis

A. Start amlodipine. 4. A65-year-old woman with a history of chronic kid-


B. Start lisinopril. ney disease stage 4, hypertension, stroke, and recently
C. Change hydrochlorothiazide to chlorthalidone. diagnosed lung cancer presents to the clinic for evalu-
D. Change hydrochlorothiazide to furosemide. ation of asymptomatic anemia. Hemoglobin is 9 g/dL.
E. Low-salt diet Iron stores are normal. Vitamin B12 and folate levels
are found to be normal. What is the next step?
2. A 30-year-old man presents to the emergency depart-
ment with a 3-week history of headaches and blurry A. Start an erythropoietin stimulating agent.
vision and is found to have a blood pressure of 270/140 B. Transfuse packed red blood cells.
mm Hg. Physical examination is notable for retinal C. Transfuse packed red blood cells and start an
hemorrhages. The complete blood count reveals a white erythropoietin stimulating agent.
blood cell count of 5,500/mm3, hemoglobin of 8 g/dL, D. Start oral iron therapy.
and platelets of 50,000/L. Serum creatinine is 19 mg/ E. Counsel the patient on vascular access for dialysis.
dL. Coagulation studies are normal. Fibrinogen is ele-
vated. Lactate dehydrogenase is elevated, reticulocyte 5. A 72-year-old woman with three-vessel coronary
index is calculated to be elevated, and haptoglobin is artery disease returns from the operating room after
low. A review of the peripheral smear reveals schisto- undergoing coronary artery bypass grafting with
cytes. The urine sediment is remarkable for several red the intraoperative use of cardiopulmonary bypass,
blood cell casts. Electrocardiography reveals the pres- with postoperative labs revealing acute kidney injury
ence of left ventricular hypertrophy. and hyperkalemia with a potassium of 6.5 mEq/L.
What is the diagnosis? Electrocardiogram shows peaked T waves and widened
QRS complexes. Calcium, insulin, and dextrose are
A. Atypical hemloytic uremic syndrome administered. What is the next best management step
B. Glomerulonephritis for definitive treatment of hyperkalemia?
C. Hypertensive emergency
D. Thrombotic thrombocytopenic purpura A. Furosemide
E. Disseminated intravascular coagulation B. Sodium polystyrene sulfonate

1 25
C. Sodium polystyrene sulfonate with sorbitol 9. A41-year-old man presents with resistant hyperten-
D. Sodium bicarbonate sion and hypokalemia. Physical examination is remark-
E. Hemodialysis able for a blood pressure of 180/110 mm Hg despite
three antihypertensive agents at maximal doses. There
6. A 47-year-old African American man with recently is no peripheral edema on examination. Secondary
diagnosed HIV infection in the setting of esopha- hypertension workup reveals an aldosterone to plasma
geal candidiasis is referred for evaluation of bilateral renin activity ratio of 35, with a serum aldosterone
lower-extremity edema associated with a serum cre- level of 30 ng/dL. After confirmatory testing is per-
atinine of 2 mg/dL and 3+ albumin by urine dipstick. formed to demonstrate inappropriately high aldoste-
A24-hour urine collection reveals 6 grams of protein- rone secretion, computed tomography (CT) imaging of
uria. Testing for hepatitis B and C is negative. After con- the abdomen and pelvis with contrast is performed and
firming the suspected diagnosis by renal biopsy, which reveals no evidence of adrenal pathology. What is the
of the following treatment options should be pursued in next step?
addition to an ACE inhibitor?
A. Adrenal vein sampling
A. Steroids alone B. MRI with gadolinium
B. HAART alone C. Addition of spironolactone
C. HAART + steroids D. Renal artery stenting
D. Steroids + mycophenolate mofetil E. Renal sympathetic denervation
E. Plasmapheresis
10. A 48-year-old woman with rheumatoid arthritis
7. A 71-year-old man with chronic kidney disease pres- and Crohns disease presents to the clinic with bloody
ents for a routine visit. He is currently asymptomatic. diarrhea and abdominal pain, suspected to be related to
On physical examination, his jugular vein is not dis- Crohns flare. She is initiated on steroids in the clinic
tended and he has no peripheral edema. Laboratory and sent to the emergency department. Laboratory tests
data are remarkable for serum sodium of 130 mEq/L, are revealing for sodium of 135 mEq/L, potassium 3.0
potassium 4 mEq/L, chloride 104 mEq/L, bicarbonate mEq/L, chloride 110 mEq/L, bicarbonate 16 mEq/L,
24 mEq/L, blood urea nitrogen 28 mg/dL, creatinine 3 blood urea nitrogen 20 mg/dL, and creatinine 1.2 mg/
mg/dL, and glucose 90 mg/dL. An osmolal gap is pres- dL. Urinalysis is within normal limits. Urine pH is 6.5.
ent. Urine dipstick analysis is negative for protein. Of The urine anion gap is positive. What is the likely cause
the following, which is the most likely explanation of of the patients hypokalemia?
the patients hyponatremia?
A. Loss of potassium and bicarbonate in stool
A. Hypovolemia B. Hypokalemia from corticosteroid administration
B. Low solute intake C. Distal (Type 1)RTA
C. Multiple myeloma D. Proximal (Type 2)RTA
D. SIADH E. Type 4 RTA
E. Hypothyroidism
11. A 29-year-old woman with lupus presents with a
8. A 26-year-old woman is referred to the emergency new onset of 3+ proteinuria on urine dipstick. Aspot
department for evaluation of acute kidney injury. urine protein to creatinine ratio is 5.Serum creatinine
Physical examination is remarkable for a blood pres- is stable at 0.9 mg/dL. The patient has low C3 and C4,
sure of 130/80, diffuse skin thickening, and sclerodac- elevated dsDNA titers, and evidence of red blood cell
tyly. Laboratory tests reveal an acute rise in creatinine casts on urine microscopy. What is the next step?
from 1 mg/dL to 4.5 mg/dL. White blood cell count
is 4,500/L, hemoglobin is 7 g/dL, and platelets are A. Renal biopsy
110,000/L. LDH is elevated and haptoglobin is low. B. ACEI inhibitor
Schistocytes are present on peripheral smear. What is C. Prednisone
the next best step? D. Prednisone and mycophenolate mofetil
E. Prednisone and cyclophosphamide
A. Renal biopsy
B. Plasmapheresis 12. An 88-year-old man with a history of uncontrolled
C. Captopril hypertension and stroke was recently seen in the clinic
D. IVIG and started on chlorthalidone 12.5 mg daily and bena-
E. Methylprednisolone zepril 20 mg daily. He now presents to the clinic with a

1 2 6 T h e B r i g h a m I n t e n s i v e R e v i e w o f I n t e r n a l M e d i c i n e Q u e s t i o n a n d A n s w e r C o m pa n i o n
blood pressure of 145/70mm Hg. What is the next step D. Switch fluids to isotonic saline.
in management? E. Switch fluids to half-normal saline.

A. Increase chlorthalidone to 25 mg daily. 16. A 44-year-old woman with a history of asthma and
B. Increase benazepril to 40 mg daily. gastroesophageal reflux disorder (GERD) and chronic
C. Start extended-release metoprolol once daily. cough presents with a fever, rash, and a newly noted eosin-
D. Counsel on low-salt diet. ophilia, associated with acute kidney injury. Urinalysis
E. Perform a workup for secondary causes of is remarkable for cellular casts. She has been on inhaled
hypertension. steroids for treatment of her asthma and was recently ini-
tiated on treatment for GERD in the setting of a persis-
13. A 52-year-old woman presents to the emergency tent cough. The patient monitors her peak flows and these
room after a motor vehicle accident in which she suffered have been normal. She otherwise has no symptoms. What
a crush injury and is found to have acute kidney injury is the likely cause of the patients acute kidney injury?
3 days after admission to the hospital. Laboratory data
reveal a sodium of 137 mEq/L, potassium 4.3 mEq/L, A. Acute tubular necrosis
chloride 105 mEq/L, bicarbonate 25 mEq/L, creati- B. Acute interstitial nephritis
nine of 3 mg/dL, calcium 6.5 mg/dL, and phosphorus C. Acute glomerulonephritis
of 6 mg/dL. Creatine kinase is found to be 10,000 U/L. D. Henoch-Schonlein purpura
What is the most appropriate treatment? E. Drug Reaction with Eosinophilic and Systemic
Symptoms (DRESS) syndrome
A. Isotonic bicarbonate
B. Isotonic saline 17. A 70-year-old man with a history of congestive
C. Furosemide heart failure, coronary artery disease, hypertension,
D. Hemodialysis and total body volume overload presents with recur-
E. Mannitol rent admissions for pulmonary edema. His outpatient
blood pressures are well controlled on a combination of
14. An 18-year-old man presents with acute kidney furosemide, carvedilol, amlodipine, and lisinopril. He
injury 3 days after an upper respiratory tract infec- is compliant with his medications. Secondary workup
tion, with rise in creatinine from 0.9 to 1.6 mg/dL. The of the patients hypertension reveals unilateral athero-
patients urinalysis reveals 2+ blood and 2+ protein. C3 sclerotic renal artery stenosis. What is the next step?
and C4 are normal. Serologic workup is negative for
ANCA and anti-GBM. SPEP and UPEP are within nor- A. Percutaneous renal artery angioplasty with stent
mal limits. What is the likely diagnosis? placement
B. Renal artery surgical revascularization
A. IgA nephropathy C. Adding a thiazide diuretic
B. Postinfectious glomerulonephritis D. Increasing the dose of furosemide
C. Thin basement membrane disease E. Referring for heart transplantation
D. Alport syndrome
E. Granulomatous polyangiitis 18. A55-year-old woman with a history of long-standing
diabetes with macroalbuminuria and hypertension
15. A 71-year-old man with a history of COPD, smok- presents to the clinic with ongoing hypertension. She
ing, and chronic hyponatremia attributed to SIADH takes lisinopril 40 mg daily for management of hyper-
presents to the emergency department with altered tension, but blood pressure remains at 150/85 mm
mental status, tongue biting, and urinary incontinence. Hg. She reports compliance with a low-salt diet. Urine
Serum sodium is found to be 105 mEq/L, down from microalbumin to creatinine ratio reveals ongoing mac-
125 mEq/L, which was noted on laboratory testing roalbuminuria. What is the next best agent to add?
approximately 2 weeks prior to presentation. Therapy
is initiated with hypertonic saline, and a repeat serum A. Add valsartan.
sodium in 10 hours is found to be 120 mEq/L. His men- B. Add chlorthalidone.
tal status has now improved back to baseline. What is C. Add metoprolol.
the next step in therapy? D. Add aliskerin.
E. Add benazepril.
A. Continue hypertonic saline.
B. Stop hypertonic saline and start D5W and ddAVP. 19. A 32-year-old man returns from travel to India with
C. Stop hypertonic saline and monitor closely. symptoms of productive cough, night sweats, chills,

6 . N e ph r o l o g y a n d H y pe r t e n s i o n 1 2 7
and unintentional weight loss. He does not initially C. Glomerulonephritis
seek medical care and develops worsening malaise, D. Glomerular hypertrophy
nausea, and lightheadedness. When he presents to the E. Chronic interstitial nephritis
emergency department, his blood pressure is 84/45 mm
Hg. Laboratory data are remarkable for a sodium of 122 23. A 60-year-old patient with autosomal dominant
mEq/L and potassium of 6.1 mEq/L. Serum creatinine polycystic kidney disease underwent a living unrelated
is 1.1 mg/dL. How would you treat acutely? kidney transplant 10 years ago. He had been on stable
doses of tacrolimus, mycophenolate mofetil, and predni-
A. Fluid restriction sone, but he accidentally took double the dose of tacroli-
B. Hypertonic saline mus over the past 2 weeks. Which of these findings is not
C. Isotonic saline alone associated with calcineurin inhibitor toxicity?
D. Isotonic saline with hydrocortisone
E. Tolvaptan A. Acute kidney injury
B. Hyperkalemia
20. A30-year-old woman presents to the clinic for evalu- C. Hypomagnesemia
ation of an enlarging jaw mass. Labs reveal a potassium D. Hypercalcemia
of 5.5 mEq/L, calcium 6.5 mg/dL, phosphorus 11 mg/ E. Hyperlipidemia
dL, and a creatinine of 1.4 mg/dL. Uric acid is 15 mg/
dL. In addition to intravenous fluids, what is the next 24. A 38-year-old woman presents with newly diag-
step in the appropriate management of this condition? nosed Hodgkins lymphoma associated with bilateral
lower-extremity edema. Workup reveals 10 grams of
A. Rasburicase
proteinuria noted on a 24-hour urine collection. Which
B. Allopurinol
of the following pathologic entities most likely explains
C. Febuxostat
the presence of proteinuria in this patient?
D. Sodium polystyrene sulfonate
E. Hemodialysis
A. Membranous nephropathy
B. Minimal change disease
21. A 62-year-old man with a history of type 2 diabetes mel-
C. Focal segmental glomerulosclerosis
litus, coronary artery disease, peripheral arterial disease,
D. IgA nephropathy
and hypertension presents to the clinic for routine follow-
E. Amyloidosis
up. He is currently on three antihypertensive agents. His
blood pressure is 130/70 mm Hg. Serum creatinine is 0.7
25. A49-year-old man with untreated hepatitis C infec-
mg/dL. There is no evidence of microalbuminuria. Serum
tion develops persistent proteinuria. Which of the
aldosterone and plasma renin activity are mildly elevated.
following diagnoses is not typically associated with
A renal duplex study reveals bilateral renal artery stenosis.
associated with hepatitisC?
What is the next step in managing this finding?

A. Percutaneous renal artery angioplasty with stent A. Membranous nephropathy


placement B. Focal segmental glomerulosclerosis
B. Renal artery surgical revascularization C. Membranoproliferative glomerulonephritis
C. Adding an additional antihypertensive D. Cryoglobulinemia
D. Stopping an antihypertensive to permissively raise E. Polyarteritis nodosa
the blood pressure closer to 140
E. Do nothing 26. A 43-year-old man presents with bilateral
lower-extremity edema associated with 3 grams per day
22. A 56-year-old man presents for evaluation of anxi- of proteinuria with preserved renal function. He under-
ety, depression, and a peripheral neuropathy. He has goes renal biopsy, which demonstrates membranous
been employed as a painter for most of his adult life. nephropathy. Infectious workup for hepatitis B, C, and
Laboratory testing reveals an estimated GFR of 25 mL/ syphilis is negative. Complements are normal. Lupus
min/1.73m2, a nongap metabolic acidosis associated serologies are unremarkable. Age-appropriate cancer
with a positive urine anion gap, and a microcytic anemia. screening is unremarkable. In addition to ACEI inhibi-
A serum lead level was found to be elevated. What would tor, what therapy should be offered?
a renal biopsy be expected to demonstrate in this patient?
A. Nothing
A. Acute interstitial nephritis B. Steroids
B. Acute tubular necrosis C. Steroids and tacrolimus

1 2 8 T h e B r i g h a m I n t e n s i v e R e v i e w o f I n t e r n a l M e d i c i n e Q u e s t i o n a n d A n s w e r C o m pa n i o n
D. Steroids and cyclophosphamide A. Rituximab
E. Rituximab B. Steroids and mycophenolate mofetil
C. Steroids and cyclophosphamide
27. A92-year-old woman presents to the emergency depart- D. Steroids and tacrolimus
ment with symptoms of right-sided weakness and slurred E. Discuss dialysis and transplantation
speech that were noted upon awakening from sleep. MRI
imaging confirms an acute ischemic stroke. Her initial 31. A62-year-old woman presents with a rise in creat-
blood pressure on presentation was 240/120mm Hg, but inine from 1 to 6 mg/L in the span of a week, associ-
it has since come down to 200/100mm Hg. She contin- ated with a new onset of hemoptysis. Serologic workup
ues to have slurred speech and right-sided weakness but reveals positive c-ANCA and anti-PR3 antibody. What
appears to be alert and following commands. Which of is the appropriate treatment?
the following is the most appropriate blood pressure goal
in mm Hg in this setting? A. Steroids
B. Steroids and cyclophosphamide
A. Less than 220/120 C. Plasmapheresis, steroids, and cyclophosphamide
B. Less than 180/105 D. Rituximab
C. Less than 150/90 E. Dialysis
D. Less than 120/80
32. A65-year-old man with a family history of autoso-
28. A 77-year-old man with hypertension, restrictive mal dominant polycystic kidney disease is diagnosed
cardiomyopathy, recently diagnosed diabetes, and with polycystic kidney disease based on CT imaging.
peripheral neuropathy develops 10 grams per day of Which of the following treatments could be expected to
proteinuria as measured by a 24-hour urine collection. reduce the annual increase in kidney size?
The onset of peripheral neuropathy preceded the diag- A. Vasopressin receptor antagonism
nosis of diabetes. Physical examination is remarkable B. Antihypertensive therapy
for periorbital purpura and lateral scalloping of the C. Statin therapy
tongue. What is the likely explanation for the patients D. Low-protein diet
nephrotic range proteinuria? E. Steroid therapy
A. AA amyloid 33. A 54-year-old woman with a history of chronic kid-
B. AL amyloidosis ney disease stage 4 secondary to uncontrolled diabetes
C. FSGS mellitus presents with the following laboratory values:
D. Diabetic nephropathy calcium 7.1 mg/dL, phosphorus 6.3 mg/dL, PTH 1000
E. Hypertensive nephropathy pg/mL, and a normal 25-hydroxy-vitamin D level. In
addition to starting a phophorus binder, what is the next
29. An 18-year-old man presents with hemolytic ane- step in management?
mia, thrombocytopenia, and acute kidney injury, in the
absence of a diarrheal prodrome. C3 is low, and C4 is A. Start 25-hydroxy-vitamin D supplementation.
normal. CH50 is <10% and AH50 is <10%. ADAMTS13 B. Start 1,25-hydroxy-vitamin D supplementation.
activity is somewhat low at 40%. How should this con- C. Start cinacalcet, a calcimimetic.
dition be treated? D. Start calcium supplementation.
E. Refer for parathyroidectomy.
A. Plasmapheresis
B. IVIG 34. A 68-year-old woman with a history of coronary
C. Steroids artery disease, peripheral vascular disease, cerebrovas-
D. Rituximab cular disease with known strokes, diabetes mellitus,
E. Eculizumab and hypertension presents to the clinic for follow-up
of hypertension. She also reports a history of falls
30. A36-year-old woman with a history of lupus nephri- after episodes of orthostasis. Her medications include
tis presents with a rising creatinine level and worsening metoprolol, lisinopril, and hydrochlorothiazide. What
proteinuria. She recently completed a course of treat- would be an appropriate systolic blood pressure goal for
ment with steroids and mycophenolate mofetil for this patient in mm Hg?
ISN/RPS classIV and classV lupus nephritis. Arepeat
biopsy is performed and reveals classVI nephropathy. A. Less than 100
What is the most appropriate treatment? B. Less than 120

6 . N e ph r o l o g y a n d H y pe r t e n s i o n 1 2 9
C. Less than 140 C. Chronic interstitial nephritis
D. Less than 150 D. Hypocalcemia
E. Nephrotic syndrome
35. A 42-year-old man with a diagnosis of hyperten-
sion, coronary artery disease, congestive heart failure, 39. A 42-year-old man with a history of alcohol abuse
and obstructive sleep apnea presents for evaluation of presents to the emergency department with a seizure
ongoing hypertension. The patient is already on three in the setting of alcohol withdrawal. He is initiated on
antihypertensives. A secondary workup for causes of a lorazepam intravenous drip and transferred to the
hypertension is performed. Endocrine workup is nega- intensive care unit. Weaning the drip results in recur-
tive. Serum aldosterone and plasma renin activity are rent seizures, so lorazepam is restarted. Serum chem-
within normal limits. Arenal ultrasound duplex study istries reveal a sodium of 138 mEq/L, potassium 3.6
reveals no evidence of renal artery stenosis. What is the mEq/L, chloride 98 mEq/L, bicarbonate 22 mEq/L,
best next step in management? blood urea nitrogen 13 mg/dL, creatinine 0.9 g/dL,
and glucose 90 mg/dL. Ethanol level is undetectable.
A. Add a fourth antihypertensive medication. Measured serum osmolality is 298 mOsm/kg. Serum
B. Recommend continuous positive airway pressure at ketones are negative. Serum lactic acid is within normal
night. limits. Urinalysis is within normal limits. What is the
C. Refer for renal sympathetic denervation. likely diagnosis of the patients anion gap acidosis?
D. Counsel on DASH diet.
E. Counsel on low-salt diet. A. Lorazepam
B. Rhabdomyolysis related to seizure
36. A68-year-old man with a history of coronary artery C. Alcoholic ketoacidosis
disease with multiple prior myocardial infarctions, D. Uremia
peripheral arterial disease, and diabetes mellitus pres- E. D-lactic acidosis
ents with ongoing hypertension despite the use of bena-
zepril. Which of the following agents should be added 40. A35-year-old woman presents with a new onset of
next for the management of this patients hypertension? hypertension, and a secondary workup of hypertension
is performed. Renal ultrasound with duplex is notable
A. Hydrochlorothiazide for an increased peak velocity and tortuosity in the mid
B. Amlodipine and distal renal artery in the left kidney. Which of the
C. Furosemide following statements is true about this condition?
D. Amiloride
E. Minoxidil A. The condition is more common in men.
B. Extracranial cerebrovascular involvement is
37. A58-year-old woman with no known medical history common.
is sent to the emergency department for evaluation of C. MR angiogram is the preferred diagnostic study.
hypertension, with a blood pressure of 210/110mm Hg. D. Surgical management is the preferred treatment for
The patient currently has no complaints. Labs are unre- the renal lesions.
markable, with the exception of a urinalysis that reveals E. Atherosclerosis is the primary cause of the disease.
trace protein. Electrocardiogram reveals left ventricular
hypertrophy. What is the next step in management? 41. A55-year-old man with a history of cirrhosis from
fatty liver disease presents with a gastrointestinal bleed,
A. Oral chlorthalidone and lisinopril and follow-up as followed by the development of oliguric acute kidney
an outpatient injury with no proteinuria and a normal urine sediment.
B. Intravenous labetalol Urine sodium is less than 10 mEq/L. Volume repletion
C. Intravenous nitroprusside with blood products and crystalloid solution results in
D. Intravenous nitroglycerin no improvement in the renal function. Which of the fol-
E. Intravenous hydralazine and transition to oral lowing is true about the likely clinical diagnosis?
hydralazine
A. Octreotide and midodrine can be used for definitive
38. Which of the following is not a manifestation of management of this condition.
lithium nephrotoxicity? B. Liver transplant is contraindicated.
C. Hemodialysis, if indicated, is typically well tolerated.
A. Diabetes insipidus D. Renal biopsy would be expected to reveal a normal
B. Renal tubular acidosis kidney.

13 0 T h e B r i g h a m I n t e n s i v e R e v i e w o f I n t e r n a l M e d i c i n e Q u e s t i o n a n d A n s w e r C o m pa n i o n
E. If evaluating the patient for a liver transplant, a urine, and sputum cultures are sent, and he is started
kidney transplant would also be indicated. on empiric therapy with broad spectrum antibiotics.
Despite negative culture data and several days of anti-
42. A28-year-old man presents with arm and leg cramps biotics, the lactic acidosis is persistent. How would you
and polyuria and is found to have a blood pressure of classify this patients anion gap acidosis
~90/50 mm Hg associated with a serum potassium of
3.1 mEq/L and a magnesium of 0.9 mEq/L. Urinary A. Type Alactic acidosis
calcium is low and urinary chloride is elevated. What is B. Type B lactic acidosis
the likely diagnosis? C. D-lactic acidosis
D. Options A and B
A. Surreptitious vomiting E. Options B and C
B. Bartter syndrome
C. Gitelman syndrome 47. A53-year-old woman with a history of alcohol abuse
D. Primary hyperaldosteronism presents to the emergency department in an inebri-
E. Cushings syndrome ated state after drinking antifreeze solution. Labs are
remarkable for a negative ethanol level. There is no
43. An 18-year-old man with a history of deafness and anion gap, though an osmolal gap is noted. Urinalysis
hematuria presents with a progressively rising creatinine reveals positive ketones. What is the agent that is most
level. His sister, mother, and father are all healthy, though likely responsible for the patients symptoms?
he does recall a maternal grandfather who died of com-
plications of kidney disease. What is the likely diagnosis? A. Ethanol
B. Ethylene glycol
A. Thin basement membrane disease C. Methanol
B. Alport syndrome D. Isopropyl alcohol
C. Fabry disease E. Propylene glycol
D. Amyloidosis
E. Tubulointerstital nephritis with uveitis 48. An 88-year-old man with decompensated heart fail-
ure presents with nonoliguric acute kidney injury and
44. Which of the following treatments would be least volume overload. Which of the following is true about
likely to reduce the risk of recurrent stone formation in ultrafiltration therapy as compared to an aggressive
a patient with known calcium oxalate nephrolithiasis? stepped approach of medical management (i.e., loop
diuretics with addition of metolazone and selective use
A. Increasing fluid intake of inotrope or vasodilator therapy)?
B. Decreasing sodium intake
C. Increasing calcium intake A. Ultrafiltration is first-line therapy for cardiorenal
D. Decreasing calcium intake syndrome.
E. Decreasing oxalate intake B. Medical management is associated with more serious
adverse events.
45. A 19-year-old man with a history of stroke in the C. Ultrafiltration is more effective in removing fluid
absence of hypertension, left ventricular hypertrophy, than aggressive medical management.
and angiokeratomas (vascular cutaneous lesions) pres- D. Medical management preserves renal function more
ents for evaluation of chronic kidney disease. Renal than ultrafiltration.
imaging demonstrates multiple renal sinus cysts. What E. Ultrafiltration should not be performed in patients
is the likely diagnosis? with cardiorenal syndrome.

A. Thin basement membrane disease 49. A17-year-old otherwise healthy woman develops eye
B. Alport syndrome pain, followed by development of flank pain, hematuria,
C. Fabry disease subnephrotic range proteinuria, and glycosuria in the
D. Amyloidosis absence of hyperglycemia. What is the likely diagnosis?
E. Tubulointerstital nephritis with uveitis
A. Thin basement membrane disease
46. A39-year-old man with newly diagnosed Hodgkins B. Alport syndrome
lymphoma develops dyspnea and is noted to have C. Fabry disease
Kussmaul respirations. He is noted to have an anion D. Amyloidosis
gap acidosis and workup reveals a lactic acidosis. Blood, E. Tubulointerstital nephritis with uveitis

6 . N e ph r o l o g y a n d H y pe r t e n s i o n 131
50. A 55-year-old man with hypertension-associated A. Line sepsis
chronic kidney disease stage 5 and diastolic heart fail- B. Meningoencephalitis
ure is initiated on hemodialysis. During his first ses- C. Acute hyponatremia
sion of dialysis, he develops a headache, nausea, blurry D. Dialysis disequilibrium syndrome
vision, and altered mental status a few minutes into E. Uremic encephalopathy
treatment. He is afebrile. Which of the following diag-
noses is most likely?

13 2 T h e B r i g h a m I n t e n s i v e R e v i e w o f I n t e r n a l M e d i c i n e Q u e s t i o n a n d A n s w e r C o m pa n i o n
CH A PT ER6 A NSW ER S features of thrombotic microangiopathy. The objective
of treatment is to rapidly lower the mean arterial pressure
1. ANSWER:D. Change hydrochlorothiazide to by 25%. Nitroprusside should be avoided in the setting of
furosemide. renal failure due to the risk of cyanide toxicity. Labetalol
would be a reasonable choice for an antihypertensive in this
Amlodipine is incorrect because it may worsen the patients patient administered as a continuous drip. Plasmapheresis
lower-extremity edema. Though ACE inhibitors such as serves no role in the management of hypertensive emer-
lisinopril are recommended in patients with proteinuric gency. Though dialysis will likely be needed if renal func-
chronic kidney disease, they do not provide a preferential tion does not improve with control of the blood pressure,
benefit in treating patients with nonproteinuric chronic there is no information given in the question stem that sug-
kidney disease. Though chlorthalidone has been purported gests an emergent indication for it.
to be more effective than hydrochlorothiazide based on
meta-analyses (due to a longer duration of action), thiazide
diuretics are less effective than loop diuretics in chronic
kidney disease. For this reason, the correct answer is to 4. ANSWER:E. Counsel the patient on vascular
switch the thiazide to a loop diuretic (furosemide) to treat access for dialysis.
the patients apparent volume expansion. A low-salt diet
should always be recommended because a high-salt diet can Based on the 2012 Kidney Disease Improving Global
lead to diuretic resistance, but in this patient it is not likely Outcomes (KDIGO) guidelines, there should be care-
the reason for the patients diuretic resistance. ful consideration on whether erythropoietin stimulating
agent (ESA) therapy should be started for hemoglobin val-
ues >10 g/dL. The risks and benefits should be weighed.
Risks of ESAs include hypertension, thrombosis, stroke,
2. ANSWER:C. Hypertensive emergency and the observation of increased mortality in patients
with malignancy. In patients with hemoglobin values
The presence of markers of hemolysis (anemia, elevated less than 10 g/dL, ESA therapy needs to be individual-
LDH, low haptoglobin, and elevated reticulocyte index) ized. The absence of anemic symptoms and the presence
combined with the presence of schistocytes in the periph- of lung cancer would both argue against the use of ESA
eral smear raises concern for microangiopathic hemolytic therapy. Though packed red cell transfusion would be safe
anemia. Although these findings, in combination with to administer from the standpoint of malignancy, there is
thrombocytopenia, raise concern for hemolytic uremic no clear indication for transfusion in the absence of symp-
syndrome and thrombotic thrombocytopenic purpura, the toms. Iron therapy is not warranted in the setting of nor-
renal disease seen is usually much milder and hypertension mal iron stores. All patients with CKD stage 4 should be
of this severity is not typical. On the other hand, hyperten- counseled on vascular (and peritoneal) access for dialysis,
sive emergency can cause a thrombotic microangiopathy so this is the correct answer.
and is the likely explanation for this patients clinical pre-
sentation. The presence of left ventricular hypertrophy on
electrocardiogram (ECG) suggests that the hypertension
has been longstanding. Atypical hemolytic uremic syn- 5. ANSWER:E. Hemodialysis
drome describes a variant of hemolytic uremic syndrome
(HUS) that occurs in the absence of diarrhea and can be In this patient with hyperkalemia with peaked T waves
explained by other infections or complement dysregulation. and widened QRS complexes, temporizing measures and
Though red blood cell casts are seen in glomerulonephri- definitive management of hyperkalemia are strongly indi-
tis, they can also be seen in thrombotic microangiopathies. cated. Sodium polystyrene sulfonate with or without sorbi-
Disseminated intravenous coagulopathy (DIC) is less likely tol carries a risk of instestinal necrosis, which is greatest in
in the setting of normal coagulation studies and an elevated the postoperative setting and thus should be avoided in this
fibrinogen. patient. Sodium bicarbonate may aid in shifting potassium
intracellularly, but it does not remove potassium from the
body. Although a loop diuretic could be considered with
less severe hyperkalemia, the evidence of cardiotoxicity on
3. ANSWER:B. Labetalol drip intravenously electrocardiogram (EKG) warrants hemodialysis for cor-
rection of hyperkalemia. Indeed, furosemide take several
Based on the explanation for Question 2, the patients hours to be effective and will not resolve this patients EKG
diagnosis is hypertensive emergency, which presents with changes acutely.

6 . N e ph r o l o g y a n d H y pe r t e n s i o n 133
6. ANSWER:B. HAART alone recovery with discontinuation of dialysis and improved
mortality observed in patients treated with ACE inhibitors.
HIV-associated nephropathy (HIVAN) is seen most com-
monly in African Americans, especially in the setting
of advanced HIV with a low CD4 count associated with
acute kidney injury and nephrotic range proteinuria. Renal 9. ANSWER:A. Adrenal vein sampling
biopsy is required to confirm the diagnosis, as nearly 40% of
individuals with suspected HIVAN will have an alternate In this patient with an elevated aldosterone to PRA ratio
diagnosis by biopsy. Highly active antiretroviral therapy (>30) and elevated serum aldosterone (>20 ng/dL) with
(HAART) is the mainstay of treatment. Steroids are not positive confirmatory testing, the diagnosis is primary
typically indicated, but they may be considered if renal func- hyperaldosteronism. In cases of hyperaldosteronism with
tion worsens despite the initiation of HAART along with normal adrenal imaging, the possibilities include a nonvi-
RAS inhibition. Steroid sparing agents are not indicated. sualized adrenal mass and bilateral adrenal hyperplasia. The
Steroids plus mycophenolate mofetil is also not indicated. only way to distinguish between this in the face of normal
Plasmapheresis plays no role in the managment of HIVAN. CT imaging is to pursue adrenal vein sampling. CT imag-
Though HIV can be associated with a thrombotic microan- ing has superior spatial resolution as compared to MRI in
giopathy, the suspected diagnosis in this patient is HIVAN. the visualization of adrenal glands, so an MRI would not be
Furthermore, plasmapheresis should not be combined with helpful. Spironolactone would be appropriate only if bilat-
an ACE inhibitor because the combination can result in a eral adrenal hyperplasia were confirmed by adrenal vein
syndrome of abdominal pain, flushing, and hypotension. sampling. The other answers are not treatments for primary
hyperaldosteronism.

7. ANSWER:C. Multiple myeloma


10. ANSWER:C. Distal (Type 1)RTA
The first step in analyzing a patient with hyponatremia is to
differentiate pseudohyponatremia from true hyponatremia A positive urine anion gap (urine Na + K Cl) suggests
by evaluating the serum osmolality. Though this patients that impaired renal excretion of ammonium (which is an
calculated serum osmalality ([2 x Na] + [BUN/2.8] + [glu- unmeasured cation in the urine) is the cause of the patients
cose/18]) is on the lower end of normal at 275 mOsm/kg, metabolic acidosis, as opposed to loss of bicarbonate in
the presence of an osmolal gap suggests that the measured the stool, which would result in a negative urine anion
osmolality is actually normal to elevated, raising concern gap. The presence of a low serum bicarbonate with a nor-
for a pseudohyponatremia. Combined with a low anion mal anion gap favors a diagnosis of RTA as opposed to a
gap, which can be seen with hypercalcemia or a positively steroid-induced hypokalemia, which would be expected to
charged immunoglobulin, these findings are strongly sug- be associated with a metabolic alkalosis from mineralocor-
gestive of a diagnosis of multiple myeloma. Urine protein by ticoid receptor binding by the corticosteroid. Type 4 RTA
dipstick is specific for albumin and would not be expected is associated with hyperkalemia. Absence of glycosuria on
to reveal the presence of a paraprotein. All other answer urinalysis argues against Fanconi syndrome, which often
choices reflect causes of true hyponatremia. accompanies a proximal RTA from injury to the proximal
tubule. Distal RTA (seen with urine pH > 5.5 and hypoka-
lemia) related to rheumatoid arthritis is the likely explana-
tion of the hypokalemia.
8. ANSWER:C. Captopril

Scleroderma renal crisis (SRC) develops in 10 to 20% of


patients with diffuse systemic sclerosis. Clinically, it resem- 11. ANSWER:A. Renal biopsy
bles TTP-HUS and can present with hemolytic anemia,
mild thrombocytopenia, and a thrombotic microangiopa- Renal biopsy is required in most patients with known lupus
thy on renal biopsy, but the physical examination features of and evidence of renal involvement because the severity on
scleroderma help distinguish this entity from TTP-HUS. clinical presentation may be discordant with the true degree
Renal biopsy is not helpful because it will not distinguish of renal involvement by biopsy, and because the histologic
SRC from TTP-HUS. The use of high-dose steroids is a risk subtype along with indices of activity and chronicity help
factor for the development of SRC. The mainstay of therapy guide the treatment. Once the pathological classification is
is ACE inhibition (captopril) even in advanced renal failure determined, therapy may be initiated. ACEI inhibitor is an
requiring dialysis, as there is a much greater chance of renal important adjunctive therapy. Prednisone along with either

13 4 T h e B r i g h a m I n t e n s i v e R e v i e w o f I n t e r n a l M e d i c i n e Q u e s t i o n a n d A n s w e r C o m pa n i o n
mycophenolate mofetil or cyclophosphamide are both 15. ANSWER:B. Stop hypertonic saline and start
appropriate options for treatment of proliferative lupus D5W and ddAVP.
nephritis, though cyclophosphamide would pose a greater
risk of infertility in this patient. Rapid overcorrection of chronic hyponatremia (rise in Na
>10 mEq/24 hours) can result in osmotic demyelination syn-
drome. The symptoms of osmotic demyelination typically do
not occur for 26 days after the sodium has been overcor-
12. ANSWER:D. Counsel on low-salt diet. rected. Animal models show a decreased incidence of osmotic
demyelination when sodium that has been overcorrected is
In the very elderly (patients older than 80years), the benefits relowered. Human studies show that relowering is safe. The
of controlling blood pressure aggressively are not known. best answer in this patient would be to stop the hypertonic
Based on the HYVET trial, it is known that treating sys- saline and to relower the sodium using D5W and ddAVP.
tolic blood pressures to less than 150 decreases the risk of
fatal strokes, heart failure, and all-cause mortality. Since
this patients blood pressure is already under 150, medi-
cations should not be intensifed and a secondary workup 16. ANSWER:B. Acute interstitial nephritis
should not be performed. It is always reasonable to counsel
hypertensive patients on a low-salt diet, which may increase Though the triad of fever, eosinophilia, and rash is only seen
the efficacy of diuretics. in a small percentage of patients with allergic interstitial
nephritis (AIN), any combination of these signs should raise
suspicion for it. Allergic interstial nephritis (causing acute
interstitial nephritis) is the likely diagnosis in this patient,
13. ANSWER:B. Isotonic saline and a proton pump inhibitor for treatment of the patients
GERD is the likely culprit. Cellular casts raise concern for
Isotonic saline is the mainstay of therapy for management glomerulonephritis, but the type of cells was not specified
of rhabdomyolysis. Isotonic bicarbonate should not be used in the question stem. White blood cellcasts can be seen in
in the setting of hypocalcemia, as the use of bicarbonate AIN, whereas typically red blood cell casts are seen in glo-
can reduce the ionized fraction of calcium as a result of merulonephritis. DRESS syndrome can result from a drug
increased albumin binding of calcium, leading to symptom- hypersensitivity as well, but the absence of other systemic
atic hypocalcemia. Loop diuretics such as furosemide can symptoms makes this diagnosis unlikely.
be used if there is volume overload, but they can also worsen
hypocalcemia and should not be used as first-line therapy.
The role of mannitol in the management of rhabdomyolysis
remains uncertain. 17. ANSWER:A. Percutaneous renal artery
angioplasty with stent placement

Though the largest trial comparing renal artery stenting to


14. ANSWER:A. IgA nephropathy medical management (ASTRAL trial) found that stenting
conferred little benefit and much risk in the management
IgA nephropathy can present as a rapidly progressing glo- of renal artery stenosis, the enrollment criteria for the study
merulonephritis (biopsy with crescentic IgA nephropathy), excluded all patients whose physicians felt they would defi-
as a latent finding of hematuria and proteinuria, or in the nitely benefit from stenting. Though stenting should not
setting of an upper respiratory tract infection (synpharyn- generally be performed in asymptomatic patients, refrac-
gitic hematuria), as is the case in this patient. IgA nephrop- tory heart failure or recurrent flash pulmonary edema is an
athy typically occurs 13 days after an upper respiratory ACC/AHA ClassIindication for renal artery revasculariza-
tract infection, as opposed to postinfectious GN, which tion. The safest way to achieve this is via percutaneous angio-
occurs 721days after an infection. The presence of normal plasty and stent placement, with surgical revascularization
complements also helps to distinguish IgA nephropathy reserved for patients with complex anatomy. The other
from postinfectious GN. Thin basement membrane dis- options are not appropriate at this time.
ease can be associated with some hematuria but does not
fit the clinical picture. Alport syndrome can be associated
with proteinuria, hearing loss, and progression to ESRD, 18. ANSWER:B. Add chlorthalidone.
but it does not fit the clinical picture. Granulomatous poly-
angiitis (formerly known as Wegeners granulomatosis) is In diabetic patients, compelling medications for hyper-
unlikely with a negative ANCA. tensive management include thiazides, beta blockers,

6 . N e ph r o l o g y a n d H y pe r t e n s i o n 135
ACEIs, ARBS, and calcium channel blockers. Though indication to pursue renal artery revascularization. There
the COOPERATE trial suggested a benefit to dual RAS are considerable risks with both percutaneous angioplasty
blockade with ACEIs and ARBs (such as valsartan), it was and surgical revascularization, so these procedures should be
later found that the authors had engaged in serious scientifc reserved for patients with compelling indications. A blood
misconduct, resulting in the retraction of the paper from pressure of 130/70 mm Hg in a diabetic patient is appropri-
Lancet. The ONTARGET trial demonstrated no benefit to ate and requires no adjustment of antihypertensives.
combination therapy with an ACEI and an ARB. Diabetic
proteinuria tends to improve with lowering of the blood
pressure, and the addition of chlorthalidone would be a rea-
sonable choice for a second antihypertensive. Metoprolol is 22. ANSWER:E. Chronic interstitial nephritis
unlikely to lower the blood pressure as much as a diuretic.
Lead nephropathy can present acutely with a Fanconi syn-
drome from injury to the proximal tubules with intranuclear
inclusion bodies made up of lead-protein complexes. Chronic
19. ANSWER:C. Isotonic saline with hydrocortisone injury is characterized by the finding of chronic interstitial
nephritis. Glomerulosclerosis may also be seen on biopsy.
The patients symptomatology is highly concerning for
tuberculosis, and the acute development of hypotension,
hyponatremia, and hypokalemia are all concerning for
23. ANSWER:D. Hypercalcemia
adrenal crisis likely from tuberculous infiltration of the
adrenal glands. Though the patient will ultimately benefit
Acute kidney injury, hyperkalemia, hypomagnesemia, and
from multidrug therapy for tuberculosis, the acute man-
hyperlipidemia are all associated with the use of calcineurin
agement should focus on volume rescuscitation and steroid
inhibitors such as tacrolimus. Hypercalcemia is not typi-
administration for management of acute adrenal crisis.
cally associated with tacrolimus toxicity.
Fluid restriction is inappropriate in a patient who appears
hypovolemic. Hypertonic saline should be reserved for
patients with symptomatic hyponatremia and sodium val-
ues less than 120 mEq/L. Tolvaptan is contraindicated in 24. ANSWER:B. Minimal change disease
hypovolemic patients as it induces a free water diuresis that
may worsen hypovolemia. The most common cause of nephrotic syndrome in patients
with Hodgkins lymphoma is minimal change disease,
though focal segmental glomerulosclerosis has also been
described. The other conditions listed do not share an
20. ANSWER:A. Rasburicase association with Hodgkins disease. Approximately 0.4%
of patients with Hodgkins lymphoma develop minimal
This patient has spontaneous tumor lysis syndrome second- change disease, and the degree of proteinuria typically par-
ary to Burkitts lymphoma. Intravenous fluids are impor- allels the course of the malignancy.
tant in preventing this condition when administered with
chemotherapy, but rasburicase is the safest and fastest way
to break down uric acid (to the water-soluble compound
allantoin) to enhance its clearance. Allopurinol and febux- 25. ANSWER:B. Focal segmental glomerulosclerosis
ostat would not be helpful in the acute setting. The hyper-
kalemia is being driven by breakdown of tumor cells and Hepatitis Cassociated renal diseases include mixed cryo-
is not likely to respond significantly to sodium polystyrene globulinemia, membranoproliferative glomerulonephri-
sulfonate. Indications for hemodialysis in tumor lysis syn- tis (MPGN), membranous nephropathy, and polyarteritis
drome include severe oliguria, persistent hyperkalemia, and nodosa (PAN). Membranous nephropathy, MPGN, and
hyperphosphatemia-induced symptomatic hypocalcemia, PAN can also be observed in hepatitis B.Focal segmental glo-
none of which are present at this time. merulosclerosis is not particularly associated with hepatitis C.

21. ANSWER:E. Do nothing. 26. ANSWER:A. Nothing

In the absence of renovascular disease (given normal cre- Phospholipase A2 receptor (PLA2R) has been identified
atinine) and refractory hypertension, there is no acute as a key pathogenic antigen in idiopathic membranous

13 6 T h e B r i g h a m I n t e n s i v e R e v i e w o f I n t e r n a l M e d i c i n e Q u e s t i o n a n d A n s w e r C o m pa n i o n
nephropathy, as appears to be the diagnosis in this patient nephrotic syndrome but cannot explain the patients associ-
without a clear secondary cause. Treatment of idiopathic ated symptoms.
membranous nephropathy is stratified by the clinical sever-
ity. Patients with normal renal function and proteinuria <4
g/day (such as the patient in this case) can be safely observed
without immunosuppression, as they often undergo spon- 29. ANSWER:E. Eculizumab
taneous partial or complete remission. Glucocorticoids in
combination with either cyclophosphamide or a calcineurin The patients diagnosis is diarrhea-negative hemolytic
inhibitor may be effective in treating patients at moderate uremic syndrome (HUS), also known as atypical HUS.
and higher risk for progression of renal disease. Rituximab Atypical HUS is characterized by a low C3. When it is
may be used in refractory cases. caused by Factor H or Ideficiency, it is associated with a low
CH50 and AH50. ADAMTS13 activity may be low but is
typically greater than 10%, which helps to distinguish this
condition from thrombotic thrombocytopenic purpura.
27. ANSWER: A. Less than 220/120 mm Hg Blocking the terminal complement cascade is the most
effective therapy for this condition, which can be achieved
In a patient suffering from an acute ischemic stroke, the with eculizumab.
blood pressure goal is determined by candidacy for throm-
bolytic therapy. Because the patients stroke symptoms
occurred while she was asleep, the timing of symptom onset
cannot be determined. Thus, systemic thrombolytic ther- 30. ANSWER:E. Discuss dialysis and transplantation.
apy cannot be safely offered. In a patient who is not a can-
didate for thrombolytic therapy (such as the patient in this ISN/RPS class VI lupus nephritis denotes a biopsy with
case), most consensus guidelines suggest lowering the blood global sclerosis noted on more than 90% of the glomeruli.
pressure only if it is above 220/120 mm Hg. This strategy In this setting, the kidney is burned out and immunosup-
is described as permissive hypertension. For candidates of pressive therapy is not warranted. The appropriate course of
thrombolytic therapy, blood pressure must be maintained action in this setting would be to discuss dialysis and trans-
at <180/105 mm Hg to minimize the risk of hemorrhagic plantation, which are likely in the patients near future,
conversion. After the acute stroke episode has resolved, a even if no acute indications for dialysis exist at this time.
systolic blood pressure goal of 150 mm Hg would be appro-
priate in this very elderly patient (>80 years of age) as per
the HYVET trial.
31. ANSWER:C. Plasmapheresis, steroids, and
cyclophosphamide

28. ANSWER:B. AL amyloidosis Based on the results of the MEPEX trial, in patients with
an ANCA-associated vasculitis and a creatinine greater
Amyloidosis typically presents with a combination of symp- than 5.8 mg/dL, an approach of plasma exchange, prednis-
toms, signs, and lab abnormalities, including nephrotic olone, and cyclophosphamide was superior to an approach
syndrome, restrictive cardiomyopathy, peripheral neuropa- of a methylprednisolone pulse followed by prednisolone
thy, hepatomegaly, macroglossia, purpura, and abnormal and cyclophosphamide with regard to mortality and renal
bleeding. The presence of lateral scalloping of the tongue function. All of the patients in the MEPEX trial were
as seen in this patient can be a result of the lateral edges switched to azathioprine for maintenance therapy after
of the tongue being flattened by the teeth in the setting 3months. Dialysis may ultimately be needed, but based on
of macroglossia. AA amyloid tends to occur secondary to the information provided in the question, it is not emer-
chronic inflammation, which does not appear to be present gently indicated.
in this patient. AL amyloidosis refers to a monoclonal gam-
mopathy complicated by the formation of fibrils made up of
light chain fragments. The presence of purpura, especially
in the periorbital area, is characteristic of AL amyloidosis. 32. ANSWER:A. Vasopressin receptor antagonism
Diabetes mellitus can cause both a peripheral neuropathy
as well as nephrotic range proteinuria but usually needs to CKD is a risk factor for cardiovascular disease, so control
be present for several years prior to onset of these compli- of hypertension and lipids is certainly important. However,
cations. Hypertensive nephropathy is not associated with the only therapy listed above that has been shown (in
nephrotic range proteinuria. FSGS can be associated with the TEMPO trial) to reduce the annual increase in total

6 . N e ph r o l o g y a n d H y pe r t e n s i o n 13 7
kidney volume and slow the decline in kidney function is modest improvement in blood pressure. ADASH diet and
tolvaptan, a vasopressin receptor antagonist. A low-protein a low-salt diet may also be helpful, but the OSA needs to be
diet can slow the progression of CKD. Steroid therapy has addressed first.
no role in the treatment of polycystic kidney disease.

36. ANSWER:B. Amlodipine


33. ANSWER:B. Start 1,25-hydroxy-vitamin D
supplementation Based on the results of the ACCOMPLISH trial published
in the New England Journal of Medicine in 2008, the com-
This patient is suffering from secondary hyperparathyroid- bination of benazepril and amlodipine was superior to
ism associated with chronic kidney disease, in which the ele- the combination of benazepril and hydrochlorothiazide
vation in PTH results from a chronically high phosphorus in reducing cardiovascular events in patients with hyper-
and low calcium concentration. The goal of reducing PTH is tension with a high risk for such events. The other agents
to prevent high turnover renal bone disease. Overreduction would not be first- or second-line therapies for hyperten-
of PTH may result in a low turnover bone disease, so PTH sion. Furosemide can be used as a diuretic in the presence
is not typically lowered all the way down to normal. The of chonic kidney disease when thiazide diuretics lose their
first line of therapy in lowering PTH is to control high potency, which is not the case in this patient.
phosphorus via institution of a low-phosphorus diet and a
phosphorus-binding agent. The next line of therapy is to
correct for deficiencies in 25-hydroxy-vitamin D levels. The
next line of therapy would be to use 1,25-hydroxy-vitamin 37. ANSWER:A. Oral chlorthalidone and lisinopril
D to suppress PTH. Cinacalcet is reserved for refractory and follow-up as an outpatient
cases, often as a bridge to parathyroidectomy.
Hypertensive urgency is defined by severe hypertension in the
relative absence of symptoms, which is the diagnosis in this
patient. As opposed to hypertensive emergency, the manage-
34. ANSWER:C. Less than 140mmHg ment of hypertensive urgency involves the use of oral hyper-
tensives to achieve a reduction of blood pressure to goal over
Based on the results of the ACCORD blood pressure trial, hours to days. Given the severe elevation of blood pressure,
intensive blood pressure control (<120 mm Hg) was not a two-drug regimen should be started, and a thiazide-type
found to reduce either the composite outcome of stroke and diuretic combined with an ACE inhibitor would be a reason-
nonfatal cardiovascular events or the outcome of all-cause able regimen. Hydralazine is not a first-line antihypertensive
mortality when compared to standard blood pressure con- and intravenous hydralazine is not indicated.
trol (<140 mm Hg). Though there was a lower annual rate
of stroke in the intensive therapy arm, there was a higher
rate of serious adverse events in the intensive therapy arm.
Though this patient has a known history of strokes, inten- 38. ANSWER:D. Hypocalcemia
sive therapy should be avoided due to episodes of orthosta-
sis. As such, a systolic blood pressure goal less than 140 mm Nephrogenic diabetes insipidus (and central diabetes insip-
Hg would be appropriate. The JNC 8 guidelines recom- idus) can occur in patients with lithium nephrotoxicity.
mend a systolic blood pressure goal of less than 140 mm Hg Distal RTA, chronic interstitial nephritis, and nephrotic
in diabetics. syndrome (related to minimal change disease and focal seg-
mental glomerulosclerosis) have been described in patients
with lithium nephrotoxicity. Whereas hyperparathyroid-
ism with resultant hypercalcemia has been described in
35. ANSWER:B. Recommend continuous positive patients taking lithium therapy, hypocalcemia is not a man-
airway pressure (CPAP) at night ifestation of lithium toxicity.

Obstructive sleep apnea (OSA) is a known secondary


cause of hypertension. The severity of sleep apnea corre-
lates with the severity of hypertension, and approximately 39. ANSWER:A. Lorazepam
three quarters of patients referred with resistant hyper-
tension who were referred for sleep studies were found to The salient features in this patients clinical presentation
have OSA. Treating the OSA with CPAP may result in a include an anion gap acidosis and an osmolal gap (measured

13 8 T h e B r i g h a m I n t e n s i v e R e v i e w o f I n t e r n a l M e d i c i n e Q u e s t i o n a n d A n s w e r C o m pa n i o n
osmolality 298 mOsm/kg vs. calculated osmolality 285 42. ANSWER:C. Gitelman syndrome
mOsm/kg). This suggests an unmeasured osmole. This
occurs most commonly with ethylene glycol and methanol The diagnosis of Gitelman syndrome is present in this
poisoning. Propylene glycol toxicity can also present this patient. While both Gitelman and Bartter syndrome are
way, with most reported cases occurring due to administra- associated with hypotension and hypokalemia, the pres-
tion of medications that contain a propylene glycol solvent, ence of low urine calcium favors a diagnosis of Gitelman
including lorazepam and phenobarbitol. The other answer syndrome. If one can remember that Gitelman syndrome
choices would not cause an osmolal gap. Though acute resembles the administration of thiazide diuretics (whereas
alcohol poisoning can result in an osmolal gap, the nega- Bartter syndrome resembles loop diuretics), it is easy to
tive ethanol level suggests that the ethanol is not directly remember that Gitelman syndrome is associated with a
responsible for the osmolal gap. low urine calcium, as it is precisely this property of thia-
zide diuretics that makes them useful for the treatment of
calcium oxalate stone disease by minimizing calcium excre-
tion in the urine. Surreptitious vomiting would be expected
40. ANSWER: B. Extracranial cerebrovascular to result in a low urinary chloride from volume depletion.
involvement is common. Both primary hyperaldosteronism and Cushings syndrome
are associated with hypokalemia, and these patients are
In patients with established renal fibromuscular dys- typically hypertensive. In the case of Cushings syndrome,
plasia (FMD), as many as 65% of patients can be found physical examination findings may also be readily apparent.
to have carotid and verbral artery involvement. FMD is
more common in women. Though CT angiogram has
excellent diagnostic utility, MR angiography is poorly
sensitive for the diagnosis of FMD due to poor spatial 43. ANSWER:B. Alport syndrome
resolution. In patients with newly diagnosed hyperten-
sion as a result of FMD, percutaneous transluminal Alport syndrome is an inherited form of glomerular dys-
angioplasty is the preferred approach if opting for defini- function associated with sensorineural hearing loss that is
tive therapy. Unlikely atherosclerotic renal artery steno- most frequently X-linked in transmission. The syndrome
sis, the pathology lesion most commonly found in FMD affects males out of proportion to females. Symptoms may
is medial fibroplasia. begin with episodes of gross hematuria but ultimately prog-
ress to proteinuria, hypertension, chronic kidney disease,
and end-stage renal disease. The other diagnoses would not
account for the patients hearing loss.
41. ANSWER:D. Renal biopsy would be expected to
reveal a normal kidney.

Hepatorenal syndrome (HRS) is a feared complication of 44. ANSWER:D. Decreasing calcium intake
end-stage liver disease that carries a great degree of mor-
bidity and mortality. Hepatorenal syndrome is a disor- Decreasing calcium intake may paradoxically increase the
der marked by splanchnic vasodilitation and renal artery risk of calcium oxalate stone formation because there will
vasoconstriction, with a decrease in renal perfusion caus- be less intestinal calcium available to bind up oxalate, which
ing reductions in glomerular filtration rate and sodium may result in increased oxalate absorption and increased
excretion. It can be precipitated by infections or bleeding. risk of calcium oxalate stone formation. All the other
The renal parenchyma would be expected to be preserved measures listed should be recommended to patients with
early in the diagnosis. If a liver transplant was being con- known calcium oxalate stone disease to minimize the risk
sidered, a kidney transplant would not be indicated early in of recurrence.
the process, as there should be no irreversible injury pres-
ent. Octreotide and midodrine are temporizing measures
at best, as liver transplant is the only cure for type 1 HRS
unless there is spontaneous improvement in the underly- 45. ANSWER:C. Fabry disease
ing liver function. Hemodialysis may be difficult to per-
form in HRS patients, as they are frequently hypotensive Fabry disease is an X-linked lysosomal storage disorder that
at baseline. Continuous venovenous hemofiltration may be may be associated with renal dysfunction in half of patients
needed as a bridge to liver transplant if renal replacement by the age of 35years. Fabry disease should be suspected in
therapy is required. young adult patients with the following findings:chronic

6 . N e ph r o l o g y a n d H y pe r t e n s i o n 13 9
kidney disease of unclear etiology, multiple renal sinus and compared to diuretic therapy, the diuretic dosing used in
parapelvic cysts, decreased perspiration, cutaneous vascular this trial was not very aggressive. The more aggressive diuri-
lesions, left ventricular hypertrophy of unclear etiology, and etic dosing used in the CARRESS-HF trial demonstrated
stroke of unclear etiology. equivalent weight loss at 96 hours in both arms of treatment,
with fewer serious adverse events noted in the medical man-
agement arm. Renal function was less likely to be preserved
in patients who received ultrafiltration. Ultrafiltration does
46. ANSWER:B. Type B lactic acidosis serve a role in cardiorenal syndrome, but this is mainly lim-
ited to patients with diuretic resistance with persistent vol-
Type Alactic acidois is the most commonly seen and occurs ume overload.
as a result of cell breakdown from tissue hypoperfusion, as
is present in septic shock. This patient has no clinical signs
of septic shock, which is documented based on negative cul-
ture data and lack of response to broad spectrum antibiot- 49. ANSWER:E. Tubulointerstital nephritis with
ics. Type B lactic acid can be seen with medications (e.g., uveitis
metformin and certain HIV medications), malignancy, and
alcoholism. D-lactic acidosis may be seen in patients with Tubulointerstital nephritis with uveitis (TINU) syndrome
short gut syndrome as a result of bacterial metabolism in is found most frequently in adolescent women. The uveitis
the gut. This patient most likely has type B lactic acidosis is primarily anterior and may present with eye pain or red-
related to Hodgkins lymphoma, which may respond to ness. The other answer choices do not account for the pres-
chemotherapy. ence of eye symptoms. The presence of Fanconi syndrome
(glycosuria without hyperglycemia) is a marker of proxi-
mal tubular injury, as could be seen in tubulointerstitial
nephritis.
47. ANSWER:D. Isopropyl alcohol

The absence of an anion gap rules out ethylene glycol, meth-


anol, and propylene glycol. Anegative serum ethanol level 50. ANSWER:D. Dialysis disequilibrium syndrome
in an acutely intoxicated patient suggests this is less likely.
The patient is most likely suffering from isopropyl alcohol Dialysis disequilibrium syndrome classically occurs
toxicity, which causes an osmolal gap without an anion shortly after the initiation of hemodialysis as a result of
gap. The primary metabolite of isopropyl alcohol is acetone, rapid urea removal from the extracellular space, causing
which can be found on urinalysis with a positive study for water to shift into cells with resultant cerebral edema.
ketones. Though the patient swallowed antifreeze solution, Many strategies are undertaken when patients are first
this in itself is not helpful, as ethylene glycol, methanol, and initiated on dialysis to minimize the chances of develop-
isopropyl alcohol can all be found in antifreeze solutions. ing this syndrome, including using short treatment times,
minimizing ultrafiltration, using low-flux dialyzers, and
the use of mannitol. Infectious etiologies are unlikely in
the absence of fever. Uremic encephalopathy would be
48. ANSWER:D. Medical management preserves expected to improve, not worsen, after initiation of dial-
renal function more than ultrafiltration ysis though it may require several treatments before this
effect is seen. Hyponatremia is unlikely to develop from
Though the UNLOAD trial found greater amounts of initiation of dialysis, as the typical dialysate bath contains
fluid removal with ultrafiltration in the first 48 hours as 140 mEq/L of sodium.

14 0 T h e B r i g h a m I n t e n s i v e R e v i e w o f I n t e r n a l M e d i c i n e Q u e s t i o n a n d A n s w e r C o m pa n i o n
7.
DIGESTIV E DISEASES A ND DISOR DER S
OFTHEPA NCR EAS A ND LI V ER

Robert S. Burakoff, Muthoka Mutinga, and Molly L. Perencevich

1. A54-year-old man with a long history of gastroesoph- A. Digoxin may predispose patients to bowel ischemia.
ageal reflux disease undergoes an esophagogastroduo- B. Urgent angiography is indicated in order to identify
denoscopy (EGD), which reveals a 6 cm segment of a culprit blood vesse.l
salmon-colored mucosa extending proximally from the C. This disorder has the highest mortality rate of the
gastroesophageal junction. Biopsies demonstrate the ischemic gut disorders.
presence of Barretts esophagus (intestinal metaplasia of D. The diagnosis can be made on computed
the esophagus), but no dysplasia is noted. Surveillance tomography (CT) scan.
biopsies 1year later reveal no dysplasia. What is the cur- E. Serum lactate levels are markedly elevated in this
rent surveillance guideline for this patient? disorder.

A. Biopsy of the Barretts segment every 3months 4. A 56-year-old former intravenous heroin addict is
B. Biopsy of the Barretts segment every 612months noted to have mild transaminase elevation on routine
C. Biopsy of the Barretts segment every 35years labs obtained during a routine physical examination to
D. Biopsy of the Barretts segment every 10years established care. He reports sharing needles 25 years
E. No further biopsies are necessary. ago with someone he later learned had chronic hepa-
titis C. He has not sought medical care for more than
2. A19-year-old woman with a history of intermittent 30years. He is not overweight and does consume alco-
abdominal pain, nonbloody diarrhea, and no systemic hol or take medications or use supplements. The hepa-
symptoms is diagnosed with diarrhea-predominant titis C antibody test is negative, but a hepatitis C viral
irritable bowel syndrome after further evaluation load, checked because of his risk factors, is found to be
excludes an organic cause of her symptoms. Which of positive. Afalse-negative hepatitis C antibody test is sus-
the following treatment options in unlikely to result in pected. Which of the following conditions may be asso-
better symptom control? ciated with a false-positive hepatitis C antibody test?

A. Hyoscyamine A. Essential mixed cryoglobulinemia


B. Dietary modification B. Autoimmune disorders
C. Rifaximin C. HIV infection
D. Mesalamine D. Renal failure
E. Psyllium E. Chronic immunosuppressive therapy

3. A67-year-old man with a history of congestive heart 5. A 41-year-old man with AIDS whose CD4 count is
failure and peripheral vascular disease presents with 80 cells/mm3 presents with waxing and waning right
48 hours of left lower quadrant pain, diarrhea with upper quadrant abdominal pain for the past 2 weeks. He
intermittent bleeding, and low-grade fever. Stool cul- is afebrile and other vital signs are normal. Laboratory
tures are negative for infection and ischemic colitis is studies are notable for a total bilirubin of 2.7 mg/dL,
considered as a possible cause of his symptoms. Which direct bilirubin 1.7 mg/dL, alkaline phosphatase 380
one of the following is true of the disorder? U/L, AST 84 U/L, and AST 104 U/L. His white blood

141
count is normal. Abdominal ultrasound imaging is C. The causative organism, Tropheryma whipplei, is
notable for mild gallbladder wall thickening but no easily cultured.
pericholecystic fluid or gallstones. The common bile D. The disease is more common in women than men.
duct diameter is 9mm. What would be an appropriate E. Colonoscopy with biopsy is the diagnostic test of
next step in the management of this patient? choice.

A. Liver biopsy 9. Apatient with chronic viral hepatitis C, suspected to


B. Endoscopic retrograde cholangiopancreatography have been acquired from blood transfusions in 1983 fol-
C. Stool cultures lowing a serious motor vehicle accident, seeks further
D. Intravenous antibiotics evaluation. He has not previously received antiviral
E. Surgical consultation for cholecystectomy therapy and does not know the viral genotype. However,
he has read on the Internet that there are some new
6. A 61-year-old woman, seen on several occasions by treatments for hepatitisC.
her dentist for symptoms of dry mouth, is referred to Viral-specific protease inhibitor therapy, when com-
you for further evaluation after several recommended bined with pegylated interferon and ribavirin, has led
therapies fail to relieve her symptoms. She was also to advances in treatment of which hepatitis C genotype?
recently diagnosed with Hashimotos thyroiditis after
undergoing evaluation of fatigue. On review of systems A. Genotype 1
she reports dry eyes and occasional pruritis. Which of B. Genotype 2
the following gastrointestinal disorders is frequently C. Genotype 3
associated with secondary Sjgrens syndrome? D. Genotype 4
E. Genotype 5 and 6
A. Gastroesophageal reflux disease
B. Small intestinal bacterial overgrowth 10. A24-year-old man is alarmed at the development of
C. Gallstone pancreatitis swelling and pain of several joints in his hands and feet a
D. Primary biliary cirrhosis few weeks after recovery from an acute diarrheal illness.
E. Ulcerative colitis He is seen in the emergency department for evaluation.
He is afebrile, anxious, but not ill appearing. Adiagno-
7. A 29-year-old woman with occasional vague right sis of reactive arthritis is suspected. Which of the fol-
upper quadrant discomfort for several years is noted lowing is not typical of this syndrome?
to have a liver mass on abdominal ultrasound imag-
ing. She has had no prior imaging and has no history of A. Intestinal infection with ameba has been associated
chronic viral hepatitis or malignancy. She takes an oral with this syndrome.
contraceptive pill. Abenign lesion is suspected. Which B. There is a higher prevalence of HLA-B27 antigen.
of the following is not a common benign cause of a focal C. There is a high male-to-female ratio.
liver lesion? D. It may be associated with a triad of arthritis,
conjunctivitis, and urethritis.
A. Focal nodular hyperplasia E. Synovial fluid shows an elevated leukocyte count.
B. Cavernous hemangioma
C. Gastrointestinal stromal tumor 11. A38-year-old woman is seen in your office with com-
D. Hepatic adenoma plaints of sensation of a lump in her throat for the past
E. Nodular regenerative hyperplasia 2months. Physical examination of her neck and throat
are unremarkable. A barium esophagram is normal.
8. Apatient with a constellation of symptoms, including Which of the following is unlikely to be associated with
chronic diarrhea, ataxia, and weight loss, is discussed at a globus sensation?
a teaching conference. An evaluation by several special-
ists is unrevealing, and he is admitted because of con- A. Goiter
tinued clinical deterioration. Adiagnosis of Whipples B. Early hypopharyngeal cancer
disease is suspected. Which of the following is true C. Anxiety disorder
regarding Whipples disease? D. Myasthenia gravis
E. Gastroesophageal reflux disease
A. Chronic diarrhea is common, but extraintestinal
symptoms are rare. 12. A 33-year-old woman is seen in your office for evalu-
B. Prolonged antibiotic treatment is required to ation of fatigue, malaise, and mild right upper quadrant
eradicate the causative organism. discomfort, ongoing for the past 1 month. Her medical

14 2 T h e B r i g h a m I n t e n s i v e R e v i e w o f I n t e r n a l M e d i c i n e Q u e s t i o n a n d A n s w e r C o m pa n i o n
history is notable for hypothyroidism, and her only med- one of the following is not a common gastrointestinal
ication is levothyroxine. Her examination is only notable complication of cystic fibrosis?
for mildly tender hepatomegaly. On further questioning,
she reports no risk factors for viral hepatitis. She does not A. Small intestinal bacterial overgrowth
consume alcohol or illicit drugs, nor does she take herbs B. Gastroesophageal reflux disease (GERD)
or supplements. Her mother has lupus and a maternal C. Pancreatic insufficiency
aunt has rheumatoid arthritis. Laboratory tests are nota- D. Diarrhea
ble for ALT 480 U/L, AST 211 U/L, total bilirubin 0.4 E. Cirrhosis
mg/dL, alkaline phosphatase 123 U/L, total protein 9.1
g/dL, and TSH 2.1 U/mL. The antismooth muscle anti- 16. A28-year-old man with eczema undergoes an urgent
body titer is 1:320. Which of the following statements upper endoscopy in the emergency room to dislodge an
about her likely diagnosis is true? esophageal food impaction. Examination of his esopha-
gus after removal of the food bolus reveals multiple thin
A. This condition most often occurs in men. rings and linear furrows along most of the esophagus. No
B. The treatment of choice is prednisone with or hiatal hernia, mass, luminal narrowing, or visible inflam-
without azathioprine. mation is seen. Midesophageal biopsies are obtained.
C. Other autoantibodies are uncommon. Further questioning reveals a several-year history of
D. There is a very low risk of developing cirrhosis. intermittent dysphagia for solids with several prior epi-
E. It does not recur after liver transplantation. sodes of food impaction that resolved without need for
medical attention. He has not experienced weight loss
13. A31-year old woman with recurrent urinary tract infec- or frequent heartburn. His hematocrit is normal. What
tions is diagnosed with a C.difficile infection after under- is his likely diagnosis?
going evaluation of persistent diarrhea following recent
completion of a course of antibiotics. Which of the follow- A. Esophageal lichen planus
ing statements regarding C.difficile infection is false? B. Schatzkis ring
C. Eosinophilic esophagitis
A. Stool testing for cure should be performed after D. Peptic stricture
completion of treatment, E. Plummer-Vinson syndrome
B. There is a hypervirulent strain associated with lower
clinical cure rates and increased recurrence rates, 17. A44-year-old woman with intermittent mild right
C. All patients with inflammatory bowel disease (IBD) upper quadrant pain is very worried that she has gall-
who are hospitalized with a disease flare should bladder cancer after performing an Internet search of
undergo testing, her symptoms. She has no family history of hepatobili-
D. Repeat testing after a negative test should be ary disorders and ultrasound imaging is unremarkable.
discouraged, You reassure her that her risk of developing gallbladder
E. Hand hygiene with soap and water is more effective cancer is low. Which of the following is not a risk factor
than alcohol-based hand sanitizers, for gallbladder cancer?

14. A78-year-old man presents for evaluation of inter- A. Obesity


mittent episodes of choking when consuming both B. Porcelain gallbladder
liquids and solids. His wife has noticed that he has hali- C. Gallstones
tosis recently. He also occasionally experiences a gur- D. Tobacco use
gling sensation in his neck when he eats. What is the E. Chronic infection with Salmonella typhi
best test to make his diagnosis?
18. A 31-year-old man with AIDS and a CD4 count
A. Esophagogastroduodenoscopy (EGD) of 35 undergoes an esophagogastroduodenoscopy
B. Barium swallow (EGD) for evaluation of odynophagia, which failed to
C. CT scan respond to empiric therapy for candidal esophagitis.
D. Esophageal motility study Asolitary, well-circumscribed, large ulcer is seen in the
E. Laryngoscopy mid-esophagus. Biopsies are obtained for further eval-
uation. Which of the following is an unlikely cause of
15. The infant of a patient you manage was recently diag- esophageal ulceration?
nosed with cystic fibrosis after presenting with meco-
nium ileus. Your patient inquires about other common A. Pill-induced esophageal ulcer
gastrointestinal complications of this disease. Which B. Idiopathic esophageal ulcer

7. D i g e s t i v e D i s e a s e s a n d D i s o r d e r s o f t h e Pa n c r e a s a n d L i v e r 14 3
C. Herpes simplex virus (HSV) associated ulcer or over-the-counter medications. Her food intake has
D. Cytomegalovirus (CMV) associated ulcer been very poor over the past few months. What vitamin
E. Mycobacterium avium complex (MAC) associated deficiency is she likely to have?
ulcer
A. Vitamin C deficiency
19. A28-year-old man with a 12-year history of ulcer- B. Niacin deficiency
ative colitis undergoes a routine biennial surveillance C. Thiamine deficiency
colonoscopy exam. No visible colonic inflammation, D. Vitamin Adeficiency
polypoid lesion, or mass is noted. Surveillance biopsies E. Vitamin K deficiency
obtained every 10cm reveal focal high-grade dysplasia
in the distal colon. Which of the following is not true 22. A 53-year-old man with recently diagnosed and
regarding screening for colonic dysplasia in patients untreated hereditary hemochromatosis presents to the
with inflammatory bowel disease (IBD)? emergency room for evaluation of fever, weakness, and
malaise. He is hypotensive and is noted to have some
A. Screening colonoscopy exams for patients who hemorrhagic bullous skin lesions on his arms. He has
only have ulcerative proctitis should begin after no signs of skin infection or trauma. Of note, the illness
1012years of disease. began shortly after returning from a summer weekend
B. Patients with Crohns colitis are at increased risk of trip to the Chesapeake Bay area. He reports no sick con-
developing colorectal cancer. tacts and does not recall any tick bites or contact with
C. Patients with left-sided ulcerative colitis or pancolitis sick animals. What infection is the most likely cause of
have a higher risk of colorectal cancer than the his illness?
general population.
D. Patients who are noted to have high-grade dysplasia, A. Lyme disease
confirmed by a second pathologist, should undergo B. Tularemia
colectomy. C. Babesiosis
E. Patients with IBD and primary sclerosing D. Vibrio vulnificus
cholangitis (PSC) should undergo surveillance E. Norovirus
colonoscopy every 12years from the time of
diagnosis of PSC. 23. A 53-year-old man who underwent a Roux-en-Y gas-
tric bypass surgery 1 year ago is seen for evaluation of
20. A72-year-old man with a history of coronary artery fatigue. He has not experienced melena, hematochezia,
disease and arthritis presents to the emergency depart- or upper abdominal pain and does not take aspirin or
ment with melena. His wife reports that he takes both NSAIDs. He takes a proton pump inhibitor and iron
aspirin and ibuprofen. She also noticed that he seemed supplement, but he has not been taking a multivitamin
confused today. On physical exam, heart rate is 98 that had been prescribed following the surgery. Mild
beats/min and blood pressure is 85/43mm Hg. His labs muscle weakness is noted on exam. Laboratory tests are
are notable for a hematocrit of 26% (baseline 42%), nor- notable for mild neutropenia and microcytic anemia. A
mal INR and platelet count, normal albumin, and nor- colonoscopy performed 1 year ago was normal. What is
mal other components of the comprehensive metabolic the likely cause of his anemia?
panel. His calculated AIMS65 score is 3.Which of the
following is not used to calculate the AIMS65 score? A. Folate deficiency
B. Iron deficiency
A. Presence of melena C. Copper deficiency
B. Age >65 D. Vitamin B12 deficiency
C. Systolic blood pressure <90mm Hg E. Calcium deficiency
D. INR >1.5
E. Albumin <3.0 g/dL 24. A19-year-old man with frequent, painful oral aph-
thous ulcers, loose stools, and unexplained iron defi-
21. A23-year-old female with anorexia nervosa is seen ciency undergoes endoscopic evaluation for further
in your office for evaluation of a rash. Her examination workup. The entire colon and terminal ileum is nor-
is notable for a symmetric erythematous, pruritic, blis- mal. The upper endoscopy is notable for atrophic duo-
tering rash on exposed skin, resembling a sunburn, even denal folds and histologic examination reveals villous
though it is late winter. She reports intermittent diar- atrophy, crypt hyperplasia, and increased intraepithe-
rhea and her parents have noted that she appears disori- lial lymphocytes, suggestive of celiac sprue. The tissue
ented at times. She denies use of laxatives, illicit drugs, transglutaminase antibody is strongly positive. Which

14 4 T h e B r i g h a m I n t e n s i v e R e v i e w o f I n t e r n a l M e d i c i n e Q u e s t i o n a n d A n s w e r C o m pa n i o n
of the following extraintestinal conditions is not associ- goal and is no longer taking acid-suppressive therapy.
ated with celiac sprue? Which statement regarding GERD is false?

A. Pericarditis A. Surgical therapy is not recommended in patients


B. Osteoporosis who do not respond to proton pump inhibitor (PPI)
C. Hyposplenism therapy.
D. Idiopathic pulmonary hemosiderosis B. Routine biopsies from the distal esophagus are
E. Elevated liver enzymes recommended to diagnose GERD.
C. The treatment of choice for symptomatic relief and
25. A55-year-old man with a history of alcoholism and healing of erosive esophagitis is an 8-week course of a
cirrhosis complicated by ascites, variceal bleeding, and PPI.
hepatic encephalopathy is referred to you for outpa- D. Barium radiographs should not be performed to
tient care following a prolonged hospitalization. He has diagnose GERD.
abstained from alcohol use for 1year and participates in E. Obesity may contribute to the development of
substance abuse counseling. Liver transplantation eval- GERD.
uation is initiated by a hepatologist. Which statement is
true regarding care of patients with cirrhosis? 28. A56-year-old woman undergoes CT scan imaging
to evaluate persistent vague upper abdominal pain after
A. Bare stents are preferred for transjugular conservative management fails to relieve the symptoms
intrahepatic portosystemic shunts (TIPSs). and laboratory evaluation does not reveal the cause
B. Patients with ascites fluid total protein <1.1 g/dL of her symptoms. Acystic lesion is noted in the tail of
and serum bilirubin >2.5 mg/dL should receive her pancreas. Which of the following cystic pancreatic
prophylactic antibiotics. lesions has the lowest malignant potential?
C. Endoscopic variceal ligation is the preferred option
for secondary prophylaxis of variceal hemorrhage. A. Mucinous cystic neoplasm
D. Ambulatory patients who have an ascites fluid B. Main-branch intraductal papillary mucinous
PMN count 150 cells/mm3 should receive empiric neoplasm
antibiotics within 24 hours of their test result. C. Solid pseudopapillary neoplasm
E. The Model for End-Stage Liver Disease (MELD) D. Serous cystadenoma
score is based on albumin, INR, and total bilirubin. E. Side-branch intraductal papillary mucinous
neoplasm
26. A48-year-old attorney meets with a gastroenterolo-
gist to discuss further evaluation of a common bile duct 29. A28-year-old woman with a strong family history of
stricture noted on MRCP imaging for evaluation of an colon cancer at a young age has a colonoscopy for evalu-
elevated alkaline phosphatase. An ERCP with brush- ation of intermittent rectal bleeding. Nine medium-size
ings of the stricture is planned and the risks, benefits, polyps are found scattered throughout her colon and
and alternatives are discussed with the patient. The are resected. Microscopic examination of the resected
patient is very concerned about the risk of post-ERCP polyps reveals that they are hamartomatous polyps.
pancreatitis. Which of the following medications may Which of the following intestinal polyposis syndrome
reduce the risk of developing post-ERCP pancreatitis? is not characterized by hamartomatous polypsis?

A. N-acetyl cysteine A. Peutz-Jeghers syndrome


B. Octreotide B. MUTYH-associated polyposis (MAP)
C. Rectal indomethacin C. Juvenile polyposis syndrome
D. Allopurinol D. Cowdens syndrome
E. Pentoxifylline E. Cronkhite-Canada syndrome

27. A38-year-old man with long-standing gastroesoph- 30. A 49-year-old man with idiopathic cardiomyopa-
ageal reflux disease (GERD) is seen in your office for an thy undergoes evaluation for heart transplantation.
annual physical exam. Review of prior records indicates He is noted to have abnormal liver biochemical tests.
that an upper endoscopy examination performed 2years He does not consume alcohol. Lab tests to assess for
prior showed no evidence of Barretts esophagus, reflux chronic viral hepatitis and metabolic and autoimmune
esophagitis, or a hiatal hernia. At his last physical exam, liver disease are unrevealing. His medications are not
you advised him to lose weight for management of mild thought to be the cause of the laboratory abnormalities.
obesity. He successfully accomplished the weight loss Which of the following clinical or laboratory findings

7. D i g e s t i v e D i s e a s e s a n d D i s o r d e r s o f t h e Pa n c r e a s a n d L i v e r 145
are not generally associated with congestive heart fail- looked a little jaundiced 1 month prior. The patient
ure (CHF)-associated liver disease? has no risk factors for viral hepatitis; takes no medi-
cations, illicit drugs, or supplements; and consumes
A. Absence of portosystemic shunts such as esophageal alcohol infrequently. Laboratory and clinical features
varices are suggestive of acute liver failure. Which of the fol-
B. Low total protein (<2.5 g/dL) in ascites lowing metabolic liver diseases may result in acute liver
C. Lack of clinically overt jaundice failure?
D. Serum-to-ascites albumin gradient (SAAG)
>1.1 g/dL A. Nonalcoholic steatohepatitis
E. Presence of hepatojugular reflux B. Hemochromatosis
C. Alpha-1 antitrypsin deficiency
31. A 33-year-old woman with chronic hepatitis C D. Wilsons disease
(HCV) acquired from prior intravenous drug is very E. Glycogenic hepatopathy
concerned about passing the infection to her newborn
(vertical transmission). She is HIV negative and is in 34. Cholestatic liver diseases such as primary biliary
her third trimester of pregnancy. She has not had prior cirrhosis (PBC) and primary sclerosing cholangitis
treatment for the chronic hepatitis C.Which of the fol- (PSC) may result in fat-soluble vitamin deficiencies,
lowing statements regarding vertical transmission of especially when advanced liver disease is also present.
HCV is false? Symptoms of which fat-soluble vitamin may result in
loss of proprioception?
A. The mode of delivery has no effect on the risk of
transmission. A. Vitamin A
B. HIV coinfection does not affect the risk of B. Vitamin D
transmission. C. Vitamin E
C. Breastfeeding is not associated with increased risk of D. Vitamin K
transmission. E. Vitamin B12
D. Patients with HCV genotype 1 have similar risk of
transmission compared to other genotypes. 35. A 54-year-old man with cirrhosis due to chronic
E. Maternal HCV viremia is associated with an hepatitis C presents with a third episodes of esophageal
increased risk of transmission. variceal bleeding over the past year. He does not con-
sume alcohol nor does he use aspirin or NSAIDs. He is
32. A45-year-old woman is seen for evaluation of per- compliant with medications, which include a nonselec-
sistent epigastric pain, vomiting, and generalized tive -blocker and oral nitrate. After successful man-
weakness. Lab tests and a breath test for H.pylori are agement of the acute bleeding with esophageal variceal
all unremarkable. She is referred for an upper endos- band ligation, transjugular intrahepatic portosystemic
copy, which is notable for diffusely enlarged gastric shunt (TIPS) placement is considered to reduce the
folds. Multiple biopsies are obtained and a diagnosis of risk of recurrent variceal bleeding. TIPS would not be
Menetriers disease is suspected. Which of the follow- appropriate for the management of which of the follow-
ing statements regarding Menetriers disease in adults ing conditions?
is false?
A. Refractory ascites in a patient with right heart
A. It is generally a self-limited illness. failure
B. Abdominal pain, weight loss, occult blood loss, and B. Acute refractory variceal bleeding not controlled
hypoalbuminemia may be present. with pharmacologic and endoscopic therapy
C. The etiology is unclear. C. Hemorrhage from inaccessible gastric or intestinal
D. Treatment with a H2 receptor blocker may provide varices
symptomatic relief. D. Bleeding from portal hypertensive gastropathy
E. Afull-thickness biopsy of the stomach is usually E. Refractory hepatic hydrothorax
required for diagnosis.
36. A62-year-old man with a history of cardiomyopathy
33. A 22-year-old man with no significant past medi- of unclear etiology presents with difficulty swallowing
cal history is transported to the emergency department solids and liquids for the past 7 months. He also has
after being noted to be confused and jaundiced. Over regurgitation and has lost 15 pounds. Abarium swallow
the past year his parents has noticed that his penman- shows a dilated esophagus with a distal birds beak
ship has deteriorated. Friends also noticed that his skin narrowing. He is originally from Brazil but has been

14 6 T h e B r i g h a m I n t e n s i v e R e v i e w o f I n t e r n a l M e d i c i n e Q u e s t i o n a n d A n s w e r C o m pa n i o n
living in the United States for the last 20years. Which which of the following would you recommend with
of the following tests is not appropriate at this time? regard to her hepatitis B infection?

A. Upper endoscopy A. No treatment at this time


B. Antibodies for Trypanosoma cruzi B. Tenofovir
C. Anti-Scl-70 antibody C. Pegylated interferon
D. Esophageal manometry D. Cesarean delivery at term
E. Chest and abdominal CT E. Lamivudine

37. A 45-year-old woman presents with postprandial 40. A32-year-old woman with ulcerative colitis comes to
epigastric pain for the past 8 months. She does not your office for evaluation of symptoms. She was diagnosed
have dysphagia, anemia, nausea, or weight loss. Her with mildmoderate pancolitis last year. She initially
medical history is notable for diabetes. AHelicobacter responded well to induction and maintenance therapy
pylori stool antigen was negative. An empiric trial of with mesalamine. However, 2months ago she developed
a proton pump inhibitor for 8 weeks did not improve bloody diarrhea occurring 6 times per day, as well as
her symptoms. An esophagogastroduodenoscopy lower abdominal cramping. Treatment with prednisone
(EGD) was unremarkable, including gastric biopsies, 40 mg daily resulted in improved symptoms, but symp-
which are negative for H.pylori. Labs and imaging are toms recurred when the prednisone dose was decreased
not suggestive of biliary or pancreatic disease. Which to 10 mg daily. On physical examination her vital signs
of the following is the most appropriate next step in are normal and she has minimal left lower quandrant ten-
management? derness. Labratory studies reveal a white blood cell count
of 11,000/L, hematocrit of 34.5%, and stool testing for
A. Alow-fat diet Clostridium difficile is negative. Which of the following
B. Ranitidine is the most appropriate next step in treating this patient?
C. Metoclopramide
D. Amitriptyline A. Switch to balsalazide.
E. Probiotics B. Add budesonide.
C. Continue prednisone at 20 mg indefinitely.
38. A 69-year-old man with coronary artery dis- D. Increase prednisone to 40 mg daily and add
ease status-post myocardial infarction and per- azathioprine.
cutaneous coronary intervention 2 years ago E. Add ciprofloxacin.
presents with melena and hemorrhage-related anemia.
Esophagogastroduodenoscopy (EGD) identified an 41. A61-year-old woman presented to the hospital with
8mm clean-based ulcer in the duodenal bulb. In addi- new-onset abdominal distension and jaundice. She has
tion to his usual aspirin, he recently started taking a long history of drinking 23 glasses of wine per day
ibuprofen for back pain. Omeprazole is prescribed to and was drinking more than this for the past few months
facilitate ulcer healing. Which of the following would in the context of social stressors. She denies abdominal
you not recommend at this time? pain, rectal bleeding, melena, confusion, fever, or chillls.
On physical examination her vital signs are normal; she
A. Resumption of aspirin with omeprazole is mildly jaundiced and appears to have ascites, but no
B. Avoidance of regular NSAID use leg edema, spider angiomas, or asterixis. Her labora-
C. Abreath test for H.pylori after he completes an tory studies show ALT 66 U/L, AST 424 U/L, alkaline
8-week course of omeprazole phosphatase 285 U/L, total bilirubin 6.3 U/L, albumin
D. Low-dose acetaminophen for back pain 3 g/dL, white blood cell count 15,000/L, hematocrit
E. Repeat EGD in 8 weeks 35%, platelets 345,000/L, and INR 1.3. Testing for viral
hepatitis is negative. An abdominal ultrasound reveals
39. A24-year-old woman from Algeria is pregnant with moderate ascites, fatty liver with Doppler measurements
her first child. Routine screening identifies her as hav- suggesting portal hypertension, but no splenomegaly or
ing a positive HBsAg. Additional testing shows nega- evidence of portosystemic shunts. Aparacentesis showed
tive HBsAb, negative HBeAg, and positive HBeAb. ascites fluid albumin of 0.9 g/dL and white blood cell
Her hepatitis B viral load is 1,186 IU/mL at the end of count at 195mm3. Which of the following is the most
the second trimester. Liver enzymes, INR, and abdomi- appropriate next step in management?
nal ultrasound are normal. In addition to following her
liver enzymes and providing the infant with hepatitis B A. Cefotaxime
immune globulin (HBIG) and hepatitis B vaccination, B. Supportive care

7. D i g e s t i v e D i s e a s e s a n d D i s o r d e r s o f t h e Pa n c r e a s a n d L i v e r 14 7
C. Prednisolone D. Urinary 5-hydroxyindoleacetic acid
D. Nadolol E. Fat pad biopsy with Congo red staining
E. Pentoxifylline
45. A67-year-old man with cirrhosis due to hepatitis C
42. A 54-year-old man is hospitalized with pancreati- presents to the emergency room with hematemesis and
tis related to alcohol use. Five days after admission he hematochezia. He is initially hypotensive but is stabi-
continues to have epigastric pain and nausea, and he has lized with intravenous fluids and blood transfusions.
not been able to tolerate much oral intake. On physical On physical examination he is jaundiced, moderately
examination his vital signs are normal, but he has dif- encephalopathic, has spider angiomas on his chest,
fuse abdominal pain with palpation. Laboratory stud- and moderate ascites. Concurrently with intravenous
ies show a white blood cell count of 14,300 per L, ALT octreotide infusion and PPI therapy, which of the fol-
89 U/L, AST 345 U/L, alkaline phosphatase 176 U/L, low treatments should be initiated at this time?
and total bilirubin 2.3 U/L. Abdominal CT with intra-
venous contrast shows a diffusely edematous pancreas A. Penicillin
with multiple small peripancreatic fluid collections, no B. Intravenous albumin
pancreatic necrosis, and no gallstones or biliary dila- C. Ceftriaxone
tion. Which of the following is the most appropriate D. N-acetylcysteine
next step in management? E. Vasopressin

A. Parental nutrition 46. A78-year-old man with hypertension and diabetes


B. Enteral nutrition with a nasojejunal tube presents for evaluation of ongoing anemia for the past
C. Imipenem 5months. He denies abdominal pain, acid reflux, nau-
D. Pancreatic debridement sea, rectal bleeding, or melena. He has felt more tired
E. Interventional radiology drainage of fluid collections and short of breath with exertion but had not had chest
pain, cough, or lightheadedness. His medications are
43. A 44-year-old woman with autoimmune thyroid- lisinopril, insulin, and aspirin.
itis presents with recurrent episodes of pancreatitis. On physical examination his vital signs are nor-
Abdominal ultrasound shows no evidence of gallstones mal. He has no scleral icterus, lymphadenopathy, or
and she denies significant alcohol use. Abdominal cardiac murmurs. His lungs are clear and abdomen is
CT shows a diffusely enlarged pancreas with no focal soft, nontender, nondistended, with no organomegaly.
lesions. Which of the following is not likely to be found Laboratory testing shows a hematocrit of 28% (his
in this patients evaluation? prior baseline was 40%) with a mean corpuscular vol-
ume of 72 fL, but a normal white blood cell count,
A. Sacroiliitis platelets, and coagulation studies. Ferritin is 7 ng/mL.
B. Elevated serum IgG4 Fecal occult blood testing is positive. Colonoscopy and
C. Enlarged salivary glands esophagogastroduodenoscopy (EGD) did not identify a
D. Pancreatic biopsy showing a lymphoplasmacytic bleeding source. CT enterography was unremarkable.
infliltrate He received 2 units of blood transfusion for symptom-
E. Interstitial nephritis atic anemia. Which of the following is the next most
appropriate step in evaluation?
44. A73-year-old woman presents with persistent diar-
rhea for the past 6months. The diarrhea is watery, occurs A. Push enteroscopy
810 times per day, including at night, and does not B. CT angiography
improve with fasting. She also reports episodes of flush- C. Wireless capsule endoscopy
ing, as well as fatigue and dyspnea on exertion. Stool D. Double-balloon enteroscopy
studies are negative for infection and her blood count is E. Tagged red blood cell scan
normal. An electrocardiogram shows low-voltage QRS,
a chest X-ray shows cardiomegaly, and an echocardio- 47. A53-year-old woman with chronic pancreatitis and
gram shows moderatesevere tricuspid regurgitation. diabetes presents for evaluation of loose stool for the
Which of the following tests is most likely to be make a past 4 months. She reports 45 bowel movements per
diagnosis in this patient? day, including occasional nocturnal symptoms. She
has bloating and chronic abdominal pain, but no rec-
A. Serum vasoactive intestinal peptide tal bleeding or nausea. Her medical history includes
B. Serum calcitonin chronic pancreatitis due to prior alcoholism, diabetes
C. Serum gastrin mellitus, and seasonal allergies. She takes lisinopril,

14 8 T h e B r i g h a m I n t e n s i v e R e v i e w o f I n t e r n a l M e d i c i n e Q u e s t i o n a n d A n s w e r C o m pa n i o n
insulin, aspirin, oxycodone, pancreatic enzyme supple- department with epigastric pain for the past 36 hours.
mentation, and loratadine. Her sister has celiac disease. The pain is constant and radiates to her back. She also
She reports no change in diet or increase in fatty food has nausea, vomiting, and decreased oral intake. Her
consumption. medications are hydrochlorothiazide and occasional
On physical examination her vital signs are normal. acetaminophen. On physical exam, her temperature
She has mild epigastric pain with palpation but is oth- is 100.3F, heart rate 95 beats per minute, blood pres-
erwise not distended with no organomegaly. She has no sure 136/84 mm Hg, respiration rate 14 breaths per
rashes. Laboratory studies reveal normal liver enzymes minute, and BMI 31kg/m2 . She has slceral icterus and
and lipase, hematocrit 33% with mean corpuscular vol- dry mucous membranes. Her abdomen is tender in the
ume 102 fL, and normal white blood cell count and epigastrium and right upper quandrant, but there is no
platelets. Vitamin B12 is 198 pg/mL, folate 31 ng/mL, rebound or guarding. She does not have ascites or spi-
and tissue transglutaminase IgA antibody is normal der angiomas. Laboratory testing reveals a white blood
(as is the total serum IgA level). Stool culture, ova cell count of 13,600/L, lipase 3578 U/L, alanine ami-
and parasites examination, and a Giardia antigen are notransferase 347 U/L, aspartate aminotransferase 266
negative. Colonoscopy, including the terminal ileum, U/L, alkaline phophatase 272 U/L, and total bilirubin
is normal, including random biopsies of the colon. 5.2 mg/dL. Abdominal ultrasonography shows cho-
Esophagogastroduodenoscopy (EGD), including biop- lelithiasis and a dilated common bile duct to 13 mm,
sies of the duodenum, is normal. Abdominal imaging but no choledocholithiasis. The following day her pain
shows stable features of chronic pancreatitis, but no remains unchanged, her total bilirubin is 5.8 mg/dL,
intestinal inflammation or strictures. An increase in and her temperature is 101.1F
the dose of her pancreatic enzyme supplementation Which of the following is the most appropriate next
does not improve her symptoms. Which of the follow- step in management?
ing is the most likely diagnosis?
A. Abdominal CT
A. Steatorrhea due to pancreatic exocrine dysfunction B. Magnetic resonance cholangiopancreatography
B. Irritable bowel syndrome C. Cholecystectomy
C. Celiac disease D. Ursodiol
D. Small intestinal bacterial overgrowth E. Endoscopic retrograde cholangiopancreatography
E. Crohns disease
50. A 35-year-old woman is being evaluated for
48. A 29-year-old woman with Crohns disease and chronic abdominal pain and constipation for the past
prior small bowel resections presented with severe left 3 years. The pain is located in the lower abdomen,
flank pain. Urinalysis reveals 100200 red blood cells occurs approximately once per week, and improves
per high-powered field and no bacteria. Akidney stone after defecation. She reports having a bowel movement
is identified on CT scan. She reports no recent change approximately every 3 days, although the frequency
in her medications (infliximab, calcium, vitamin D), varies. The stools are hard and she often has a sense of
but she had been eating more spinach salads recently. incomplete evacuation. She denies nausea, vomiting,
In addition to increasing her fluid intake, what recom- rectal bleeding, weight loss, or neurologic symptoms.
mendations would you give this patient to prevent addi- She has one child. Her mother had colorectal cancer
tional stone formation? at age 59.
On physical exam, her vital signs are normal. Her
A. Low-oxalate diet and stop the calcium abdominal examination is unremarkable and rectal
supplementation tone is normal. Laboratory evaluation shows normal
B. Low-oxalate diet and continue the calcium renal function, electrolytes, complete blood count,
supplementation and thyroid-stimulating hormone. A colonoscopy and
C. High-oxalate diet and stop the calcium radiopaque marker study are normal. What is the most
supplementation likely diagnosis for this patient?
D. High-oxalate diet and continue the calcium
supplementation A. Colonic inertia
E. Potassium citrate B. Dyssynergic defecation
C. Constipation-predominant irritable bowel syndrome
49. A 61-year-old woman with a history of hyperten- D. Multiple sclerosis
sion and low back pain presents to the emergency E. Chronic intestinal pseudo-obstruction

7. D i g e s t i v e D i s e a s e s a n d D i s o r d e r s o f t h e Pa n c r e a s a n d L i v e r 14 9
CH A PT ER7 A NSW ER S 4. ANSWER:B. Autoimmune disorders

1. ANSWER:C. Biopsy of the Barretts segment every False-positive HCV antibody test may occur in patients with
35years autoimmune disorders. On the other hand, false-negative
HCV antibody test may occur in patients who have immu-
Current Barretts surveillance guidelines recommend EGD nocompromised states such as HIV and renal failure or
with biopsies every 35years if the initial biopsies followed who have malignancies such as lymphoma. Clarification of
by surveillance biopsies obtained 1year later do not reveal a suspected false-positive or false-negative HCV antibody
dysplasia. Barretts surveillance every 3 months is recom- test can be made by performing a HCV RNA test.
mended for patients with high-grade dysplasia who do not
wish to undergo ablative or surgical therapy. Asurveillance
interval of every 612months is recommended for patients
with low-grade dysplasia. No subsequent surveillance 5. ANSWER:B. Endoscopic retrograde
and surveillance every 10years are not acceptable for this cholangiopancreatography
patient, who may be at long-term risk of developing adeno-
carcinoma of the esophagus. This patient has HIV cholangiopathy. Biliary tract infec-
tion with CMV, cryptosporidia, and microsporidia are
thought to play a role in this disorder. Endoscopic retro-
grade cholangiopancreatography (ERCP) is the diagnostic
2. ANSWER:D. Mesalamine test of choice and facilitates detection of bile duct abnor-
malities such as papillary stenosis, strictures, and sclerosing
Irritable bowel syndrome (IBS) is a functional bowel dis- cholangitis. Endoscopic sphincterotomy, stent placement,
order characterized by episodic abdominal discomfort and or balloon dilation of the papilla may all lead to decreased
altered bowel habits. Bloating, nausea, and fatigue may also symptoms in patients with HIV cholangiopathy. A liver
be present. Visceral hypersensitivity is thought to play a role. biopsy and stool culture are not needed to make the diag-
Treatment options, including stool-bulking agents such as nosis of HIV cholangiopathy. The patient is afebrile and has
psyllium, dietary modification, and antispasmodic agents a normal white blood count, making bacterial cholangitis
such as hyoscyamine, may be beneficial in some patients. unlikely; hence, intravenous antibiotics are not warranted.
Rifaximin, a poorly absorbed, topically acting antibiotic, Lastly, the patient has no evidence of acute cholecystitis and
has been shown to reduce symptoms in some patients with hence does not require surgical consultation.
IBS. Mesalamine, while effective for treatment of inflam-
matory bowel disease (IBD), is not useful for management
of IBS.
6. ANSWER:D. Primary biliary cirrhosis

Secondary Sjgrens syndrome may develop in as many as


3. ANSWER:A. Digoxin may predispose patients to 40%65% with primary biliary cirrhosis (PBC). Typical
bowel ischemia. symptoms include dry mouth (xerostomia) and dry eyes
(keratoconjunctivitis), and may precede PBC-related symp-
Ischemic colitis is a form of gut ischemia most often toms. Gastroesophageal reflux disease, small intestinal
affecting the descending and sigmoid colon. It results bacterial overgrowth, and gallstone pancreatitis are not
from low vascular flow to a colonic segment, rather associated with Sjgrens syndrome. Ulcerative colitis is
than from an embolic event or large vessel thrombosis. associated with several extraintestinal manifestations, but
It is generally associated with low mortality. Digoxin, a secondary Sjgrens syndrome is not one of them.
splanchnic vasoconstrictor, can predispose to both mes-
enteric ischemia and ischemic colitis. CT scan imaging
may show colonic thickening, but it cannot distinguish
ischemic colitis from other inflammatory processes. 7. ANSWER:C. Gastrointestinal stromal tumor
Angiography is of limited value in ischemic colitis and is
more useful for evaluation of suspected mesenteric isch- Gastrointestinal tumors (GISTs) are generally benign
emia. Serum lactate elevation is uncommon in patients tumors, but they are most often found in the stomach and
with ischemic colitis and, if present, suggests the presence small intestine, not in the liver. Cavernous hemangiomas
of bowel infarction and necrosis. are the most common benign focal hepatic lesions. Focal

15 0 T h e B r i g h a m I n t e n s i v e R e v i e w o f I n t e r n a l M e d i c i n e Q u e s t i o n a n d A n s w e r C o m pa n i o n
nodular hyperplasia, 90% of which occur in women, are the common in men than women. Shigella sp. is the most com-
second most common benign liver tumors. Hepatic adeno- monly associated enteric organism, although Salmonella
mas are another common focal hepatic lesion that are usu- sp, Campylobacter jejuni, Yersinia enterocolitica, and even
ally associated with oral contraceptive use and sometimes Clostridium difficile have been implicated. Amebic intes-
undergo malignant transformation. Nodular regenerative tinal infections are not typically associated with reactive
hyperplasia is uncommon and is characterized by develop- arthritis.
ment of small regenerative nodules.

11. ANSWER:D. Myasthenia gravis


8. ANSWER:B. Prolonged antibiotic treatment is
required to eradicate the causative organism. Globus refers to the sensation of a lump or foreign body in
the throat in the absence of dysphagia and odynophagia.
Whipples disease requires prolonged antibiotic therapy, Myasthenia gravis can cause oropharyngeal dysphagia, but
usually for 12 months. Most cases occur in middle-aged not globus sensation. Gastroesophageal reflux, goiter, early
men. Extraintestinal symptoms are common and include hypopharyngeal cancer, and anxiety disorders all can be
rheumatologic, cardiac, and central nervous system associated with globus sensation.
involvement, which occurs frequently. The causative
organism, Tropheryma whippelii, a gram-positive bacil-
lus related to Actinomycetes, is difficult to culture.
Esophagogastroduodenoscopy (EGD) with small bowel 12. ANSWER:B. The treatment of choice is
biopsies is the diagnostic test of choice, not colonoscopy. prednisone with or without azathioprine.
Diagnosis can usually be made by identification of charac-
teristic PAS staining of macrophages in the lamina propria The patients elevated hepatic enzymes, female gender, lack
of the small intestine or with electron microscopy. of risk factors for chronic viral hepatitis, personal history
of hypothyroidism which could be autoimmune in nature,
family history of autoimmune disorders, elevated globulin
level, and high titer antismooth muscle antibody test are
9. ANSWER:A. Genotype1 strong clues suggesting autoimmune hepatitis. Autoimmune
hepatitis usually presents in the fourth decade and is more
The protease inhibitors boceprevir and telaprevir were common in women than in men. Several autoantibodies
approved for treatment of chronic hepatitis C genotype may be present such as antinuclear antibody (ANA), anti-
1 in 2011. The protease inhibitor must be administered soluble liver antigen (SLA), anti-liver-kidney-microsomal
in combination with pegylated interferon and ribavirin. type 1 (LKM-1) antibodies, and antineutrophil cytoplasmic
When compared to pegylated interferon and ribavirin antibodies (ASCAs), for example. Treatment consists of a
dual therapy, advantages of triple therapy include the fol- glucocortecoid (such as prednisone) with or without aza-
lowing: (a) improved sustained virologic response (SVR) thioprine. Undiagnosed and untreated autoimmune hepa-
in treatment-nave patients (60%80% vs. 40%50%); titis may lead to the development of cirrhosis. Autoimmune
(b) shortened therapy for some patients (2428 weeks vs. hepatitis recurs in 20%30% of patients after liver trans-
48 weeks); (c) new treatment options for prior relapsers plantation despite the immunosuppression regimen.
after antiviral therapy (SVR 60%80%) as well as partial
responders (SVR 50%) and null responders (SVR 30%) to
antiviral therapy.
13. ANSWER:A. Stool testing for cure should be
performed after completion of treatment.

10. ANSWER:A. Intestinal infection with ameba has The C.difficile toxin and PCR assays may remain positive
been associated with this syndrome. after resolution of symptoms; hence, testing for cure after
symptoms resolution is not advised. There is a hyperviru-
Postenteric reactive arthritis (formerly known as Reiters lent strain of C.difficile (NAP1/BI/027) that is associated
syndrome) usually develops 24 weeks after an acute diar- with lower clinical cure rates and increased recurrence
rheal illness. It is an immune-mediated synovitis, so the rates. There has been a significant increase in the inci-
synovial fluid shows an elevated leukocyte count, but it dence of C.difficile infection in patients with IBD. Hence,
is not infected. People who develop this syndrome have all patients with IBD who are hospitalized with a disease
a higher prevalence of HLA-B27 antigen, and it is more flare should undergo testing. Repeat C. difficile testing

7. D i g e s t i v e D i s e a s e s a n d D i s o r d e r s o f t h e Pa n c r e a s a n d L i v e r 151
after a negative test is positive <5% of the time and repeat Esophageal lichen planus can cause dysphagia due
testing increases the likelihood of a false-positive result. to stricture formation, but the disease mostly occurs in
Hand hygiene with soap and water is more effective than middle-aged women and strictures tend to be located in
alcohol-based hand sanitizers for eradicating C. difficile the proximal esophagus. A Schatzkis ring is solitary and
spores, and it is therefore recommended when caring for a located in the distal esophagus, often in association with a
patient with C.difficile infection. hiatal hernia. Peptic strictures generally occur in the distal
esophagus and cause focal luminal narrowing. In addition,
signs of acid-induced injury such as reflux esophagitis and
ulceration are commonly seen. Plummer-Vinson syndrome
14. ANSWER:B. Barium swallow consists of dysphagia, proximal esophageal webs, atrophic
glossitis, and iron deficiency anemia.
Zenkers diverticulum is an outpouching of the mucosa
immediately above the upper esophageal sphincer. The best
test to diagnose a Zenkers diverticulum is a barium swallow.
EGD may not identify them, and there is a risk-inadvertent 17. ANSWER:D. Tobacco use
perforation of the diverticulum. CT scan may identify
a large Zenkers diverticulum. Laryngoscopy is not an There is no direct association between tobacco use and
ideal method for identifying a Zenkers diverticulum. An gallbladder cancer. Obesity has been consistently shown to
esophageal motility study would not diagnose a Zenkers be associated with an increased risk of gallbladder cancer.
diverticulum, but it may provide information regarding the A porcelain gallbladder, characterized by intramucosal
underlying pathogenesis. calcification, due to chronic inflammation, is associated
with a 2%3% incidence of gallbladder cancer. Gallstones
are present in 70%90% of patients with gallbladder can-
cer. Large, solitary stones (>2.5cm) are associated with the
15. ANSWER:D. Diarrhea greatest risk. Chronic infection with Salmonella typhi is
associated with an increased risk of gallbladder cancer.
Cystic fibrosis (CF) is an inherited disease characterized by
a mutation in the cystic fibrosis transmembrane conduc-
tance regulator (CFTR). CFTR is found in all epithelia of
the gastrointestinal tract, including the pancreas and liver. 18. ANSWER:C. Herpes simplex virus (HSV)
Constipation and obstipation rather than diarrhea are com- associated ulcer
mon in patients with CF and are related to intestinal dys-
motility and decreased water secretion due to the CFTR Herpes simplex virus is usually associated with multiple
defect. Intestinal dysmotility, use of acid-suppressing small, shallow ulcers, rather than a solitary large ulcer.
agents, and chronic antibiotic therapy predispose CF Certain antiretroviral medications such as didanosine (ddI)
patients to develop small intestinal bacterial overgrowth and zidovudine (AZT) can cause pill esophagitis. These
(SIBO). SIBO occurs in 30%55% of CF patients. GERD is ulcers are typically solitary and are often found in the proxi-
reported in 30% of adult CF patients primarily due to inap- mal to mid-esophagus. CMV and idiopathic ulcers are the
propriate lower esophageal sphincter relaxation. More than most common cause of esophageal ulceration in AIDS and
85% of CF patients have pancreatic insufficiency related to are usually large, solitary, and well circumscribed. MAC
pancreatic ductal obstruction caused by thick secretions as can occasionally cause esophageal ulceration in patients
a result of the disease. Liver disease in patients with CF can with AIDS.
include asymptomatic elevation in liver enzyme tests, hepa-
tosplenomegaly, steatosis, and cirrhosis.

19. ANSWER: A.Screening colonoscopy exams for


patients who only have ulcerative proctitis should
16. ANSWER:C. Eosinophilic esophagitis begin after 1012years of disease.

Eosinophilic esophagitis typically occurs in young men with Colorectal cancer is one of the most dreaded complications
a history of atopic disorders such as eczema and asthma. of inflammatory bowel disease (IBD), accounting for 15% of
Endoscopic findings include multiple thin rings with linear IBD-related deaths. Patients with isolated ulcerative procti-
furrows. Eosinophilic esophagitis is a chronic inflammatory, tis (rectal involvement) are not considered at increased risk
immune-mediated condition in which dense eosinophilic infil- of developing colorectal cancer. Long-standing Crohns
trates are seen on esophageal biopsies (>15/high-power field). colitis is associated with an increased risk of colorectal

152 T h e B r i g h a m I n t e n s i v e R e v i e w o f I n t e r n a l M e d i c i n e Q u e s t i o n a n d A n s w e r C o m pa n i o n
cancer. Similarly, patients with long-standing ulcerative bacteria is in part dependent on the availability of iron.
colitis, including left-sided disease as well as pancolitis, are Vibrio vulnificus is a Gram-negative bacterium that causes
at increased risk of developing colorectal cancer. Screening gastroenteritis, wound infections, and septicemia. It can
for dysplasia and cancer should begin after 8years of disease also cause bullous skin lesions. Consumption of shellfish
in patients who have pancolitis and who are surgical candi- is a risk factor for acquiring Vibrio vulnificus infection.
dates, per current guidelines. Colectomy is recommended if Lyme disease is a tick-borne illness causes by the bacteria
high-grade dysplasia is detected and confirmed by a second Borrelia burgdoferi. Erythema migrans skin lesions rather
pathologist. Such patients carry a 43% risk of synchronous than bullous skin lesions are associated with Lyme dis-
malignancy. Patients with IBD and PSC have a higher risk ease, and sepsis is atypical. Tularemia is a zoonotic illness
of developing colorectal cancer, so it is recommended that caused by a Gram-negative bacterium, Francisella tularen-
they undergo surveillance colonoscopy every 12 years sis. It is transmitted by contact with infected animals or
from the time of diagnosis of PSC. invertebrate vectors. Most cases in the United States occur
in Arkansas, Missouri, South Dakota, and Oklahoma.
Patients usually present with fever and a single, erythema-
tous papulo-ulcerative lesion with central eschar. Babesiosis
20. ANSWER:A. Presence of melena is a tick-borne zoonotic illness, caused by the protozoan
Babesia microti, in northern and midwestern regions of the
The AIMS65 score is used to predict inpatient mortality in United States. Skin rash is atypical. Severe illness leading to
patients with upper gastrointestinal bleeding. The score is death has been reported especially in patients with signifi-
easy to calculate and relies on data readily obtainable in the cant levels of parasitemia. Norovirus can also be acquired
emergency room. from shellfish, but it typically causes a gastroenteritis and
The presence of melena is not a variable used to calcu- does not cause a characteristic skin rash.
late the score. Ascore of 1 point is given to each of the fol-
lowing five risk factors:Albumin <3.0 mg/dL, INR >1.5,
Altered mental status, Systolic blood pressure <90, and age
>65. A patient with an upper gastrointestinal bleed who 23. ANSWER:C. Copper deficiency
has an AIMS65 score of 3 has an inpatient mortality rate
of >40%, compared with the overall mortality of 6.5%, in Bariatric surgery is the most common cause of acquired
a recent study. copper deficiency. Microcytic, hypochromic anemia,
and neutropenia are typical hematologic manifestations.
Muscle weakness, skin depigmentation, abnormally formed
hair, hepatosplenomegaly, and even ataxia and neuropathy,
21. ANSWER:B. Niacin deficiency mimicking vitamin B12 deficiency, have been described.
Folate, iron, and vitamin B12 deficiency can occur in
This patient has pellagra, a common manifestation of niacin up to 40% of patients who have undergone Roux-en-Y gas-
deficiency. It is uncommon in the Western world except as tric bypass surgery, without long-term supplements. Lack
a complication of anorexia nervosa, alcoholism, and some of gastric acid impairs iron and vitamin B12 absorption.
malabsorptive disorders. The classic clinical features are diar- Bypassing the duodenum and proximal jejunum impairs at
rhea, pigmented dermatitis (as described in this patient), and folate and iron absorption as well. Folate and vitamin B12
dementia. Vitamin C deficiency, known as scurvy, causes deficiency cause macrocytic anemia. Though iron deficiency
perifollicular hemorrhage, gingivitis, anemia, and joint pain. may cause a microcytic anemia, muscle weakness and neu-
Thiamine deficiency may cause a variety of symptoms such tropenia are not typical. Furthermore, this patient has been
as polyneuropathy (dry beriberi), high-output cardiac failure taking iron supplements, making iron deficiency less likely.
(wet beriberi), as well as ataxia, nystagmus, confabulation, Poor calcium absorption can occur after gastric bypass, but
memory loss, and ophthalmoplegia (Wernicke-Korsakoff it usually causes secondary hyperparathyroidism and meta-
syndrome). Vitamin Adeficiency can cause night blindness, bolic bone disease.
corneal drying, and follicular hyperkeratosis. Vitamin K
deficiency causes easy bruising and bleeding.

24. ANSWER:A. Pericarditis

22. ANSWER:D. Vibrio vulnificus infection Some studies suggest that patients with celiac sprue may be
at increased risk of ischemic heart disease, but pericarditits
Patients with hereditary hemochromatosis are susceptible has not been reported to be associated with celiac sprue.
to sepsis due to Vibrio vulnificus, as the growth of this Impaired absorption of calcium and vitamin D can result in

7. D i g e s t i v e D i s e a s e s a n d D i s o r d e r s o f t h e Pa n c r e a s a n d L i v e r 153
osteoporosis in patients with celiac disease. Hyposplenism The greatest response to antireflux surgery is primarily seen
and elevated liver enzymes have been described in some in patients with typical symptoms of heartburn and/or
patients with celiac sprue, though the mechanisms are regurgitation, who demonstrate good response to PPI ther-
unknown. Patients with hyposplenism should receive apy, or have abnormal ambulatory pH monitoring studies
pneumococcal vaccination. The Lane-Hamilton syndrome with good symptoms correlation. PPI therapy is associated
characterized by the coexistence of celiac sprue and idio- with superior healing rates and lower relapse rates com-
pathic pulmonary hemosiderosis has been reported in some pared to H2 blockers in patients with erosive esophagitis.
patients. Adherence to a gluten-free diet has led to resolu- The overall sensitivity of barium esophagrams to diagnose
tion of pulmonary symptoms in a few patients. GERD is low; hence, it is not recommended to diagnose
GERD. Obesity is a risk factor for GERD.

25. ANSWER:B. Patients with ascites fluid total


protein <1.1 g/dL and serum bilirubin >2.5 mg/dL 28. ANSWER:D. Serous cystadenoma
should receive prophylactic antibiotics.
Serous cystadenomas are more common in women, usually
Cirrhotic patients with low total protein levels in ascites occur in people age 60 or older, predominate in the pancre-
fluid and advanced liver disease are at increased risk of atic head, and are almost always benign. They are usually
developing spontaneous bacterial peritonitis and should small lesions, calcify more than any other pancreatic neo-
therefore receive prophylactic antibiotics. Covered stents plasm, and may contain a classic central stellate scar.
are preferred to bare stents for TIPS procedures due to Mucinous cystic neoplasms are found almost exclu-
lower risk of shunt occlusion and dysfunction. A combi- sively in women, tend to occur at a younger age, predomi-
nation of a nonselective -blocker plus endoscopic variceal nate in the pancreatic tail, are solitary, and have a moderate
ligation is the preferred method for secondary prophylaxis malignant potential. Intraductal papillary cystic neoplasms
of variceal hemorrhage. The threshold for empiric antibiotic (IPMNs) occur with similar frequency in men and women,
treatment for suspected spontaneous bacterial peritonitis is and usually arise in the head of the pancreas. They appear
a PMN count >250. The Model for End-Stage Liver Disease to be more common in cigarette smokers, patients with
(MELD) score is based on creatinine, INR, and total biliru- familial adenomatous polyposis (FAP) and Peutz-Jegher
bin (albumin is not included). syndromes, and in patients with familial pancreatic car-
cinoma. Main-branch IPMNs have a higher malignant
potential than side-branch IPMNs. Solid pseudopapillary
neoplasms predominantly affect women and occur at a
26. ANSWER:C. Rectal indomethacin young age (median age of 3038years). They contain solid
and cystic components and may be difficult to differentiate
Meta-analyses of prior randomized controlled studies have from cystic pancreatic endocrine neoplasms on fine-needle
demonstrated that rectally administered indomethacin sig- aspiration (FNA) cytologic analysis. Although most solid
nificantly reduces the incidence of post-ERCP pancreatitis. pseudopapillary neoplasms are benign, some may exhibit
In addition, recent data suggests that rectally admnistered vascular and perineural invasion and are thought to have
indomethacin alone may be more effective than no pro- moderate to high malignant potential.
phylaxis and pnacreatic duct stent placement in preventing
post-ERCP pancreatitis.
N-acetylcysteine has not been shown to be beneficial in
preventing post-ERCP pancreatitis. Octreotide may reduce 29. ANSWER:B. MUTYH-associated polyposis
serum amylase elevation but studies evaluating its effect on
the incidence of post-ERCP pancreatitis are mixed. Studies MUTYH-associated polyposis (MAP) is an autosomal
of allopurinol and pentoxifylline have not supported their recessive disorder associated with biallelic mutations of the
use for the prevention of post-ERCP pancreatitis. MUTYH gene and is characterized by multiple adeno-
matous polyps. It is currently the only known autosomal
recessive hereditary colon cancer syndrome. MAP may be
phenotypically indistinguishable from attenuated famil-
27. ANSWER:B. Routine biopsies from the distal ial polyposis syndrome (AFAP) and, in a few cases, classic
esophagus are recommended to diagnose GERD. familial adenomatous polyposis syndrome (FAP).
Peutz-Jeghers syndrome is an autosomal dominant
Based on current literature, the use of routine biopsies of disorder due to a germline mutation of a serine threo-
the esophagus to diagnose GERD cannot be recommended. nine kinase (STK11 or LKB1) characterized by multiple

15 4 T h e B r i g h a m I n t e n s i v e R e v i e w o f I n t e r n a l M e d i c i n e Q u e s t i o n a n d A n s w e r C o m pa n i o n
hamartomatous polyps predominantly in the small intes- breast feeding, and viral genotype are not associated with
tine. Patients with Peutz-Jeghers syndrome have distinctive increased risk of vertical transmission of HCV.
mucocutaneous pigmentation (brown macules) not only on
the lips and perioral/buccal mucosa but also on the hands
and feet in some cases.
Juvenile polyposis syndrome is an autosomal dominant 32. ANSWER:A. It is generally a self-limited illness.
disorder associated with germline mutations of one of three
genes (BMPR1A, SMAD4, and ENG). Patients with juve- In adults, Menetriers disease typically is associated with
nile polyposis syndrome have multiple harmatomatous pol- chronic symptoms, whereas in children, it is usually a
yps in the colon and have an increased risk of developing self-limited disease. In adults, Menetriers disease is four
colorectal cancer. times more common in men than women. Symptoms of
Cowdens syndrome is an autosomal dominant disorder postprandial epigastric pain and weight loss are com-
associated with a germline mutation of the PTEN gene. mon. Patients may develop anemia due to gastrointes-
Patients with Cowdens syndrome have multiple gastroin- tinal occult blood loss. Hypoalbuminemia, peripheral
testinal hamartomatous polyps as well as some character- edema, and ascites may occur as a result of protein loss
istic dermatologic manifestations, including oral fibromas, from the hypersecretory gastric mucosa. The etiology
cutaneous verrucous papules known as trichilemmomas, of the disorder is unknown. Patients often experience
and punctate palmoplantar keratosis. relief of epigastric pain with the use of H 2 receptor
Cronkhite-Canda syndrome is a rare, nonfamilial dis- blockers. On esophagogastroduodenoscopy (EGD) there
order of unknown etiology with features including hamar- are thickened gastric folds. A full-thickness biopsy of
tomatous gastrointestinal polyposis. Other features of this the stomach is usually required for diagnosis, and the
syndrome include alopecia, onychodystrophy, cutaneous pathology shows extreme foveolar hyperplasia with glan-
hyperpigmentation, diarrhea, weight loss, and abdominal dular atrophy.
pain.

33. ANSWER:D. Wilsons disease


30. ANSWER:B. Low total protein (<2.5 g/dL) in
ascites Wilsons disease on rare occasions presents with acute liver
failure, which is uniformly fatal without liver transplanta-
Patients with CHF-associated liver disease generally have tion. Wilsons disease is a rare disorder of copper transport
preserved hepatic synthetic function and thus higher pro- with a prevalence of 1 in 30,000 people. It typically pres-
tein levels (usually >2.5 g/dL) than patients with primary ents in the second to fourth decade. Early in the disease,
liver disease. In addition, patients with CHF-associated hepatic steatosis may be present, and chronic liver disease is
liver disease rarely have evidence of portosytemic shunts common if it goes unrecognized and untreated. Neurologic
such as esophageal varices and lack clinically overt jaundice, symptoms such as tremors and choreic movements, psychi-
though up to 70% may have mild increase in unconjugated atric problems, hemolytic anemia, and other extrahepatic
bilirubin. Of note, just as in patients with primary liver manifestations may be present.
disease, patients with CHF-associated liver disease who Nonalcoholic steatohepatitis (NASH) is now increas-
have ascites have a SAAG >1.1 g/dL, as a result of portal ingly recognized as a major cause of cryptogenic cirrhosis.
hypertension. Hepatojugular reflux is generally present and Associated medical conditions include hypertension, dys-
can be useful in distinguishing hepatic congestion from pri- lipidemia, obesity, insulin resistance, or frank diabetes. It is
mary intrahepatic liver disease. not associated with acute liver failure.
Hemochromatosis is the most common inherited dis-
order among Caucasians of Northern European descent
with a prevalence of 1 in 200 people. Untreated patients
31. ANSWER:B. HIV coinfection does not affect the may develop cirrhosis, cardiac dysfunction, or diabetes, but
risk of transmission. acute liver failure has not been reported. Early detection
and treatment with phlebotomy may result in a normal life
Vertical transmission of hepatitis C (HCV) is estimated span for affected patients.
to be 20% in women coinfected with hepatitis C and Alpha-1 antitrypsin deficiency occurs in approximately
HIV, compared with a 4%7% risk in HCV monoin- 1 in 2,000 people. Over 75 different protease inhibitor (Pi)
fected patients. Vertical transmission occurs almost exclu- alleles have been described. The PiSZ and PiZZ phenotypes
sively among women who have detectable HCV viremia. can be associated with development of cirrhosis, but acute
The mode of delivery (vaginal versus cesarean section), liver failure has not been described.

7. D i g e s t i v e D i s e a s e s a n d D i s o r d e r s o f t h e Pa n c r e a s a n d L i v e r 155
Glycogenic hepatopathy occurs due to glycogen deposi- origin. Testing for Chagas is initially done with antibodies
tion in the liver in patients with poorly controlled type 1 for T. cruzi. Esophageal manometry is an appropriate test
diabetes mellitus. It causes hepatomegaly, abdominal pain, to confirm the diagnosis of achalasia. The characteristic
and elevated liver enzymes, but it does not typically cause manometry findings are poor-to-absent esophageal peristal-
acute liver failure. sis, incomplete relaxation of the LES, and elevated LES pres-
sure. The anti-Scl-70 antibody is a test for scleroderma, whose
esophageal manifestations include esophageal hypomotility
and incompetence of the lower esophageal sphincter.
34. ANSWER:C. VitaminE

Vitamin E deficiency, if severe, may result in abnormali-


ties affecting the posterior columns leading to ataxia, loss 37. ANSWER:D. Amitriptyline
of proprioception, and areflexia. However, severe vitamin
E deficiency occurs infrequently. vitamin Adeficiency may This patient appears to have functional dyspepsia. In a
lead to diminished nocturnal visual acuity. Vitamin D defi- patient with no improvement after 8 weeks of PPI ther-
ciency may result in metabolic bone disease. Vitamin K apy and no evidence of H. pylori disease, treatment with
deficiency may result in prolongation of prothrombin time a tricyclic antidepressant should be considered. Low-dose
and increased susceptibility to bleeding. Although neuro- amitriptyline or desipramine 10 to 25 mg at night is most
logic symptoms can be associated with vitamin B12 defi- commonly used. An H2-blocker is less likely to be helpful
ciency, it is not a fat-soluble vitamin. if a PPI was ineffective. Promotility agents such as meto-
clopramide can be considered if there is a concern for gas-
troparesis, although this patient does not have any clearly
predisposing conditions (such as diabetes) and a gastric
35. ANSWER:A. Refractory ascites in a patient with emptying study can be performed for further evaluation.
right heart failure There is currently no evidence to support a low-fat diet or
probiotics in the treatment of functional dyspepsia.
TIPS increases right-sided heart pressure due to the creation
of a portosystemic shunt and is therefore contraindicated
in patients with right heart failure. Acceptable indications
for TIPS include management of acute refractory variceal 38. ANSWER:E. Repeat upper endoscopy in 8 weeks
bleeding; hemorrhage from inaccessible gastric or intesti-
nal varices; bleeding from portal hypertensive gastropathy; The etiology of duodenal ulcers is most commonly related
recurrent severe variceal bleeding after endoscopic therapy; to aspirin, NSAIDs, and H. pylori. Even though he has a
refractory hepatic hydrothorax; Budd-Chiari syndrome; history of recent NSAID use, he should still be evaluated
and some other veno-occlusive disorders. for H. pylori infection as a contributing factor. Patients
with uncomplicated duodenal ulcers generally do not need
follow-up EGD due to the low likelihood of malignancy.
Since PPIs and gastrointestinal bleeding can result in
36. ANSWER:C. Anti-Scl-70 antibody false-negative testing for H.pylori, it is preferable to test a
patient with a breath test or stool antigen after a comple-
Idiopathic achalasia, pseudoachalasia, and Chagas disease tion of PPI therapy. If a patient requires aspirin for a cardiac
may present with similar symptoms and have indistinguish- indication, concurrent PPI therapy should be prescribed
able features on barium swallow imaging. Older age (espe- to reduce risk of ulcer recurrence. Interruption of aspirin
cially over age 60), weight loss, and a relatively short onset therapy my increase the risk of adverse cardiac outcomes.
of symptoms (<6months) raise concern for pseudoachalasia. However, if other noncritical medications, such as NSAIDs
Upper endoscopy should be performed to assess for a distal in this case, can be avoided, this is advisable. Low-dose acet-
esophageal or proximal gastric neoplasm. Paraneoplastic aminophen does not increase the risk of ulcer recurrence or
manifestations of extraintestinal tumors such as pancre- prevent ulcer healing.
atic, lung, and lymphoma may result in symptoms of dys-
phagia; therefore, a CT scan may be indicated if there is a
high index of suspicion. Chagas disease is caused by infec-
tion with Trypanosoma cruzi. The chronic form of Chagas 39. ANSWER:A. No treatment during pregnancy
disease can cause both cardiac and gastrointestinal disease,
including a picture consistent with achalasia, and most com- This patient is a chronic carrier of hepatitis B who appears
monly occurs in people of South and Central American to have a precore mutant strain and low levels of viremia.

15 6 T h e B r i g h a m I n t e n s i v e R e v i e w o f I n t e r n a l M e d i c i n e Q u e s t i o n a n d A n s w e r C o m pa n i o n
Antiviral therapy is not indicated for HBeAg-negative varices. Nadolol is not routinely started in the acute setting
patients with normal liver enzymes and hepatitis B viral of alcoholic hepatitis.
load <2,000 IU/mL, as in this case. This patient appears
to have inactive disease and no evidence of chronic liver
disease.
All newborns born to mothers who are carriers should 42. ANSWER:B. Enteral nutrition with a nasojejunal
receive passive-active immunization consisting of HBIG and tube
hepatitis B vaccination, a strategy that has a high protective
efficacy (95%). Tenofovir is the drug of choice for pregnant Acute fluid collections are an early complication of acute
patients requiring antiviral therapy. Pegylated interferon is pancreatitis. They generally do not require any treatment.
not recommended during pregnancy. Lamivudine is associ- The patient has not had any nutrition in 5days and is not
ated with rapid virologic resistance and is considered a preg- expected to be able to tolerate oral intake in the near future;
nancy classC drug (not recommended during pregnancy). thus, enteral nutrition with a nasojejunal feeding tube
Cesarean delivery does not reduce the risk of vertical trans- is recommended at this time. Enteral nutrition in acute
mission of hepatitis B.Mothers should be encouraged to fol- pancreatitis has been shown to have decreased infectious
low up after childbirth to discuss treatment for hepatitis B. complications and disease severity compared to parenteral
nutrition. Antibiotics may be indicated if there is concern
for infected pancreatic necrosis, but there is no evidence of
this on CT scan imaging. Pancreatic debridement is indi-
40. ANSWER:D. Increase prednisone to 40 mg daily cated for management of infected necrosis. Interventional
and add azathioprine. radiology drainage of fluid collections might be beneficial if
there is a dominant collection that is causing gastric outlet
Mild-to-moderate ulcerative colitis is often initially treated or intestinal obstruction, or if there is concern for a super-
with 5-aminosalicylates (5-ASA) for induction and then infected fluid collectionbut neither of these is present at
maintenance of remission. Short courses of steroids are this time.
sometimes needed to induce remission or manage flares,
but they are not recommended for maintenance therapy
due to the side effects. Immunomodulator therapy such as
6-mercaptopruine (as in this case) or azathioprine, or an 43. ANSWER:A. Sacroiliitis
antitumor necrosis factor (TNF) biologic therapy such as
infliximab should be considered for patients whose disease Autoimmune pancreatitis is characterized by elevated serum
is not effectively controlled with 5-ASA therapy. Switching IgG4 levels, characteristic imaging (most commonly a dif-
to another 5-ASA agent at this time is unlikely to improve fusely enlarged pancreas, although focal lesions and pancre-
her symptoms. A slow-release form of budesonide has atic duct strictures can also occur), and diagnostic histology
recently been approved for ulcerative colitis, but it is used in (which often includes a lymphplasmacytic infliltrate with
a similar way to prednisone in this case to attempt to induce IgG4-positive cells). IgG4-related disease can also involve
remission and not as a maintenance or steroid-sparing the biliary tract and salivary glands, and it may cause lung
agent. Antibiotics have not been shown to be effective in nodules, autoimmune thyroiditis, and interstitial nephritis.
the treatment of ulcerative colitis. Sacroiliitis is not associated with autoimmune pancreatitis
and IgG4-related disease, although it can be seen with other
autoimmune diseases such as inflammatory bowel disease.

41. ANSWER:B. Supportive care

This patient has alcoholic hepatitis based on her history 44. ANSWER:D. Urinary 5-hydroxyindoleacetic acid
and consistent laboratory findings and should be advised
to abstain from alcohol use. Her paracentesis is not sug- This patient most likely has carcinoid syndrome based
gestive of spontaneous bacterial peritonitis, so she should on the secretory diarrhea, flushing, and evidence of heart
not receive cefotaxime at this time. Her Maddrey discrimi- failure with right-sided valvular lesions. The most useful
nant function is less than 32 based on her total bilirubin initial test for carcinoid syndrome is a 24-hour urine for
and INR, so she should not receive prednisolone or pent- 5-hydroxyindoleacetic acid (5-HIAA), although it is more
oxifylline at this time. Alcoholic hepatitis can cause portal useful for midgut carcinoid tumors compared to foregut and
hypertension, but this will often reverse with cessation of hindgut tumors. Vasoactive intestinal peptide is elevated in
alcohol. She does not have clear evidence of cirrhosis at this VIPomas, which can also cause secretory diarrhea and flush-
time and has not had an upper endoscopy to evaluate for ing, but are not associated with right-sided valvular lesions.

7. D i g e s t i v e D i s e a s e s a n d D i s o r d e r s o f t h e Pa n c r e a s a n d L i v e r 157
Calcitonin is elevated in medullary cancer of the thyroid Steatorrhea due to exocrine pancreatic insufficiency is pos-
and gastrin is elevated in gastrinomas, both of which can sible but typically improves with increased enzyme supple-
cause a secretory diarrhea. A fat pad biopsy with Congo mentation and is not associated with increased folate levels.
red staining is performed to diagnose amyloid, which could Crohns disease can result in similar symptoms, but colonos-
cause diarrhea and heart failure with a low-amplitude QRS copy to the terminal ileum and abdominal imaging is not
on electrocardiogram, but would not be expected to cause suggestive of this. Celiac disease is more common in patients
the tricuspid regurgitation and flushing. with diabetes, but the normal tissue transglutaminase IgA
antibody (and serum IgA level) as well as the normal duo-
denal biopsies make this diagnosis highly unlikely. Irritable
bowel syndrome would not explain the vitamin B12 defi-
45. ANSWER:C. Ceftriaxone ciency, anemia, and nocturnal stools.

This patient has upper gastrointestinal bleeding concerning


for variceal bleeding in light of the history of cirrhosis with
evidence of portal hypertension. Octreotide is the preferred 48. ANSWER:B. Low-oxalate diet and continue the
pharmacologic treatment in the United States. Vasopressin calcium supplementation
can also be used in this setting, but it has more side effects
and would not be administered concurrently with octreo- Patients with small intestinal Crohns disease, especially
tide. Short-term (maximum 7days) spontaneous bacterial those with small bowel resections, are predisposed to neph-
peritonitits (SBP) antibiotic prophylaxis should be given in rolithiasis, particularly calcium oxalate stones. As a result
any patient with cirrhosis and gastrointestinal hemorrhage, of bile salt malabsorption, fat binds to calcium in the intes-
especially those that have ascites. Intravenous ceftriaxone tines, leaving oxalate free to be absorbed and deposited
or ciprofloxacin is the preferred initial antibiotic prophy- in the kidney, where it can form into stones. Spinach is a
laxis. Penicillin is not recommended for SBP prophylaxis. high oxalate-containing food, and eating large amounts
Intravenous albumin is not generally given in the setting of this may have contributed to stone formation. In addi-
of gastrointestinal or variceal bleeding. N-acetylcysteine tion to maintaining adequate hydration, the treatment for
is used in acute liver failure due to acetaminophen toxicity calcium oxalate stones is generally to eat a low-oxalate diet
but not for gastrointestinal or variceal bleeding. and ensure adequate calcium intake. Calcium intake should
not be reduced (unless it is excessive), as a decrease in free
calcium in the intestines can lead to increased absorption
of oxalate due to decreased binding of oxalate by calcium.
46. ANSWER:C. Wireless capsule endoscopy Potassium citrate is sometimes used to treat calcium stones
that are thought to form due to hypocitraturia, which is not
This patient has obscure, occult gastrointestinal bleed- thought to be a common mechanism for nephrolithiasis
ing, given the negative upper endoscopy, colonoscopy, and formation in Crohns disease.
small bowel imaging with CT enterography. Asmall bowel
source, such as angiodysplasia, is the most likely cause. The
next test would therefore be a wireless capsule endoscopy.
CT angiography and tagged red blood cell scan are not 49. ANSWER:E. Endoscopic retrograde
sensitive for detection of occult gastrointestinal bleeding. cholangiopancreatography
Double-balloon and push enteroscopy are generally per-
formed for further diagnosis and treatment of a small bowel This patient has acute pancreatitis most likely due to gall-
bleeding source identified on capsule endoscopy or another stones based on the elevated liver enzymes as well as the
study. gallstones and dilated common bile duct seen on the ultra-
sound. The ultrasound did not identify gallstones in the
common bile duct, but it has relatively poor sensitivity for
detecting choledocholithiasis. The persistent pain, elevated
47. ANSWER:D. Small intestinal bacterial bilirubin, and fever suggest that she has cholangitis and
overgrowth persistent biliary obstruction. The most appropriate treat-
ment is endoscopic retrograde cholangiopancreatography
Small intestinal bacterial overgrowth (SIBO) typically causes (ERCP), which will provide both diagnostic and therapeu-
loose stool and bloating, and can also result in v itamin B12 tic benefit. It has been shown to decrease biliary sepsis if
deficiency (bacteria consume vitamin B12) and elevated performed within 72 hours.
serum folate (bacteria synthesize folate). Chronic pancre- Abdominal CT scan and magnetic resonance cholangi-
atitis and diabetes can predispose to SIBO due to stasis. opancreatography (MRCP) would show acute pancreatitis

15 8 T h e B r i g h a m I n t e n s i v e R e v i e w o f I n t e r n a l M e d i c i n e Q u e s t i o n a n d A n s w e r C o m pa n i o n
(and may still be too early to show pancreatic necrosis) following: improvement with defecation, onset associated
and may detect choledocholithiasis but offers no therapy. with a change in frequency of stool, and onset associated
An elective cholecystectomy should be considered in this with a change in form (appearance) of stool. She likely
patient who has a history of gallstone-induced complication. has constipaton-predominant IBS. Colonic inertia (or
Ursodiol can dissolve small gallbladder stones, but is not a slow-transit constipation) and dyssynergic defecation (or
suitable therapeutic option for treating choledocholithiasis. pelvic floor dysfunction) are not typically associated with
the abdominal pain symptoms, and they are typically asso-
ciated with retention of radiopaque markers in the right side
of the colon or rectum, respectively. Anorectal manometry
50. ANSWER:C. Constipation-predominant irritable can be performed to further differentiate between these
bowel syndrome two entities. Multiple sclerosis can cause chronic constipa-
tion, but she does not appear to have any other symptoms of
This patient meets the Rome III criteria for constipa- this. Chronic intestinal pseudo-obstruction typically causes
tion predominant irritable bowel syndrome (IBS), which obstructive symptoms (nausea, vomiting) and is associated
is recurrent abdominally at least 3 days per month in with underlying neuropathic or myopathic diseases, which
the last 3 months associated with two or more of the she does not appear to have.

7. D i g e s t i v e D i s e a s e s a n d D i s o r d e r s o f t h e Pa n c r e a s a n d L i v e r 159
8.
CAR DIOVASCULAR DISEASE

Thomas S. Metkus Jr., Patrick OGara, and Donna M. Polk

1. A32-year-old woman presents for evaluation of pal- 3. A52-year-old gentleman with hypertension, dyslip-
pitations. She notes the intermittent sensation of her idemia, and a smoking history presents with 6 hours
heart flip-flopping in her chest for the past month. of severe, unrelenting chest burning. On examination,
Review of systems is notable for fatigue and subjectively the blood pressure is 88/66mm Hg. The jugular venous
decreased energy. On physical examination, she is well pressure is 14 cm of water and increases with inspira-
appearing. The jugular venous pressure is 5cm of water. tion. The lungs are clear. Cardiac auscultation reveals
There is a parasternal lift and a grade 2 of 6 mid peak- normal first and second heart sounds without murmur.
ing systolic murmur at the left upper sternal border. The An electrocardiogram is shown in Figure 8.2.
second heart sound is widely split without respiratory In addition to administering aspirin and activating
variation, and pulmonic component of the second heart the cardiac catheterization laboratory, the most appro-
sound is prominent. The extremities are warm without priate management is:
edema. An electrocardiogram demonstrates a right-
ward axis, narrow QRS complex, and incomplete right A. Infusion of normal saline
bundle branch block. B. Nitroglycerine infusion
What is the most likely diagnosis? C. Furosemide bolus
D. Neosynephrine infusion
A. Wolff-Parkinson-White syndrome E. Esmolol infusion
B. Arrhythmogenic right ventricular cardiomyopathy
C. Atrial septal defect 4. A 36-year-old woman who is 22 weeks pregnant
D. Mitral stenosis presents with dyspnea and palpitations. Her dyspnea
E. Hyperthyroidism began at 16 weeks of pregnancy but has progressed
to the point where she is dyspneic at rest. She sleeps
2. A 42-year-old corporate executive presents to the upright in a chair. On examination, the heart rate is
emergency department with 2 hours of unrelenting 108 beats per minute with an irregularly irregular
chest pain. He has a salient medical history of tobacco rhythm. The blood pressure is 90/64mm Hg. The first
use and dyslipidemia. The pain is severe and exacer- heart sound is louder than the second heart sound at
bated by movement. On physical examination, the the base. There is a high-pitched, snapping, discrete,
patient appears uncomfortable. The cardiopulmonary early diastolic sound that follows the second heart
examination is normal. A bedside echocardiogram sound. There is a diastolic rumbling murmur heard
reveals normal left ventricular function, a trace pericar- over the apex with the patient positioned in the left
dial effusion, and no wall motion abnormality. An elec- lateral decubitus position. Pulmonary auscultation
trocardiogram is shown in Figure 8.1. reveals bibasilar wet rales. The jugular venous pressure
What is the most appropriate next step in management? is 16cm of water.
What is the most likely diagnosis?
A. Aspirin and thrombolysis
B. Aspirin, heparin, and clopidogrel A. Pregnancy-induced cardiomyopathy
C. Prednisone B. Pulmonary embolism
D. Ibuprofen C. Dyspnea secondary to normal progression of
E. Ativan and nitroglycerine pregnancy

16 0
Figure 8.1 Electrocardiogram for patient in Question 2.

Figure 8.2 Electrocardiogram for patient in Question 3.

D. Thyroid storm respiration. The blood pressure is 88/62mm Hg. The jug-
E. Mitral stenosis ular venous pressure is 15cm of water. The heart sounds
are quiet without murmur or gallop. The lungs are clear.
5. A 64-year-old woman presents with increasing dys- The strength is 5 out of 5 in both upper and lower extrem-
pnea and fatigue. She has a medical history notable for ities, and an electrocardiogram reveals sinus tachycardia
small cell lung cancer metastatic to liver and bone and with low voltage in the limb leads and precordial leads.
is currently undergoing chemotherapy. Her symptoms Chest radiograph reveals a mass in the left upper lobe,
have been progressive over the past week; initially she was which is increased in size compared to 2 weeks prior.
dyspneic climbing stairs, and now she is dyspneic walk- What is the most appropriate next diagnostic test?
ing across the room. On examination, the heart rate is
128 beats per minute with a regular rhythm. The inten- A. Electromyography
sity and amplitude of the radial pulse are variable with B. Echocardiography

8 . C a r d i ova s c u l a r D i s e a s e 161
C. CT pulmonary angiography he has noted the abrupt onset of a racing heartbeat and
D. Blood cultures a sensation of pounding in the neck. Three of the epi-
E. Whole body FDG-PET CT scan sodes terminated after 1 minute, but the most recent
episode lasted for 15 minutes before terminating.
6. A 67-year-old gentleman presents for preoperative The physical examination reveals a thin, tall young
evaluation. He has a history of progressive osteoarthri- man who is not distressed. The palate is slightly high
tis of the right knee, which has been refractory to medi- arched. The arm span is normal. The first and second
cal treatment, and total knee replacement is planned. heart sounds are normal; there is a grade 1 out of 6
The remainder of the patients medical history includes systolic murmur at the left upper sternal border that
type 2 diabetes, maintained on insulin therapy; dia- diminishes with handgrip. The remainder of the exam-
betic nephropathy with baseline creatinine of 2.5; ination is normal. The electrocardiogram is shown in
hypertension; dyslipidemia; and ST segment elevation Figure 8.3.
myocardial infarction 10 years prior, treated success- What is the best next step?
fully with thrombolytics. Knee pain limits his ability to
ambulate, and he walks with a limp. He has no angina or A. Electrophysiology study and mapping
symptoms of heart failure. Medications include aspirin, B. Signal averaged electrocardiogram
metoprolol, simvastatin, NPH insulin, glyburide, and C. Exercise stress test
lisinopril. Physical examination includes heart rate of D. Holter monitor
88 beats per minute and blood pressure of 126/86mm E. CT coronary angiogram
Hg and is otherwise normal.
What is the most appropriate recommendation? 8. A 52-year-old woman is referred to you by her den-
tist. She presented with multiple dental caries and is in
A. Pharmacologic stress test prior to total knee need of several fillings. Her medical history is notable
replacement for hypertension and mitral valve prolapse. Other than
B. Increase dosage of metoprolol for a goal heart rate of tooth pain, she feels well. On examination, there is a
5565, then proceed with total knee replacement. mid systolic click and late systolic murmur heard over
C. Echocardiogram prior to total knee replacement the apex.
D. Recommend against total knee replacement due to What is the most appropriate next step?
medical comorbidities.
E. Coronary angiography prior to total knee A. Clindamycin
replacement B. Amoxicillin
C. Repeat echocardiogram
7. A22-year-old collegiate cross country runner pres- D. Proceed with planned dental procedure.
ents with palpitations. Four times over the past month E. Blood cultures

Figure 8.3 Electrocardiogram for patient in Question 7.

162 T h e B r i g h a m I n t e n s i v e R e v i e w o f I n t e r n a l M e d i c i n e Q u e s t i o n a n d A n s w e r C o m pa n i o n
9. A68-year-old man presents for evaluation of chest first diagonal branch of the left anterior descending are
pain. He has a history of hypertension, diabetes, dys- demonstrated. Medical management in lieu of revas-
lipidemia, and cigarette smoking. Within the past cularization is advised. In addition to aspirin, beta
24 hours, he had two severe episodes of substernal blocker, statin therapy, and smoking cessation, you
chest pain at rest. On examination, a fourth heart prescribe:
sound is noted without murmurs. The extremities are
warm without edema. The jugular venous pressure is A. Fish oil
5 cm of water. The electrocardiogram reveals sinus B. Ranolazine
rhythm with T wave inversions and 1mm of ST seg- C. Isosorbide mononitrate
ment depression in leads V5 and V6. He is pain-free D. Clopidogrel
and comfortable at the time of the evaluation. E. Warfarin
Troponin-T measurement is 0.3 ng/mL (normal range
00.1 ng/mL). 12. A 63-year-old long-term patient of yours presents
In addition to aspirin and clopidogrel, what is the most with worsening breathlessness and palpitations. He
appropriate next step in management? has a known history of mitral valve prolapse diagnosed
years prior on the basis of auscultatory findings of a
A. Tenecteplase and unfractionated heparin click-murmur complex and confirmed on echocardio-
B. Enoxaparin, nitrates, and coronary angiography in gram. For the past 2 weeks he describes progressive
72 hours breathlessness, orthopnea, and palpitations. On exami-
C. Unfractionated heparin, nitrates, coronary nation, he appears uncomfortable and is dyspneic with
angiography in 72 hours conversation. The heart rate is approximately 100 beats
D. Fondaparinux, and coronary angiography within 24 per minute and irregular. The jugular venous pressure is
hours 16cm of water. There is a grade 4 of 6 pansystolic mur-
E. Unfractionated heparin, and coronary angiography mur with a thrill appreciated over the apex. The mur-
within 24 hours mur is blowing in quality and is heard throughout the
precordium with particular radiation toward the base
10. A 36-year-old woman presents for evaluation of a of the heart. The apex beat is hyperdynamic. An elec-
murmur. She feels well, without symptoms of dyspnea, trocardiogram demonstrates atrial flutter with variable
chest pain, or palpitations. Review of systems is oth- A-V conduction.
erwise negative. On examination, she is well appear- In addition to anticoagulation, what is the next step
ing and slightly anxious. The oropharynx is normal in diagnosis and management?
and jugular venous pressure is low. S1 is normal, and
S2 splits with inspiration. The pulmonic aspect of the A. TEE and DC cardioversion
second heart sound is softer than the aortic component. B. Beta blockade
There is no gallop, and there is a grade 2 out of 6 systolic C. Transthoracic echocardiogram
murmur heard best at the left upper sternal border. The D. Exercise treadmill test
murmur peaks in mid systole and S2 is heard clearly. E. Thyroid function testing and nocturnal
The intensity of the murmur diminishes with handgrip polysomnogram
and does not change appreciably with squat to stand.
Her electrocardiogram is normal. 13. A42-year-old woman with type 1 diabetes mellitus
What is the most appropriate next step? presents for a new patient visit. She feels overall well. On
examination, the blood pressure is 150/88. Funduscopy
A. Chest radiograph reveals evidence of micro-hemorrhages. The first and
B. Exercise treadmill test second heart sounds are normal, and a fourth heart
C. Transthoracic echocardiogram sound is appreciated. There are no abdominal bruits.
D. Carotid ultrasound Urinalysis is notable for 2+ protein. The hemoglobin
E. Reassurance A1C is 9.2%. Her medications include NPH and aspart
insulin. In addition to uptitration of her insulin regi-
11. A67-year-old man with a history of cigarette smok- men, you prescribe:
ing presents with accelerating anginal chest pain. On
initial evaluation, troponin-T is elevated to 2.0 (00.1), A. Fish oil
and the electrocardiogram demonstrates T wave inver- B. Lisinopril
sions in leads I and aVL. He is referred for coronary C. Chlorthalidone
angiography whereupon a 60% stenosis in the mid D. Amlodipine
right coronary artery and an 80% stenosis in a small E. Metformin

8 . C a r d i ova s c u l a r D i s e a s e 163
Figure 8.4 Electrocardiogram for patient in Question 14.

14. A 32-year-old woman presents with 35 minutes of C. Amiodarone


palpitations. Symptoms began abruptly. She has slight D. Labetalol
lightheadedness but no syncope, dyspnea, or chest pain. E. Normal saline
Physical examination reveals a tachycardic, regular heart
rhythm, blood pressure of 132/82mm Hg, and is other- 15. A45-year-old gentleman with a heavy smoking history
wise normal. An electrocardiogram is shown in Figure 8.4. presents by ambulance with 3 hours of crushing substernal
Valsalvas maneuver and carotid sinus pressure fail to chest pain. On examination, he appears uncomfortable.
terminate the arrhythmia. The next best step in man- The jugular venous pressure is 10cm of water. The first and
agement is: second heart sounds are normal. There are scant bibasilar
rales. The electrocardiogram is shown in Figure 8.5.
A. DC cardioversion A cardiac catheterization lab is 2.5 hours away,
B. Adenosine accounting for transfer time.

Figure 8.5 Electrocardiogram for patient in Question 15.

16 4 T h e B r i g h a m I n t e n s i v e R e v i e w o f I n t e r n a l M e d i c i n e Q u e s t i o n a n d A n s w e r C o m pa n i o n
In addition to aspirin, clopidogrel, morphine, oxy- An electrocardiogram demonstrates left bundle branch
gen, and nitrate therapy, what is the most appropriate block with QRS duration of 155 ms. Serum creatinine is
next step in management? 1.4 mg/dL, which is the baseline level.
What is the best next step in management?
A. Transfer for primary percutaneous coronary
angiography (PCI). A. Increase furosemide dose.
B. Administer tenecteplase and transfer to a B. Add metolazone.
PCI-capable facility. C. Refer for cardiac resynchronization therapy
C. Prescibe colchicine and aspirin. (biventricular pacemaker placement).
D. Administer half-dose thrombolytics and transfer to D. Evaluate for placement of a left ventricular assist
a PCI-capable facility. device.
E. Administer intravenous metoprolol boluses for goal E. Add hydralazine.
heart rate of 55.
18. A 34-year-old woman presents for follow-up. She
16. A58-year-old woman with hypertension and diabe- is in week 32 of her first pregnancy. She feels overall
tes presents for follow-up. She was recently hospitalized well and has been walking 20 minutes daily for exer-
with dyspnea, elevated neck veins, and pulmonary rales. cise. She has suffered several episodes of nocturnal
Brain naturietic peptide levels were markedly elevated heartburn. On review of systems, she notes bilateral
on admission. She responded well to diuresis with intra- lower-extremity edema most pronounced at the end of
venous furosemide and was discharged. An echocardio- her workday and improved by elevating the legs. On
gram in the hospital revealed moderate left ventricular examination, the blood pressure is 96/52 mm Hg and
hypertrophy and a left ventricular ejection fraction of heart rate is 88 beats per minute; the uterus is gravid.
65% with no significant valvular disease. Since her dis- The jugular venous pressure is 5cm of water. Avenous
charge, she feels well. On examination, the blood pres- hum is appreciated over the right clavicular fossa. S1 is
sure is 162/94 and heart rate is 90. The body-mass index normal and S2 splits with inspiration. There is a grade 2
is 32.3. The jugular venous pressure is 10cm of water. of 6 mid peaking systolic murmur at the left upper ster-
The lungs are clear. S1 and S2 are normal, and a fourth nal border. Athird heart sound is appreciated. There is
heart sound is appreciated. Her medications include 1+ bilateral pitting edema at the ankle.
lisinopril, amlodipine, metformin, and NPH insulin. What is the most appropriate next step?
The next step in management would include:
A. Urinalysis and liver function testing
A. Refer for implantable cardioverter defibrillator B. Echocardiogram
placement. C. Routine follow-up in 2 weeks
B. Check repeat BNP level and prescribe furosemide if D. Lower-extremity venous ultrasound
persistently elevated. E. Holter monitor
C. Prescribe nitrates.
D. Increase antihypertensive therapy. 19. A48-year-old gentleman is seen prior to discharge after
E. Refer to an advanced heart failure specialist. being admitted with myocardial infarction. He presented
with a large anterolateral ST segment elevation myocardial
17. A 48-year-old Caucasian woman presents for infarction complicated by heart failure. An occlusion of the
follow-up after a recent hospitalization for heart fail- proximal left anterior descending artery was treated with
ure. She has a salient history of nonischemic cardio- PCI. At present, he feels well. Physical examination reveals
myopathy with ejection fraction of 25%. She presented euvolemia, a third heart sound, and warm extremities. The
with volume overload and dyspnea, which responded renal function is normal. Electrocardiogram reveals ante-
well to diuresis. Since discharge, however, she remains rior Q waves. An echocardiogram demonstrates a left ven-
dyspneic, climbing less than one flight of stairs. Her tricular ejection fraction of 35%. The medications include
medications include lisinopril 40 mg daily, carvedilol metoprolol, lisinopril, furosemide, atorvastatin, aspirin
25 mg twice daily, spironolactone 25 mg daily, furose- 81 mg, and clopidogrel 75 mg.
mide 40 mg twice daily, and potassium supplementa- What is the most optimal next step in management?
tion. On examination, she appears fatigued. Heart rate
is 68beats per minute and blood pressure is 100/60mm A. Increase aspirin from 81 to 325 mg daily.
Hg. The jugular venous pressure is 6cm of water. The B. Increase clopidogrel to 150 mg twice daily.
first and second heart sounds are normal, and a third C. Ranolazine
heart sound is appreciated. There is scant bilateral pit- D. Eplerenone
ting edema of the lower extremities. The lungs are clear. E. Metolazone

8 . C a r d i ova s c u l a r D i s e a s e 16 5
20. An 82-year-old gentleman is referred for preopera- water. There is a tricomponent friction rub heard at the
tive risk stratification prior to planned total hip replace- left lower sternal border. An electrocardiogram reveals
ment for severely painful, limiting osteoarthritis. His diffuse ST segment elevation, which is concave upwards
past medical history includes hypertension. On review in morphology. An echocardiogram reveals a trace peri-
of systems, he suffered an episode of syncope 3months cardial effusion and normal left ventricular function
prior for which he did not seek care, attributing the without wall motion abnormalities.
event to dehydration secondary to the warm summer What is the most optimal management?
weather. On examination, he appears well. The carotid
upstrokes are normal in contour and volume. The first A. Aspirin and prednisone
and second heart sounds are normal. There is a grade B. Prednisone
2 of 6 midpeaking systolic murmur at the left upper C. Prednisone and ibuprofen
sternal border and a grade 1 of 6 pansystolic murmur D. Aspirin
at the apex. A single pause is heard during the period E. Ibuprofen and colchicine
of auscultation. An electrocardiogram demonstrates
sinus rhythm at a rate of 75 beats per minute, left bundle 23. A16-year-old young man is evaluated for a prepar-
branch block, and Mobitz type II AV block. ticipation sports physical for swimming. He feels well.
What is the next step in management? His family history is negative for sudden cardiac death;
an uncle has aortic stenosis. On examination, he is tall
A. Initiate metoprolol and uptitrate for a goal heart rate and thin. The first heart sound is normal. There is an
of 5565 perioperatively. early systolic click and a grade 2 out of 6 mid peaking
B. Proceed with surgery without further diagnostic systolic murmur. There is a grade 1 out of 6 diastolic
testing. murmur at the left upper sternal border. There is a harsh
C. Echocardiogram systolic bruit heard best over the posterior chest. There
D. Prescribe 4 L of fluid intake daily preoperatively. is slight radio-femoral pulse delay. Blood pressure is
E. Refer for pacemaker placement prior to surgery. 128/82 mm Hg in the right arm. An echocardiogram
reveals a bicuspid aortic valve with no stenosis and mild
21. A 76-year-old woman seeks care for palpitations. regurgitation, normal aortic root diameter, normal left
She notes sustained palpitations with exertion for the ventricular chamber size, and normal left ventricular
past 2 weeks. Her medical history includes obstruc- function.
tive sleep apnea, hypertension, type 2 diabetes, and What is the next best step?
chronic renal insufficiency with baseline creatinine of
2.6. She underwent carotid endarterectomy 8years ago A. Refer for aortic valve replacement.
for symptomatic L carotid stenosis. On examination, B. Proceed with sports participation with serial
the heart rate is 118 beats per minute and the rhythm is echocardiographic monitoring.
irregularly irregular. There is a soft left carotid bruit and C. Aspirin 81 mg
a well-healed surgical scar over the left neck. The first D. Magnetic resonance angiography of the thoracic and
heart sound has variable intensity, and the second heart abdominal aorta
sound splits with inspiration. There are no murmurs or E. Losartan
gallops. There is trace pitting edema at both ankles. An
electrocardiogram confirms atrial fibrillation. 24. A24-year-old woman is evaluated for hypertension.
Which of the following strategies would best prevent She presented initially to her primary care physician
future stroke? with headaches. Her family history includes both par-
ents and an older sibling with hypertension controlled
A. Aspirin 325 mg with drug therapy. On examination, she is thin and well
B. Warfarin dosed for INR 23 indefinitely appearing. Blood pressure is 180/122mm Hg and sym-
C. Dabigatran 150 mg twice daily metric in both arms and a leg. The first and second heart
D. Aspirin 325 mg and clopidogrel 75 mg sounds are normal, and a fourth heart sound is appre-
E. DC cardioversion followed by warfarin for 1month ciated. An abdominal bruit is appreciated over the left
upper quadrant. Measurements of serum creatinine and
22. A35-year-old woman presents with 2days of severe thyroid-stimulating hormone are normal. The serum
chest pain. She had an upper respiratory infection 1 potassium is 3.2 mEq/L.
week ago. Pain is worsened with deep inspiration and What is the next best next step in diagnosis?
relieved by leaning forward. Review of systems is nota-
ble for cough. On examination, she appears uncomfort- A. Renal vein renin levels
able. The jugular venous pressure is less than 5 cm of B. Serum toxicology screen

16 6 T h e B r i g h a m I n t e n s i v e R e v i e w o f I n t e r n a l M e d i c i n e Q u e s t i o n a n d A n s w e r C o m pa n i o n
C. 24-hour urinary free cortisol What is the next step in management?
D. Renal artery Doppler studies
E. Serum renin and aldosterone levels A. Proceed with surgery, maintaining the current
medication regimen.
25. A62-year-old gentleman with diabetes and tobacco B. Discontinue clopidogrel and proceed with surgery,
use presents with 6 hours of stuttering substernal chest continuing aspirin through the perioperative period.
pressure. On initial evaluation, there are ST segment C. Discontinue aspirin and clopidogrel prior to
depressions in leads V5 and V6 of the electrocardiogram proceeding with surgery.
with a troponin-T of 1.0 (00.1) ng/mL. He is referred D. Discontinue clopidogrel and atorvastatin prior to
for coronary angiography and a 95% stenosis of the first proceeding with surgery.
obtuse marginal coronary artery is treated with percu- E. Perform an exercise treadmill test prior to issuing a
taneous coronary intervention. Post procedure, he feels recommendation regarding the proposed surgery.
well, and chest pain has abated. He is prescribed meto-
prolol 25 mg, fluvastatin 10 mg daily, aspirin 81 mg 27. A 32-year-old woman presents with what she
daily, and prasugrel 10 mg. describes as spells of anxiety. Symptoms occur primar-
Which of the following would you recommend? ily when she finds herself in large crowds. Spells last
for seconds to hours and abate spontaneously. She oth-
A. Change prasugrel to clopidogrel. erwise feels well, without syncope, chest pain, or dys-
B. Change fluvastatin to high-dose atorvastatin. pnea. There are no fevers, rash, or arthralgias. The past
C. Increase aspirin 81 mg to 325 mg. medical history is unremarkable. Examination reveals a
D. Add ranolazine. regularly irregular heart rhythm at a rate of 60 beats per
E. Add isosorbide mononitrate. minute. Heart rate with exercise increases to 140 beats
per minute and is regular. An electrocardiogram at rest
26. A64-year-old gentleman is seen in consultation for is shown in Figure 8.6.
preoperative risk assessment prior to cholecystectomy What is the next step in management?
for biliary colic. He has a salient medical history of cor-
onary artery disease presenting with NSTEMI 3years A. Doxycycline
prior. At the time, he was treated with placement of a B. Refer for pacemaker placement.
drug-eluting stent to a 90% stenosis of the right coro- C. Refer for cognitive-behavioral therapy.
nary artery. Since then, he has felt well without angina, D. Refer for invasive EP testing.
dyspnea, or syncope. He plays singles tennis twice per E. Implant a dual-chamber pacemaker.
week in a competitive league. Medications include
metoprolol, aspirin, clopidogrel, and atorvastatin. 28. A72-year-old gentleman with a history of peripheral
Physical examination is normal. arterial disease requiring femoral-popliteal bypass and

Figure 8.6 Electrocardiogram for patient in Question 27.

8 . C a r d i ova s c u l a r D i s e a s e 167
a long-standing smoking history presents with subster- 25%. Coronary angiography was normal, and no other
nal chest pain. Pain is evoked by walking four blocks reversible cause of cardiomyopathy was discovered.
briskly and abates readily with several minutes of rest. Since then, he has been maintained on furosemide, lisin-
He has never had pain with activities of self-care or at opril, carvedilol, spironolactone, and multivitamin. He
rest. Coronary angiogram revealed a 60% stenosis in has one hospitalization for volume overload attributed
the mid right coronary artery (FFR=0.92), a 70% ste- to dietary indiscretion. He feels winded shoveling snow
nosis in a small first diagonal branch of the left anterior and working out on his exercise bicycle but otherwise
descending artery, and a 40% stenosis in the mid left cir- has no current symptoms of heart failure. On exami-
cumflex artery. Left ventriculography revealed normal nation, the blood pressure is 110/60mm Hg and heart
left ventricular function. Medications include meto- rate is 70 beats per minute. The jugular venous pressure
prolol, aspirin, and atorvastatin. On examination, the is 5cm of water. The first and second heart sounds are
heart rate is 60 beats per minute at rest, increasing to 80 normal and a third heart sound is noted. An electrocar-
with ambulation. Blood pressure is 132/82mm Hg. The diogram reveals sinus rhythm with a single premature
cardiac and pulmonary examinations are normal. ventricular beat. QRS duration is 100 ms.
What is the next step in management? What is the best next step in management?

A. APCI with stent placement of the first diagonal A. Refer for cardiac resynchronization therapy (CRT).
branch B. Add digoxin.
B. Increase dosage of metoprolol. C. Refer for implantable cardioverter defibrillator.
C. Prescribe isosorbide mononitrate. D. Add metolazone.
D. Prescribe ranolazine. E. Refer for cardiac transplantation evaluation.
E. Prescribe clopidogrel.
31. A48-year-old man presents with dyspnea and dizzi-
29. A72-year-old gentleman recently underwent coro- ness. He notes several months of progressive dyspnea,
nary artery bypass surgery and is seen in follow-up. He which has limited his ability to perform yardwork. In
presented 4 weeks prior with non-ST segment myocar- addition, he notes pronounced dizziness and presyn-
dial infarction and was found to have three-vessel coro- cope when moving from squatting to standing position
nary disease with greater than 80% stenoses in the right as he weeds his garden. His past medical history includes
coronary, left circumflex, and proximal left anterior hypertension. His medications include amlodipine. On
descending arteries. He had complete revascularization examination, he appears well. The jugular venous pres-
with placement of three bypass grafts. Postoperatively, sure is 8cm of water. The first heart sound is normal, and
he did well and was discharged to a brief inpatient reha- the second heart sound splits with inspiration. Afourth
bilitation stay. Currently at home, he is asymptomatic heart sound is present. There is a grade 1 of 6 pansys-
with activities of daily living and can climb two flights tolic murmur at the apex radiating to the back. There is
of stairs without difficulty. He is participating in a car- a grade 2 of 6 harsh, late-peaking diamond-shaped sys-
diac rehabilitation program and progressing appropri- tolic murmur at the base with radiation to the clavicles
ately. He asks when he can resume sexual activity. and carotid arteries. The murmur increases in intensity
You recommend: with Valsalva and with squat-to-stand maneuvers. The
lungs are clear. Extremities are warm and well perfused.
A. Resume sexual activity now. What is the most likely diagnosis?
B. Resume sexual activity in 1month after follow-up
with cardiac surgery. A. Dilated cardiomyopathy
C. Do not resume sexual activity. B. Mitral stenosis
D. Exercise stress echocardiogram prior to resumption C. Aortic stenosis
of sexual activity. D. Hypertrophic obstructive cardiomyopathy
E. Ankle-brachial index with pulse-volume recording E. Atrial septal defect
prior to resumption of sexual activity
32. A32-year-old woman presents with fever. She has a
30. A 42-year-old gentleman with known cardiomy- salient medical history of intravenous drug abuse. On
opathy presents for follow-up. His family history is examination, the temperature is 102F. There is a grade
notable for dilated cardiomyopathy in his father and 2 of 6 diastolic blowing murmur at the base. There are
one brother. Apaternal uncle died suddenly at a young splinter hemorrhages in the nails. Blood cultures grow
age of unknown cause. He presented 1year prior with methicillin-sensitive S. aureus. Electrocardiogram
dyspnea on exertion; transthoracic echocardiogram reveals normal sinus rhythm. Transthoracic echocar-
revealed a dilated left ventricle with ejection fraction of diogram reveals moderate aortic regurgitation and an

16 8 T h e B r i g h a m I n t e n s i v e R e v i e w o f I n t e r n a l M e d i c i n e Q u e s t i o n a n d A n s w e r C o m pa n i o n
8mm vegetation on the aortic valve. She is hospitalized She feels like she is slightly more limited in her activi-
for evaluation and treatment and initiated on nafcillin. ties of daily living as compared to 1year ago; she feels
On hospital day 3, fevers persist. Electrocardiogram dyspneic playing with her 2-year-old grandchild. She has
demonstrates a prolonged PR interval with periods of no chest pain, lower-extremity edema, or paroxysmal
Mobitz II AV block. nocturnal dyspnea. On examination, the blood pressure
What is the next step in diagnosis? is 100/40mm Hg and heart rate is 70 beats per minute.
The carotid impulses are brisk with a rapid rise. The first
A. Electrophysiology testing heart sound is normal, and the second heart sound splits
B. Brain MRI with inspiration. There is an early systolic click and a
C. Transesophageal echocardiogram grade 2 out of 6 mid peaking systolic murmur. There is
D. Request blood cultures to be incubated for 14days. a grade 2 out of 4 early diastolic murmur with a blowing
E. Serologies for Coxiella burnetii quality. The apex beat is laterally displaced. An echocar-
diogram demonstrates a left ventricular ejection fraction
33. A65-year-old woman presents with chest and arm of 70%. The left ventricle is dilated with an end-diastolic
pain. Pain is substernal with radiation to the right diameter of 69mm. The aortic valve is bicuspid with a
arm and is evoked by exertion but not predictably wide jet of aortic insufficiency, which is graded as severe.
relieved with rest. She has a history of severe lum- There is systolic flow reversal noted in the descending
bar disc disease and osteoarthritis of the knee, which thoracic aorta. The aortic root diameter is 30mm.
limit her functional capacity. In addition, she has What is the next step in management?
hypertension and depression. Examination reveals a
well-appearing, slightly overweight woman. The first A. Prescribe losartan
and second heart sounds are normal and the lungs are B. Aortic valve replacement
clear. Electrocardiogram demonstrates a left bundle C. Echocardiogram in 6months
branch block, which was noted on prior electrocardio- D. Exercise treadmill testing
grams obtained before treatment with antidepressant E. Carotid ultrasound
therapy.
What is the best next diagnostic step? 36. A 58-year-old gentleman is seen in follow-up for
management of hypertension. He has been maintained
A. Exercise treadmill test on pharmacotherapy for 8 months with persistently
B. Exercise stress echocardiogram poor blood pressure control. He feels well and is asymp-
C. Dobutamine stress echocardiogram tomatic. The past medical history includes osteoar-
D. Nuclear stress test with regadenoson thritis for which he takes over-the-counter analgesics.
E. Reassurance Medications otherwise include amlodipine, hydro-
chlorothiazide, and lisinopril. On examination, he is
34. A76-year-old gentleman with a significant smoking an anxious-appearing, middle-aged gentleman not dis-
history presents with leg pain. He notes left-sided leg tressed. His body-mass index is 28kg/m2 . Blood pres-
heaviness that occurs when he walks more than three sure in the right arm is 172/102. Afourth heart sound
blocks and resolves predictably with several minutes is appreciated. There are no murmurs and no peripheral
of rest, after which he can resume walking. He has no bruits. There is no lower-extremity edema.
chest pain or dyspnea. On physical examination, there What is the best next step in evaluation?
is a left femoral bruit and diminished left dorsalis pedis
and posterior tibial pulses. Ankle brachial index is 0.72 A. 24-hour urine free cortisol, serum renin, and
on the left and 0.9 on the right. aldosterone levels; thyroid function testing; and
In addition to treating hypertension and dyslipid- serum metanephrines
emia to target and prescribing aspirin, what is the best B. CT angiography of the renal and mesenteric arteries
next step? C. Captopril renal scan
D. Obtain detailed over-the-counter medication use
A. Supervised exercise program history.
B. Pentoxyfylline E. 24-hour Holter monitor
C. Lower-extremity angiography and stenting
D. Peripheral arterial bypass surgery 37. A 66-year-old woman presents with arm pain and
E. Amlodipine abdominal bloating that is evoked by climbing two
flights of stairs. The past medical history includes diabe-
35. A 60-year-old woman with a bicuspid aortic valve tes and hypertension. Medications include simvastatin,
and known aortic regurgitation presents for follow-up. aspirin, and losartan. She is referred for an exercise

8 . C a r d i ova s c u l a r D i s e a s e 16 9
treadmill test, which demonstrates diffuse ST segment 40. A 67-year-old woman presents for follow-up. She
depressions of 23mm at 70% of maximum predicted presented 2 years ago with an ST segment myocardial
heart rate. Coronary angiography demonstrates a 90% infarction and underwent successful stenting of a com-
stenosis of the proximal right coronary artery, a 40% plete right coronary artery occlusion. Since then, she
stenosis of a first obtuse marginal branch of the left cir- has felt well. Medications include metoprolol, atorv-
cumflex artery, and an 80% stenosis of the proximal left astatin, and aspirin. Blood pressure is 110/60mm Hg.
anterior descending artery. Left ventriculogram dem- Physical examination is normal. The last LDL choles-
onstrates normal left ventricular function. terol was 68 mg/dL.
What is the optimal approach to management? What is the next step in management?

A. Add metoprolol and isosorbide mononitrate. A. Exercise treadmill testing with imaging
B. PCI with stent placement of the LAD B. Routine follow-up in 12months
C. PCI with stent placement of the RCA and LAD C. Echocardiogram
D. Refer for coronary artery bypass grafting of the LAD D. hs-CRP measurement
and RCA. E. Measurement of carotid intima media thickness
E. Myocardial viability study (CIMT)

38. A77-year-old gentleman with hypertension and a 60 41. A 74-year-old gentleman with hypertension, coro-
pack/year smoking history presents for evaluation and nary artery disease, GERD, and osteoarthritis presents
routine follow-up. He feels well. Medications include for follow-up. He suffered an ST segment myocar-
lisinopril, aspirin, amlodipine, and atorvastatin. On dial infarction 2years prior and underwent successful
examination, there is a soft bruit over the epigastrium, stenting of a complete LAD occlusion. For the past 3
and the aortic pulsation is prominent. An abdominal weeks, he has noted worsening dyspnea on light exer-
ultrasound reveals an abdominal aortic aneurysm mea- tion coupled with lower-extremity swelling. He has had
suring 4.1cm in transverse diameter. no recurrent chest pain. Medications include metopro-
In addition to smoking cessation counseling, what is lol, nifedipine, aspirin, and rosuvastatin. On examina-
the next step in management? tion, blood pressure is 126/80mm Hg. The heart rate
is 70 beats per minute. The jugular venous pressure is
A. Refer for open surgical repair. 14 cm of water. The first and second heart sounds are
B. Refer for endovascular repair. normal, and a third heart sound is appreciated. There is
C. Urgent CT angiogram of the abdominal aorta. lower-extremity edema to the knee bilaterally. Astress
D. Repeat abdominal ultrasound in 6months. echocardiogram reveals mild anterior wall hypokinesis
E. Exercise treadmill test at rest, and all walls augment appropriately with stress.
The left ventricular ejection fraction at rest is estimated
39. A 48-year-old woman presents with palpitations. at 40%.
She describes the sensation as a flip-flop in the chest, In addition to diuresis and discontinuation of nifedip-
which occurs several times weekly. She is concerned ine, what is the most appropriate management?
because her father had a heart attack at a young age. She
has not had chest pain, syncope, or dyspnea. She is oth- A. Add hydralazine and isosorbide mononitrate.
erwise healthy and takes no medications. She drinks B. Add clopidogrel.
alcohol rarely, has one cup of black tea per week, and C. Add lisinopril.
does not smoke cigarettes. Physical examination is nor- D. Add spironolactone.
mal with the exception of several extrasystoles heard on E. Add digoxin.
cardiac examination. Electrocardiogram reveals nor-
mal sinus rhythm with a single premature ventricular 42. A82-year-old woman is seen for evaluation of a mur-
complex. A 24-hour Holter monitor reveals multiple mur. She has a past history of GERD and osteoarthri-
premature ventricular complexes that correlate with tis. She feels well without cardiovascular symptoms and
symptoms. maintains an active lifestyle gardening and babysitting
What is the next step in management? grandchildren. On examination, the blood pressure
is 130/86 mm Hg and heart rate is 68 beats per min-
A. Exercise treadmill test ute. The jugular venous pressure is 5cm of water. The
B. Reassurance carotid upstrokes are slightly delayed and decreased in
C. Prescribe metoprolol volume. There is a grade 3 of 6 late-peaking murmur
D. Echocardiogram heard at the base, which has a harsh quality and radi-
E. Flecainide to be taken as needed ates to both clavicles. The first heart sound is normal,

170 T h e B r i g h a m I n t e n s i v e R e v i e w o f I n t e r n a l M e d i c i n e Q u e s t i o n a n d A n s w e r C o m pa n i o n
and the second heart sound single. An echocardiogram 45. A26-year-old medical student is seen in the office.
reveals a calcified aortic valve with decreased leaflet In the midst of a workshop with his classmates learning
mobility; the mean aortic valve gradient is calculated at to perform bedside cardiac ultrasound, an abnormal-
42mm Hg; the aortic valve area is estimated at 0.9cm2. ity was appreciated. He feels entirely well, and physi-
What is the next step in management? cal examination is normal. A formal echocardiogram
reveals a left atrial mass measuring 2.1 2.3 cm. The
A. Echocardiography in 1year and as needed with mass is pedunculated, mobile, and attached to the inter-
development of new symptoms atrial septum by a thin stalk. The remainder of the echo-
B. Referral for aortic valve replacement cardiogram is normal.
C. Prescribe high-dose atorvastatin. What is the next step in management?
D. Refer for balloon aortic valvuloplasty.
E. Refer for cardiac catheterization with direct A. Echocardiography in 6months
measurement of aortic and left ventricular B. Two sets of blood cultures 12 hours apart
pressure. C. Anticoagulation with warfarin, for goal INR of 23
D. Referral for surgical removal
43. A48-year-old gentleman presents with chest pres- E. PET-CT of the chest, abdomen, and pelvis
sure. He has a history of diabetes, dyslipidemia, hyper-
tension, and active tobacco use. For the past 3 weeks, 46. An 88-year-old gentleman with atrial fibrillation
he notes substernal chest pressure that is evoked by and dyslipidemia presents for follow-up. He feels well,
climbing two flights of stairs briskly and relieved over without palpitations, dyspnea, or chest pain. He is able
several minutes with rest. His family history includes to exercise and perform activities of daily living with-
two brothers with myocardial infarction at age 50 out difficulty. Medications include verapamil, warfarin,
and 52, respectively. Physical examination is nor- and simvastatin. On examination, the blood pressure is
mal. Electrocardiogram (ECG) reveals normal sinus 124/72mm Hg. The heart rate is 88 beats per minute and
rhythm with no ischemic change and normal axis and irregularly irregular. The first heart sound is variable and
intervals. the second heart sound is normal. Lungs are clear.
What is the best diagnostic step? What is the next step in management?

A. Cardiac MRI A. Transition from warfarin to dabigatran.


B. Coronary artery calcium score B. Increase dosage of verapamil.
C. Exercise treadmill test C. Add amiodarone.
D. Coronary angiography D. Clinical follow-up in 6months
E. Pharmacologic stress myocardial perfusion imaging E. Add digoxin.

44. A78-year-old woman is seen in the office for a pre- 47. An 84-year-old woman presents with fatigue and
operative evaluation. She has a notable history of low dyspnea for 3 months. While previously active, she
back pain secondary to spinal stenosis, and it has been notes a decreased ability to garden, shop, and perform
recommended she undergo lumbar laminectomy. She activities of daily living. She has no chest pain, orthop-
is unable to ascend and descend stairs secondary to low nea, or lower-extremity edema. The past medical history
back pain. Her medical history includes hypertension is unremarkable, and her only medication is a multivita-
and diabetes leading to chronic kidney disease with min. On examination, the heart rate is 42 beats per min-
baseline creatinine of 2.1. She had a transient ischemic ute and regular. The blood pressure is 128/70mm Hg,
attack 2years prior. Medications include aspirin, NPH and oxygen saturation is 97% breathing room air. The
insulin, rosuvastatin, and amlodipine. Physical exami- skin is warm and dry, venous pressure is low, and first
nation reveals heart rate of 78 beats per minute, blood and second heart sounds are normal. Lungs are clear.
pressure of 138/68 mm Hg, and is otherwise normal. An electrocardiogram reveals sinus bradycardia with a
Electrocardiogram reveals sinus rhythm, left anterior narrow QRS complex. With ambulation, oxygen satu-
fascicular block, and Q waves in leads V1 and V2. ration is 96% and heart rate increases to 50 beats per
What is the next step in diagnosis and management? minute. Thyroid function testing is normal.
What is the next step in management?
A. Exercise treadmill test
B. Dobutamine stress echocardiogram A. 48-hour Holter monitor
C. Coronary CT angiogram B. 30-day event monitor
D. Proceed with surgery. C. Electrophysiology study with measurement of sinus
E. Prescribe metoprolol. node recovery time

8 . C a r d i ova s c u l a r D i s e a s e 171
D. Serum Lyme titer Anechocardiogram reveals a dilated left ventricle, mod-
E. Referral for pacemaker placement erate mitral regurgitation, and global left ventricular
dysfunction with an LVEF of 35%. Serum electrolytes
48. A32-year-old man is seen in follow-up. He feels well. are normal as are thyroid function tests.
At a recent Health Day event at his office, blood pres- What is the next step in diagnosis?
sure was 120/60mm Hg and total cholesterol was mea-
sured at 160 mg/dL. The past medical history is notable A. Urine and serum metanephrines
for tension headache well controlled with as-needed B. Coronary angiography
acetaminophen. On examination, the heart rate is 103 C. Transesophageal echocardiogram
and blood pressure 156/98 mm Hg. The funduscopic D. Cardiac MRI
examination reveals normal optic disc and normal reti- E. Serum protein electrophoresis
nal vessels. The thyroid gland is slightly full to palpa-
tion. The carotids are normal in contour and volume. 50. A62-year-old gentleman with diabetes and tobacco
The first and second heart sounds are normal. There use history presents for follow-up of chest pain. He ini-
are no abdominal or flank bruits and no radio-femoral tially presented with chest pain 5 months prior; coro-
pulse delay. nary angiography demonstrated a 60% stenosis of the
What is the best next diagnostic step? mid left anterior descending artery and an 80% stenosis
of a moderate-sized obtuse marginal branch of the left
A. Serum potassium and measurement of circumflex. He was initiated on aspirin, metoprolol,
renin-aldosterone ratio and isosorbide mononitrate. For persistent chest pain
B. Renal artery Doppler ultrasound with moderate exertion, ranolazine was added. He has
C. Home blood pressure diary, measure TSH, and undergone two courses of supervised cardiac rehabilita-
repeat office blood pressure in 3months tion, yet symptoms persist. At present, he gets chest pain
D. Prescribe chlorthalidone. with moderate exertion, which precludes him from play-
E. Measure serum free metanephrines. ing doubles tennis and bowling. Medications include
metformin, metoprolol, rosuvastatin, isosorbide mono-
49. A 46-year-old gentleman presents with dyspnea and nitrate, ranolazine, and lisinopril. On examination, the
lower-extremity edema. For the past 4 weeks, he has heart rate is 56 beats per minute, the blood pressure is
noted progressive symptoms. The past medical history 100/60mm Hg. The cardiopulmonary examinations are
includes hypertension. Medications include chlorthali- normal. The total cholesterol is 100 mg/dL with HDL
done. He drinks 2 glasses of wine per week. On exami- cholesterol of 38 mg/dL. The hemoglobin A1C is 6.2%.
nation, the blood pressure is 100/50 mm Hg, with heart What is the best next step in management?
rate of 80 beats per minute. The jugular venous pressure
is 10 cm of water. The first heart sound is normal, and A. Refer for coronary artery bypass surgery.
the second heart sound splits with inspiration. A third B. Refer for PCI of the obtuse marginal stenosis.
heart sound is present, as is a 2 out of 6 pansystolic mur- C. Increase metoprolol.
mur at the apex with radiation to the axilla. The extrem- D. Increase isosorbide mononitrate.
ities are warm, and there is pitting edema to the knee. E. Refer for cardiac rehabilitation.

17 2 T h e B r i g h a m I n t e n s i v e R e v i e w o f I n t e r n a l M e d i c i n e Q u e s t i o n a n d A n s w e r C o m pa n i o n
CH A PT ER8 A NSW ER S and lack of a wall motion abnormality also favor pericar-
ditis rather than STEMI. Nonsteroidal anti-inflammatory
1. ANSWER:C. Atrial septal defect agents, often coupled with colchicine, are the treatment of
choice. Answer A would be appropriate for a patient sus-
Explanation:This patient presents with symptoms of pal- pected of having STEMI, in which case urgent reperfusion
pitations, a common complaint. The physical examination therapy would be indicated. Answer B would be appro-
in such cases should be focused on identifying underlying priate for patients with high-risk non-ST elevation acute
structural heart disease. The findings of a parasternal lift, coronary syndromes. Answer C, prednisone, has been
widened second heart sound with a fixed split, prominent shown to increase the risk of recurrent disease when pre-
pulmonic closure sound, and a pulmonary flow murmur scribed in patients with pericarditis and should be avoided.
are all classic findings of a secundum atrial septal defect Answer E is the management of choice for patients with
(ASD). Assuming normal right and left ventricular com- cocaine-induced myocardial ischemia, which the history
pliances, ASDs cause left-to-right shunting of blood and does not support in this case.
chronic volume loading of the right atrium and ventricle
leading to progressive dilation and predisposing to arrhyth-
mias, including atrial fibrillation, atrial tachycardia, and
ventricular rhythms. The increased right-sided flow gives 3. ANSWER:A. Volume loading with normal saline
rise to the pulmonary valve murmur and fixed splitting of
the second heart sound. The ECG findings of rightward This patient with multiple cardiac risk factors presents with
axis and incomplete right bundle branch block are typical an inferior ST segment myocardial infarction. Inferior
of a secundum ASD. Wolff-Parkinson-White syndrome STEMI can be due to occlusion of the right coronary or
would present with a short PR interval and delta waves on left circumflex coronary arteries; in this case, the ST seg-
the ECG, while the ECG of patients with arrhythmogenic ment elevation in lead III that is of greater magnitude than
right ventricular cardiomyopathy classically demonstrates in lead II coupled with the significant reciprocal depres-
right ventricular conduction delay, low voltage, and T wave sions in lead aVL suggests occlusion of the right coronary
inversions in the septal, anterior, and inferior leads. The artery. The physical examination reveals a triad of hypoten-
diastolic rumbling murmur, loud S1, and opening snap of sion, jugular venous distention with Kussmauls sign, and
mitral stenosis are not appreciated; finally, hyperthyroidism clear lungs. This triad suggests a clinical diagnosis of right
can present with arrhythmia and murmur related to the ventricular myocardial infarction. When the right coro-
hyperdynamic state, but other findings of hyperthyroidism, nary artery is occluded proximally, the acute RV marginal
including weight loss, stare, diaphoresis, and tremor, are not branches of the RCA receive no flow leading to RV infarc-
appreciated. tion. The hypocontractile, infarcted RV leads to elevated
right atrial and jugular venous pressure, and the decreased
right ventricular compliance gives rise to Kussmauls sign.
The right ventricle is unable to maintain left ventricular
2. ANSWER:D. Ibuprofen preload leading to hypotension. The hypotension can be
worsened markedly with nitrate therapy. The appropri-
This patient presents with chest pain and ECG findings of ate hemodynamic management includes judicious volume
ST segment elevationspecifically, the ECG reveals sinus loading, often guided by a pulmonary artery catheter. The
tachycardia and ST segment elevations of 13mm in leads low left ventricular preload would be exacerbated by diure-
I, II, III, aVF, and V2-V6. There are PR segment depressions sis. Intravenous beta blockers should not be administered
most prominent in leads I, II, and V6. The differential diag- to patients in cardiogenic shock with acute myocardial
nosis for ST segment elevation on the electrocardiogram infarction and would worsen hemodynamics in this case;
includes ST segment elevation, myocardial infarction, acute esmolol can be a useful agent to control blood pressure in
pericarditis, early repolarization, and left ventricular aneu- the management of acute aortic dissection. Finally, neo-
rysm. In this case, the diffuse nature of the ST segment ele- synephrine, an alpha agonist, causes increased afterload
vation, the concave-upward morphology of the ST segment, and can worsen myocardial performance in the setting of
the lack of reciprocal ST segment depressions, and the PR cardiogenic shock.
segment depressions on the electrocardiogram are all most
consistent with acute pericarditis rather than STEMI.
Patients with acute pericarditis often assume a position of
sitting up leaning forward, which minimizes pericardial 4. ANSWER:E. Mitral stenosis
pain. Making the distinction between acute pericarditis
and acute STEMI is sometimes difficult, and bedside echo- This patient who is 22 weeks pregnant presents with dys-
cardiography can be helpfulhere, the pericardial effusion pnea; dyspnea is a common complaint in pregnancy and

8 . C a r d i ova s c u l a r D i s e a s e 17 3
has myriad causes. Salient symptoms here include orthop- 6. ANSWER:B. Increase dosage of metoprolol for a
nea, which is a historical feature with high specificity for goal heart rate of 5565 beats per minute, then proceed
elevated left heart-filling pressures. The examination sug- with total knee replacement.
gests atrial fibrillation with an irregularly irregular heart
rhythm. The loud S1, opening snap, and diastolic murmur This patient is referred for preoperative cardiac risk stratifi-
are all cardinal physical features of mitral stenosis. The state cation prior to noncardiac surgery. The first step in assessing
of pregnancy leads to significant hemodynamic changes for operative risk is to assess whether the planned surgery is
even in normal cases, including decreased systemic vas- emergent and life-or-limb saving, in which case the surgi-
cular resistance, increased plasma volume, and increased cal procedure should proceed without delay from further
cardiac output. These physiologic changes can lead to cardiovascular diagnostics or therapeutics. In this case,
physical findings of systolic flow murmurs, a third heart surgery is elective. Next, assess whether unstable cardio-
sound, a venous hum, and a mammary souffle, which are vascular conditions are present that should be addressed
normal findings in pregnancy. In patients with preexist- prior to elective surgerythese conditions would include
ing heart disease, however, the hemodynamic stressors can unstable tachy or bradyarrhythmias, unstable ischemic syn-
lead to significant decompensation. Stenotic valve lesions dromes, decompensated heart failure, and critical, symp-
in particular tolerate pregnancy poorly, and it is often tomatic stenotic valvular disease. None of these syndromes
that patients who had well tolerated asymptomatic occult are present. Next, assess the risk of the planned surgery. For
mitral stenosis prior to pregnancy present with symptoms low-risk procedures (endoscopy, dermatologic surgery, cata-
as pregnancy progresses. Pregnancy-induced cardiomyop- ract surgery, and breast surgery), no further cardiovascular
athy and pulmonary embolism are both important differ- diagnostics or therapeutics are necessary prior to the pro-
ential diagnoses for the dyspneic pregnant patient, but the cedure. Orthopedic procedures, intraabdominal surgery,
physical findings in this case are inconsistent with these and intrathoracic surgery are considered intermediate-risk
diagnoses. Similarly, thyroid storm can present with new procedures. Next, assess functional capacity. If a patient can
atrial arrhythmia and high-output heart failure but would achieve greater than 4 METS of activity (corresponding
not be associated with a systolic flow murmur rather than roughly to walking briskly up two flights of stairs) without
a diastolic rumble and opening snap. This patients physi- cardiovascular limitation, the risk of perioperative cardio-
cal examination is not consistent with normal physical vascular events is low and the planned operation can pro-
findings of pregnancy. ceed. In this case, the functional capacity is limited by knee
pain. Finally, the Revised Clinical Risk Index can be cal-
culatedthis Index assesses for patient-specific risk factors
for a cardiovascular event in the perioperative period. These
5. ANSWER:B. Echocardiography include (1)history of heart failure, (2)history of stroke or
TIA, (3)history of ischemic heart disease (prior infarction,
This patient with known malignancy presents with dys- known CAD, or Q waves on the ECG), (4)serum creati-
pnea. The differential diagnosis of a cancer patient pre- nine greater than 2.0, and (5) diabetes requiring insulin.
senting with dyspnea includes hematologic abnormalities, Depending on the RCRI and the risk of the planned proce-
pulmonary embolism, paraneoplastic syndromes, infec- dure, no therapy, beta blockade, or, rarely, stress testing and
tion, and cardiac causes. Tachycardia, hypotension, and possible revascularization could be considered. This patient
elevated jugular venous pressure are consistent with both has RCRI of 3 (including coronary disease, diabetes, and
pulmonary embolism and cardiac tamponade. Kussmauls renal insufficiency) and is already maintained on beta block-
sign can be seen in both conditions. The low electrocar- ade. The most optimal course of therapy is to uptitrate beta
diogram voltage and quiet heart sounds coupled with the blockade over several weeks for goal heart rate of 5565 and
aforementioned abnormalities suggests cardiac tampon- thereafter proceed with total knee replacement.
ade as the cause of this patients symptoms, which is best
evaluated with an echocardiogram. EMG may be indi-
cated to evaluate for neuromuscular paraneoplastic syn-
dromes in the setting of small cell lung cancer, but there 7. ANSWER:A. Electrophysiology study and mapping
is no evidence of weakness on exam. Cardiac tamponade
secondary to cancer can present insidiously with gradual This young athlete presents with palpitations. Historical
onset of dyspnea, in contrast to the acute tamponade features favoring a paroxysmal supraventricular tachycardia
observed after cardiac surgery or intracardiac procedures. include abrupt onset and offset as well as the presence of neck
Cancers of lung, breast, and kidney as well as hematologic pounding, which can be caused by retrograde atrial activation
malignancies are the most likely malignancies to metasta- in the setting of AVNRT or AVRT. The electrocardiogram in
size to the pericardium. this case is diagnostic, revealing sinus bradycardia with sinus

174 T h e B r i g h a m I n t e n s i v e R e v i e w o f I n t e r n a l M e d i c i n e Q u e s t i o n a n d A n s w e r C o m pa n i o n
arrhythmia that is a normal finding in a young, highly condi- coronary syndromes are comprised of unstable angina and
tioned athlete. The PR interval is short, less than 120 ms, and non-ST segment myocardial infarction; serum biomarkers
there are delta waves visible as broad slurred initial portion of myocardial necrosis (troponin in this case) are not pres-
of the QRS best seen in leads II, III, and aVF and leads V2 ent in unstable angina and are present in NSTEMI. It has
and V3. Lead aVL also has a short PR interval with a Q wave been established that there is no benefit to thrombolysis in
that represents a negative delta wave rather than evidence of NSTEMI; in contrast to STEMI, the infarct-related artery
prior infarction; this is a so-called pseudoinfarction pattern. is partially rather than completely occluded. A selectively
A short PR interval and delta waves on the ECG coupled invasive strategy consists of medical management of the
with symptoms suggestive of arrhythmia is diagnostic of the patient with antiplatelet, antithrombin, and anti-ischemic
Wolff Parkinson White syndrome. First-line therapy for this therapy followed by noninvasive risk stratification with stress
condition is referral to electrophysiology for EP study with testing. An early invasive strategy includes aggressive medi-
mapping and ablation of the accessory pathway. This proce- cal therapy and coronary angiography with revascularization
dure is curative in the vast majority of cases. Signal-averaged within 2448 hours of hospitalization. It has been shown
ECG can be used in diagnosis of arrhythmogenic right ven- that patients with higher risk NSTEACS have proportion-
tricular cardiomyopathy, but the 12-lead ECG is inconsistent ally more benefit from an early invasive strategy of man-
with this diagnosis. An exercise stress test or CT coronary agement with angiography within this time frame. There
angiogram could be helpful in diagnosing anomalous coro- are several validated scores available for risk stratification of
nary or premature coronary obstruction. The Q wave on patients with NSTEACS, including the GRACE score and
the ECG in lead aVL in this case is more consistent with a TIMI score. This patients age, multiple risk factors, severe
pseudo-infarct pattern from WPW. AHolter monitor could angina at rest, ST depression on the ECG, and positive tropo-
document presence of SVT; however, with symptoms and a nin all place him at high risk, and an early invasive strategy of
12-lead ECG consistent with WPW, there are sufficient data management would be favored. Fondaparanux is less favored
to recommend EP study as the initial step. as an antithrombin agent in patients going to the cath lab due
to risk of catheter-associated thrombus that is not present
when unfractionated heparin is used.

8. ANSWER:D. Proceed with planned dental


procedure.
10. ANSWER:E. Reassurance
This patient has known valvular heart disease with mitral
valve prolapse and is in need of dental work. For some Normal states, benign conditions, and cardiac and extracar-
patients with heart disease, the sequelae of infectious endo- diac pathology can all give rise to murmurs. This patient has
carditis mandate antibiotic prophylaxis before routine a systolic murmur that is soft and not associated with other
dental work. Patients with prior infectious endocarditis, adventitial sounds such as clicks or gallops. The second heart
prosthetic heart valves, cyanotic congenital heart disease sound splits normally and the loudness of A2 and P2 are nor-
that is unrepaired, or congenital heart disease repaired mal. There are no cardiovascular symptoms and an increase
with prosthetic material and patients status post cardiac in afterload causes decreased intensitythese are all features
transplantation all mandate antibiotics prior to dental pro- of a benign systolic flow murmur. In such a case, there is no
cedures that manipulate gingival tissue or involve the peri- need for further evaluation with echocardiography, and the
apical area of the tooth. The patient in this case meets none patient can be reassured. Any diastolic murmur, the pres-
of these criteria, and the optimal management is to proceed ence of clicks or gallops, a diamond-shaped systolic murmur
with planned dental work without antibiotic therapy. There louder than grade II/VI, and any pansystolic murmur would
is no indication for repeat echocardiogram for mitral valve be appropriate to investigate with echocardiography. In addi-
prolapse in the absence of a change in the physical exami- tion, if there are any symptoms referable to structural heart
nation or the development of interval cardiac symptoms. disease, echocardiogram should be obtained. Atreadmill test
There is no indication for blood cultures, as there is no cur- could be ordered if coronary disease or exercise-induced dys-
rent evidence of infectious endocarditis. rhythmia was suspected. Carotid ultrasound could confirm
the cause of a carotid bruit if one were appreciated.

9. ANSWER:E. Unfractionated heparin and coronary


angiography within 24 hours 11. ANSWER:D. Clopidogrel

This patient presents with high-risk non-ST segment eleva- This patient with cardiac risk factors presents with a non-ST
tion myocardial infarction. The non-ST segment acute segment myocardial infarction; the single severe stenosis

8 . C a r d i ova s c u l a r D i s e a s e 175
demonstrated on angiography was not amenable to percuta- Uncontrolled hypertension is a major risk factor for pro-
neous coronary intervention, and a course of medical manage- gression of diabetic nephropathy and retinopathy, and an
ment was undertaken. Aspirin, statin therapy, beta blockade, aggressive approach to blood pressure control is mandatory.
smoking cessation, and cardiac rehabilitation should be The Joint National Commissions 7th report on diagnosis
prescribed, and lipids and blood pressure should be treated and management of hypertension (JNC-8) recommends
to current goals. In addition, a large randomized controlled that patients with diabetes and patients with chronic kidney
trial (CURE trial; NEJM 2001)demonstrated the additive disease (this patient has both!) be treated to goal blood pres-
benefit of thienopyridine therapy added to aspirin therapy sure of 140/90 mm Hg. Given this patients diabetes and
in patients with NSTEMI. Clopidogrel therapy in addition proteinuria, the first-line therapy should be an ace-inhibitor
to aspirin reduced a composite endpoint of cardiovascular or angiotensin receptor blocker. The ACE/ARB should be
death, nonfatal myocardial infarction, and stroke. Thus, for uptitrated and, if a second agent is needed, amlodipine or
this patient with NSTEMI managed medically, clopidogrel chlorthalidone could be considered, although thiazides
should be added to aspirin therapy for 12months, assuming can worsen glucose control and may be less optimal in this
there is not a high risk of bleeding. Ranolazine and isosorbide setting. Metformin would not be used in a type 1 diabetic
mononitrate are anti-ischemic therapies that can improve patient.
anginal symptoms but have no effect on cardiovascular mor-
tality. There is no role for fish oil or warfarin in managing
NSTEMI absent another indication for these therapies.
14. ANSWER:B. Adenosine

This young patient presents with a supraventricular tachy-


12. ANSWER:C. Transthoracic echocardiogram cardia. The electrocardiogram demonstrates a narrow com-
plex, regular tachycardia at approximately 150 beats per
This patient has a history of mitral valve prolapse, which is a minute. No clear atrial activity is seen, and the differential
common cardiac valve lesion; myxomatous valve degeneration includes AVRT, AVNRT, atrial flutter, and ectopic atrial
leads to redundant chordal tissue and billowing of the leaflets tachycardia. Vagal maneuvers were an appropriate first step
back into the left atrium during ventricular systole, causing but were unsuccessful. Adenosine should be administered,
the characteristic click-murmur complex. Consequences of which will be both diagnostic and possibly therapeutic
MVP include progressive mitral regurgitation, an increased terminating AV node-dependent arrhythmias (AVNRT
risk of infective endocarditis, and chordal rupture leading and AVRT) and unmasking occult atrial activity in atrial
to subacute decline. This patient presents with new atrial flutter, ectopic atrial rhythm, or sinus tachycardia. There
arrhythmia, heart failure, and auscultory findings of severe is no need for DC cardioversion, as this patient is hemo-
mitral regurgitation rather than the known click-murmur dynamically stable. Amiodarone is not indicated in this
complex, all suggesting that he has suffered a ruptured chord, case. Normal saline would be prescribed if there were sinus
leading to subacute decompensation. Echocardiogram tachycardia secondary to volume depletion. Labetalol is a
should be ordered to confirm the diagnosis, and it is likely drug better suited to hypertensive urgency than supraven-
that surgical correction will be necessary. A surgeon with tricular tachycardia.
experience in mitral valve repair should be consulted. In
this circumstance, mitral valve repair is preferred over mitral
valve replacement. Warfarin and DC cardioversion could be
considered for new atrial fibrillation of recent onset; however, 15. ANSWER:B. Administer tenecteplase and
that would not address the underlying cause of dysrhythmia transfer to a PCI-capable facility.
in this case. Similarly, thyroid testing and screening of sleep
apnea are reasonable in an uncomplicated case of new atrial This patient is suffering an infero-postero-lateral ST seg-
fibrillation or atrial flutter, but neither is the most likely diag- ment myocardial infarction. Timely reperfusion therapy
nosis here. An exercise treadmill test is not recommended is paramounttime is muscle. The time standards for
for a patient in decompensated heart failure. Beta blockade reperfusion in the setting of STEMI include a door to nee-
would not be indicated. dle time of 30 minutes or less if thrombolytics are chosen
as the reperfusion strategy and a door to balloon time of
90 minutes or less if primary PCI is chosen as the reper-
fusion strategy. Timely PCI has been demonstrated to be
13. ANSWER:B. Lisinopril superior to timely lytic therapy with improved patency of
the infarct related artery, less reinfarction, and less hemor-
This patient presents with diabetes complicated by protein- rhagic complications. If a patient with STEMI presents to
uria. Hypertension is noted on the physical examination. a non-PCI center, transfer for PCI should be considered if

176 T h e B r i g h a m I n t e n s i v e R e v i e w o f I n t e r n a l M e d i c i n e Q u e s t i o n a n d A n s w e r C o m pa n i o n
the first medical contact to device time is less than 120 min- fraction. For patients with EF <35%, left bundle branch
utes. First medical contact is defined as the time at which block with QRS duration greater than 120 ms, and NYHA
the EMTs arrive at the bedside. In this case, there would classIII symptoms despite optimal medical therapy, a biven-
be unacceptable delay (2.5 hours) associated with transfer tricular pacemaker should be placed for cardiac resynchro-
and on-site full-dose tenecteplase should be administered. nization. Of note, the trials of CRT enrolled patients with
Recent guidelines recommend transfer to a PCI-capable QRS duration greater than 120 ms, but in subgroup anal-
facility even if on-site thrombolysis is successful, and not ysis, the most profound benefit was seen in patients with
just for failure of lytic therapy or for early reocclusion. QRS duration greater than 150 ms. Arecent trial demon-
Intravenous beta blockers are no longer recommended strated benefit of CRT in patients with left bundle branch
acutely for myocardial infarction because of the potential block and NHYA class II symptoms, but this indication
to precipitate low cardiac output states. Half-dose throm- has yet to be incorporated into guidelines. Augmentation of
bolytics prior to transfer for PCI (with or without a glyco- diuretic therapy in this patient is not indicated, as there are
protein 2B/3A antagonist) is not recommended. no stigmata of volume overload. Referral for LVAD place-
ment would be premature.

16. ANSWER:D. Increase antihypertensive therapy.


18. ANSWER:C. Routine follow-up in 2 weeks
This patient presents after an exacerbation of a syndrome
of heart failure; echocardiogram demonstrates normal This pregnant patient presents with mild lower-extremity
left ventricular function. Thus, this patients diagnosis is edema, a venous hum and flow murmur, and a third heart
heart failure with preserved ejection fraction. Risk factors sound. These are normal physical findings in pregnancy
for this condition include diabetes, obesity, hypertension, and, in the absence of symptoms or other signs of cardio-
and sleep apnea. Unlike heart failure with reduced ejection vascular disease, no further testing is indicated. In nor-
fraction, there are no therapies proven to decrease mortal- mal pregnancy, circulating plasma volume nearly doubles,
ity for patients with heart failure with preserved ejection systemic vascular resistance decreases, and cardiac output
fraction. Therapy is directed towards treating underlying increases. These changes lead to an exaggerated early dia-
and contributing causes aggressively- managing blood pres- stolic filling phase in the cardiac cycle and increased out-
sure, obesity, diabetes, sleep apnea, treating atrial fibril- flow tract flow with attendant S3 and pulmonary flow
lation if present, treating coronary disease if present. For murmur in many patients. Venous hums and mammary
this patient, anti-hypertensive therapy should be uptitrated souffle murmurs are also benign findings. Lower-extremity
aggressively, particularly given the presence of LVH on the venous pressure increases due to compression of the pelvic
echocardiogram. Use of an implantable cardiac defibrillator veins, leading to edema. Urinalysis and liver function tests
(ICD) is indicated for primary prevention of sudden death could be obtained if pre-eclampsia were suspected, on the
in patients with EF less than 30% after 90days of optimal basis of significant edema, hypertension, and headache. An
medical therapy for heart failure. BNP-guided diuresis is echocardiogram could be obtained if peripartum cardio-
actively being investigated as a management strategy but at myopathy were suspected, but the patients good functional
present is not standard of care. For patients who have multi- capacity and low venous pressure makes this less likely.
ple hospitalizations for heart failure, an advanced heart fail- Lower-extremity ultrasound would be indicated to evaluate
ure specialist can be helpful in consideration of advanced for DVT if leg pain or unilateral edema, warmth, and red-
therapies, but this patient has not yet had optimal manage- ness were present, as pregnancy is a hypercoagulable state.
ment of underlying comorbidities. Finally, there is no indication of arrhythmia, and a Holter
monitor would not be indicated.

17. ANSWER:C. Refer for cardiac resynchronization


therapy (biventricular pacemaker placement). 19. ANSWER:D. Add eplerenone.

This patient with cardiomyopathy has persistent symptoms This patient presented with a large myocardial infarction
of dyspnea despite optimal medical therapy. Using the complicated by heart failure and depressed ejection frac-
New York Heart Association symptom scale, this patient tion. The EPHESUS trial demonstrated the benefit of the
would be categorized as NYHA ClassIII. Patients with left addition of an aldosterone antagonist to such patients
bundle branch block have delayed depolarization of the lat- trial inclusion criteria included an ejection fraction of 40%
eral left ventricle, and this dyssynchony can worsen symp- or less and a syndrome of heart failure post myocardial
toms of heart failure in patients with depressed ejection infarction. Randomization occurred between 3 and 14days

8 . C a r d i ova s c u l a r D i s e a s e 17 7
post myocardial infarction, and there was reduction in total treatment include prevention of stroke and control of symp-
mortality and cardiovascular mortality and sudden car- toms. With regard to the latter, beta blockers and calcium
diac death in the eplerenone arm. Importantly, aldosterone channel blockers can be prescribed for rate control and
antagonists increase the risk of hyperkalemia, and so cre- anti-arrhythmic drugs and atrial fibrillation ablation can be
atinine and potassium should be monitored closely. There utilized to maintain sinus rhythm. With regard to preven-
is no evidence that aspirin 325 mg per day has superior effi- tion of stroke, the first step in choosing a therapy is assessing
cacy to aspirin 81 mg per day in this setting, and there is the baseline risk of stroke. The CHADS2 score is a scoring
an increased risk of bleeding with the higher dose. Ahigher system to determine stroke risk. One point is assigned for
dose of clopidogrel was tested in the OASIS-7 trial, which each of a history of congestive heart failure, hypertension,
assessed whether a higher loading dose of clopidogrel fol- age >75years, diabetes, and 2 points are assigned for prior
lowed by double dose clopidogrel for 1 week was superior stroke or transient ischemic attack. Patients with a score
to standard dosing of 75 mg daily; the primary endpoint in of 0 can be maintained on aspirin or no anticoagulation.
this trial was negative; however, there may be a decreased Ascore greater than or equal to 2 would be an indication
risk of stent thrombosis with the increased dosing regi- for warfarin therapy in the absence of a high risk of bleed-
men. There is no indication for ranolazine in this patient, ing. Ascore of 1 is indeterminate and a discussion should
and given that he is euvolemic, no indication for augmented be had with the patient about the risks and benefits of anti-
diuresis with metolazone. coagulation. The CHADS2 score has been further refined
with publication of the CHADS2-VASc score, which gives
additional points for age between 65 and 75, female gender,
and vascular disease. This patients scores on both scoring
20. ANSWER:E. Refer for pacemaker placement prior systems suggest that anticoagulation should be pursued.
to surgery. Warfarin is favored over dabigatran given her age and renal
insufficiency. Even if cardioversion were pursued, antico-
This patient is planning elective noncardiac surgery. An agulation should be maintained indefinitely given her high
important role of the perioperative medicine evaluation is risk of stroke.
an assessment for unstable cardiac conditions that mandate
treatment prior to surgery. This patient presents with syn-
cope and evidence of significant conduction system disease
on the 12-lead electrocardiogram with left bundle branch 22. ANSWER:E. Ibuprofen and colchicine
block and type II AV block. He has a high risk of progres-
sion to complete heart block, and a pacemaker should be This patient presents with acute pericarditis on the basis of
placed prior to elective surgery. Other unstable cardiac con- typical history, friction rub on physical examination, and
ditions that should be addressed prior to elective noncar- characteristic electrocardiographic and echocardiographic
diac surgery include unstable cardiac ischemic syndromes, findings. Coronary angiography is not needed in such typi-
including unstable and accelerating angina, STEMI, and cal cases, although it is reasonable in cases in which the dis-
NSTEMI, symptomatic bradycardia and supraventricular tinction between pericarditis and ST segment myocardial
tachycardias, ventricular tachycardia, decompensated heart infarction is unclear. The management of acute pericardi-
failure, and critical symptomatic stenotic valvular heart dis- tis should include both a nonsteroidal anti-inflammatory
ease (aortic and mitral stenosis). Beta blockade is indicated agent and colchicine; the combination reduced recurrence
to prevent perioperative events in patients at high risk but of symptoms in the pivotal COPE trial and was superior
would be inappropriate in this patient with symptomatic to NSAIDs alone. Prednisone is effective for acute treat-
heart block. An echocardiogram could be ordered if severe ment but is associated with an increased risk of relapses and
stenotic heart disease is suspected, but this patients mur- should not be utlized routinely.
murs are not consistent with severe aortic or mitral stenosis.
The syncope is likely due to heart block rather than dehy-
dration and so fluid loading would not address the underly-
ing cause. 23. ANSWER:D. Magnetic resonance angiography of
the thoracic and abdominal aorta

This young man presents with a bicuspid aortic valve, a


21. ANSWER:B. Warfarin dosed for INR 23 common congenital cardiac condition. Regarding the valve
indefinitely itself, there is only mild regurgitation and no stenosis, and
so no specific therapy is indicated for aortic valve disease.
This patient presents with atrial fibrillation. Atrial fibrilla- Aortic root dilation and aortic coarctation are both associ-
tion is a common arrhythmia with aging; the main goals of ated with bicuspid aortic valve disease. This patients aortic

17 8 T h e B r i g h a m I n t e n s i v e R e v i e w o f I n t e r n a l M e d i c i n e Q u e s t i o n a n d A n s w e r C o m pa n i o n
root is normal size; however, the presence of a posterior bruit or ranolazine. High-dose atorvastatin therapy has been
and a radio-femoral pulse delay suggest that there may be shown to be superior to moderate dose statin therapy when
concomitant coarctation. Magnetic resonance angiography administered early to acute coronary syndrome patients
would visualize the coarctation and enable quantification in the PROVE-IT trial. Patients receiving atorvastatin 80
of severity. This should be performed prior to clearance for mg had improvement in a composite endpoint, including
sports participation. Losartan has been shown to decrease myocardial infarction and all-cause mortality, with the ini-
aortic root diameter and aortic dilation associated with tial improvements beginning within 24 hours of random-
Marfan and Loeys-Dietz syndromes but is not indicated in ization. Hence, this patient should be transitioned from a
this case. low-potency statin to high-dose atorvastatin.

24. ANSWER:D. Renal artery Doppler studies 26. ANSWER:B. Discontinue clopidogrel and
proceed with surgery, continuing aspirin through the
This young patient presents with severe hypertension. perioperative period.
Hypertension can be essential or secondary to another dis-
ease process. It is appropriate to consider secondary causes This patient with coronary artery disease and prior PCI
of hypertension in patients presenting initially with hyper- presents prior to elective noncardiac surgery. He has been
tension at very young or very old ages and in patients with maintained on dual antiplatelet therapy with aspirin and
known hypertension whose disease worsens abruptly. The clopidogrel since his PCI. Guidelines suggest that dual
secondary causes of hypertension can be categorized as antiplatelet therapy be strictly maintained for 30days after
renal (secondary to intrinsic renal disease), renovascular placement of a bare metal stent and for 1year after place-
(secondary to renal arterial obstruction leading to decreased ment of a drug-eluting stent to reduce the risk of acute
renal perfusion and increased renin release), and endocrine/ stent thrombosis, a catastrophic and life-threatening event.
metabolic (primary hyperaldosteronism, thyroid disease). Given that this patients stent placement was 3years prior, it
Important additional causes of secondary hypertension is permissible to discontinue clopidogrel. Aspirin should be
include substance use, obstructive sleep apnea, and medica- maintained perioperatively, if possible, as the inflammatory
tion effect, including NSAIDs. This young, female patient milieu of the postsurgical state is prothrombotic. There is
has severe hypertension, hypokalemia, and an abdominal emerging data that statin therapy in high-risk patients can
bruit suggesting renal artery stenosis; given her demograph- reduce perioperative events, and statins should be contin-
ics, the stenosis is most likely secondary to fibromuscular ued in patients already taking them. Given this patients
dysplasia. Arenal artery imaging study should be ordered excellent functional status (playing competitive singles ten-
to confirm. Measurement of serum renin and aldosterone nis), there is no role for exercise stress testing or other diag-
levels will likely confirm elevated renin and aldosterone; nostic testing prior to surgery.
however, it will not yield diagnostic anatomic informa-
tion. Neither captopril-renal scintigraphy nor assessment
of renal vein renin levels is recommended as a diagnostic
strategy. There is no evidence of substance use or Cushings 27. ANSWER:C. Refer for cognitive-behavioral
syndrome. therapy.

This patient presents with symptoms of agoraphobia. On


examination, there is a regularly irregular heart rhythm
25. ANSWER:B. Change fluvastatin to high-dose noted, and an electrocardiogram confirms the presence
atorvastatin. of Mobitz IAV block (Wenckebach block) demonstrating
sinus rhythm and cycles of progressive PR interval prolonga-
This patient with diabetes presents with NSTEMI and tion followed by a dropped beat. Mobitz IAV block is com-
underwent successful PCI. Several medications have been mon in young healthy individuals with high vagal tone and
shown to improve outcome in patients with diabetes and represents conduction delay at the AV node. In this case,
NSTEMI. Compared with clopidogrel, prasugrel has been with exercise and attendant vagal withdrawal, the block
shown to improve a composite endpoint of cardiovascular abates and heart rate increases appropriately. Lyme disease
death and repeat myocardial infarction with a greater ben- can cause AV block, but this patient lacks evidence of fever,
efit in patients with diabetes. There is no advantage to aspi- arthralgias, or erythema migrans rash to suggest Lyme dis-
rin 325 mg per day as compared to 81 mg per day at a higher ease. A pacemaker would be indicated if complete heart
cost of greater bleeding. Given that this patient has no block or Mobitz type II AV block were present. The pres-
residual angina, there is no role for isosorbide mononitrate ence of bifascicular block (right bundle branch block with

8 . C a r d i ova s c u l a r D i s e a s e 17 9
left fascicular block) with a prolonged PR interval and syn- 30. ANSWER:C. Refer for implantable cardioverter
cope would also be an indication for pacemaker placement defibrillator.
as would symptoms that are convincingly demonstrated to
be secondary to Mobitz Iheart block. In this case of a young This patient has nonischemic dilated cardiomyopathy,
patient with symptoms not referable to heart block and with likely familial, and is doing overall well on appropriate
appropriate increase in heart rate and amelioration of AV medical therapy with good functional status. For patients
block with exercise, no specific therapy is indicated. with systolic dysfunction secondary to nonischemic cardio-
myopathy, ejection fraction less than 35% despite optimal
medical therapy, and functional status that is New York
Heart Association class IIIII, placement of an implant-
28. ANSWER:C. Prescribe isosorbide mononitrate. able cardioverter-defibrillator is indicated for primary pre-
vention of sudden cardiac death. The pivotal SCD-HeFT
This patient presents with typical angina that has a stable trial demonstrated benefit of an ICD in such patients. This
pattern; that is, the angina is predictably evoked by a stable patient has a narrow QRS and as such has no indication
level of exertion and readily relieved with rest. His angio- for CRT, which could be considered if the QRS were pro-
gram reveals nonobstructive RCA and LCx disease and a longed with a left bundle branch block pattern. The venous
70% stenosis in a small branch of the left anterior descend- pressure is low and there would be no benefit to adding an
ing artery. In addition, FFR across the RCA lesion indi- additional diuretic metolazone. Given his good functional
cates a non-ischemia-producing stenosis. Revascularization, status, referral for transplantation would be premature.
either with PCI and stent placement or bypass surgery, in Digoxin improves symptoms of heart failure without an
this circumstance does not provide any survival advantage effect on mortality and can be considered for patients with
over medical therapy, and as such, a trial of medical therapy persistent symptoms despite optimal medical therapy.
should be attempted. Medical therapies for patients with
stable angina should include agents to decrease myocardial
oxygen demand, including beta blockers, nitrates, and cal-
cium channel blockers. This patients heart rate is well con- 31. ANSWER:D. Hypertrophic obstructive
trolled and increasing the dose of beta blockers is unlikely cardiomyopathy
to be helpful. Along-acting nitrate would be a good choice
to decrease myocardial oxygen consumption by decreasing This patient presents with dizziness evoked by squatting to
preload and wall stress. Ranolazine can be considered as standing and dyspnea. The examination reveals a harsh systolic
an anti-anginal agent but typically is added after nitrates, murmur that becomes louder with maneuvers that decrease
beta blockers, and calcium channel blockers are maximized. preloada cardinal physical examination finding present in
There is no indication for dual antiplatelet therapy with hypertrophic obstructive cardiomyopathy. In this condition,
aspirin and clopidogrel in this patient. there is left ventricular outflow tract obstruction caused by
a hypertrophied interventricular septum. Decreased preload
decreases left ventricular cavity size and worsens obstruction
with attendant increase in the loudness of the murmur. The
29. ANSWER:B. Resume sexual activity in 1month obstruction can cause a Venturi effect, leading to systolic
after follow-up with cardiac surgery. anterior motion of the mitral valve and mitral regurgitation,
as is also appreciated in this patient. An echocardiogram
This patient presented with myocardial infarction and has should be ordered to confirm the diagnosis. Dilated cardio-
undergone complete revascularization with bypass surgery. myopathy can present with a third heart sound and murmurs
He is progressing well and is asymptomatic postoperatively. of mitral and tricuspid regurgitation. Mitral stenosis presents
The American Heart Association has issued guidelines with a diastolic rumbling murmur at the apex. The harsh sys-
addressing sexual activity and cardiovascular disease. For tolic murmur of aortic stenosis would be expected to decrease
patients post coronary artery bypass grafting, sexual activ- with Valsalva. Atrial septal defect presents with fixed splitting
ity can be resumed 68 weeks postoperatively, assuming of the second heart sound and murmurs of tricuspid regurgi-
there are no or minimal symptoms and the sternotomy is tation and increased pulmonary artery flow.
well healed. Stress testing prior to resumption of sexual
activity is reasonable if incomplete revascularization was
noted (i.e., if some of the coronary stenoses were unable to
be grafted at the time of surgery). Sexual dysfunction can 32. ANSWER:C. Transesophageal echocardiogram
coexist with peripheral arterial disease (Leriche syndrome),
which can be diagnosed with ABI/PVR, but this diagnos- This patient presents with staph aureus bacteremia and
tic test would not be indicated in this case. a vegetation on the aortic valve consistent with bacterial

18 0 T h e B r i g h a m I n t e n s i v e R e v i e w o f I n t e r n a l M e d i c i n e Q u e s t i o n a n d A n s w e r C o m pa n i o n
endocarditis. Appropriate antibiotic therapy for MSSA peripheral arterial disease (PAD). Patients with PAD are
was initiated with nafcillin. Vancomycin is appropriate also at risk for atherosclerosis of other arterial beds, and so
therapy if MRSA is suspected; however, nafcillin has bac- an aggressive regimen of antihypertensive, antiplatelet, and
tericidal activity against MSSA and is a preferred agent. lipid therapy is mandatory. To treat PAD that is moderate
Despite therapy, however, the patient develops evidence and not limb threatening, an exercise program has been
of new heart block. The aortic valve and aortic root are demonstrated to increase pain-free walking distance and is
in close anatomic proximity to the Bundle of His, and an an effective initial therapy. The data for efficacy of pentoxy-
aortic root abscess complicating bacterial endocarditis can fylline are marginal; cilostazol can be effective in increasing
progress to complete heart block. Serial monitoring of AV walking distance, but it should be avoided in patients with
conduction in patients with bacterial endocarditis is man- heart failure. Cilostazol can be used in conjunction with
datory to identify this complication early. Transesophageal an exercise program. Detailed anatomic imaging should be
echo should be performed to confirm the suspected aortic pursued if limb-threatening ischemia is present or if medical
root abscess and provide anatomic information to guide therapy fails and revascularization is being considered; refer-
surgical drainage. Surgical intervention for bacterial endo- ral for revascularization would be premature at this time.
carditis is indicated for abscess formation, severe valvular
destruction with heart failure, failure to clear the infection
and persistently positive blood cultures, and for large veg-
etation with significant risk for embolism. Blood cultures 35. ANSWER:B. Refer for aortic valve replacement.
can be incubated for a prolonged period to increase yield if
endocarditis associated with the fastidious HACEK organ- This patient presents with bicuspid aortic valve and severe
isms is suspected; similarly, serologies for C.burnetii can be aortic regurgitation (AR). Patients with AR should be
requested if Q fever, a cause of culture-negative endocardi- referred to surgery if symptoms are present, as in this case. If
tis, is suspected. Implantation of a pacemaker in this patient patients have asymptomatic severe AR, they should undergo
with active infection would not be the treatment of choice. valve replacement when the ejection fraction begins to fall
or when the left ventricle begins to dilate; repair at this
stage forestalls the eventual development of dilated cardio-
myopathy associated with chronic volume loading of the
33. ANSWER:D. Nuclear stress test with left ventricle. Bicuspid valve can be associated with aortic
regadenoson root dilation that in some instances mandates concomitant
aortic root repair, but aortic root diameter in this case is
This woman presents with a chest pain syndrome. She has normal. Trials of afterload reduction in aortic insufficiency
an intermediate pretest probability of coronary artery dis- have been negative, and medical therapy specifically for AR
ease based on her age and the characteristics of her pain. is not indicated. Serial echocardiogram would not be appro-
Hence, a form of stress testing is appropriate rather than priate in this patient with symptomatic severe AR. Finally,
reassurance. Her severe knee osteoarthritis and lumbar the brisk carotid impulses are an expected finding in severe
disc disease make it unlikely that exercise as a stressor will AR and do not mandate a carotid imaging study.
yield diagnostic informationthat is, it is unlikely she
could exercise on a treadmill to reach her target heart rate.
Adobutamine stress echocardiogram could be considered,
but the presence of left bundle branch block makes image 36. ANSWER:D. Obtain detailed over-the-counter
interpretation in this setting more difficult, although not medication use history.
impossible. Nuclear stress testing with exercise in patients
with left bundle branch block can produce a perfusion This patient presents with apparent resistant hyperten-
abnormality in the septum, and therefore pharmacologic sion, defined as blood pressure that is uncontrolled despite
stress testing is preferred in these patients. The best diag- use of three medications at good doses, one of which is a
nostic test for this patient who cannot exercise with a left diuretic. The first step in management of resistant hyper-
bundle branch block is a pharmacologic nuclear stress test. tension is to confirm the diagnosis, and home blood
pressure diary or 24-hour ambulatory blood pressure mon-
itoring can be used for confirmation. Next, ensure good
medication compliance, as noncompliance is a frequent
34. ANSWER:A. Recommend a supervised exercise cause of apparently resistant hypertension. Medications
program. that can contribute to hypertension, including possible
NSAID use in this case and sympathomimetics, should
This patient with tobacco use history presents with typical be minimized. Alcohol intake and obstructive sleep apnea
claudication and ankle-brachial index confirming moderate also can contribute to poorly controlled hypertension.

8 . C a r d i ova s c u l a r D i s e a s e 181
Secondary causes of hypertension can be evaluated, 39. ANSWER:B. Reassurance
including Cushings syndrome, hyperaldosteronism, thy-
roid disease, and renal artery disease, although these con- This patient presents with symptomatic premature ven-
ditions are less common than the aforementioned causes, tricular contractions (PVCs). Single, infrequent PVCs, as
which should be excluded first. in this patient, do not connote an adverse prognosis and she
can be reassured. Conditions that predispose to PVCs can
include alcohol intake, structural heart disease, and sympa-
thomimetic use, none of which are present in this case. If
37. ANSWER:D. Refer for coronary artery bypass symptoms are frequent and troublesome to the patient, beta
grafting of the LAD and RCA. blockers can be prescribed to suppress PVC. Antiarrhythmic
agents or catheter ablation in the electrophysiology lab can
This patient with diabetes and normal left ventricular func- be used for frequent, symptomatic PVC. Given the rarity
tion presents with obstructive coronary stenoses in two ves- of PVC and mild symptoms, neither would be indicated in
sels, one of which is the proximal left anterior descending this case. Given the lack of dyspnea and chest pain and nor-
artery. Coronary artery bypass grafting has been demon- mal physical examination, there would be no indication for
strated to be superior to medical therapy in such patients exercise treadmill testing or an echocardiogam.
and is a ClassIrecommendation in current guidelines. PCI
of one or both lesions could be considered if the patient
were a poor candidate for surgery, in which case physiologic
information, such as with stress MPI or FFR, about the 40. ANSWER:B. Routine follow-up in 12months
extent and anatomic distribution of ischemia may be help-
ful to guide revascularization. Regardless of the method This patient with coronary artery disease is doing well. Blood
of revascularization, aggressive medical therapy should be pressure and lipids are at goal, and there are no symptoms of
instituted with good control of blood pressure and lipids angina or heart failure and physical examination is normal.
and antiplatelet therapy with aspirin. Other ClassIindica- In this circumstance, further cardiovascular risk stratifica-
tions for coronary bypass surgery include patients with ste- tion with stress testing or echocardiogram is not needed. If
noses greater than 70% in three coronary vessels and those there were new symptoms or signs of cardiovascular disease,
patients with left main coronary artery stenosis greater than further testing would be indicated. hs-CRP can be a useful
50%. In a subset of patients with anatomically amenable test in patients with LDL < 130 mg/dL but other risk factors
coronary artery disease who are high risk for bypass surgery, for coronary disease that may benefit from statin therapy,
PCI could be considered. but this patient is already taking atorvastatin. Measurement
of CIMT may be useful in further risk stratifying patients
with an intermediate risk of coronary disease in the primary
prevention setting, but it is not useful in this patients case,
38. ANSWER:D. Repeat abdominal ultrasound in as she has already had a coronary event.
6months.

This patient presents with an asymptomatic abdominal


aortic aneurysm (AAA) measuring 4.1 cm. AAA should 41. ANSWER:C. Add lisinopril.
be repaired electively when greater than 5.5 cm or when
expanding greater than 0.5 cm in 6 months or 1.0 cm in This patient with prior myocardial infarction now presents
1year. If repair is planned, detailed anatomic information with systolic heart failure. In addition to a diagnostic evalua-
should be obtained with an imaging study, for example, a tion to determine underlying contributors to systolic dysfunc-
CT angiogram, which can help guide the choice of endo- tion, appropriate pharmacotherapy should be provided. The
vascular versus open repair. Prior to repair, patients should calcium channel blocker may impair LV function and should
be optimized medically, although stress testing should only be stopped. First-line therapy for patients with systolic dysfunc-
be recommended if it would affect management and lead tion include blockade of the renin-angiotensin-aldosterone
to preoperative coronary angiography and revasculariza- system with an angiotensin-converting enzyme inhibitor or
tion. AAA between 4 and 5.5cm should be followed with an angiotensin receptor blocker coupled with a beta blocker
an imaging study every 6 to 12months; AAA smaller than (carvedilol, extended-release metoprolol, bisoprolol, and nev-
4 cm can be followed at a lesser interval, every 13 years ibulol are the agents that have been evaluated in randomized
unless new symptoms emerge. AAA is considered a coro- controlled trials for use in heart failure). This patient has been
nary risk equivalent, and all patients should undergo maintained on beta blockers for coronary disease, and lisinopril
aggressive therapy of hypertension and dyslipidemia as well should be added and uptitrated as able while following blood
as smoking cessation. pressure, serum potassium, and renal function. Hydralazine

18 2 T h e B r i g h a m I n t e n s i v e R e v i e w o f I n t e r n a l M e d i c i n e Q u e s t i o n a n d A n s w e r C o m pa n i o n
and nitrates can be used in patients intolerant of ACE and likelihood of disease. Research studies have yielded tools
ARB and also have been shown to have incremental benefit in that can assist in estimating the pretest probability of coro-
African American patients with heart failure, but they would nary disease [Sox HC Jr, Hickam DH, Marton KI, et al.
not be first line in this case. There is no indication for dual anti- Am J Med. 1990;89:7]. Coronary artery calcium score can
platelet therapy with aspirin and clopidogrel. Spironolactone be used to refine risk prediction in the primary prevention
should be prescribed for heart failure patients with NYHA setting for patients at intermediate pretest risk of coronary
class II or greater symptoms despite adequate doses of ACE disease, but at present it should not be used for patients
inhibitors and beta blockers and normal potassium handing with active cardiac symptoms. Proceeding directly to coro-
there is a survival benefit with spironolactone treatment in nary angiography would be premature. Imaging with car-
this population. Digoxin can improve symptoms but offers no diac MRI can be useful in evaluation of pericardial disease
mortality benefit, and other agents should be optimized first. of myopathic diseases; however, a stress test is more appro-
priate for this patient. Finally, an exercise test yields more
hemodynamic data than a pharmacologic stress test and
should be first line in a patient who can exercise.
42. ANSWER:A. Echocardiography in 1year and as
needed with development of new symptoms

This patient presents with a murmur consistent with aortic 44. ANSWER:E. Prescribe metoprolol.
stenosis. Physical findings of decrease carotid pulse ampli-
tude and a late peaking murmur as well as mean gradient This patient presents for preoperative risk stratification prior
greater than 40 and aortic valve area less than 1.0 are all con- to lumbar laminectomy, an intermediate-risk noncardiac
sistent with severe aortic stenosis. Despite her valve disease, surgical procedure. There is no evidence of unstable car-
she remains asymptomatic with good functional status and diac conditions such as acute coronary syndrome, tachy or
as such can be followed expectantlyechocardiogram every brady-dysrhythmia, or severe stenotic valvular disease that
year or sooner if symptoms develop is recommended for sur- would preclude surgery, and the surgery is not emergent. The
veillance. If symptoms, subtle or overt, were to develop, she patients functional capacity is limited secondary to back
would be a candidate for aortic valve replacement. Aortic pain. She has multiple risk factors for a perioperative coronary
valvuloplasty can be used as a bridge to definite therapy event, including creatinine >2.0, diabetes requiring insulin,
for patients with severe AS who are critically ill; however, and a history of cerebrovascular disease. Other risk factors
in this patient valvuloplasty is not appropriate given her age include a history of heart failure and a history of coronary
and that she is asymptomatic with good functional capacity. artery disease. Of note, when left anterior fascicular block is
Statin therapy has been studied to retard the progression of present, Q waves must be visualized from V1 through V3 to
aortic stenosis but has not been shown to be effective and diagnose septal myocardial infarction. For patients with three
should not be used for this indication. In the majority of AS of these risk factors, beta blockers should be prescribed and
cases, assessment of severity of disease by echocardiography uptitrated over the weeks prior to the perioperative period for
is sufficient and direct measurement of the aortic valve gra- a goal heart rate of 5565 beats per minute. Stress testing and
dient in the catheterization lab is not necessary. coronary imaging should only be pursued as part of the pre-
operative evaluation if it would affect managementthat is, if
revascularization with coronary artery bypass grafting or PCI
would follow if needed, and no trial to date has demonstrated
43. ANSWER:C. Exercise treadmill test the benefit of routine revascularization prior to noncardiac
surgery. Given this patients high risk, however, proceeding to
This patient with multiple cardiac risk factors presents with surgery without beta blockade would not be appropriate.
substernal chest pain evoked by exertion and relieved with
rest. In evaluation of the chest pain patient, the pretest prob-
ability of coronary disease should be assessed. The patients
age, gender, and the characteristics of the pain inform the 45. ANSWER:D. Referral for surgical removal
pretest probability. In this case, the pretest probability of
coronary artery disease is considered intermediate to high, This young patient presents with a cardiac mass that was inci-
and further evaluation with stress testing is appropriate. In dentally discovered. Causes of intracardiac masses include
patients where the pretest probability of coronary disease thrombi, infectious and noninfectious vegetations, and car-
is low (<10%), a stress test that is positive likely represents diac tumors. Given lack of fever or other symptoms and lack
a false positive. For patients with an intermediate pretest of comorbidities, this is unlikely to represent a thrombus or
probability of coronary disease, stress testing is appropriate vegetation. The description of a peduculated mass attached
as the test characteristics meaningfully affect the posttest to the interatrial septum by a thin stalk is consistent with

8 . C a r d i ova s c u l a r D i s e a s e 183
atrial myxoma, which is the most common primary tumor pressure reading outside of the office. The funduscopic
of the heart. Myxomas should be removed surgically due to a examination reveals no evidence of chronic hypertension;
risk of embolism; atrial myxomas can also secrete cytokines, these facts should raise the question of whether white coat
leading to constitutional symptoms and para-immune phe- hypertension is present. A home blood pressure diary, or
nomena. Prognosis is good with surgical removal. ambulatory 24-hour blood pressure measurement if avail-
able, may help to clarify whether sustained hypertension is
present, and office blood pressure should be repeated at a
follow-up visit. Given the abnormal thyroid on examina-
46. ANSWER:D. Clinical follow-up in 6months tion, hyperthyroidism as a cause of elevated blood pressure
should be excluded. Patients with white coat hypertension
This patient with permanent atrial fibrillation presents have a risk of developing sustained hypertension and should
with heart rate of 88. A recent trial compared strict rate be followed. There are no symptoms to suggest pheochro-
control of resting heart rate under 80 beats per minute to mocytoma. White coat hypertension should be excluded
lenient rate control of resting heart rate under 110 beats before further evaluation for hyperaldosteronism and renal
per minute. There was no difference between groups with artery stenosis is undertaken. Empiric therapy is not indi-
regard to hospitalization, mortality, or symptoms. In this cated, given that only a single elevated blood pressure has
older patient who may be predisposed to orthostasis and is been documented.
currently asymptomatic, there would be no advantage to
increasing dosage of nodal agents or adding a second agent.
Dabigatran is superior to warfarin with fewer bleeding
events; however, in older patients, there may be increased 49. ANSWER:B. Coronary angiography
bleeding. As such, this patient should be maintained on
warfarin therapy. Amiodarone could be considered if a This patient presents with new-onset dilated cardiomyopa-
rhythm control strategy were pursued. Digoxin has some thy. Although young, his hypertension is a risk factor for
benefit in controlling resting heart rate; however, it is inef- coronary disease, which should be excluded as a cause of
fective at controlling ambulatory heart rate. heart failure, as revascularization could possibly improve
myocardial performance. Transesophageal echocardiogram
could be considered if a shunt lesion not visible on transtho-
racic echo were considered, but this is unlikely to be respon-
47. ANSWER:E. Referral for pacemaker placement sible for this patients left heart failure. Cardiac MRI can be
considered if infiltrative diseases were suspected, but coro-
This patient presents with symptomatic sinus bradycardia nary artery disease should be excluded first. Similarly, there
and failure to augment the heart rate with exercise. Causes are no symptoms to suggest pheochromocytoma or amyloid
of sinus bradycardia include hypothyroidism, medications, cardiomyopathy.
and degenerative conduction system diseasethe sick sinus
syndrome. Given that thyroid function studies are normal
and the patient is on no agents that would depress sinus node
function, the most likely diagnosis is sick sinus syndrome with 50. ANSWER:B. Refer for PCI of the obtuse marginal
chronotropic incompetence. Pacemaker implantation is indi- stenosis.
cated for the symptomatic bradycardia. There is no indication
for further confirmation and correlation of symptoms with This patient presented originally with chest pain and was
arrhythmia using a 48-hour Holter monitor. Event monitors diagnosed with coronary disease with moderate mid LAD
are best for diagnosis of paroxysmal symptoms hypothesized stenosis and obstructive disease in the left circumflex sys-
to be related to dysrhythmia. There are no symptoms to sug- tem. Despite aggressive medical therapy, lifestyle-limiting
gest Lyme disease. Sick sinus syndrome is a clinical diagnosis, angina persists; because of refractory symptoms, revas-
and invasive measurement of the sinus node recovery time (a cularization should be pursued. Given only a single vessel
marker of sinus node function) is rarely performed. is affected with greater than 70% stenosis, PCI would be
favored over bypass surgery. Increasing the dose of metopro-
lol would be limited by heart rate, and increase of nitrates
would be limited by blood pressure. Cardiac rehabilita-
48. ANSWER:C. Home blood pressure diary, measure tion could be considered, but this patient is already fit and
TSH, and repeat office blood pressure in 3months active and conditioning is unlikely to lead to abatement of
symptoms.
This patient presents with a single elevated blood pressure
reading in the office in the setting of recent normal blood

18 4 T h e B r i g h a m I n t e n s i v e R e v i e w o f I n t e r n a l M e d i c i n e Q u e s t i o n a n d A n s w e r C o m pa n i o n
9.
NEUROLOGY

Galen V. Henderson

1. A 61-year-old man is evaluated in the emergency Which of the following is the most appropriate next
department for mild left-sided weakness and left visual step in this patients management?
field loss that began about 6 hours ago. He has a his-
tory of diet-controlled hypertension. His current blood A. Start aspirin.
pressure is 160/75mm Hg, and heart rate is 110 beats B. Start intravenous heparin.
per minute and irregular. C. Start clopidogrel.
Physical examination reveals an irregularly D. Start intravenous tissue plasminogen activator.
irregular cardiac rhythm but is otherwise normal. E. Lower blood pressure to 140/90mm Hg.
Neurologic examination reveals a partial left homony-
mous hemianopsia, a left central facial palsy, mild left 3. A 32-year-old man is evaluated for what he calls a
upper-extremity weakness, and mild left sensory loss. sinus headache. The headache occurs two or three
Complete blood count and serum electrolytes and times a month and is accompanied by facial pressure
glucose are normal. Electrocardiography reveals atrial and occasional rhinorrhea; it worsens with move-
fibrillation. Computed tomography (CT) scan of the ment. Resting in a dark, quiet room results in subjec-
brain shows early hypodensity of the right basal ganglia tive improvement. The symptoms resolve in 1 or 2days
and insula. regardless of treatment. He has tried multiple varieties
Which of the following is the most appropriate of decongestants and antihistamines without success.
therapy? Acetaminophen-aspirin-caffeine preparations offer
minimal relief. He is currently symptomatic.
A. Warfarin On examination, the patient is pale and moderately
B. Enoxaparin distressed. Temperature is 37.1C (98.8F), heart rate
C. Clopidogrel 84 beats per minute, respiration rate of 16 breaths per
D. Adenosine minute, and blood pressure 132/75 mm Hg. His face is
tender on palpation.
What is the most likely diagnosis?
2. A71-year-old man presents to the emergency depart-
ment at the instruction of his primary care physician. A. Cluster headache
The patient felt well when he went to bed at midnight B. Migraine without aura
but awoke at 8:00 a.m. with left upper-extremity weak- C. Sinus headache
ness and numbness. He called his physician, who told D. Tension headache
him to go to the emergency department. He arrives at
the emergency department at 9:00 a.m. 4. A 22-year-old woman is evaluated for daily headaches
The patients medical history includes hypertension that seemed to be initially worse in the supine position for
and hyperlipidemia for which he takes a thiazide diuretic 1 to 2 weeks but now have been present continuously for
and a statin. His blood pressure is 178/92 mm Hg; the last month. She previously had occasional headache
physical examination reveals mild left-sided neglect, with photo- and phonophobia beginning at age 14years.
a mild left central facial palsy, mild left upper- and Therapy with triptans for her current headaches is only
lower-extremity weakness, and a mild left hemisensory mildly effective; she is on no other medications. She has had
deficit. Complete blood count and serum electrolytes intermittent blurred vision for the past month and notes a
and glucose are normal. CT scan of the brain is normal. pulsatile sound in both ears when she lies in a quiet room.

185
Physical examination is significant only for obesity Which of the following is the most appropriate next
(BMI 30kg/m2). Funduscopic examination reveals the diagnostic step?
presence of bilateral papilledema. No other abnormal
findings are seen on neurologic examination. Magnetic A. Lumbar puncture
resonance imaging (MRI) of the brain with contrast B. Neuropsychological evaluation
and magnetic resonance venography of the brain are C. Serial follow-up
normal. D. Analysis for presenilin-1 mutation
Which of the following is the most appropriate next E. Diffusion-weighted MRI of the brain
step in the management of this patients headache?
7. A78-year-old woman is evaluated for progressive impair-
A. Acetazolamide ments of memory and other aspects of intellect, which
B. Amitriptyline have developed insidiously over 3years. The patients score
C. Lumbar puncture on the MiniMental State Examination is 22/30, and she
D. Neurosugical consultation recalls none of three words after a 3-minute delay. There
are no other significant findings on physical examination.
5. A32-year-old woman is evaluated in the emergency Which of the following is most likely to improve this
department for a 2-day history of progressive numbness patients symptoms?
and weakness in the legs; she has also noted urinary
incontinence since awakening this morning. She has A. Vitamin E
no significant medical history and has not had recent B. Ginkgo biloba
infections or vaccinations. C. Central cholinesterase inhibitor
Physical examination shows normal mental status D. Memantine
and cranial nerves. Strength is normal in the arms. Both E. Estrogen
legs are very weak diffusely (2/5). Sensation to pinprick
is diminished from the umbilicus down, and vibratory 8. A 32-year-old woman is evaluated for a 2-week his-
sensation is diminished in the toes. Reflexes are 2+ in tory of weakness of the right arm and left leg. The ini-
the biceps and triceps, 3+ in the knees and ankles, with tial symptom was acute right wrist drop associated with
a bilateral extensor plantar response. Sagittal MRI of sensory loss in a radial nerve distribution and severe
the spinal cord shows an enhancing lesion within the pain. One week later, she developed similar symptoms
spinal cord, with edema, extending from the T8 to the in the left peroneal nerve distribution. The patient has
T10 level. systemic lupus erythematosus, and her medications
Which of the following is the most appropriate ini- include prednisone and hydroxychloroquine.
tial management of this patient? Which of the following is the most likely diagnosis?

A. Intramuscular interferon-beta A. GuillainBarr syndrome


B. Intravenous acyclovir B. Toxic neuropathy
C. Intravenous methylprednisolone C. Motor neuron disease
D. Neurosurgical decompression D. Vasculitic neuropathy
E. Oral prednisone E. Lyme disease

6. A75-year-old man is evaluated for memory loss occur- 9. A32-year-old man is evaluated for a 3-month history
ring over the past 3months. He has a family history of of progressive, symmetric proximal and distal weak-
Alzheimers disease, with three first-degree relatives ness. He was diagnosed with HIV infection 2 weeks ago
affected in their 50s. His memory loss is not apparent and is taking combination therapy with efavirenz, zid-
to his spouse or close friends and has not affected his ovudine, and lamivudine.
ability to perform his duties as a financial officer. His On examination, he has diffuse extremity weakness,
medical history includes treated hypertension and areflexia, and decreased vibratory sensation in the dis-
osteoarthritis. tal upper and lower extremities.
Physical examination is normal. The patients score on What is the most likely diagnosis?
the MiniMental State Examination is 28/30, and he
recalls one of three words after a delay. There are no A. Vasculitic neuropathy
abnormalities in other areas of cognition, and there is B. Chronic inflammatory demyelinating
no evidence of depression. Laboratory evaluation shows polyneuropathy
normal levels of serum thyroid-stimulating hormone C. Cytomegalovirus polyradiculopathy
and vitamin B12. MRI of the brain is normal. D. Toxic neuropathy

18 6 T h e B r i g h a m I n t e n s i v e R e v i e w o f I n t e r n a l M e d i c i n e Q u e s t i o n a n d A n s w e r C o m pa n i o n
10. A previously healthy 54-year-old woman is evalu- D. Transient ischemic attack (TIA)
ated in the emergency room for worsening shortness of
breath and an inability to swallow over the past 2days. 13. A 45-year-old man is admitted to the hospital
She also has had generalized fatigue, difficulty climb- after having three recent TIA episodes of aphasia and
ing stairs, difficulty holding her eyelids open, and right-hand weakness. One week before onset of these
intermittent double vision for the past month. Physical signs and symptoms, he developed persistent left jaw/
examination reveals nasal speech, bilateral ptosis, mild face pain. On the physical examination, vital signs are
proximal weakness, and normal reflexes. Body mass normal, there is a carotid bruit heard on auscultation,
index is 20kg/m2 , and forced vital capacity is 1.5L. and a left miosis and left ptosis are present. Laboratory
Which of the following is the most appropriate ini- studies and brain CT are normal.
tial treatment for this patient? Which of the following is the most likely diagnosis?

A. High-dose prednisone A. Cluster headache


B. Plasmapheresis B. Giant cell arteritis
C. Pyridostigmine C. Spontaeous left internal carotid artery dissection
D. Azathioprine D. Spontaneous left vertebral artery occlusion
E. Riluzole
14. A 60-year-old man is brought to the emergency
11. A 30-year-old man is evaluated in the emergency room by ambulance with a 1 hour history of witnessed
room for a 3-day history of confusion and visual loss onset of aphasia and right hemipariesis. He has a his-
and has a 4-day history of gradually increasing head- tory of hypertension and atrial fibrillation. His current
ache. One week ago, he had a bout of severe gastrenteri- medications include anithypertensives and wafarin. On
tis with diarrhea. A review of the medical records shows examination, his blood pressure is 165/90mm Hg and
that he has Factor V Leiden heterozygote. His exami- his heart rate is 72 beats per minute and irregular. The
nation is remarkable for papilledema, a right pronator neurological examination confirms the nonfluent apha-
drift, right homonymous hemianopsia, and fluent apha- sia, right hemiparesis of the arm and leg and extensor
sia. The CT scan of the brain and laboratory studies are plantor response on the right. The results of the labo-
normal. ratory studies performed were remarkable for an INR
Which of the following is the most appropriate next of 1.5 and an electrocardiogram (EKG), which confirms
diagnostic test in this patient? the presence of atrial fibrillation. CT scan of his head
obtained within 1 hour of his arrival reveals no early
A. Carotid ultrasonography ischemic changes.
B. Electroencephalography Which of the following is the best treatment?
C. Lumbar puncture
D. Magnetic resonance venography A. Aspirin
B. Continous IV heparin
12. A70-year-old woman is evaluated in the emergency C. Intra-arterial recombinant tissue plasminogen
room 1 hour after the acute onset of language problems activation (rtPA)
with an associated right-sided weakness. This event was D. Intravenous labetalol
witnessed by the family, and they say that the onset of E. Intravenous rtPA
symptoms progressed over minutes and that there was
some nausea and vomiting. They describe the patient as 15. A35-year-old woman is evaluated in the emergency
holding her head as if she was in pain. Her medical his- department for a 2-hour hisory of severe headache that
tory is remarkable for hypertension. During the exam, started suddenly. The headache involves the left fron-
the patient becomes more lethargic. Her blood pressure tal temporal region and the left ear and jaw. The pain
is 220/110mm Hg, heart rate is 105 beats per minute, is pulsatile and very severe. There are no other associ-
and respirations rate is 16 breaths per minute. There ated symptoms and there is no history of headaches.
is no nucal rigidity and no carotid bruits or jugular On examination, the patient ia afebrile at a blood
venous distention. The remainder of the examination pressure of 145/70mm Hg, heart rate of 70 beats per
is remarkable for a global aphasia and right hemiplegia. minute, and the BMI is 26kg/m2 . The patient has a left
What is the most likely diagnosis? ptosis and a mild anisocoria (OS 3mm and OD 4mm).
There is no meningismus and she has a normal exam.
A. Intracerebral hemorrhage Laboratory studies show normal complete blood count,
B. Ischemic stroke ESR, and serum chemistries. An unenhanced CT scan
C. Meningitis of the head and neck are normal. Amagnetic resonance

9. N e u r o l o g y 18 7
angiogram of the head and neck shows a carotid artery 18. A60-year-old man is admitted to the hospital after
dissection. sustaining a severe left hemipheric ischemic stroke
Which of the following is the most appropriate next while alone at home. His son found him collapsed in
step in treatment? the living room when he went to visit him. The patient
has hypertension and takes enalapril. On exam, blood
A. Intra-arterial nimodipine pressure is 190/100mm Hg and heart rate is 80 beats
B. Intravenous dihydroergotamine per minute. There is a right hemiparesis, right facial,
C. Intravenous heparin aphasia, and dysarthria. The laboratory tests are
D. Intravenous nitroglycerin normal and CT of the brain shows ischemic changes
E. Stent-assisted aneurysm coiling that occupy most of the left middle cerebral artery
territory.
16. A 65-year-old man is evaluated in the emergency Which of the following is the most appropriate treat-
department 6 hours after having sudden onset of a ment of his hypertension at this time?
severe headache. There is no other medical history and
the patient takes no medications. On physical exam, he A. Intravenous labetalol
is in obvious distress from the headache. He is afebrile B. Intravenous nicardipine
he has a blood pressure of 180/110mm Hg, heart rate of C. Oral difedipine
120 beats per minute, and respirations of 14 breaths per D. Withholding of all antihypertinsive medications
minute. There is no meningismus, and no subhyaloid
retinal hemorrhages are noted. The neurological exami- 19. A 68-year-old male is admitted to the hospital for
nation is normal. Results of the laboratory studies and evaluation of a 15-minute transient episode of left facial
CT of the brain without contrast are normal. droop, slurred speech, and left arm weakness. Three years
Which of the following is the most appropriate next prior, the patient had a radical neck dissection to treat
diagnostic test? head and neck cancer and subsequently had neck radia-
tion. He also has hypertension, hyperlipidemia, and a
A. CT angiogram of the head 40 pack per year smoking history. He takes lisinopril,
B. Lumbar puncture atorvastain, and aspirin. On examination, he is afebrile,
C. MRI of the brain with a blood pressure of 150/80 mm Hg, heart rate of 88
D. MR venogram of the brain beats per minute, and respiration rate of 14 breaths per
E. MRI of the brain minute. A right carotid bruit is heard on auscultation.
No abnormal findings are noted on neurologic examina-
17. An 80-year-old man was admitted to the emergency tion. An MRI of the brain reveals a small wedge-shaped
room 5 hours after he developed slurred speech, diffi- cortical diffusion-weighted positive region of signal
culty swallowing food and drink, and left hemiparesis. change in the right hemisphere. An MRA of the neck
A right hemispheric ischemic stroke was diagnosed shows 80% stenosis of the right internal carotid artery.
in the emergency department and the brain CT con- In addition to the ASA therapy, which of the follow-
firmed the diagnosis. Because the time of stroke onset ing is the most appropriate next step in treatment?
could not be determined, no recominant tissue plas-
minogen activator was administered. The patient has A. Carotid angioplasty and stenting
no other medical problems and takes no medication. B. Carotid endarectomy
On examination, the blood pressure is 170/80mm Hg C. External carotid to internal carotid artery bypass
and the heart rate of 80 beats per minute. The neurolog- surgery
ical assessment reveals dysarthria, dysphagia, left facial D. Intravenous administration of heparin
droop, and a left hemiparesis. Laboratory studies show
a plasma LDL cholesterol is 160 mg/dL but no other 20. A 40-year-old woman is admitted to the hospital
abnormalities. with a severe left hemiplegia, hemineglect, and dysar-
Which of the following is the most appropriate first thria. She has a history of antiphospholipid antibody
step in management after transfer is completed? syndrome diagnosed 3 years ago after an episode of
iliofemoral venous thrombosis. She was treated with
A. Bedside screening for dysphagia 18 months with warfarin but elected to discontinue
B. Oral administration of an ACE for blood pressure the treatment. There is no relevant family history and
control she only takes aspirin. Examination reveals severe left
C. Oral administration of a statin hemiplegia and that her head and eyes are deviated to
D. Physical therapy and rehabilitation consultation the right. CT of the head reveals a large hypodense
region occupying the entire right middle cerebral artery

18 8 T h e B r i g h a m I n t e n s i v e R e v i e w o f I n t e r n a l M e d i c i n e Q u e s t i o n a n d A n s w e r C o m pa n i o n
territory, a local mass effect, and a 7mm midline shift vomits, and then becomes rapidly more progressively
from the right to left. weak and eventually becomes obtunded, requiring intu-
bation and mechanical ventilation. On physical examina-
A. Which of the following is the most appropriate tion, she is afebrile; blood pressure is 188/100mm Hg,
therapy at this time? and heart rate is 120 beats per minute. The patient exhib-
B. Decompressive hemicraniectomy its flaccid quadriplegia, and meningismus is present.
C. Intra-arterial thrombolysis Both pupils are 4mm and nonreactive; theoculocephalic
D. Intravenous heparin reflex is absent, and the corneal reflex is absent bilaterally.
E. Use of an endovascular mechanical clot retrieval She has a depressed level of consciousness and a Glascow
device Coma Scale (GCS) score of 3.Subhyaloid hemorrhages
are present bilaterally. Complete blood count (CBC) and
21. An 80-year-old man is re-evaluated after a 5mm left the remainder of the blood work are normal. CT of the
middle cerebral artery aneurysm is discovered inciden- head shows an extensive acute subarachnoid and mild
tally on an MRI of the brain obtained due to headaches. prominence of the temporal tips of the lateral ventricles.
The patient has no other relevant personal or family Which of the following neurologic complica-
medical history and takes no medications. On physi- tions is most likely to have caused this patients rapid
cal examination his blood pressure is 140/80mm Hg, deterioration?
and his heart rate is 80 beats per minute. Results of the
physical and neurological exams are normal. An MRA A. Hydrocephalus
shows an unruptured aneurysm but no additional intra- B. Rebleeding
cranial aneurysms. No hemorrhage, infarction, mass, C. Syndrome of inappropriate antidiuretic hormone
or mass effect is evident. D. Vasospasm
Which of the following is the most appropriate next
step in the management of this patients aneurysm? 24. A 60-year-old man is admitted to the hospital for
a generalized tonic clonic seizure that began 30 min-
A. Annual MRA utes ago. Clinical seizure activity continues for another
B. Endovascular coling of the aneurysm 60minutes, during which time the patient is intubated,
C. Nimodipine administration placed on a ventilator and given lorazepam and phe-
D. Surgical clipping of the aneurysm nytoin, both intravenously. After his seizures stop, he
is transferred to the ICU where he remains comatose.
22. An 80-year-old male is evaluated in the office for an The patient has been receiving chemotherapy and whole
episode of hesitancy in speech and word-finding diffi- brain radiation therapy for recently diagnosed small
culty, right facial weakness, and weakness in the right cell lung cancer. He indicated at the time of his diagno-
arm. The episode occurred early yesterday, lasted 20 sis that he wants everything done to prolong his life. On
minutes, and was witnessed by his wife. The patient examination, the patient is comatose, with no response
has a history of coronary artery disease, hypertension, to deep pain stimulation; cranial nerve examination
and hyperlipidemia. Current medications are metopro- is significant for reactive pupils. Results of laboratory
lol, aspirin, hydrochlorothiazide, and lovastatin. On studies are noncontributory. Other than sinus tachycar-
examination, his blood pressure is 150/80mm Hg and dia, the electrocardiogram is normal.
his heart rate is 70 beats per minute. The neurological Which of the following is the most appropriate next
examination is normal. step in management?
Which of the following is the most appropriate next
step in management? A. Continuous EEG monitoring
B. Discussion of withdrawal of care with the family
A. Add clopidogrel. C. IV barbiturate therapy
B. Admit to hospital. D. MRI of the brain
C. Order outpatient diagnostic studies.
D. Schedule a follow-up visit in 1 week. 25. Apatient has 3 hours of dysarthria and arm/hand
weakness and then symptoms completely resolve.
23. A73-year-old woman is evaluated in the emergency The most likely diagnosis is:
room for sudden onset of explosive headache starting
8 hours ago, which is unchanged after resting in a dark A. Stroke
room. The patient has a history of hypertension con- B. Transient ischemic attack (TIA)
trolled by lisinopiril, and family history is noncontribu- C. Reversible ischemic neurologic deficit
tory. In the emergency room, she becomes nauseated, D. Peripheral neuropathy

9. N e u r o l o g y 18 9
26. What is the most practical cerebral imaging study 31. A20-year-old male college student is evaluated for a
within 6 hours of stroke-like symptoms? single generalized tonic clonic seizure that began when
he was asleep in his dormitory and resolved unevent-
A. CT of the brain fully. He has no history of head trauma, meningitis,
B. CT of the brain and CTA of the head and neck or prior seizure and no family history of epilepsy. He
C. MRI of the brain takes no medications. Results of the physical examina-
D. MRI of the brain and MRA of the head and neck tion, including a neurological examination, are normal.
Results of laboratory studies, including a complete
27. Which antiplatelet/antithombotic should be used blood count, a serum electrolyte panel, and a urine toxi-
within 48 hours of a stroke? cology screen are also normal. An MRI of the brain and
the EEG show no abnormalities.
A. Heparin, intravenously Which of the following is the most appropriate man-
B. Aspirin agement of this patients seizures?
C. Aspirin/dypridamole combination
D. Clopidogral A. Initiate no drug therapy at this time.
B. Initiate therapy with carbamazepine.
28. For patients discharged with stroke or TIA, dis- C. Initiate therapy with lamotrigine.
charge instructions include: D. Initiate therapy with valproic acid.
E. Refer to an epilepsy specialist.
A. Antiplatelets
B. Statins 32. A 55-year-old man is evaluated for a 2-year history
C. Antihypertinsives of burning feet. Symptoms are constant and are worse
D. Lifestyle changes at night. The patient is overweight and has a history
E. All of the above of hypertension treated with lisinopril. There is no
family history of peripheral neuropathy. On physical
29. The patient had slurred speech and arm weak- examination, the patient is abefrile, blood pressure is
ness for 45 minutes, and now there are no symptoms 130/80 mm Hg, heart rate is 66 beats per minute, res-
and physical examination is normal. The MRI shows pirations rate is 12 breaths per minute, and the BMI is
diffusion-weighted changes. The most likely diagnosis is: 25 kg/m2 . Neurological examination shows diminished
pinprick and temperature sensation on the dorsal and
A. Stroke plantar surfaces of both feet. Cranial nerve examina-
B. Transient ischemic attack (TIA) tion and testing of the manual muscle strength, deep
C. Reversible ischemic neurologic deficit tendon reflexes, proprioception, and coordination
D. Peripheral neuropathy reveal no abnormalities. Laboratory studies show a fast-
ing plasma glucose of 100 mg/dL. Results of the CBC,
30. A 20-year-old man is evaluated in the emergency vitamin B12, and serum protein electrophoresis are all
room for a 12-hour history of mild headache, nausea, normal. EMG testing shows a mild reduction in the sen-
and vomiting. His roommate had similar symptoms sory nerve action potential in the legs, compatible with
the previous day. He is given intravenous fluids and a mild peripheral neuropathy. An MRI of the lumbar
prochlorperazine and begins to feel better until his spine is normal.
head suddenly becomes stiff and turns to the right; Which of the following is the most appropriate next
he cannot move it to the midline or to the left, and diagnostic step?
he reports cramping and aching in the right neck
muscles. Neurological examination shows the head to A. Cerebrospinal fluid examination
be turned to the right with sustained contraction of B. Genetic testing for Charcot-Marie-Tooth disease
the left sternocleidomastoid muscle but is otherwise C. Glucose tolerance test
unremarkable. D. Skin biopsy
Which of the following is the best treatment for this
patient? 33. A 60-year-old woman is evaluated in the office for
a 6-month history of progressive weakness of the lower
A. Benztropine extremities. She says she has difficulty rising from a
B. Botulinum toxin seated position and walking up stairs and also has epi-
C. Phenytoin sodes of dry eyes, dry mouth, and erectile dysfunction.
D. Recombinant TPA The patient reports no ptosis, diplopia, dysphagia, or
E. Tetanus immune globulin dyspnea. She had a 15-year history of hypertension and

19 0 T h e B r i g h a m I n t e n s i v e R e v i e w o f I n t e r n a l M e d i c i n e Q u e s t i o n a n d A n s w e r C o m pa n i o n
a 42-pack/year history of smoking. Family history is physical examination, blood pressure is 120/60 mm
unremarkable. Her only medication is hydrochloro- Hg, heart rate is 68 beats per minute, respiration rate
thiazide. On physical examination, vital signs are nor- is 22 breaths per minute, and arterial oxygen saturation
mal. Manual muscle strength testing shows weakness in on ambient air is 98%. Her lungs are clear to ausculta-
the proximal upper and lower limb muscles. Deep ten- tion. She has moderate bilateral proximal weakness in
don reflexes are absent diffusely. Plantar responses are the upper and lower limbs with mild weakness distally.
flexor. Asensory examination shows no abnormalities, Deep tendon reflexes are normal and there are no appen-
and cranial nerve function is normal. Laboratory stud- dicular or truncal ataxia. She had fluctuating ptosis and
ies show normal serum electrolytes and creatine kinase. diminished facial strength. Speech is nasal and slurred.
Results of liver chemistry are also normal. Her language is normal.
Which of the following is the best diagnostic test for Which of the following is the best treatment option
this patient? for this patient?

A. Measurement of acetylcholine receptor antibody A. Plasma exchange


level B. Prednisone
B. Parathyroid hormone level C. Pyridostigmine
C. Voltage-gated P/Q type calcium channel antibody D. Repeat ciprofloxacin therapy
level
D. Muscle biopsy 36. A36-year-old woman is evaluated in the office for a
6-year history of headache. The patient says her head-
34. A35-year-old woman is evaluated in the office for aches occur approximately three times each month, are
a 6-week history of severe left facial pain. She says the associated with occasional nausea and pain that can be
pain occurs multiple times each day, is confined to the moderately severe and disabling, have a squeezing qual-
left cheek and jaw, and is stabbing in quality. Each pain ity, and begin in the neck, radiating forward to involve
episode lasts 2 seconds, but she may experience mul- the frontal head region bilaterally. Her headaches are
tiple consecutive episodes. The pain can be triggered preceded by yawning and irritability, which last up to
by drinking cold fluid, chewing on the left side of the 24 hours, and can be triggered by stress or changes in
mouth, or by touching the left cheek. Three years ago, barometric pressure. She has a history of mild depres-
she had intermittent paresthesias of the hands and sion treated with fluoxetine. Her mother and sister have
feet, which led to the diagnosis of anxiety. She has been a history of sinus headaches. On physical examination,
taking ibuprofen for pain control since her current blood pressure is 120/86mm Hg, heart rate is 78 beats
symptoms began, but it was ineffective. On physical per minute, and BMI is 21 kg/m2 . Results of general
examination, vital signs are normal. Pain was triggered physical and neurologic examination are normal. The
during the examination by touching the left cheek. She patients mood appears euthymic.
has no long tract pyramidal signs or pathologic reflexes. Which of the following is the most diagnosis?
Facial sensation and strength are normal.
Which of the following is the most appropriate next A. Chronic migraine
step in management? B. Medical overuse headache
C. Migraine without aura
A. Administration of baclofen D. Sinus headache
B. CT of the head E. Tension type headache
C. Lumbar puncture
D. MRI of the brain and MRA of the head and neck 37. A30-year-old male has a recent diagnosis of multiple
E. Nerve conduction studies of the extremities sclerosis (MS). He experienced two transient neurologic
episodes in the past 6months, one involving optic neu-
35. A60-year-old woman is evaluated in the emergency ritis and the other minor partial myelitis. He recovered
room for a 2-week history of progressive shortness of completely from both events and is currently asymptom-
breath, which has culminated over the past 4days in a atic. MS was diagnosed after an MRI of the brain, which
change in the sound of her speech and occasional swal- showed white matter signs typical of the disease. He has
lowing difficulties. She says that approximately 7 weeks no other pertinent personal or family medical history.
ago, she began to have blurred vision late in the day, par- Which of the following MS subtypes best describes
ticularly when driving home from work and occasional the course of his disease?
weakness in the upper and lower limbs. Three weeks
ago, she had a urinary tract infection treated with cip- A. Benign
rofloxacin. The patient takes no other medications. On B. Primary progressive

9. N e u r o l o g y 191
C. Relapsing remitting There is no history of trauma. The patient has degen-
D. Secondary progressive erative arthritis of the lumbosacral spine. He is a cur-
rent smoker with a 50 pack per year smoking history.
38. A 25-year-old woman comes to the office to ask The patient takes occasional ibuprofen or acetamino-
about discontinuing her epilepsy medication because phen for back pain. On physical examination, tempera-
she is concerned about its potential long-term adverse ture is normal, blood pressure is 140/85mm Hg, heart
effects. She has a 9-year history of generalized tonic rate is 80 beats per minute, and respiration rate is 14
clonic seizures and rapid, shock-like body jerks consis- breaths per minute. Neurologic examination shows left
tent with myoclonic seizures on awakening. After her lower-extremity weakness, hyperreflexia, and extensor
first seizures, she had an MRI with normal findings and plantar response, bilateral sensory impairment below
EEG with generalized polyspike and wave abnormali- the T7 dermatome, and gait atazia. Findings from men-
ties. She was started on lamotrigine and had done rela- tal status, cranial nerve, and upper-extremity motor and
tively well, having only rare seizures when she does not sensory examinations are normal. The complete blood
get adequate sleep or indulge in binge alcohol drinking. count, coagulation panel, and ESR are normal. Achest
Her last generalized tonic seizure occurred 1year ago. X-ray shows a parenchymal lesion in the right lung apex.
Results of the physical and neurological examination Which of the following is the most likely diagnosis?
are normal.
Which of the following is the best advice regarding A. Acute L5 disc herniation
discontinuation of the lamotrigine? B. Epidural abscess
C. Epidural hematoma
A. Begin a slow taper over the next 3months. D. Epidural metastases
B. Continue lifelong treatment.
C. Discontinue after she is seizure-free for 2years. 41. A 70-year-old man is admitted to the intensive care
D. Discontinue now. unit because of exacerbation of chronic obstructive pul-
monary disease. His course becomes complicated over
39. A 55-year-old man is evaluated in the office for a the next 3 days by pneumonia and acute kidney injury,
4-month history of tremor. The tremor affects both and he requires noninvasive positive pressure ventila-
upper extremities and is present most of the time. tion with bilevel positive airway pressure. After 10 days,
He has a 15-year history of type 2 diabetes mellitus; his medical condition stabilizes but profound weakness
he also has a history of history of hypertension, gas- of the extremities is noticed. Besides chronic obstructive
troparesis, and chronic kidney disease. Medications pulmonary disease (COPD), the patient had a history
are insulin glargine, insulin, lispro, lisinopril, hydro- of hypertension, hypothyroidism, and hyperlipidemia.
chlorothiazide, and metoclopramide. On examination, One week before admission, he was started on predni-
he has diminished pedal pulses, but speech, language, sone, 60 mg/day, for his COPD. His other medications
and mental status are normal. Cranial nerve function is include piperacillin-toazobactam, mitoprosol, levo-
normal, although a paucity of facial expression is noted. thyroxine, and simvastatin. On physical examination,
Movements are slow and there is a mild bilateral upper the blood pressure is 130/90 mm Hg, heart rate is 90
and lower rigidity. Deep tendon reflexes are normal, as beats per minute, and arterial oxygen saturation is 96%
are results of manual muscle strength testing. Sensory on nasal oxygen 2 L/min. Neurological examination
examination reveals distal sensory loss. He had a mild shows profound symmetric weakness of bilateral upper-
stooped posture but no postural instability. A 4-Hz and lower-extremity muscles. Deep tendon reflexes and
resting tremor in body upper extremities is noted as is a cranial nerve funtion are normal. Laboratory studies
prominent postural tremor. show a serum creatinine level at 1.5 mg/dL. His blood
Which of the following is the most likely diagnosis? chemistries, liver function, and creatine kinase are all
normal.
A. Dementia with Lewy body Which of the following is the most appropriate ther-
B. Drug-induced parkinsonism apy for this patient?
C. Multiple-system atrophy
D. Parkinsons disease A. Increased dosage of prednisone
B. Intravenous administration of immune globulin
40. A 75-year-old man is evaluated in the emergency C. Physical and occupational therapy
department for a 2-day history of impaired balance, left D. Plasma exchange
leg weakness, and urinary urgency and a 3-day history of
constant midthoracic pain with occasional shooting pain 42. A 50-year-old woman is evaluated in the office for
in the left thorax at approximately the T7 dermatome. memory loss of gradual onset and progression over the

19 2 T h e B r i g h a m I n t e n s i v e R e v i e w o f I n t e r n a l M e d i c i n e Q u e s t i o n a n d A n s w e r C o m pa n i o n
past year. She says she has difficulty recalling names of related to alcohol intoxication. He has end-stage liver
familiar people, has misplaced her glasses on numer- disease secondary to alcoholic cirrhosis but has been
ous occasions, and is slower to find her car in large or sober for the past 2years and is waiting for a liver trans-
crowded parking lots. The patient now requires help plant. His kidney function is normal. His current med-
from her daughter to manage her finances and prepare ications include spironolactone and furosemide. On
large meals. She has no other problems or personal med- physical examination, the patient is awake and alert,
ical history, but several members of her family devel- if afebrile, and has a normal blood pressure and pulse
oped dementia between age 46 and 54 years, including rate. Neurologic examination findings are normal. The
her mother, maternal uncle, and maternal grandfather. general physical examination reveals changes consis-
Her only medication is a daily multivitamin. On physi- tent with chronic liver disease, including jaundice and
cal examination, temperature is normal and blood pres- ascites. Laboratory studies show a serum creatnine level
sure is 115/75 mm Hg, heart rate of 72 beats per minute, of 0.6 mg/dL and no blood ethanol. Serum electrolyte
respiration rate of 14 breaths per minute, and BMI is levels are normal. An MRI of the brain shows an area
24 kg/m2. Her level of alertness, speech, and gait are of chronic encephalomalacia in the left frontotemporal
normal. She scores 24/30 on the Folstein Mini-Mental head region consistent with old trauma. An EEG shows
State Examination, losing 2 points on the orientation left temporal sharp waves.
section for misstating the date and year, one point in Which of the following is the best treatment for this
the serial calculation portion, and all three points on patient?
the recall portion. Results of her CBC, basic metabolic
panel, serum vitamin B12 measurement, and thyroid A. Levetiracetam
function are normal. Genetic testing is positive for the B. Oxcarbazepine
presenilin 1 mutation. C. Phynytoin
Which of the following is the most likely diagnosis? D. Valproic acid

A. Autosomal recessive Parkinsons disease with 45. A 40-year-old woman is evaluated for a 4-month his-
dementia tory of worsening gait and bilateral leg numbness. She
B. Creuzfeldt-Jacob disease underwent gastric bypass surgery 8 years ago but has no
C. Early-onset familial Alzheimer dementia other relevant personal history of family history. Her
D. Frontotemporal demenita only medications are a daily multivitamin and vitamin
E. Vascular dementia B12. Her vital signs were normal and her BMI was 29
kg/m2 . The patient has moderate gait ataxia, lower-
43. A 30-year-old man is evaluated in the office for an extremity spasticity, hyperreflexia, and bilateral exten-
8-month history of intensely painful headaches, which sor plantar responses. Strength is normal, but vibration
occur up to 10 times per day and last approximately 5 to and proprioceptive sensation is impaired in both feet.
10 minutes each. The pain is most severe around the left Results of the laboratory studies show mild normocytic
eye, and he has no pain between attacks. Each attack is anemia. Serum vitamin B12, vitamin E, methylmalo-
associated with rhinorrhea, lacrimation, and conjunctival cic acid, and homocysteine levels are all normal. Nerve
injection. The patient has a 12 pack year smoking history. conduction studies and EMG reveal a moderate axonal
He takes a combination of acetaminophen, caffeine, and peripheral neuropathy. MRI of the cervical spine shows
aspirin, usually taking a total of five tablets daily. Results no abnormalities.
of a physical and neurological examination are normal. Which of the following is the most appropriate next
Which of the following is the most likely diagnosis? diagnostic study?

A. Cluster headache A. Antinuclear antibody testing


B. Medication overuse headache B. Measurement of serum copper and zinc levels
C. Paroxysmal hemicrania C. MRI of the brain
D. SUNCT syndrome (short-lasting unilateral D. Serum neuromyelitis optica autoantibody test
neuralgiform headache attack with conjunctival
injection and tearing) 46. A30-year-old man is evaluated for a 3-day history
of worsening weakness and numbness of the right arm
44. A 50-year-old man is admitted to the hospital and leg. He has a 5-year history of multiple sclero-
after having two generalized tonic clonic seizures in a sis. His only current medication is glatiramer acetate.
24-hour period. He has had seizures in the past, which On physical examination, temperature is normal, and
were always attributed to alcohol withdrawal. Ten years blood pressure, heart rate, and respirations are normal.
ago, he was in a major motor vehicle collision that was Moderate right arm and leg weakness, hyperreflexia,

9. N e u r o l o g y 19 3
and extensor plantar response, and vibratory sense C. Parkinsons disease
impairment are noted. D. Wilsons disease
Which of the following should this patient receive to
treat his acute relapse? 49. A50-year-old woman is evaluated in the emergency
department after having a witnessed a generalized
A. Empiric antibiotic therapy tonic-clonic seizure. Her husband reports hearing her
B. Immune globulin, intravenously fall and finding her on the kitchen floor convulsing. The
C. Methylprednisolone, intravenously event lasted 2 minutes and stopped spontaneously. The
D. Plasmapherisis patient has never experienced a similar episode, has no
E. Prednisone, orally personal history of experiencing a similar episode, has
no personal history of head trauma or meningitis, and
47. A 26-year-old woman is evaluated in the office for has no family history of a seizures disorder. She has a
a change in migraine symptoms. She began having history of hypertension treated with hydrochlorothia-
migraine attacks shortly after menarche at age 13years, zide. On examination, the patient is lethargic but coop-
experiencing an attack approximately every 2months. erative. She is afebrile with normal blood pressure. She
For the past 8 weeks, these attacks have been associated had mild weakness of the left face and arm that resolves
with visual aura. The neurological symptoms evolve over the next 3 hours. Results of laboratory studies are
over a period of 10 minutes, lasts less than 60 minutes, normal. An MRI of the brain shows chronic small vessel
and are followed within 30 minutes by severe, uni- ischemic change but no acute abnormality.
lateral throbbing headaches associated with nausea. Which of the following findings predicts a greater
Sumatriptan relieves the headache within 30 minutes. risk of future seizure in this patient?
The patient also has asthma. Her mother and sister have
a history of migraine. Current medications are an oral A. Age
contraceptive pill started 9 weeks ago, sumatriptan B. Hypertension
as needed, a daily inhaled corticostroid, and inhaled C. Presence of secondary generalization
beta-agonist as needed. Results of physical examina- D. Small vessel ischemic change on the MRI
tion, including neurologic examination, are normal. E. Todd paralysis
Complete blood count, ESR, serum chemistry studies,
TSH, and anticardiolipin and ANA antibody levels are 50. A65-year-old man is evaluated for worsening of gait,
normal. An MRI of the brain is normal. unsteadiness, and falls. He first noticed unsteadiness
Which of the following is the most appropriate next 1year ago while working and has started to fall recently,
step in management? falling four times in the past 2 weeks. Approximately
3 years ago, he developed erectile dysfunction and
A. Add propranolol. has had increasing constipation ever since that time.
B. Add verapamil. On examination, vital signs are normal except for the
C. Discontinue the oral contraceptive pill. supine blood pressure, which is 190/105 mm Hg, and
D. Discontinue the sumatriptan. blood pressure decreases to 76/50 mm Hg when he
E. Measure surum lactate and pyruvate levels. stands without a compensatory increase in the heart
rate. Results of mental status testing are normal. He has
48. A 24-year-old woman is evaluated for a 2-month mildly slurred speech. Testing of cranial nerve function,
history of abnormal movements affecting both arms. including testing of extraocular movements, reveals no
She describes these movements as intermittent and abnormalities. Manual muscle strength in the upper
irregular writhing movements and says she cannot sup- and lower extremities is normal, but he had slight rigid-
press them. The patient had an unprovoked deep venous ity of the extremities and mild appendicular ataxia. His
thrombosis in her right leg 2years ago and a miscarriage gait is slow with a reduced stride length and arm swing,
8months ago but has had no other medical problems. and he has marked postural instability.
She has no known family history of neurologic disease Which of the following is the most likely diagnosis?
or thrombophilia and takes no medications. Physical
examination is normal. Neurological examination A. Dementia with Lewy body
reveals random irregular movements of the hands and B. Multiple-system atrophy
arms but is otherwise unremarkable. C. Parkinsons disease
Which of the following is the most likely diagnosis? D. Progressive supranuclear palsy

A. Antiphospholipid antibody syndrome


B. Huntingtons disease

19 4 T h e B r i g h a m I n t e n s i v e R e v i e w o f I n t e r n a l M e d i c i n e Q u e s t i o n a n d A n s w e r C o m pa n i o n
CH A PT ER9 A NSW ER S this patient include worsening of the headache with move-
ment, limitation of activities, and requiring absence of light
1. ANSWER:A. Warfarin and sound (dark, quiet room).
Although some autonomic features are present (conges-
The patient has an acute infarction involving the right mid- tion/rhinorrhea), the headache is not cluster type because it
dle cerebral artery territory. Although he has no history of lasts longer than 180 minutes (1 to 2days in this patients
atrial fibrillation, it was likely present but previously undi- case). In addition, although not part of the absolute criteria
agnosed. He is not eligible for acute thrombolytic therapy for cluster headache, patients with cluster headache prefer
because he could not be treated within 3 hours of symptom to be mobile, because resting causes worsening of the pain.
onset; however, the patient would benefit from early inter- The patient does not have the secondary headache of sinus
vention of secondary prevention measures. He is at high infection because of the lack of fever or discolored nasal dis-
risk for future cardioembolic strokes and would achieve charge. Although sinus symptoms are not part of the formal
substantial risk reduction with warfarin to maintain an criteria for migraine, they are quite common and can com-
INR between 2.0 and 3.0. plicate the diagnosis. Tension-type headache can be ruled
Early use of either unfractionated or low-molecular- out because of the disabling characteristic of the headache
weight heparin is dangerous in acute ischemic stroke, and and the presence of both photo- and phonophobia.
therefore enoxaparin is inappropriate. Clopidogrel has not
been shown to be effective in either acute stroke or in the
prophylaxis of stroke due to atrial fibrillation, although it
is effective in secondary prevention of noncardioembolic 4. ANSWER:C. Lumbar puncture
stroke. Adenosine does not effectively treat atrial fibrilla-
tion nor prevent stroke. This patients symptoms and signs of headache, intermit-
tent blurred vision, pulsatile tinnitus, and papilledema are
most consistent with increased intracranial pressure. Her
normal imaging studies exclude a mass lesion, hydrocepha-
2. ANSWER:A. Start aspirin. lus, or venous sinus thrombosis, leaving idiopathic intracra-
nial hypertension (pseudotumor cerebri) as the most likely
This patient had an acute ischemic stroke in the right middle diagnosis. This condition is most commonly seen in obese
cerebral artery territory. The time of onset is unknown, but women of childbearing age.
he was last known to be well at midnight, and he is therefore Because a mass lesion has been excluded by imaging,
not eligible for intravenous thrombolytic therapy, which is lumbar puncture is indicated to confirm increased cerebro-
indicated if therapy is started within 3 hours of onset of spinal fluid pressure and to initiate treatment by removal
stroke symptoms or when the patient was last known to be of cerebrospinal fluid. Urgent ophthalmologic consulta-
well. Patients often do not recognize stroke symptoms and tion also is important for formal assessment and subse-
at-risk patients should be educated about these symptoms. quent monitoring of visual fields, because visual loss is a
Early administration of aspirin, 160 to 325 mg daily, results potential complication of this condition. Acetazolamide
in a modest reduction in the risk of recurrent stroke in the therapy to reduce cerebrospinal fluid production should be
short term, and slightly less death and disability in the long started only after lumbar puncture confirms the diagnosis.
term. Amitriptyline therapy is not appropriate, because her cur-
Early administration of parenteral anticoagulants has rent headaches are not consistent with migraine and this
no net benefit for patients with acute stroke. Clopidogrel is therapy may cause increased weight gain. Neurosurgical
not beneficial as acute stroke therapy. Early blood pressure consultation is not appropriate at this time, although
lowering is not recommended for most patients with acute patients with idiopathic intracranial hypertension who
stroke unless they are being considered for thrombolytic develop visual field loss despite medical management may
therapy or are suffering from a concomitant myocardial require surgical intervention with optic nerve sheath fenes-
infarction or aortic dissection. In such cases, some experts tration or cerebrospinal fluid shunting procedures.
aim for a target mean arterial pressure of 140 mm Hg,
though without definitive evidence that this is beneficial.

5. ANSWER:C. Intravenous methylprednisolone

3. ANSWER:B. Migraine without aura This patient presents with a subacute onset of bilateral sen-
sory and motor dysfunction in the legs and urinary incon-
This patient presents with typical symptoms of migraine tinence; her examination shows sensory loss below the T10
headache. The symptoms meeting criteria for migraine in level as well as upper motor neuron signs in the legs. These

9. N e u r o l o g y 19 5
signs and symptoms suggest a lesion of the spinal cord and Although there may be an autosomal-dominant form
are consistent with the lesion seen within her thoracic cord of Alzheimers disease in this patients family, affected
on MRI. The clinical presentation of an acutely, or sub- members had disease onset when they were 20 years
acutely, evolving process involving the spinal cord causing younger than this patient. Thus, if the patient has
severe motor, sensory, and autonomic dysfunction below Alzheimers disease, it is more likely to be a sporadic than
that level is called a transverse myelopathy. The lesion on familial case, and genetic testing for presenilin-1 is not
MRI most likely represents an acute inflammatory or likely to be helpful.
demyelinative process involving the cord (transverse myeli- Diffusion-weighted MRI is sensitive for acute ischemic
tis), for which the most appropriate treatment is high-dose stroke, and perhaps for subacutely progressive dementia due
corticosteroids, usually given in the form of intravenous to CreutzfeldtJakob disease, but it has no role in routine
methylprednisolone for 3 to 5days. This therapy can be fol- evaluation of suspected degenerative disease.
lowed by a tapering course of oral prednisone.
This lesion is unlikely to represent tumor, given the
patients rapidly evolving symptoms. In addition, surgi-
cal decompression is not appropriate because the lesion is 7. ANSWER:C. Central cholinesterase inhibitor
within the spinal cord. Although transverse myelitis can
occur in the setting of infectious, postinfectious, or post- This patient has symptoms consistent with progressive
vaccination processes, specific causative agents are often not dementia, most likely Alzheimers dementia. Clinical trials
apparent; in this case, there is no evidence to support the of cholinesterase inhibitors (including galantamine, done-
use of an antiviral agent, such as acyclovir. Although this pezil, and rivastigmine) in patients with mild to moderate
patients syndrome might potentially represent the first Alzheimers disease consistently demonstrate a modest effi-
attack of multiple sclerosis, there is not enough evidence cacy on cognitive and global function scales. Cholinesterase
for multiple sclerosis in this patient to support the use of a inhibitors may also alleviate psychiatric symptoms, includ-
chronic immunomodulating agent, such as interferon-beta. ing agitation, apathy, and hallucinations, although there is
Even if such evidence was given in this case, the acute treat- no evidence that these agents delay the natural history of
ment would still be high-dose corticosteroids. Alzheimers disease.
Although some evidence suggests that vitamin E and
selegiline delay the progression of Alzheimers disease,
these agents do not directly relieve cognitive or psychiatric
6. ANSWER:B. Neuropsychological evaluation symptoms. Ginkgo biloba probably exerts modest effects on
cognitive symptoms in patients with Alzheimers disease,
This patient has isolated mild amnesia, and no impair- but it has not been shown to improve global or psychiatric
ment of interpersonal, occupational, or daily living activi- function. In addition, no standard preparation of Ginkgo
ties. His condition should be classified as mild cognitive biloba is available, and it is therefore not the agent of choice
impairment, rather than dementia. There are several rea- in mild to moderate Alzheimers disease.
sons to perform neuropsychological testing. First, depres- Memantine has been shown to be efficacious for relief
sion is often difficult to diagnose, especially in an articulate, of cognitive symptoms in moderate to severe, but not mild,
well-compensated person. Second, testing is indicated Alzheimers disease. Estrogen replacement in postmeno-
because of the patients position of responsibility. The pausal women with Alzheimers dementia has not been
evaluation will provide a basis for deciding whether or not shown to have any benefit for cognitive symptoms or dis-
to increase his level of supervision or reduce his level of ease progression.
responsibility. Third, neuropsychological examination will
also provide an objective baseline. Follow-up evaluation in
1 year is appropriate, as many affected patients may have
predemential Alzheimers disease. The conversion rate from 8. ANSWER:D. Vasculitic neuropathy
mild cognitive impairment to mild dementia is 10% to 15%
per year. Multiple mononeuropathies, as seen in this patient with
Routine lumbar puncture is normal in patients with involvement of the right radial and left peroneal nerves,
mild cognitive impairment and Alzheimers disease. No most commonly occur in patients with diabetes mellitus
biomarkers have been shown to have sufficient specificity and vasculitis. Vasculitic neuropathies (mononeuritis mul-
and sensitivity to assist diagnosis of dementia. tiplex) typically present with the acute onset of asymmet-
Serial follow-up is important because patients with mild ric weakness and sensory loss associated with severe pain.
cognitive impairment progress to mild dementia at a rate of Multiple mononeuropathies have been reported in leprosy,
about 15% per year. However, first establishing a baseline sarcoidosis, amyloidosis, and a genetic disease known as
with neuropsychological testing is appropriate. hereditary neuropathy with predisposition to pressure

19 6 T h e B r i g h a m I n t e n s i v e R e v i e w o f I n t e r n a l M e d i c i n e Q u e s t i o n a n d A n s w e r C o m pa n i o n
palsies (HNPP) but have not been described in association the likelihood of aspiration. Azathioprine is an effective
with Lyme disease. long-term treatment for myasthenia gravis, but clinical
Although GuillainBarr syndrome and toxic neuropa- improvement frequently takes 6 to 12months; therefore, it
thies may present acutely, the distribution of weakness and is not appropriate for myasthenic crisis. Riluzole is a treat-
sensory loss is usually symmetric. Motor neuron disease ment for amyotrophic lateral sclerosis, which rarely presents
can present with asymmetric weakness but is not associated with respiratory failure.
with pain or sensory symptoms.

11. ANSWER:D. Magnetic resonance


9. ANSWER:B. Chronic inflammatory demyelinating venography
polyneuropathy
The patient should undergo a magnetic resonance veno-
Chronic inflammatory demyelinating polyneuropathy is gram. The most likely diagnosis is venous sinus thrombo-
the chronic form of GuillainBarr syndrome in which sis given his known hypercoagulability, dehydration, and
symptoms progress for at least 8 weeks. The clinical fea- symptoms of mounting intracranial pressure and eventual
tures are similar to those of GuillainBarr syndrome and focal deficits. Carotid ultrasonography does not assist in
include proximal and distal weakness, areflexia, and distal the assessment of the intracranial vasculature. Seizures
sensory loss. The two disorders can both occur early in the are a recognized complication of venous sinus thrombo-
course of HIV infection, in which case cerebrospinal fluid sis; however, electroencephalography does not assist with
pleocytosis may occur along with an elevated cerebrospinal the establishment of a preliminary diagnosis and should
fluid protein concentration. not be performed. Alumbar puncture is contraindicated
Although vasculitic neuropathy may present sub- in this patient with evidence of elevated intracranial
acutely, it is predominantly asymmetric and associated with pressure.
severe pain. Cytomegalovirus polyradiculopathy occurs in
patients with end-stage AIDS and generally presents with
symptoms of leg weakness, saddle anesthesia, and urinary
retention. Toxic neuropathies may also present acutely but 12. ANSWER:A. Intracerebral hemorrhage
are generally axonal and present with distal sensory loss
and weakness with only loss of the Achilles tendon reflexes The patient most likely has had a cerebral hemorrhage. The
(rather than diffuse areflexia). classic presentation of the cerebral hemorrhage is the sud-
den onset of focal neurological deficits with subsequent
symptomatic progression over minutes to hours. Headache,
vomiting, hypertension, and an impaired level of conscious-
10. ANSWER:B. Plasmapheresis ness are its most common clinical features compared to
cerebral hemorrhage.
This patients symptoms and neurologic examination are
consistent with myasthenia gravis. Affected patients may
occasionally present in myasthenic crisis, which consists
of severe respiratory insufficiency requiring noninvasive or 13. ANSWER:C. Spontaneous left internal carotid
mechanical ventilation and/or dysphagia requiring naso- artery dissection
gastric feeding. Plasmapheresis and intravenous immuno-
globulin both rapidly improve respiratory function and The most likely clinical diagnosis to explain the neurologic
muscle strength, and either one is the treatment of choice symptoms is a left internal carotid artery dissection with
for myasthenic crisis. resultant transient ischemic attacks.
High-dose prednisone may transiently worsen symp-
toms in up to 30% of patients and may take several weeks
before resulting in clinical improvement. High-dose pred-
nisone can be initiated safely, however, if plasmapher- 14. ANSWER:E. Intravenous rtPA
esis is also being used for the initial treatment of crisis.
Pyridostigmine can be used for mild myasthenic symptoms This patient should receive intravenous TPA. He has clini-
but is generally not effective in the treatment of crisis. If cal symptoms and signs and radiographic evidence of an
patients are already taking pyridostigmine, it should be dis- acute left hemispheric stroke. The probable mechanism of
continued if they require ventilatory support because the stroke is cardioembolic due to the history of atrial fibrilla-
medication can increase oral secretions, thereby increasing tion and his INR is subtherapeutic.

9. N e u r o l o g y 19 7
15. ANSWER:C. Intravenous heparin should undergo carotid angioplasty and stenting. Although
carotid endarectomy is still considered the gold standard of
The patient has a carotid dissection, for which he should surgical therapies for patients with such stenosis, it cannot
receive heparin. Acute headache with oculosympathetic be performed in those who have stenosis that is difficult to
paresis (Horner syndrome) is common with carotid dissec- access surgically (above the C2 level), medical conditions
tion and must be assumed to be a carotid dissection until that greatly increase the risk of surgery or other specific con-
proven otherwise. ditions, such as radiation-induced stenosis or restenosis after
carotid endarcectomy, or other specific conditions, such as
radiation-induced stenosis or restenosis after carotid endar-
ectomy. For such patients, the less invasive combination of
16. ANSWER:B. Lumbar puncture carotid angioplasty and stenting is preferable. The FDA has
approved carotid angioplasty and stenting for patients with
This patient should have a lumbar puncture. Athunderclap symptomatic severe carotid artery stenosis who are classi-
headache is a severe and explosive headache that is maximal fied as high surgical risk or who have unfavorable anatomy
in intensity at work within 60 seconds of onset. CT scan- that precludes a surgical approach.
ning is the first test to be performed in patients with thun-
derclap headache in whom a subarachnoid hemorrhage is
suspected. If the initial CT reveals nothing, a lumbar punc-
ture should be performed to evaluate for xanthochromia in 20. ANSWER:A. Decompressive hemicraniectomy
the cerebrospinal fluid.
This patient has malignant brain edema as a result of her
ischemic stroke and should be treated with decompressive
hemicranectomy. Young patients with major infarctions
17. ANSWER:A. Bedside screening for dysphagia affecting the cerebral hemispheres or cerebellum have a
heightened risk of brain edema and increased intracranial
On admission to the hospital ward, a patient with stroke pressure. Reducing any edema and close monitoring for
should be kept NPO until a swallowing assessment is con- signs of neurologic worsening, particularly during the first
ducted. Dysphagia screening is appropriate for all patients 35 days after the stroke when the edema maximizes are
who have difficulty swallowing. Bedside screening of swal- recommended. Meta-analysis of randomized trials have
lowing should be completed before oral intake of any medi- shown that decompressive hemicraniectomy for malignant
cation or food because of the risk of aspiration, which can stroke reduced morbidity and mortality.
result in pneumonia and death.

21. ANSWER:A. Annual MRA


18. ANSWER:D. Withholding of all antihypertensive
medications This patient should have an annual MRA or CT angiogram
to monitor aneurysmal growth. For patients without a
For strokes without concurrent acute coronary artery dis- prior subarachnoid hemorrhage, the lowest risk aneurysms
ease or heart failure, consensus exists that antihypertinsive are those in the anterior circulation and less than 7mm in
medications, such as intravenous labetalol or nimcadip- diameter. The annual risk of rupture for an aneurysm of
ine, should be withheld if the systolic blood pressure is the size of this patients is 0.05% annually. The risk of neu-
less than 220mm Hg or the diastolic blood pressure is less rologic disability associated with intervention exceeds the
than 120 mm Hg, unless there are other manifestations potential benefit. After 3 successive years of annual moni-
of end-organ damage. This patients systolic and diastolic toring, an MRA or CTA obtained once every 3years will
blood pressure levels are below these limits. Many patients be sufficient.
have spontaneous declines in blood pressure during the first
24 hours after stroke onset.

22. ANSWER:B. Admit to hospital.

19. ANSWER:A. Carotid angioplasty and stenting This patient should be admitted to the hospital. His diag-
nosis is consistent with TIA. His ABCD2 score is elevated.
The patient who has an ischemic stroke and has asymptom- Several studies have identified risk factors for stroke after
atic severe internal carotid artery stenosis (>70% stenosis) TIA, which may be useful in making initial management

19 8 T h e B r i g h a m I n t e n s i v e R e v i e w o f I n t e r n a l M e d i c i n e Q u e s t i o n a n d A n s w e r C o m pa n i o n
decisions. Three very similar formal prediction rules have evaluation. MRI of the brain with diffusion-weighted
been developed and cross-validated in northern California imaging (DWI) should be considered superior to noncon-
and Oxfordshire. The California score and the ABCD score trast CT scan for the diagnosis of acute ischemic stroke in
both predict short-term risk of stroke well in independent patients presenting within 12 hours of symptom onset but
populations of patients presenting acutely after a TIA. The is not usually available for acute treatments.
newer ABCD2 score was derived to provide a more robust
prediction standard and incorporates elements from both
prior scores. Patients with TIA score points (indicated in
parentheses) for each of the following factors: age 60 years 27. ANSWER:B. Aspirin
(1); blood pressure 140/90 mm Hg on first evaluation (1);
clinical symptoms of focal weakness with the spell (2) or Aspirin (50 to 325 mg/d) monotherapy, the combination of
speech impairment without weakness (1); duration 60 min- aspirin and extended-release dipyridamole, and clopidogrel
utes (2)or 10 to 59 minutes (1); and diabetes (1). In combined monotherapy are all acceptable options for initial therapy.
validation cohorts, the 2-day risk of stroke was 0% for scores Aspirin is the only medication that has been examined
of 0 or 1, 1.3% for 2 or 3, 4.1% for 4 or 5, and 8.1% for 6 or 7. within 48 hours and can be given rectally until the patient
has passed the screening dysphagia exam.

23. ANSWER:B. Rebleeding


28. ANSWER:E. All of the above
The most likely complication to have caused this patients
rapid deterioration is rebleeding. In the first few hours after For secondary risk reduction measures, all measures have
an initial hemorrhage, up to 15% of affected patients have a been useful for stroke prevention.
sudden deterioration of consciousness, which strongly sug-
gests rebleeding. In patients who survive the first day, the
rebleeding risk is evenly distributed during the next 4 weeks,
with a cumulative risk of 40% without surgical or endovas- 29. ANSWER:A. Stroke
cular interventions. Occlusion of the responsible aneurysm
is thus the first aim in the management of the subarachnoid Diffusion-weighted magnetic resonance imaging
hemorrhage and is usually performed by coiling or clipping. (DW-MRI) is a highly sensitive tool for the detection of early
changes in water diffusion that characterize many brain
pathologies, including acute ischemic stroke. These changes
represent variations in the random motion of water mol-
24. ANSWER:A. Continuous EEG monitoring ecules in tissues. They are expressed, in diffusion-weighted
images, as changes in MRI signal intensity or as varia-
Continuous EEG monitoring is indicated for patients who tions in the apparent diffusion coefficient (ADC) of water.
remain unresponsive after resolution of clinical status epi- Acute brain lesions like ischemic stroke are associated with
lepticus to distinguish nonconvulsive status epilepticus reduced water diffusion, and they can be detected as bright
from a postictal state. areas of signal hyperintensity in diffusion-weighted images,
or as dark areas of signal hypointensity in ADC maps.

25. ANSWER:B. Transient ischemic attack (TIA)


30. ANSWER:A. Benztropine
TIA is a brief episode of neurologic dysfunction caused by
focal brain or retinal ischemia, with clinical symptoms typi- Medications that block dopamine receptors can cause
cally lasting less than 1 hour and without evidence of acute acute dystonic reactions, which can be readily treated with
infarction. benztropine.

26. ANSWER:B. CT of the brain and CTA of the


head and neck 31. ANSWER:A. Initiate no drug therapy at this time.

The CT of the brain and CTA of the head and neck still Unless special circumstances exist, drug therapy is generally
remain the most practical cerebrovascular imaging for not started in patients with a single unprovoked seizure.

9. N e u r o l o g y 19 9
32. ANSWER:C. Glucose tolerance test greater than 50% risk of developing a secondary progressive
disease course.
A history of burning or lancination distal extremity pain
and examination findings showing only sensory loss sug-
gest a small fiber peripheral neuropathy, which is most
frequently associated with diabetes mellitus and impaired 38. ANSWER:B. Continue lifelong treatment.
glucose tolerance.
Juvenile myoclonic epilepsy requires lifelong antiepileptic
drug therapy.

33. ANSWER:C. Voltage-gated P/Q type calcium


channel antibody level
39. ANSWER:B. Drug-induced parkinsonism
The diagnosis of Lambert Eaton myasthenic syndrome, a
neuromuscular junction disorder that causes progressive Drug-induced parkinsonism is a potential complication of
proximal muscle weakness and areflexia, precedes the clini- dopamine-blocking medications, including metaclopramide.
cal recognition of cancer in up to 50% of patients. This diagnosis was based on the history of the patient.
Patients with Lewy body disease usually present with hallu-
cinations or extreme mental status changes. Multiple-system
atrophy and Parkinsons disease can be very similar and can
34. ANSWER:D. MRI of the brain and MRA of the only be diagnosed after medical effect has been ruled out.
head and neck

Trigeminal neuralgia usually presents after 40years, and its


diagnosis in a younger patient should prompt an evaluation 40. ANSWER:D. Epidural metastases
for secondary cause, such as multiple sclerosis, posterior
fossa tumors, and vascular or aneurysmal compression of New and progressive symptoms and progressive symptoms
the trigeminal nerve. referable to the spinal cord represent a neurologic emer-
gency that should prompt evaluation for a compressive
lesion. It is not an epidural abscess because there is no fever
or elevated ESR or CBC. Epidural hematoma is not likely
35. ANSWER:A. Plasma exchange because there is no coagulopathy or history of recent surgi-
cal procedure in the affected area.
Myasthenic crisis, a potentially life-threatening neuro-
logic emergency characterized by muscle weakness severe
enough to necessitate intubation, typically requires plasma
exchange therapy. 41. ANSWER: C. Physical and occupational therapy

Therapy for critical illness myopathy, a frequent complica-


tion in sepsis or multiorgan failure that can cause failure
36. ANSWER:C. Migraine without aura to wean from mechanical ventilation and limb weakness,
involves physical and occupational rehabilitation therapies
Approximately 90% of patients who present in an ambula- and avoidance of corticosteroids. There is no effective treat-
tory care setting with chief symptoms of recurrent, moder- ment other than physical and occupational therapy.
ately severe headache have a form of migraine.

42. ANSWER:C. Early-onset familial Alzheimer


37. ANSWER:C. Relapsing remitting dementia

Multiple sclerosis begins as a relapsing remitting disorder The presenilin-1 mutation is specific for early-onset famil-
in 85% of patients and a primary progressive disorder in ial Alzheimer dementia. At this time, this is not a rou-
15% of patients. Those with relapsing remitting type have a tine clinical test and still considered as research. Patients

2 0 0 T h e B r i g h a m I n t e n s i v e R e v i e w o f I n t e r n a l M e d i c i n e Q u e s t i o n a n d A n s w e r C o m pa n i o n
with vascular dementia have a history of strokes or TIAs. 46. ANSWER:C. Methylprednisolone, intravenously
Frontotemporal dementias usually present with behavioral
problems such as disinhibition. This is not Parkinsons dis- Multiple sclerosis relapses may resolve more rapidly with
ease because there is no movement disorder. intravenous methylprednisolone therapy.

43. ANSWER: C. Paroxysmal hemicrania 47. ANSWER:C. Discontinue the oral contraceptive
pill.
The various trigeminal autonomic cephalalgias, character-
ized by pain referred to the first division of the trigeminal Oral contraceptive pills are contraindicated in women with
nerve and accompanying cranial autonomic symptoms, migraine with aura, especially if the aura involves more
can be distinguished by the duration and frequency of than just simple visual aura, if there are additional stroke
each attack. Paroxysmal hemicrania attacks are of shorter risk factors, or if the aura begins after the initiation of oral
duration as compared to cluster headaches (cluster head- contraception.
aches usually >15 minutes to 3 hours in duration). Short-
lasting unilateral neuralgiform headache with conjunctival
injection and tearing (SUNCT) is a rare type of primary
headache that belongs to the group of headaches called tri- 48. ANSWER:A. Antiphospholipid antibody
geminal autonomic cephalalgia (TACs). TACs are caused syndrome
by activation of the autonomic nervous system of the tri-
geminal nerve in the face. Patients experience excruciat- Chorea can occur in patients with antiphospholipid anti-
ing burning, stabbing, or electrical headache mainly in the body syndrome or systemic lupus erythematosus. There are
orbital area only on one side of the body along with cranial mental status changes in Huntingtons disease. There is no
autonomic signs that are unique to SUNCT. Each attack chorea with Parkinsons disease. The movement disorder of
can last from 5 seconds to 6 minutes and may occur up to Wilsons disease is more proximal compared to distal.
200 times daily. Onset of the symptoms usually come later
in life, at an average age of about 50. Although the major-
ity of patients are males above age 50, it is not uncommon
to find SUNCT present among other age groups, including 49. ANSWER:E. Todd paralysis
children and infants.
After a single unprovoked seizure, a greater risk of recur-
rence is predicted if the event was a partial seizure of if the
patient has Todd paralysis, stratus epilepticus on presenta-
44. ANSWER:A. Levetiracetam tion, an age greater than 65years, or abnormal findings on
neurological examination.
For patients who should avoid hepatically metabolized
antiepileptic drugs, either because of drug interaction or
underlying liver disease, levetiracetam, gabapentin, and
pregabalin can be used. 50. ANSWER: B. Multiple-system atrophy

Multiple-system atrophy is a sporadic, hetrogenous, neuro-


degenerative disorder that causes impairment of multiple
45. ANSWER:B. Measurement of serum copper and neurologic systems, including the autonomic nervous sys-
zinc levels tem, the extrapyramidal system, and the cerebellum.

Vitamin B12 and copper deficiencies are associated with


malabsorption syndromes related to gastric bypass surgical
procedures and can cause anemia and a syndrome of pro-
gressie myeloneuropathy.

9. N e u r o l o g y 2 01
10.
GENER A L INTER NA L MEDICINE

Nikhil Wagle, Christopher Gibson, Ami Bhatt, Molly L.Perencevich, William Martinez,
JasonOjeda, Rose Kakoza, and Lindsay King

1. A55-year-old African American male comes for his cytological result is atypical squamous cells of undeter-
first primary care office visit. He is without complaints mined significance (ASC-US). Reflex DNA testing for
but presents because his father had a myocardial infarc- high-risk human papillomavirus (HPV) is performed
tion recently at age 75, and his mother is healthy at 74. He and is negative.
does not smoke. His blood pressure is 120/71mm Hg on What is the best next step in management?
hydrochlorothiazide. His total cholesterol is 200 mg/dL,
and is HDL is 25 mg/dL. His 10-year risk for develop- A. Refer for colposcopy.
ing atherosclerotic cardiovascular disease, according to B. Repeat cytology in 6months.
Pooled Cohort Equations, is 11.9%. C. Repeat cytology in 1year.
Which of the following is the most appropriate D. Repeat cytology in 3years.
management? E. Refer for an endometrial biopsy.

A. Recommend lifestyle change; follow-up in 6months 4. A35-year-old female presents to the clinic with her
B. Start statin, husband due to difficulty becoming pregnant. The
C. Obtain Coronary Calcium Score, couple has been trying to become pregnant for the last
D. Refer for cardiac stress test, 6 months without success despite having intercourse
E. Refer for cardiac catheterization, 3 times per week consistently. Her periods are usu-
ally regular on a 26-day cycle. Her partner had semen
2. A 54-year-old woman with a remote history of analysis, which was normal. Her physical examination
migraines (but none in years) presents to her primary is unremarkable.
care physician with a new-onset headache. The pain is What is the most appropriate next step in
retro-orbital on the right side only and has been pro- management?
gressive for 3 weeks with intermittent responsiveness to
acetaminophen. She denies visual changes, fevers, chills, A. Reassurance and workup in 6months if still not
jaw claudication, or weakness. Her examination is nor- pregnant
mal, including thorough neurological examination. B. Day 20 serum progesterone
What is the most appropriate next step in C. Hysterosalpingogram
management? D. Referral for in vitro fertilization
E. Day 20 LH and FSH
A. Prescribe oxycodone.
B. Order a head MRI/MRA. 5. A30-year-old female presents to the clinic complain-
C. Prescribe amitriptyline. ing of bilateral breast pain. She states that the pain is
D. Prescribe sumatriptan. worse toward the end of her menstrual cycle and also
E. Refer to physical therapy. in the evening. She has had the pain for approximately
3 months and describes it as an aching, diffuse pain.
3. A 25-year-old monogamous woman with no past Her examination is notable for a BMI of 28kg/m2, pen-
medical history undergoes her annual Pap smear. The dulous breasts without erythema, masses, or discharge.

202
She has no lymphadenopathy or skin changes. She is a basic metabolic, complete blood count, live function
concerned she may have cancer and tells you that her test, and TSH are unremarkable.
maternal grandmother had cancer at age 76. What is the What is the nest most appropriate step in
best next step in management? management?

A. Bilateral mammography A. Refer to Psychiatry for psychogenic itch.


B. Bilateral breast MRI B. Start an antihistamine at night.
C. Bilateral breast ultrasound C. Start gabapentin.
D. BRCA testing D. Obtain a chest X-ray and HIV test.
E. Reassurance and a more supportive bra
9. A69-year-old male is found to have an alkaline phos-
6. A 31-year-old male with a family history of phatase level of 365 U/L. Radiographs and radionuclide
Hashimotos thyroiditis presents to the clinic for evalu- bone scan are consistent with Pagets disease of the bone
ation of hair loss. He states that he noticed a small bald involving the left humerus, right ischium, and several
spot on his occiput several months ago. Hair did regrow vertebrae. The patient is asymptomatic.
in that area but was white instead of black. He now pres- What is the next best step in management at this
ents to clinic because he has three well-circumscribed time?
round areas of complete hair loss on his scalp as well as
one in his beard. He otherwise denies any complaints. A. No further workup or treatment
Thyroid-stimulating hormone (TSH) was within nor- B. Start an oral bisphosphonate.
mal range. C. Start an intravenous bisphosphonate.
What is the best next step in management? D. Start calcitonin.
E. Refer for orthopedic surgery.
A. Intradermal injection of triamcinolone
B. Minoxidil 2% solution applied BI 10. A 28-year-old woman calls the office with symptoms
C. Biopsy of the bald area of dysuria, urgency, and frequency. She had a recent uri-
D. Ketaconazole shampoo nary tract infection a few weeks ago that was treated with
trimethoprimsulfamethoxazole with resolution of her
7. A 27-year-old female presents for evaluation of sec- symptoms. Urine cultures were not sent at that time.
ondary amenorrhea. She had onset of menarche at age What is the next most appropriate step in
15 and had regular periods up until a few years ago. management?
She denies any other symptoms and states that she runs
approximately 1015 miles a day. There is no evidence of A. No further workup or treatment
an eating disorder. Her pregnancy test is negative, and B. Prescribe a 3-day course of
her prolactin and TSH are normal. Her FSH, LH, and trimethoprimsulfamethoxazole.
estradiol levels are in the low-normal range. Her BMI C. Prescribe a 3-day course of amoxicillin.
is 22kg/m2, and her examination is unremarkable. The D. Prescribe a 5-day course of nitrofurantoin.
female athlete triad is suspected, and a bone density E. Prescribe a 3-day course of ciprofloxacin.
shows significant bone loss.
What is the best treatment for the patient? 11. A 62-year-old man with a history of gout on allo-
purinol 300 mg presents after a recently inflamed first
A. Start oral contraceptive pills. metatarsal joint. A joint aspiration showed monoso-
B. Start a bisphosphonate. dium urate crystals within neutrophils. He is currently
C. Encourage increasing exercise along with calcium asymptomatic, but this is his second gout flare in the
and vitamin D. last 3 months. He has a history of hypertension and is
D. Start leptin. being treated with lisinopril. He states that he is taking
all of his medications regularly. His serum creatinine is
8. A43-year-old man presents to the clinic with several 0.9 mg/dL and serum uric acid is 8.3 mg/dL.
months of diffuse itch. He states that the itching has What is the next best step in management?
been progressive over the last several months and is not
interfering with his sleep. He has not noted any rash. On A. No further workup or treatment
examination, there is evidence of scattered excoriations B. Increase allopurinol with concurrent colchicine.
and some areas of lichen simplex chronicus. He is not C. Colchicine alone
on any medications and has not seen a doctor for many D. Switch allopurinol to febuxostat.
years but has no notable past medical history. Results of E. Swtich allopurinol to probenacid.

10 . G e n e r a l I n t e r n a l M e d i c i n e 2 0 3
12. A 37-year-old man presented to his primary care minute. Her pelvic examination shows copious muco-
physician with epigastric discomfort and was found to purulent discharge from a red, inflamed cervix. She
be H. pylori positive. He did not have melena, weight has tenderness on palpation of cervix but no adnexal or
loss, or any other concerning features. He was treated uterine tenderness. Apregnancy test is negative and you
with omeprazole 20 mg twice daily, amoxicillin 1g send a GC/chlamydia probe. A wet prep shows white
twice daily, and clarithromycin 500 mg twice daily for cells; KOH is negative.
10 days. He now represents with the same epigastric What is the next management step?
discomfort.
What is the next best step in management? A. Ceftriaxone 125 mg intramuscular injection 1
B. Ceftriaxone 250 mg intramuscular injection 1
A. Empirically treat with the same regimen for 14days. C. Ceftriaxone 125 mg intramuscular injection 1 +
B. Empirically treat with omeprazole 20 mg twice daily, azithromycin 1 g orally 1
tripotassium dicitratobismuthate 120 mg four times D. Ceftriaxone 250 mg intramuscular injection 1 +
per day, tetracycline 500 mg four times daily, and azithromycin 1 g orally 1
metronidazole 250 mg four times per day. E. Doxycycline 100 mg orally twice daily 7 days
C. Order a repeat serology to confirm persistence of
H.pylori. 16. A 35-year-old male presents to urgent care with a
D. Order a urea breath test to confirm persistence of 2-week history of worsening purulent rhinorrhea and
H.pylori. right-sided maxillary sinus pain. His symptoms have
persisted despite Tylenol, fluticasone propionate, and
13. A 31-year-old monogamous woman with no past saline irrigation. Examination is notable for a temper-
medical history presents for routine Papanicolaou (Pap) ature of 101.5F, copious rhinorrhea, and tenderness
smear. The cytological result is negative for intraepi- with percussion over maxillary sinuses. Cardiac and
thelial lesion or malignancy. Reflex DNA testing for pulmonary exams are unremarkable.
high-risk human papillomavirus (HPV) is performed He takes hydrochlorothiazide 25 mg daily and amlo-
and is negative. dipine 5 mg daily for hypertension. He has no known
What is the next step in management? drug allergies.
What is the most appropriate next step in
A. Repeat cytology in 6months. management?
B. Repeat cytology in 1year.
C. Repeat cytology in 2years. A. Amoxicillin 500 mg three times daily 57 days
D. Repeat cytology in 3years. B. Amoxicillin/clavulanate 500/125 mg three times
E. Repeat cytology in 5years. daily 57 days
C. Amoxicillin/clavulanate 2 g/125 mg extended
14. A 22-year-old woman sustained a compound tib- release twice daily 57 days
ial fracture during a motor vehicle accident requir- D. Doxycycline 200 mg once daily 57 days
ing surgery. Her hospital course was complicated by a E. Levofloxacin 100 mg twice daily 57 days
lower-extremity deep venous thrombosis (DVT). She
has no family history of pulmonary embolism or DVT 17. A34-year-old woman is seen in the emergency depart-
and a hypercoaguable workup is negative. She is not ment with confusion, malaise, and lower-extremity
taking any medication. rash. Past medical history is notable for allergic rhini-
She is started on low molecular weight heparin fol- tis. Medications include Loratadine and Flonase.
lowed by warfarin and achieves a target INR of 2.5. The patient is alert and oriented to self only. Physical
How long should she be anticoagulated? examination is notable for jaundice and bilateral
lower-extremity petechiae.
A. 1months Laboratory data are notable for a hemoglobin of
B. 3months 7 g/dL, reticulocyte count of 15%, and platelets of
C. 612months 45,000/L. LDH is 1,500 mg/dL. Coagulation studies
D. Lifelong anticoagulation are normal. Her serum creatinine is 3.6 mg/dL.
A peripheral smear is shown in Figure 10.1.
15. A 19-year-old woman presents with 3 days of Which of the following is the best next step?
increasing vaginal discharge. She is sexually active with
one partner. Her last menstrual period was 4days ago. A. Intravenous immune globulin
Physical examination shows temperature 99F, blood B. ANA test
pressure 100/60mm Hg, and heart rate of 90 beats per C. Plasma exchange

2 0 4 T h e B r i g h a m I n t e n s i v e R e v i e w o f I n t e r n a l M e d i c i n e Q u e s t i o n a n d A n s w e r C o m pa n i o n
Which of the following diseases is most likely to be
the cause of the patients presentation?

A. Granulomatosis with polyangitis (formerly


Wegeners granulomatosis)
B. Temporal arteritis
C. Schistosomiasis
D. Eosinophilic granulomatosis with polyangiitis
E. Hodgkins disease

21. An 18-year-old man presents for a physical examina-


tion prior to joining his college basketball team. AII/
VI crescendo-decrescendo murmur without radiation
is heard at the left lower sternal border on cardiac exam-
Figure 10.1 Peripheral blood smear for patient in Question 17. ination. The murmur increases with Valsalva maneuvers
and there is an extra heart sound preceding S1.
What is the most likely underlying etiology?
D. Transthoracic echocardiogram
E. Direct antiglobulin (Coombs) test A. Congenital aortic stenosis
B. Marfans syndrome
18. A42-year-old man with AIDS (CD4 188 cells/L) C. Hypertrophic cardiomyopathy
presents with new headaches. CT scan of the head D. Early-onset hypertension
reveals two ring-enhancing lesions with mass effect. E. Rheumatic heart disease
Which of the following is least likely to be the etio-
logic agent? 22. A 22-year-old male college football player with-
out a significant past medical history presents to the
A. Toxoplasmosis emergency room with a small abscess on his neck. The
B. Primary central nervous system lymphoma collection is drained and a Gram stain demonstrates
C. Progressive multifocal leukoencephalopathy Gram-positive cocci in clusters.
D. Tuberculosis What treatment would you prescribe?
E. Staphylococcus
A. Oral vancomycin
19. A51-year-old male from Western Massachusetts pres- B. Dicloxacillin
ents to his primary care physician with a fever and rash of C. Oral trimethoprim-sulfamethoxazole
2 weeks duration. He has no other symptoms. On exami- D. Oral penicillin
nation the rash is consistent with erythema migrans. His E. Intravenous nafcillin
Lyme ELISA and Western blot tests are positive.
What is your next best step in management? 23. A 23-year-old gentleman with a history of mild
asthma presents for evaluation of intermittent dyspha-
A. Doxycycline 200 mg 1 gia for solid foods. He denies heartburn. He has not
B. Azithromycin 500 mg 1day then 250 mg 4days lost weight. He reports that 1 year ago he underwent
C. Amoxicillin 500 mg three times per day 2 weeks an upper endoscopy for removal of pieces of steak. He
D. Doxycycline 100 mg twice daily 2 weeks has taken 40 mg of omeprazole twice daily for the last
E. Ceftriaxone 2 g IV daily 3 weeks month, but his symptoms have persisted.
His physical examination was unremarkable. An
20. A68-year-old woman presents to her primary care upper endoscopy revealed circular rings in the mid
doctor with parasthesias in her right foot, progressive esophagus. A biopsy showed a dense eosinophilic
right-sided foot drop, and erythematous rash over both infiltrate.
lower extremities. Five months ago she had cough, spu- Which of the following is the most appropriate
tum production, and chest X-ray with patchy infiltrates, first-line therapy?
prompting treatment with antibiotics. She was diag-
nosed with asthma 2years earlier. Laboratory data show A. Increase the omeprazole to 80 mg twice daily.
white blood cell count 9,600/L, hematocrit 36.2%, B. Esophageal dilation
platelets 271,000/L, with 58% neutrophils, 9% lym- C. Topical swallowed fluticasone
phocytes, and 31% eosinophils. D. Oral nifedipine

10 . G e n e r a l I n t e r n a l M e d i c i n e 2 0 5
E. Botulinum toxin injection into the lower esophageal D. Anti-citrullinated protein antibody
sphincter E. No additional studies are needed.

24. A 55-year-old obese (BMI 33 kg/m2) woman with 27. A 68-year-old man with a history of hypertension
a history of hyperlipidemia and hypertension presents and gout presents for his routine annual exam. He has
for follow-up following a recent liver biopsy for evalu- no complaints. He was a past smoker for 20 years but
ation of persistently abnormal aminotransferases. She quit 30years ago. He drinks one glass of red wine daily.
has no history of heavy alcohol use. Her medications He goes on long walks daily for exercise. He gets his flu
include atorvastatin and HCTZ. Her ALT was 85 IU/L shot annually and he received his pneumococcal vaccine
(ref 035) and AST was 66 IU/L (035 U/L). Viral sero- 3years ago. He had a normal colonoscopy 7years ago.
logic testing and autoimmune markers were negative. He is on amlodipine and allopurinol. His vital signs are
An abdominal ultrasound revealed hepatic steatosis. normal and his physical examination is unremarkable.
She underwent a liver biopsy that showed steatosis with Which of the following screening tests is most appro-
inflammation consistent with steatohepatitis and stage priate for this patient based on most evidence of benefit?
2 fibrosis.
In addition to weight loss, which of the following A. Coronary calcium CT imaging
would you recommend? B. Prostate-specific antigen
C. Thyroid-stimulating hormone
A. Pioglitazone D. Abdominal ultrasound
B. Vitamin E E. Exercise treadmill test
C. Metformin
D. No additional therapy 28. A 20-year-old woman with a history of systemic
E. Refer for bariatric surgery. lupus erythematosus (SLE) diagnosed 2 years earlier
presents to the emergency department with fatigue and
25. A35-year-old gentleman presents to his primary care fevers to 100.5F for several days.
physician for a routine physical exam. His only medi- Home medications include metoprolol succinate
cal history includes seasonal allergies for which he uses 25 mg daily, lisinopril 10 mg daily, prednisone 15 mg
intranasal fluticasone. He does not smoke cigarettes or daily, hydroxychloroquine 200 mg daily, azathioprine
drink alcohol. There is no family history of colorectal 50 mg daily, and dapsone 100 mg daily.
cancer, although his mother did have two adenomatous Initial evaluation reveals a young woman in no acute
polyps at age 55. distress. Vital signs are notable for a temperature of
When should he undergo his first screening 100.5F, heart rate of 60 beats per minute, blood pres-
colonoscopy? sure 110/70mm Hg, respiratory rate of 16 breaths per
minute, and an oxygen saturation of 80% on room air.
A. Now Oxygen saturation increases to 88% on 100% oxygen by
B. Age 40years nonrebreather mask. Chest X-ray and CT scan of the
C. Age 45years chest are unremarkable. Complete blood count reveals
D. Age 50years a white blood cell count of 5420/L, Hct of 29.5% (at
her baseline), and platelet count of 273/L. Chemistry
26. A 70-year-old woman presents with pain in her panel is unremarkable.
hands and wrists for 9 months. Her hands are stiff in Oxygen saturation remains 88% on 100% oxygen by
the morning for 15 minutes. She has pain with sewing nonrebreather mask; simultaneous arterial blood gas
and typing. She has not noticed swelling or warmth. shows a pH of 7.38, PCO2 32mm Hg, and PO2 527mm
Her vital signs are normal. Her bilateral proximal inter- Hg on 100% oxygen.
phalangeal joints are tender to palpation and have bony The next best step in management is:
enlargements. The first carpometacarpal joints are also
tender and the palms appear square due to malalign- A. Endotracheal intubation and mechanical ventilation
ment of the thumb base. Her metacarpal squeeze test is B. Transfuse 2 units packed red blood cells (PRBCs)
negative. The remainder of her examination is normal. C. Treat with hyperbaric oxygen.
Which of the following studies should be done to D. Stop the dapsone.
establish the diagnosis? E. Treat with bactrim and prednisone.

A. ANA 29. A37-year-old woman with no significant past medi-


B. Erythrocyte sedimentation rate cal history presents to the emergency department with
C. Bilateral X-rays of the hands 2 days of nausea, vomiting, and abdominal pain. Her

2 0 6 T h e B r i g h a m I n t e n s i v e R e v i e w o f I n t e r n a l M e d i c i n e Q u e s t i o n a n d A n s w e r C o m pa n i o n
(A) (B)

Figure10.2 (A) Noncontrast head computed tomography; (B)axial T2-FLAIR magnetic resonance imaging.

only medication is acetaminophen, which she has been he seemed not quite himself and was walking into
taking for low back pain. She has not been taking any walls, prompting her to bring him into the emergency
calcium supplements. department. In the emergency department, he under-
Laboratory tests reveal calcium 15.4 g/dL, phosphate goes head imaging, as shown in Figure 10.2A and B.
4.9 g/dL, and creatinine 2.0 mg/dL. Her PTH level is What are the most likely diagnosis and best next
low and her PTH-RP is undetectable. management choice?
All of the following would be appropriate initial
therapy in the acute setting except: A. Meningitis:treatment with ceftazidime,
vancomycin, and micafungin
A. Hydration with normal saline B. Meningitis:treatment with ceftriaxone, vancomycin,
B. Zolendronate 4 mg IV and ampicillin
C. Furosemide 40 mg IV C. Brain abscess:treatment with ceftazidime,
D. Calcitonin 4 units/kg IM vancomycin, micafungin, and acyclovir
E. Ondansetron 1 mg IV D. Ruptured brain abscess:treatment with ceftriaxone,
vancomycin, and ampicillin, with neurosurgery
30. CT scan of the chest, abdomen, and pelvis of the consultation if symptoms do not improve with 24
patient in question 29 reveal diffuse lytic lesions in the hours of antibiotics
spine, pelvis, long bones, and ribs. No other abnor- E. Ruptured brain abscess:treatment with ceftriaxone,
malities are noted. Additional workup demonstrates a vancomycin, and ampicillin, with emergency
normal serum protein electrophoresis, normal serum neurosurgery consultation
angiotensin-converting enzyme (ACE) level, and a mildly
elevated lactate dehydrogenase (LDH) of 450 mg/dL. 32. Which of the following is not indicated in the man-
The most likely diagnosis is: agement/workup of the patient from Question 31?

A. Multiple myeloma A. Blood culture


B. Sarcoidosis B. Consideration of ventricular drainage
C. Metastatic breast cancer C. Serial lumbar punctures to evaluate opening pressure
D. Diffuse large B-cell lymphoma D. TTE with bubble study
E. Langerhans cell histiocytosis E. Careful physical examination of the sinuses and
tympanic membranes
31. A 32-year-old man with no past medical history
presents with low-grade fevers, anorexia, headache, and 33. A 30-year-old woman with ulcerative colitis and
neck stiffness of 4days duration, which started shortly autoimmune hepatitis complicated by cirrhosis, asci-
after a dental procedure. The night prior to presenta- tes, and esophageal varices presents with dyspnea and
tion, he had one episode of emesis and a worsening left-sided back pain. Abdominal US shows minimal
posterior headache. This morning his wife noticed that ascites and the chest X-ray is shown in Figure 10.3.

10 . G e n e r a l I n t e r n a l M e d i c i n e 2 0 7
(A) (B)

Figure 10.3 Chest X-ray for patient in Question 33.

Which of the following would not be appropriate in On exam, he has an unsteady gait, Rombergs sign, spas-
the evaluation and management of this pleural effusion? ticity in the bilateral lower extremities, bilateral hyper-
reflexia, and Babinskis sign. There is impaired vibration
A. Thoracentesis and position sense in the feet. Pain and temperature
B. Chest tube sensation in the lower extremities are normal. His labs
C. Diuretics reveal leukopenia, neutropenia, normocytic anemia, high
D. TIPS (transjugular intrahepatic portosystemic serum zinc, and low ceruloplasmin, B12, folate, homocys-
shunt) teine, and methylmalonic acid levels are normal.
E. Evaluation for liver transplantation Which of the following is the most likely cause of his
condition?
34. A 56-year-old woman with hypertension pres-
ents to the emergency room with abdominal pain in A. Vitamin B12 deficiency
the left lower quadrant and no bowel movements for B. Paraneoplastic polyneuropathy
several days. Her medications include lisinopril and C. Copper deficiency
omeprazole. Her vital signs, including blood pressure, D. Vitamin B6 toxicity
are normal. Her abdomen is soft, nontender, and non- E. Lead toxicity
distended. Abdominal CT suggests constipation but
is otherwise normal. The CT also shows a 3.5cm right 36. A 56-year-old woman presents with dyspnea on
adrenal lesion. exertion and fatigue for 2months. She has a history of
All of the following tests for the evaluation of this hypertension and stage 4 chronic kidney disease. She
adrenal lesion are appropriate except has no nausea, anorexia, chest pain, or orthopnea. Her
weight is stable. Her medications include furosemide
A. Dexamethasone suppression test and lisinopril. Her health care maintenance is up to
B. Plasma aldosterone and renin date, including a recent colonoscopy.
C. Plasma metanephrines On exam, her blood pressure is 118/62 mm Hg. BMI is
D. Cosyntropin stimulation test 24 kg/m2. Her conjunctivae are pale. Her cardiac and
E. Adrenal protocol CT or MRI lung exams are unremarkable. She has 1+ lower-extrem-
ity edema. Her labs reveal hemoglobin 9.6 g/dL, MCV
35. A42-year-old man with a history of morbid obesity 92, eGFR 18 mL/min/1.73m2 , ferritin 190 ng/mL, iron
status-post bariatric surgery with 75 pound weight loss 57 g/dL, TIBC 257 g/dL, and transferrin saturation
presents for a follow-up visit. He complains of 5years is 22%. Urinalysis reveals 1+ protein. Stool is guaiac
of progressive gait instability and numbness and weak- negative and treatment with erythropoietin is started.
ness in his distal extremities. His family history is unre- Four weeks later, her fatigue and exercise tolerance have
markable. He takes high doses of vitamin supplements, improved and her hemoglobin is now 12.6 g/dL and
including B-complex and zinc. transferrin saturation is 21%.

2 0 8 T h e B r i g h a m I n t e n s i v e R e v i e w o f I n t e r n a l M e d i c i n e Q u e s t i o n a n d A n s w e r C o m pa n i o n
On presentation, electrocardiogram (EKG) demon-
strates 1 mm ST segment elevations in lead I, II, and
aVL. Laboratory values are notable for a troponin-T
of 0.22g/L, normal CK and CKMB, white blood cell
count 4,100/L, Hct 22%, and platelet count 11,000/L.
LDH is elevated at 1,300 mg/dL, and PT and PTT are
normal.
A peripheral blood smear is shown in Figure 10.4.
The most appropriate initial treatment for this con-
dition is:

A. Platelet transfusion
B. Cardiac catheterization
C. Rituximab
Figure 10.4 Peripheral blood smear for patient in Question 37. D. IVIG
(Courtesy of Ed Uthman, MD.) E. Plasma exchange

38. A42-year-old man presents to the emergency depart-


What is the most appropriate next step in the man- ment 1 week after developing chest pain. One week ago,
agement of this patient? he developed severe left-sided chest pain in the setting
of cocaine use. The chest pain persisted for 1 day and
A. Stop erythropoietin. then resolved. He has not had further chest pain.
B. Stop lisinopril. His EKG in the emergency department is shown in
C. Change lisinopril to hydrochlorothiazide. Figure 10.5.
D. Add IV ferrous gluconate. Cardiac biomarkers are notable for a normal CK and
E. Schedule EGD. CK-MB and an elevated troponin at 13.2g/L.
The best next step in management is:
37. A 44-year-old man is brought to the emergency
department after brief loss of consciousness at work A. Echocardiogram
lasting for approximately 30 seconds. He has had a B. Anticoagulation with heparin
5-day history of dyspnea on exertion and chest pain. On C. Urgent cardiac catheterization
the morning of presentation, he had difficulty walking D. Clopidogrel
to work because of shortness of breath and worsening E. Pharmacologic stress test
chest pain.

Figure 10.5 Electrocardiogram for patient in Question 38.

10 . G e n e r a l I n t e r n a l M e d i c i n e 2 0 9
39. A28-year-old woman with no past medical history edema for 1 week. Her baseline blood pressures are
presents with nausea and vomiting after completing her 120140/7080. Her medications include nifedipine
first marathon. She was able to complete the marathon and omeprazole.
and thereafter immediately rehydrated. She took four On exam, she is afebrile. Her heart rate is 98. Her
200 mg ibuprofen tablets and was at a postmarathon blood pressure is 170/100. She has skin thickening
party when she started to feel ill, saying unusual things over the face, hands, arms, chest, and abdomen. There
to her friends such as I made a terrible mistake and are telangiectasias on her face and palms. Her cardiac
I am drowning. Her friends brought her to the emer- examination is notable for an S4. Her lungs are clear
gency room. On physical examination, she appears tired bilaterally. She has 2+ lower-extremity edema. Her
and is mildly confused. She has an otherwise nonfocal labs reveal hemoglobin 9.8 g/dL, platelets 95,000/L,
neurological examination. Her jugular venous pressure BUN 40 mg/dL, creatinine 2.4 mg/dL, albumin
is 6cm of water. 3.4 g/dL, urinalysis shows 2+ protein and 10 RBCs
What is the best next step in the workup and man- per high-powered field. A blood smear shows 2+
agement of this patient? schistocytes.
What is the most appropriate next step in management?
A. Administration of 1L of normal saline
B. Encouraging oral rehydration with an electrolyte A. Increase the nifedipine dose.
replacement sports drink B. Start captopril.
C. STAT electrolyte panel C. Start oral labetalol.
D. Administration of hypertonic saline at a rate of D. Begin IV methylprednisolone.
1 cc/kg per hour E. Start oral prednisone.
E. Check an ibuprofen level
43. A 36-year-old man presented to his primary care
40. A36-year-old woman with depression, mild asthma, physician with a week-long history of severe pain in his
and obesity presents with 3 weeks of a nonproductive left Achilles tendon. Over the past few days, he has also
cough. She also has paroxysms of coughing and post- developed pain and swelling in his fingers and toes (see
tussive vomiting. She denies significant wheezing. She Figure 10.6). He has been having difficulty walking and
works at a day care. She received her vaccinations as a bearing weight. Of note, 2 weeks ago, he developed a
child. Vital signs, lung exam, complete metabolic panel, week-long course of diarrhea accompanied by chills and
and chest X-ray are unremarkable. sweats following a weekend camping trip.
The best treatment at this time would be: The most appropriate treatment is:

A. Albuterol inhaler A. Ceftriaxone 1 g IV


B. Azithromycin B. Solumedrol 1,000 mg IV
C. Prednisone C. Prednisone 60 mg orally
D. Antitussive agents D. Indomethacin 50 mg orally
E. Admission to the hospital for IV antibiotics E. Observation

41. A 25-year-old man who recently moved to the 44. A 39-year-old woman of Greek descent presents to
United States from Russia presents with a headache. the emergency room after experiencing a brief loss of con-
Imaging of his head diagnoses a superior sagittal sinus sciousness while at work. Workup reveals a white blood cell
thrombosis. On exam, he was noted to be tall and thin, count 4,000/L, Hct 20%, and platelet count 207,000/L.
and to have pectus excavatum and arachnodactyly. No She notes that she had a viral syndrome 1 week ago,
murmurs were appreciated on exam. which subsequently resolved. She has no history of
Which of the following tests would you expect to be bleeding. She recently moved into a new house 3months
abnormal in this patient? ago. She notes that she has had a propensity to chew ice
for the past 1year. She has no family history of anemia.
A. Activated protein C resistance Additional workup reveals:
B. Fibrillin 1 mutation MCV 55fL
C. Protein S activity Iron < assay, ferritin 1, TIBC 400g/dL
D. Homocysteine level ESR 8mm/hour
E. Prothrombin G20210A mutation Normal haptoglobin, LDH, B12, and folate levels
Blood smear shows microcytic, hypochromic cells of
42. A38-year-old woman with a history of diffuse cuta- varying shapes. Of note, a complete blood count 3years
neous systemic sclerosis presents with lower-extremity ago showed a HCT of 28% with an MCV of 85.

210 T h e B r i g h a m I n t e n s i v e R e v i e w o f I n t e r n a l M e d i c i n e Q u e s t i o n a n d A n s w e r C o m pa n i o n
(A) (B)

Figure 10.6 Photograph of lower and upper extremity for patient in Question 43.

The most likely diagnosis is: C. Check an IgE level and consider initiation of
omalizumab (anti-IgE).
A. Thalassemia D. Encourage patient to stop marijuana usage.
B. Iron deficiency anemia E. Check for FIP1L1-PDGFR alpha translocation.
C. Lead toxicity
D. Hemolysis 47. A 64-year-old man with hypertension presents
E. Anemia of chronic disease with right upper quadrant abdominal pain for 3days.
Laboratory values demonstrate an alanine aminotrans-
45. A 19-year-old man with no significant past medi- ferase (ALT) of 927 U/mL, aspartate aminotransferase
cal history presents complaining of a new, nonproduc- (AST) of 1,048 U/L, alkaline phosphatase of 132 U/L,
tive cough of 5 months duration. He was evaluated total bilirubin of 7.8 g/dL, direct bilirubin of 5.7g/dL,
by his primary care physician when the cough began, and peripheral white blood cell count of 8,620/L,
was diagnosed with bronchitis, and was treated with a hematocrit 42.5%, platelet count 270,000/L, albumin
course of antibiotics. His symptoms did not improve 3.5 g/dL, and international normalized ratio (INR)
and the cough has worsened and is now accompanied by of 1.1. He denies acetaminophen ingestion or chronic
dyspnea and wheezing. Both the dyspnea and the cough alcohol use. Hepatitis AIgM, hepatitis B surface anti-
worsen with exertional activity. The physical examina- gen, and hepatitis B core IgM are negative. Hepatitis
tion is notable for an oxygen saturation of 91% on room C viral load is 5,041,727 IU/L with genotype 1b.
air and scattered expiratory wheezes. The peripheral Antineutrophil antibody, antismooth muscle antibody,
white blood cell count is 15,600/L with an absolute and testing for herpes simplex virus, cytomegalovirus,
eosinophil count of 1,890/L. The review of systems is and Epstein-Barr virus are negative. The right upper
notable for a 25 pound weight loss in the last 5months. quadrant ultrasound shows no cirrhosis or hepatomeg-
His social history is notable for marijuana use. aly, and patent portal and hepatic veins with normal
What is the most likely diagnosis? direction of flow. He later admits to recent IV heroin
use.
A. Asthma The best management of this patient at this time is:
B. Chronic bronchitis
C. Eosinophilic granulomatosis with polyangiitis A. Recheck hepatitis C viral load in 3months and
D. Allergic bronchopulmonary aspergillosis consider treatment if virus not cleared.
E. Invasive aspergillosis B. Peginterferon and ribavirin
C. Lamivudine
46. All of the following are reasonable next steps in the D. Peginterferon, ribavirin, and telaprevir
workup and treatment of the patient in Question 45 except E. Entecavir

A. Start prednisone at a dose of 0.51 mg/kg per day. 48. A27-year-old woman, originally from Brazil, who is
B. Check strongyloides serology. 25 weeks pregnant (G1P0) presents with dyspnea, blood

10 . G e n e r a l I n t e r n a l M e d i c i n e 211
tinged-sputum, and pleuritic chest pain. Upon physical
examination, heart rate is 137 beats per minute, blood
pressure 96/53mm Hg, and oxygen saturation 82% on
room air. Examination of the pulmonary and cardiac
systems reveals diffuse rales in bilateral lung fields and
a difficult to ausculate low-pitched diastolic rumble at
the apex. The electrocardiogram shows sinus tachycar-
dia. The chest radiograph shows diffuse bilateral infil-
trates. An echocardiogram reveals a normal ejection
fraction, a diffusely thickened mitral valve, moderate
severe mitral stenosis, and elevated pulmonary artery
systolic pressure. She is intubated for respiratory sup-
port. Fetal ultrasound was reassuring.
The most appropriate regimen for medical manage-
ment is:

A. Beta-blockers and gentle diuresis


B. Digoxin
C. Dopamine
D. ACE-inhibitors
E. Hydralazine and nitrates Figure 10.7 Photograph of patients left arm in Question 50.

49. A 29-year-old woman presents to the emergency


department with sore throat, fever, and recurrent Which of the following is the most likely diagnosis:
hematuria.
She was in her usual state of health until 1 month A. Poststreptococcal glomerulonephritis
ago, when she developed a sore throat and a fever of B. IgA nephropathy
101F. The following day, she noticed frank blood in her C. Carcinoma of the bladder
urine and went to the emergency department. She was D. Urinary tract infection
diagnosed with a presumed urinary tract infection and E. Nephrolithiasis
given a 7-day course of cephalexin. After several days of
antibiotics, her fevers resolved and her urine cleared. 50. A30-year-old woman with no past medical history
She remained in good health until 1 week ago, when she was incidentally found to have a hyperpigmented, lin-
again developed fever, sore throat, and bloody urine, ear lesion on her left arm following travel to Hawaii (see
and returned to the emergency department. She notes a Figure 10.7).
similar episode about 3years ago. Which activity mostly likely led to the development
Urinalysis was notable for 3+ blood and 2+ protein. of this physical examination finding?
Urine sediment showed 493 dysmorphic red blood cells
per high-powered field. No casts were seen. Laboratory A. Handling fish in a salt-water tank
values were notable for a serum creatinine of 3.4 mg/ B. Squeezing limes while making mojitos
dL. Complements were normal. Renal ultrasound C. Applying sunscreen containing PABA
showed no evidence of obstruction, hydronephrosis, or D. Handling thorned roses
perinephric fluid collections. E. Injection drug use

21 2 T h e B r i g h a m I n t e n s i v e R e v i e w o f I n t e r n a l M e d i c i n e Q u e s t i o n a n d A n s w e r C o m pa n i o n
C H A P T E R 10 A N S W E R S DNA testing are repeating a Pap smear at 6 months and
1year, but this is unnecessary if the HPV testing is negative.
1. ANSWER:B. Start a statin.

The joint American College of Cardiology and American


Heart Association guideline on the treatment of blood 4. ANSWER:B. Day 20 serum progesterone
cholesterol identified four major groups that would benefit
from statin therapy and include individuals with (1)clini- Reassurance is inappropriate as the patient is 35years old,
cal atherosclerotic cardiovascular disease (ASCVD), so evaluation should begin after 6months of trying (1year
(2) LDL greater than or equal to 190, (3) diabetics aged if <35). Aday 20 or 21 serum progesterone can confirm the
4075years old with an LDL > 70, and (4)an estimated presence of ovulatory cycles and should be the first step in
10-year ASCVD risk of greater than or equal to 7.5%. Based the workup of female infertility. Although a hysterosalpin-
on this guideline, this patient with a 10-year risk of 11.9% gogram is useful to confirm tubal patency, it should not
would qualify for statin therapy. Lifestyle change should be precede assessing ovulation. IVF is inappropriate without
encouraged but is insufficient to mitigate the patients risk completing a workup. LH and FSH can assess ovulatory
of ASCVD. A coronary calcium score is not necessary in reserve but would be checked on day 3, not day 20.
this patient who already has an indication for statin ther-
apy. Stress testing and catheterization are not indicated in
this asymptomatic patient.
5. ANSWER:E. Reassurance and a more supportive bra

The history is consistent with cyclic mastalgia, which may


2. ANSWER:B. Order a head MRI/MRA. be due to Coopers ligament pain due to inadequate support
of her pendulous breasts. Imaging would be overly aggres-
An MRI is indicated in this patient as she is presenting with sive as would BRCA testing.
a new headache after the age of 50years old. Imaging should
be performed in those patients presenting with:

First or worst headache 6. ANSWER:A. Intradermal injection of


Increased frequency and increased severity of headache triamcinolone
New-onset headache after age 50
New-onset headache with history of cancer or Alopecia areata typically presents before age 40 with 66% of
immunodeficiency patients presenting before 30. Alopecia areata causes rapid areas
Headache with mental status changes of well-circumscribed hair loss on the scalp but can involve
Headache with fever, neck stiffness, and meningeal signs any and all hair growth. Growth of white hair in the alopecic
Headache with focal neurologic deficits, if not previously regions is a common occurrence. Intradermal injection of tri-
documented as a migraine with aura amcinolone is the best studied and most effective treatment.
Minoxidil has been used but is of limited benefit when used
Narcotics are generally not first line for headaches; ami- alone. Biopsy is unnecessary in this classic presentation.
triptyline can be used for chronic prophylaxis for migraines
and tension headaches but should not be used prior to a
proper evaluation. Sumitriptan is a first-line therapy for
migraines, but she does not seem to be experiencing a clas- 7. ANSWER: B. Encourage increasing exercise along
sic migraine. Physical therapy is not indicated. with calcium and vitamin D.

This patient has functional hypothalamic amenorrhea from


suppression of the hypothalamic-pituitary-ovarian axis due
3. ANSWER:D. Repeat cytology in 3years. to energy imbalance. The best treatment for bone loss under
these conditions is to treat the underlying energy imbal-
HPV DNA testing is recommended to be routine for ance by nutritional rehabilitation and decreasing exercise
ASC-US results. If HPV DNA is negative for high-risk along with calcium and vitamin D supplementation. OCPs
types (16, 18), it is recommended by the American cancer will not correct the bone loss, though they would cause
society to repeat cytology in 3years (for ages 2129, 5years resumption of menses. Bisphosphonates would not be a
if >29); if HPV DNA is positive, the patient should be good option in a young premenopausal woman who may
referred for colposcopy. Alternate choices to reflex HPV desire pregnancy. Leptin has been shown to restore menses

10 . G e n e r a l I n t e r n a l M e d i c i n e 213
in functional hypothalamic amenorrhea, but its effects on symptoms. Persistence of symptoms after treatment can be
bone health are unknown. related to lack of cure but may be unrelated to H.pylori. It
is reasonable to test again if symptoms persist, but serology
will remain positive even with adequate treatment, and a
urea breath test or stool antigen must be used to confirm
8. ANSWER:D. Obtain a chest X-ray and HIV test. active infection. If found to be positive for H.pylori, qua-
druple therapy as listed in option B would be appropriate.
Patients who present with diffuse itching for more than
6 weeks without an apparent dermatologic cause warrant
investigation with basic metabolic, complete blood count,
liver function tests, thyroid-stimulating hormone, human 13. ANSWER:E. Repeat cytology in 5years.
immunodeficiency virus, and a chest X-ray to attempt to find
an underlying systemic cause of the itch. Antihistamines The USPSTF recommends cervical cancer screening every
can be helpful as can gabapentin in cases of chronic itch, 3 years for women 2165 with cytology (Papanicolaou).
but a full investigation should be performed. Women 3065 years of age who desire longer screening
intervals can be screened with a combination of cytology
and HPV testing every 5years. There is sufficient evidence
that screening with HPV testing (alone or in combination
9. ANSWER:A. No further workup or treatment with cytology) confers little to no benefit among women
younger than 30years of age.
There is no reason to treat asymptomatic Pagets disease of
the bone. When symptomatic, bisphosphonates, orally or
IV, can be effective in reducing pain due to bone turnover. 14. ANSWER:B. 3months
Calcitonin may be effective but has less evidence than a
bisphosphonate. For patients with a first episode of venous thromboem-
bolism due to a reversible or time-limited risk factor (i.e.,
oral contraceptive use, surgery/prolonged immobilization,
10. ANSWER:D. Prescribe a 5-day course of trauma, pregnancy), the treatment course is 3 months.
nitrofurantoin. Anticoagulation beyond 3months is typically not required.
Idiopathic or recurrent venous thromboembolism requires
This 28-year-old female has an uncomplicated cystitis and extended and/or lifelong therapy where the exact duration
warrants treatment to relieve symptoms. The Infectious of therapy can vary based on patient history and preference.
Disease Society of America guidelines suggest that trime-
thoprimsulfamethoxazole and nitrofurantoin are first-line
options for uncomplicated cystitis. The other agents are 15. ANSWER: D. Ceftriaxone 250 mg intramuscular
second line. Given that she has already been treated with 1 + Azithromycin 1 g orally 1
trimethoprim-sulfamethoxazole, she may have a resistance.
In 2011, the CDC increased the recommended treatment
dose of ceftriaxone for N. gonorrhea from 125 mg to 250
mg (given together with azithromycin or doxycycline).
11. ANSWER:B. Increase allopurinol with
This recommendation was in response to reports of clinical
concurrent colchicine.
treatment failures with ceftriaxone 125 mg intramuscular
1 and the finding that some isolates had decreased suscep-
This 62-year-old male is having continued attacks of gout
tibility to cephalosporins (e.g., increased minimal inhibi-
while on allopurinol, but his serum uric acid level is higher
tory concentration). Note that because of the increased
than the goal level. His allopurinol should be increased to
prevalence of N. gonorrhea resistance to doxycycline when
bring his serum uric acid down to less than 6.0 mg/dL. He
compared to azithromycin, azithromycin is the preferred
should be on colchicine or an NSAID while the allopurinol
agent for treatment of potential chlamydia infection.
is increased. There is no reason to switch medications.

12. ANSWER:D. Order a urea breath test to confirm 16. ANSWER: B. Amoxicillin/clavulanate 500/125
persistence of H.pylori. mg three times daily 57 days

It is reasonable to use a test and treat strategy for H.pylori Acute bacterial rhinosinusitis is characterized by at least
in patients with epigastric discomfort without any red flag one of the following three symptoms:persistent symptoms

214 T h e B r i g h a m I n t e n s i v e R e v i e w o f I n t e r n a l M e d i c i n e Q u e s t i o n a n d A n s w e r C o m pa n i o n
(710 days), severe symptoms (fever 102F, purulent leukoencephalopathy (PML), HIV encephalopathy, and
nasal discharge or facial pain from onset lasting 34days) CMV encephalitis are conditions generally associated
or initial improvement followed by subsequent worsening of with nonenhancing central nervous system lesions. PML
symptoms. The 2012 Infectious Disease Society of America is the only etiology that would not cause a ring-enhancing
guidelines recommend amoxicillin-clavulanate as first-line lesion and is not typically associated with mass effect.
therapy, instead of amoxicillin alone. High-dose augmentin CNS lesions with mass effect are most common in severely
is indicated in those at high risk (age >65years, immuno- immunocompromised patients with CD4 count <200.
compromised, recent antibiotic use, recent hospitalization, In patients with CD4 count >500, etiologies are similar
high prevalence of resistant S. pneumonia). Doxycycline to non-HIV-infected individuals, specifically benign and
and levofloxacin are recommended for patients with a peni- malignant brain tumors, including metastases.
cillin allergy; otherwise amoxicillin/clavulanate is the rec-
ommended first-line treatment.

19. ANSWER: D. Doxycycline 100 mg twice daily 2


weeks
17. ANSWER:C. Plasma exchange
The treatment of choice for early Lyme disease is doxy-
This is a case of thrombotic thrombocytopenic purpura cycline 1021 days as it will also treat anaplasmosis.
hemolytic uremic syndrome (TTP-HUS). The smear is Amoxicillin is an alternative in pregnant women and
microangiopathic, demonstrating schistocytes (>5/hpf) and children <8 years of age. Intravenous third-generation
thrombocytopenia. The diagnosis of TTP-HUS is a clinical cephalosporins are only required in advanced heart block,
one; however, the presence of microangiopathic hemolytic meningitis, neuritis, or other more serious consequences. In
anemia and thrombocytopenia without other clear cause is Europe, studies have found azithromycin to be as effective
required for diagnosis and is sufficient, in this case, to make as doxycycline. However, in the United States studies have
an initial diagnosis. The additional findings of renal failure, found azithromycin not to be as effective as amoxicillin (and
neurologic abnormalities, and fever constitute the remaining presumably doxycycline) in the treatment of Lyme disease.
classical clinical and laboratory features of TTP-HUS. Plasma
exchange is the treatment of choice for TTP-HUS, and early
initiation is key to decrease patient morbidity. See Figure 10.8.
20. ANSWER:D. Eosinophilic granulomatosis with
polyangiitis

18. ANSWER:C. Progressive multifocal This patient displays the classic triad for eosinophilic gran-
leukoencephalopathy ulomatosis with polyangiitis: asthma, sinus disease, and
peripheral eosinophilia. The diagnosis is made primarily by
Toxoplasmosis, CNS lymphoma, and tuberculosis are clinical features, including asthma (particularly late onset),
generally associated with ring-enhancing central nervous mononeuropathy (including multiplex), or polyneuropa-
system lesions with mass effect. Progressive multifocal thy, skin disease:variable rash patterns, seen in two-thirds
of patients, paranasal sinus abnormalities (allergic rhinitis,
recurrent sinusitis, nasal polyposis), eosinophilia:>10% or
> 1,500 eosinophils/l, and migratory or transient pulmo-
nary opacities on X-ray.

21. ANSWER:C. Hypertrophic cardiomyopathy

This murmur is characteristic of hypertrophic cardiomy-


opathy (HCM)specifically the increase with Valsalva
maneuvers and lack of radiation to the carotids. HCM is an
autosomal dominant disorder of the cardiac sarcomere and
carries an increased risk of sudden cardiac death.
Aortic stenosis also has a crescendo decrescendo murmur
but decreases or does not change significantly with Valsalva
Figure10.8 Peripheral blood smear for patient in Answer 17. maneuvers. The murmur associated with aortic stenosis

10 . G e n e r a l I n t e r n a l M e d i c i n e 215
also characteristically radiates to the carotids. Marfans syn- Patients with metabolic syndrome fall into this category.
drome can be associated with aortic insufficiency, which is Weight loss is the mainstay of therapy for NAFLD. Vitamin
a diastolic murmur. Rheumatic heart disease can cause sys- E has been shown to improve liver histology in nondiabetic
tolic murmurs from aortic stenosis or mitral regurgitation. patients with biopsy-proven nonalcoholic steatohepatitis
However, the murmur described in this case is not consis- (NASH). Pioglitazone has been associated with improve-
tent with either of these valvular disorders. ment in some histologic changes of NASH. However, in the
large placebo-controlled trial, it was not superior to placebo.
Metformin has not been shown to affect liver histology or
disease progression.
22. ANSWER:C. Oral trimethoprim- Bariatric surgery is not used specifically to treat NASH
sulfamethoxazole at present. However, bariatric surgery is recommended for
patients with obesity and a BMI > 40 or a BMI > 35 and
Community-acquired MRSA (CA MRSA) is a common serious medical comorbidities.
cause of skin and soft tissue infections. Outbreaks of CA
MRSA have been seen in particular groups such as athletes
and intravenous drug users, but it should be considered in
anyone presenting with a soft tissue infection. This patient 25. ANSWER:B. Age 40years
has a characteristic presentation and appropriately had a
Gram stain and culture performed. For average risk persons, the preferred method of colon
CA MRSA is not susceptible to beta lactam antibiotics cancer screening is a colonoscopy beginning at age 50.
such as nafcillin, penicillin, and dicloxacillin. Systemically Patients with a family history of adenomatous polyps in a
ill-appearing patients with CA MRSA can be treated with first-degree relative should have a screening colonoscopy at
intravenous vancomycin. There is no role for oral vanco- age 40 or 10years before the diagnosis of adenomatous pol-
mycin besides Clostridium difficile infections because it is yps in the family member, whichever comes first.
not absorbed. When treating suspected CA MRSA in an
outpatient setting, antibiotic options include clindamycin,
trimethoprim-sulfamethoxazole, or a long-acting tetracy-
cline (e.g., minocycline or doxycycline). 26. ANSWER:E. No additional studies are needed.

This patient has first carpometacarpal joint tenderness and


squaring of the palm, consistent with osteoarthritis (OA).
23. ANSWER:C. Topical swallowed fluticasone While symmetric, polyarticular arthritis is typical of rheu-
matoid arthritis (RA), the joints involved are not consistent
The patient has eosinophilic esophagitis. The diagnostic cri- with RA, and there are no signs of inflammation on exam.
teria include clinical symptoms of esophageal dysfunction, The DIP joints are commonly involved in osteoarthritis of
biopsy showing >15 eosinophils/HPF, lack of responsiveness the hand and rarely involved in RA. Involvement of the
to high-dose PPI, and normal pH monitoring in the distal first carpometacarpal joint is almost always a sign of osteo-
esophagus. Men are affected more commonly than women. arthritis. Morning stiffness less than 30 minutes indicates
Adults with this disorder usually give a history of inter- degenerative rather than inflammatory arthritis. Answers
mittent dysphagia and food impaction. First-line therapy A through D are part of the workup for inflammatory
includes topical corticosteroids. Fluticasone propionate at a arthritis involving the hands. However, in this case, the
dose of 440500g twice daily administered for 46 weeks diagnosis of OA in the hands can be made without addi-
leads to clinical and histologic improvement. Other thera- tional testing (Answer E).
pies include elimination diets, oral budesonide, and leukot-
riene receptor antagonists. Dilation is reserved for those
patients with fixed strictures who do not respond to medical
therapy as there is a risk of esophageal tearing with dilation. 27. ANSWER:D. Abdominal ultrasound

The United States Preventive Services Task Force (USPSTF)


recommends that men between the ages of 65 and 75 with
24. ANSWER:B. VitaminE any current or past history of smoking undergo a one-time
screening for abdominal aortic aneurysm (AAA) with an
This patient has nonalcoholic fatty liver disease (NAFLD). abdominal ultrasound (Answer D). Several studies have
A biopsy is recommended in patients with NAFLD and demonstrated a survival benefit to screening, including a
increased risk of steatohepatitis and advanced fibrosis. population-based study of over 67,800 men aged 65 and 74

216 T h e B r i g h a m I n t e n s i v e R e v i e w o f I n t e r n a l M e d i c i n e Q u e s t i o n a n d A n s w e r C o m pa n i o n
who were randomized to AAA screening (with surgery for Loop diuretics such as furosemide (Answer C) are no
those found to have AAA > 5.4cm) or no screening. This longer recommended to increase calcium excretion. In the
study showed AAA-related mortality was reduced by an past, a loop diuretic was initiated routinely to increase uri-
average of 42% (95% CI, 22%58%) in the screened popu- nary calcium excretion. However, this has fallen out of favor
lation compared with the unscreened population. because of the risks of electrolyte abnormalities and volume
Several studies have shown no benefit in male nonsmok- depletion resulting from overdiuresis, as well as the avail-
ers and in women. There are insufficient data to determine ability of bisphosphanates and calcitonin. Loop diurectics
whether the association of coronary calcium with coronary should be considered in cases of concomitant fluid overload
artery disease risk warrants coronary calcium CT screening in order to restore euvolemia.
(Answer A) in asymptomatic men. Similarly, there are insuf- Ondansetron (Answer E) can be used to treat nau-
ficient data to support the use of exercise treadmill testing sea, but it does not have a role in the treatment of
(Answer E) for screening asymptomatic patients. The util- hypercalcemia.
ity of prostate cancer screening tests (Answer B) to decrease
mortality is uncertain. There is not enough evidence to rec-
ommend routine screening for thyroid disease (Answer C).
30. ANSWER:C. Metastatic breast cancer

All of the choices listed can cause lytic bone lesions and
28. ANSWER:D. Stop the dapsone. hypercalcemia. Of the choices, the most likely diagnosis in a
37-year-old woman with no other CT findings is metastatic
This is a case of methemoglobinemia, caused by dapsone, breast cancer (Answer C). A complete physical examina-
which had been started 2 weeks earlier for pneumocystis tion of both breasts, mammogram, and possibly breast MRI
prophylaxis. should be done to look for an occult primary breast cancer.
Methemoglobinemia is a form of hemoglobin that CT scanning may miss breast lesions.
does not bind oxygen, normally present at <1% due to the The median age at diagnosis for multiple myeloma
activity of antioxidation enzymatic pathways. Acquired (Answer A) is 66years, and only ~2% of patients are younger
methemoglobinemia is caused by exogeneous oxidizing than 40 years old. In addition, only approximately 3% of
drugs, including trimethoprim, sulphonamides, dapsone, cases of myeloma are nonsecretory (i.e., no M-spike on SPEP).
local anaesthetics (especially lidocaine and prilocaine), Sarcoidosis (Answer B) of bone occurs in approximately 5%
and aniline dyes. Most people are asymptomatic except for of patients with the disease, but this usually occurs in advanced
cyanosis, and possibly headache and fatigue. Severe methe- disease and rarely without pulmonary manifestations. Asmall
moglobinemia can cause shortness of breath, mental status number of patients with lytic or sclerotic lesions of the spine
changes, dysrhythmias, seizures, coma, and death. have been reported. Approximately 75% of patients with
The diagnosis should be suspected in the setting of a nor- untreated sarcoidosis have an elevated ACE level.
mal PAO2 despite a low O2 saturation +/- cyanosis. Treatment Diffuse large B-cell lymphoma (Answer D) is a rare cause
involves stopping the offending medication (Answer D). of bony lesions. The CT scan would have been expected to
Severe cases may be treated with IV methylene blue. show lymphoma.
Intubation and mechanical ventilation (Answer A) is Langerhans cell histiocytosis (Answer E) is a rare dis-
not needed, as the ABG demonstrates that she is not truly order (2 in 1million) that may present with bony lesions.
hypoxemic. Blood transfusions (Answer B) are not indi- Other presenting symptoms may include rash, diabetes
cated for treatment of methemoglobinemia. Hyperbaric insipidus, lymphadenopathy, ataxia, and memory problems.
oxygen (Answer C) is a treatment for carbon monoxide
poisoning. Bactrim and prednisone (Answer E) are used to
treat pneumocystis.
31. E.Ruptured brain abscess; treatment with
ceftriaxone, vancomycin, and ampicillin, with
emergency neurosurgery consultation
29. ANSWER:C. Furosemide 40 mgIV
The patients initial clinical syndrome of fevers, anorexia,
The recommended approach for the treatment of severe headache, and neck stiffness is consistent with a diagnosis
hypercalcemia (calcium > 14 mg/dL) includes: of meningitis. The abrupt change in symptomatology and
1. Hydration with saline (AnswerA) focal neurological symptoms reported the night prior to
2. Bisphosphonates (zolendronate or pamidronate) presentation are an indication for emergent head imaging
(AnswerB) (noncontrast head CT). Upon imaging, the head CT dem-
3. Calcitonin (AnswerD) onstrates a brain abscess with likely surrounding vasogenic

10 . G e n e r a l I n t e r n a l M e d i c i n e 217
edema, and magnetic resonance imaging (MRI) demon- 34. ANSWER:D. Cosyntropin stimulation test
strates pus within the ventricle, indicating rupture of the
brain abscess. Aruptured brain abscess is considered a neu- This is an adrenal incidentaloma in a patient with a history
rological emergency with a very high mortality rate. of hypertension. The appropriate lab evaluation includes
Given the high mortality rate of ruptured brain abscesses dexamethasone suppression test (Answer A) to evaluate
with pus draining into the ventricles, broad-spectrum anti- for Cushings disease in all patients, plasma metaneph-
biotics as well as emergent neurosurgical consultation are rines (Answer C) to evaluate for pheochromocytoma in all
indicated in this case (Answer E). patients, plasma aldosterone and renin (Answer B) if the
patient has hypertension, and an adrenal-protocol CT or
MRI (Answer E) should be performed to identify benign
features or those concerning for malignancy. Acosyntropin
32. ANSWER:C. Serial lumbar punctures to evaluate stimulation (Answer D) test evaluates for adrenal insuffi-
opening pressure ciency and is not a standard part of the work up for an adre-
nal incidentaloma.
Blood cultures (Answer A) can be helpful in identification
of causative organism of CNS infections by hematogenous
spread. Emergency neurosurgical consultation for consid-
eration of ventricular drainage (Answer B) is indicated as 35. ANSWER:C. Copper deficiency
intracranial pressure can lead to brainstem herniation.
Serial lumbar punctures (LPs) to evaluate opening pres- The patient has progressive upper motor neuron deficits.
sure (Answer C) are not indicated in the management of In a patient with history of bariatric surgery, copper and
these patients, who typically are managed with ventricular B12 deficiency should be suspected. However, the B12 level
drainage. Serial LPs are used in the management of crypto- (Answer A) in this case is normal. Zinc competes with cop-
coccal meningitis. per for absorption and the patient is taking supplemen-
Transthoracic echocardiogram (TTE) with bubble tal zinc, so this may be contributing. Copper deficiency
study (Answer D) is indicated as patients with cyanotic (Answer C) affects the corticospinal tract (hyperreflexia
heart disease (i.e., those with right-to-left shunt physiology) and Babinskis sign) and posterior column (impaired vibra-
have a higher incidence of CNS infections. The presence of tion sensation).
a right-to-left shunt in a patient with a brain abscess may be Vitamin B6 (Answer D) toxicity causes peripheral
an indication for closure of the shunt if it is intracardiac. neuropathy, not upper motor neuron signs. Lead toxicity
Careful physical examination of the sinuses and tym- (Answer E) is more common in children. Adults with lead
panic membranes (Answer E) should be performed. Both poisoning frequently have sleep disorders and may be hyper-
otitis media and infections of the paranasal/frontal sinuses somnolent. A paraneoplastic polyneuropathy (Answer B)
can occasionally spread directly to the CNS. would not explain all of his laboratory abnormalities.

33. ANSWER:B. Chest tube 36. ANSWER:A. Stop erythropoietin.

This patient most likely has a pleural effusion due to cir- Therapy with erythropoietin (Answer A) has been shown
rhosis and ascites (also called hepatic hydrothorax). It is due to effectively treat anemia in patients with chronic kidney
to a diaphragmatic defect, which can be microscopic. This disease (CKD) and may reduce fatigue and the need for
occurs on the right side 85% of the time, on the left side 13% blood transfusions. Normalizing hemoglobin levels with
of the time, and bilaterally 2% of the time. Thoracentesis erythropoiesis stimulating agents in patients with anemia
(Answer A) should be performed to assess for other pos- due to CKD is associated with increased risk of death, non-
sible etiologies of the pleural effusion and to rule out infec- fatal myocardial infarction, stroke, congestive heart failure,
tion. Diuretics (Answer C) (and salt and fluid restriction) and thrombosis. The FDA recommends that erythropoi-
can be used to manage the pleural effusion. Transjugular etin should not be used once the hemoglobin is higher than
intrahepatic portosystemic shunt (TIPS) (Answer D) is 11 g/dL. The rate of hemoglobin increase should not exceed
used to manage refractory effusions occuring in this set- 1 g/dL over 2 weeks.
ting. Patients with hepatic hydrothorax should be evaluated An iron saturation of >20% is adequate. IV iron
for liver transplantation (Answer E). Achest tube (Answer (Answer D) is not necessary. Oral iron may be beneficial
B) should never be placed in hepatic hydrothorax, as it can as increased red cell production over time may deplete iron
cause massive protein and electrolyte depletion, infection, stores in the future. ACE inhibitors (Answers B and C) can
renal failure, and bleeding. slow the progression of chronic kidney disease, particularly

218 T h e B r i g h a m I n t e n s i v e R e v i e w o f I n t e r n a l M e d i c i n e Q u e s t i o n a n d A n s w e r C o m pa n i o n
those associated with proteinuria, and should be continued. An echocardiogram (Answer A) should be performed
The patient has no signs consistent with a gastrointestinal to look for the presence of a ventricular aneurysm and ven-
bleeding, so esophagogastroduodenoscopy (EGD) (Answer tricular thrombus. Ventricular aneurysms are a common
E) is not the best next step. complication of anterior MIs. They are treated with after-
load reduction and anticoagulation.
Treatment with heparin (Answer B) may be initiated
if the patient has evidence of a ventricular aneurysm, but
37. ANSWER:E. Plasma exchange it would not be the next best step in management. Late
catheterization of STEMI (after 2448 hours) (Answer C)
This is a case of thrombotic thrombocytopenic purpura should only be done for severe heart failure, electrical or
(TTP), as evidenced by microangioapathic hemolytic anemia hemodynamic instability, or persistent ischemia. A phar-
and thrombocytopenia. While the classic pentad of micro- macologic stress test (Answer E) may be performed prior
angiopathic anemia, thrombocytopenia, renal failure, neu- to discharge for risk stratification. Clopidogrel (Answer D)
rologic changes, and fevers were common in the era before and aspirin are indicated for secondary prevention after a
effective treatment, it is now rare for all five features to be pres- STEMI.
ent in a single individual. The presence of thrombocytopenia
and microangiopathic hemolytic anemia without another
clinically apparent etiology should prompt a presumptive
diagnosis of TTP and the initiation of plasma exchange. 39. ANSWER:C. STAT electrolyte panel
Plasma exchange (Answer E) should be initiated even if there
is uncertainty about the diagnosis of TTP, since the risks of This patient has symptoms associated with acute hypo-
progressive TTP generally outweigh the risks of exchange. natremia (confusion and altered mental status) caused by
Platelet transfusion (Answer A) in TTP may lead to extreme hypotonic losses (through sweat) from exercise and
progressive consumption of infused platelets and produc- replacement of the hypotonic fluid with free water. In the
tion of thrombi, causing worsening neurologic symptoms setting of euvolemia, as indicated by the normal to slightly
and/or renal failure. Cardiac catheterization (Answer B) high jugular venous pressure of 6 cm of water, rehydra-
might be reasonable in this patient, but it should not delay tion with normal saline (Answer A) and oral rehydration
the initiation of plasma exchange. The ischemic signs and (Answer B) is inadequate to correct hyponatremia and can
symptoms are most likely due to microthrombi in the coro- result in worsening hyponatremia.
nary circulation, which are treated by exchange. Rituximab The first step in this emergency situation should be to
(Answer C) may be used in addition to plasma exchange for obtain a STAT electrolyte panel (Answer C) to confirm
refractory or recurrent TTP. IVIG (Answer D) may also be hyponatremia and to help guide further treatment.
used as an adjunctive therapy in TTP, though should not be Empiric administration of hypertonic saline (Answer D)
used as an alternative to plasma exchange. should be avoided in a clinical setting where laboratory tests
can be obtained rapidly, as overly hasty correction of hypo-
natremia can lead to an osmotic demyelination syndrome
(also known as central pontine myelinolysis).
38. ANSWER:A. Echocardiogram The patients signs and symptoms are inconsistent with
ibuprofen overdose (Answer E). While her symptoms may
The clinical history of chest pain in the setting of cocaine be consistent with very severe acute kidney injury in the set-
use 1 week ago suggests a cocaine-induced myocardial ting of potential hypovolemia and ingestion of ibuprofen as
infarction (MI). a second insult, this is much less likely.
Following an MI, cardiac biomarkers peak at 1824
hours. CK and CKMB remain elevated for 48 hours, while
troponins may remain elevated for 10days. The pattern of
biomarkers here is consistent with an MI 1 week ago. The 40. ANSWER:B. Azithromycin
EKG demonstrates ST elevations in leads V1V4, suggestive
of an anterior ST-elevation MI. There are also Q-waves in The clinical presentation suggests pertussis, which can be a
leads V1V4. The presence of anterior Q-waves plus persis- cause of persistent cough in adults, even those who received
tent ST-elevations with a clinical story of a cocaine-induced vaccinations as child. The paroxysms of coughing and post-
MI 1 week ago is most suggestive of a ventricular aneurysm. tussive vomiting are also features suggestive of pertussis.
The differential diagnosis of chronic or persistent ST eleva- The clinical course is usually less severe than in children,
tions includes early repolarization, ventricular aneurysm but treatment is advised within 4 weeks of symptoms in
(when ST elevations occur with Q-waves), a scar following order to contain the spread of infection, which is especially
a large anterior MI, and chronic pericarditis. important given that she works at a daycare. Treatment

10 . G e n e r a l I n t e r n a l M e d i c i n e 219
options for pertussis include azithromycin (Answer B), 43. ANSWER: D. Indomethacin 50 mg orally
clarithromycin, and trimethoprim-sulfamethoxazole.
Albuterol inhaler (Answer A) and prednisone (Answer This is a presentation of reactive arthritis, which presents
C) might be used for an asthma attack. Antitussive agents as an asymmetric mono/oligo-arthritis, predominantly of
(Answer D) may be helpful, but it will not treat the infec- lower-extremity joints. Classically, it also presents with enthesi-
tion. There is no clinical indication for hospitalization or IV tis (inflammation of the insertion of ligaments, tendons, joint
antibiotics for this patient (Answer E). capsule, or fascia to bonetypically the Achillies tendon) and
dactylitis (sausage digits). Extra-articular involvement may
include urethritis, conjunctivitis, uveitis, oral ulcers, and rashes.
Reactive arthritis may occur following GU or enteric infec-
41. ANSWER:D. Homocysteine level tions caused by Chlamydia trachomatis, Yersinia, Salmonella,
Shigella, Campylobacter, and possibly Clostridium difficile.
The patient likely has homocystinuria (Answer D). These Typically, there are a few days to a few weeks between infection
patients have a clinical phenotype that can be similar to and onset. Treatment is with NSAIDS, such as indomethacin
Marfans syndrome (caused by a fibrillin 1 mutation), 50 mg three times a day, for at least 2 weeks.
but there are some key differences. For example, patients Gonococcal arthritis similarly may present with the
with Marfans do not have associated venous thrombosis. abrupt onset of a mono- or oligo-arthritis, but it typically
Similar to Marfans, patients with homocysteinuria can does not cause sausage digits and frequently presents with a
be tall and thin, and have pectus excavatum and arach- rash. Ceftriaxone (Answer A) would be an appropriate ini-
nodactyly. However, they do not have the cardiovascular tial treatment for this. IV (Answer B) or oral (Answer C)
problems associated with Marfans. Patients with homo- steroids may be used to treat numerous rheumatologic con-
cysteinuria can also have neuropsychiatric disorders, ditions, but they are not the treatment of choice for reactive
osteoporosis, and skin hypopigmentation. Patients with arthritis. Observation (Answer E) is not the best answer
homcysteinuria have downward lens dislocation while given the severity of symptoms in this case.
those with Marfans have upward lens dislocation. In the
United States there is newborn screening for homocys-
teinuria. Factor V Leiden (Answer A) (activated protein
C mutation), prothrombin gene (G20210A) mutation 44. ANSWER:B. Iron deficiency anemia
(Answer E), and protein S deficiency (Answer C) are
causes of venous thrombosis, but they do not explain his This is a case of profound iron-deficiency anemia (Answer B),
other physical findings. as evidenced by microcytic anemia with very low iron levels.
Causes of microcytic anemia include iron deficiency,
thalassemia, and anemia of chronic disease. Rarer causes
include copper deficiency, lead poisoning, and sideroblastic
42. ANSWER:B. Start captopril. anemia. Iron deficiency may be subtle and insidious, with
typical presenting symptoms including fatigue, weakness,
The patient has scleroderma and newly elevated blood pres- exercise intolerance, headache, and irritability. Additional
sure, lower-extremity edema, renal failure with proteinuria, signs and symptoms include tongue pain, dry mouth, pica/
and microangiopathy, consistent with scleroderma renal pagophagia, and restless leg syndrome.
crisis. Thalassemia (Answer A) typically presents with very
The drug of choice for scleroderma renal crisis is an low MCVs, as in this case, but iron stores should be normal
angiotensin-converting-enzyme inhibitor (Answer B), to increased. The prior MCV of 85 fL also makes thalas-
which should be rapidly titrated to reduce blood pressure. semia highly unlikely, as it is an inherited disorder. Family
ACE inhibitors help to preserve or improve renal function history is often positive.
in scleroderma renal crisis and have been shown to signifi- Lead poisoning (Answer C) may cause microcytic ane-
cantly reduce mortality in this setting. mia. Basophilic stippling is often (but not always) evident
There is no evidence that this is an inflamma- on peripheral blood smear. Other manifestations include
tory process; thus, glucocorticoids (Answers D and abdominal pain, joint and muscle aches, memory problems,
E) are not indicated. In addition, high-dose glucocor- and irritability
ticoids increases the risk of scleroderma renal crisis. The normal haptoglobin and LDH make the diagnosis
Calcium-channel blockers (Answer A) may be used of hemolytic anemia (Answer D) less likely. Hemolysis usu-
as additional treatment of resistant hypertension. ally results in a normocytic anemia.
Beta-blockers (Answer C) are usually avoided in patients Anemia of chronic disease (Answer E) typically pres-
with scleroderma because of the theoretical risk of wors- ents with low iron, low TIBC, and a normal to increased
ening vasospasm. ferritin. The ESR would be expected to be elevated.

2 2 0 T h e B r i g h a m I n t e n s i v e R e v i e w o f I n t e r n a l M e d i c i n e Q u e s t i o n a n d A n s w e r C o m pa n i o n
45. ANSWER:D. Allergic bronchopulmonary 25% of patients present with symptoms). The RNA becomes
aspergillosis detectable from days up to 8 weeks after infection, while the
IgG serology is detectable after 8 weeks. There is no IgM for
Allergic bronchopulmonary aspergillosis (ABPA) (Answer hepatitisC.
D) should be suspected in a patient with new, adult-onset There is limited evidence for the treatment of acute hep-
asthma and peripheral eosinophilia. While this constella- atitis C, but the general recommendations are to recheck the
tion of symptoms can also be seen in Churg Strauss syn- viral load at 3months (Answer A). If it is negative, the viral
drome, the historical feature of marijuana use heavily sways load is checked every 3months for 1year. If it is positive,
the diagnosis in favor of ABPA, as marijuana is often con- treatment for hepatitis C is started. The rationale for wait-
taminated with Aspergillus species. ing 3months is that patients who present with symptom-
Eosinophilic granulomatosis with polyangiitis (Answer atic acute hepatitis C infection are more likely to clear the
C) is characterized by sinus disease, asthma, and end-organ infection without treatment within the first few months.
damage from peripheral eosinophilia, including cardiac The standard of care for treatment for genotype Ihep-
dysfunction and neuropathy. atitis C is with peginterferon, ribavirin, and a protease
Chronic bronchitis (Answer B) is unlikely to explain inhibitor such as telaprevir (Answer D). Therapy with
the patients eosinophilia or end-expiratory wheezing. peginterferon and ribavirin (Answer B) are still the treat-
Invasive aspergillosis (Answer E) is usually seen in ment for the other hepatitis C genotypes. New therapies are
patients with an underlying immunocompromised state being developed for the treatment of hepatitisC.
and is associated with fevers and occasionally with hemop- Lamivudine (Answer C) and entecavir (Answer E) are
tysis or blood-tinged sputum. treatments for hepatitis B.

46. ANSWER:C. Check an IgE level and consider 48. ANSWER:A. Beta-blockers and gentle diuresis
initiation of omalizumab (anti-IgE).
Given the epidemiologic features of being raised in Brazil,
The treatment for ABPA includes encouraging cessation and the examination and echocardiographic features sugges-
of offending agents and administration of oral steroids tive of mitral valve stenosis, this patient likely had rheumatic
(Answer A). Thus, encouraging cessation of marijuana use fever as a child, resulting in rheumatic heart disease and
is of particular importance in this case (Answer D). mitral stenosis, which was asympomatic until her pregnancy.
Peripheral eosinophilia can also be caused by strongy- Medical management of symptomatic mitral stenosis in
loides infection and treatment with oral steroids can lead to pregnancy involves beta-blockers (to slow the heart rate and
a life-threatening strongyloides hyperinfection syndrome. improve diastolic filling) and gentle diuresis (Answer A). If
Thus, ruling out strongyloides infection is indicated in this is not adequate, percutaneous mitral valvuloplasty can
this patient prior to initiation of steroid therapy (AnswerB) be considered.
The test for a FIP1L1-PDGFR alpha translocation is a Inotropes such as digoxin (Answer B) and dopa-
molecular diagnostic tool that is used in the diagnosis of mine (Answer C) should be avoided in mitral stenosis.
primary hypereosinophilic syndrome. Although unlikely, it Vasodilators such as ace-inhibitors (Answer D) and hydral-
is a reasonable test in this case, given the degree of eosino- azine/nitrates (Answer E) are not first-line agents in the
philia (Answer E). setting of decompensated mitral stenosis. The safety of
Omalizumab is a monoclonal antibody directed at medications during pregnancy should also be considered.
IgE that is FDA approved for the treatment of refractory
asthma in the setting of elevated IgE levels (Answer C). It is
not currently indicated in the treatment of ABPA.
49. ANSWER:B. IgA nephropathy

IgA nephropathy (Answer B) is the most common cause


47. ANSWER:A. Recheck hepatitis C viral load in of primary glomerulonephritis in the developed world. It
3months and consider treatment if virus not cleared. usually presents with recurrent gross hematuria, usually
less than 5days after a URI or bacterial tonsilitis, though
This patient likely has acute hepatitis C given the recent it may also present with microscopic hematuria and mild
exposure to IV drugs, the positive viral load, and the lack of proteinuria.
another cause for his hepatitis. Post-streptococcal GN (Answer A) is an
Hepatitis C accounts for 20% of acute hepatitis in the immune-complex disease from specific nephritogenic
United States, but it is often asymptomatic (approximately strains of GAS. It usually occurs 13 weeks after pharyngitis

10 . G e n e r a l I n t e r n a l M e d i c i n e 2 21
and 36 weeks after skin infection. Children ages 512 and phototoxic eruption that results from contact with
adults >60 are at highest risk. The most common present- light-sensitizing substances and exposure to UVA radia-
ing symptoms are edema, gross hematuria, and hyperten- tion (Answer B).
sion, though the presentation ranges from asymptomatic to Symptoms typically begin within 24 hours of expo-
microscopic hematuria to full-blown acute nephritic syn- sure, peak at 4872 hours, and may take weeks to resolve.
drome. Complements are usually decreased and this may Manifestation can range from hyperpigmentation (as is seen
persist for 48 weeks. Renal function resolves within 34 here) to bullous eruptions. The most frequently reported
weeks; hematuria may persist for 36months. Unlike IgA sensitizing agents are lime juice, mangoes, celery, occa-
nephropathy, it rarely recurs. sionally roses/grasses, and bergamot oils in perfumes with
Proteinuria and dysmorphic red cells suggest glomerular essential oils. The treatment for this condition is watchful
(rather than extraglomerular) bleeding, making the remain- waiting.
ing choices unlikely. Carcinoma of the bladder (Answer C) Given the linear pattern on the arm, lymphangitis is
typically affects older patients, with a mean age at diagno- also on the differential diagnosis, but in the absence of sys-
sis of around 70years. Urinary tract infections (Answer D) temic symptoms, this is highly unlikely. Lymphangitis can
and nephrolithiasis (Answer E) would be expected to pres- be caused by various organisms such as:
ent with additional signs and symptoms and without dys-
morphic red blood cells in the urine. Mycobacterium marinumcommon in salt-water fish
tanks (Answer A)
Sporothrix spp.classically contracted through a
rosebush thorn puncture (Answer D)
50. ANSWER:B. Squeezing limes while making Streptococcus pyogenescontracted through injection
mojitos drug use (Answer E)

This strange, often bizarre, and well-demarcated rash A reaction to sunscreen would not be expected to have
(phytophotodermatitis) is the result of a cutaneous such a linear pattern (Answer C).

2 2 2 T h e B r i g h a m I n t e n s i v e R e v i e w o f I n t e r n a l M e d i c i n e Q u e s t i o n a n d A n s w e r C o m pa n i o n
INDEX

Page numbers followed by t and f indicate tables and figures, respectively.

A allergic bronchopulmonary aspergillosis, antidepressants, tricyclic, 117 autoimmune disorders, 150


ABCD score, 199 221 anti-double-stranded DNA antibody, 68 autoimmune hepatitis, 151
ABCD2 score, 199 allergic interstitial nephritis, 135 antiepileptic drug therapy, 200, 201 autoimmune pancreatitis, 157
abdominal aorta, 178179 allergy consultation, 21 antiglutamic acid decarboxylase antibodies, avascular necrosis, 70
abdominal aortic aneurysm, 182, 216217 allopurinol, 48, 214 113 azathioprine (AZA), 7374, 151
abdominal compartment syndrome, 95 alopecia areata, 213 antihypertensive therapy, 177, 198 azithromycin, 24, 214, 219220
abdominal ultrasound, 182, 216217 alpha-1 antitrypsin deficiency, 8687, antineutrophilic cytoplasmic antibodies, 22
abscess, brain, 217218 92, 155 antiphospholipid antibody syndrome, 201 B
Accolate (zafirlukast), 87 alpha thalassemia, 39 antiplatelet therapy, 179, 199 babesiosis, 15, 25, 153
acetaminophen (paracetamol) poison- Alport syndrome, 139 antiretroviral therapy (ART), 19, 20, 2425 back pain, 72
ing, 95 alveolar hemorrhage, diffuse, 62f, 74 highly active (HAART), 134 bacteremia, S.aureus, 180181
N-acetylcysteine, 95 Alzheimer dementia, 200201 anti-Ro syndrome, 73 bacterial endocarditis, 17, 1920, 180181
achalasia, 156 Alzheimers disease, 196 anti-Scl-70 antibody, 156 bacterial meningitis, 2021
Achilles tendon rupture, 71 ambulatory blood pressure measurement, antithombotic therapy, 199 bacterial overgowth, small intestinal
acidosis 184 aortic regurgitation, 181 (SIBO), 158
anion gap, 138139 amenorrhea, 4344, 116, 213214 aortic stenosis, 183, 215216 bacterial peritonitits, spontaneous (SBP),
lactic, 140 American Diabetes Association guidelines, aortic valve, bicuspid, 178179, 181 154, 158
respiratory, 95 123 aortic valve replacement, 181 bacterial rhinosinusitis, acute, 214215
acromegaly, recurrent, 4445, 121 5-aminosalicylates, 157 apnea, 8990 Bacteroides fragilis, 1920
activated partial thromboplastin time amitriptyline, 156 argatroban, 96 barium swallow, 152
(aPTT), 40 amlodipine, 138 arterial blood gases, 99 Barretts esophagus, 150
acute bacterial rhinosinusitis, 214215 amnesia, 196 arthritis bartonellosis, 16
acute calcific tendonitis, 6869 amoxicillin-clavulanate, 214215 degenerative, 70 Bartter syndrome, 139
acute chest syndrome, 39 amphotericin B, 17 gonococcal, 220 B cell leukemia/lymphoma 2 (BCL2), 46
acute gouty arthritis, 68 ampicillin, 2021, 217218 gouty, 68 bedside screening, 198
acute interstitial nephritis, 135 amyloidosis, 137 inflammatory, 216 benztropine, 199
acute ischemic stroke, 137, 195, 199 ANCA-associated vasculitis, 137 in knee, 6970, 74 beriberi, wet, 153
acute metabolic alkalemia, 9394 androgen deprivation therapy, 41 of Lyme disease, 66 beta-blockers, 100, 221
acute-on-chronic respiratory acidosis, 95 anemia, 97 osteoarthritis, 64, 6970, 216 beta-hemolysis, 26
acute promyelocytic leukemia, 4546 of chronic disease, 218219, 220 postenteric reactive, 151 bilevel positive airway pressure, 90
acute pseudogout arthritis, 64 hemolytic, 39, 71, 220 pseudogout, 64, 70 biliary cirrhosis, primary, 150
acute renal failure, 96 iron deficiency, 220 pseudo-osteo,70 biopsy
acute respiratory distress syndrome, 94, 100 microcytic, 38, 220 pseudo-rheumatoid,70 distal esophagus, 154
acute rhinosinusitis (sinusitis), 21 aneurysm psoriatic, 67 lymph node, 43
acute viral polyarthritis, 66 abdominal aortic, 182, 216217 reactive, 71, 151, 220 renal, 134135
adalimumab, 74 middle cerebral artery, 198 rheumatoid, 64, 75 temporal artery, 64, 69
Addisons disease, 117 angina, 180 septic, 74 bites, 3, 15, 17
adenoma, benign, 117 angiography viral, 66 bladder cancer, 41
adenosine, 176 computed tomography, 198, 199 arthropathy, hemochromatosis, 51f, 65 bladder pressure, 95
adrenal crisis, 136 coronary, 175, 184 arthroplasty, knee, 6970 bladder tumors, 41
adrenal incidentaloma, 218 magnetic resonance, 178179, 198, 200 ascites, 156, 218 bleeding. See also hemorrhage
adrenal insufficiency, 117118 angioplasty, carotid, 198 ascites fluid, 154 gastrointestinal, occult, 158
adrenal masses, 117 angiotensin-converting enzyme inhibitors, aspergillosis, 17, 221 variceal, 158
adrenal myelolipoma, 117 71 aspirin-exacerbated respiratory disease, 87 blood cholesterol, 213
adrenal vein sampling, 134 animal bites, 3, 15, 17 aspirin-intolerant asthma, 87 blood pressure, 136
agoraphobia, 179180 anion gap acidosis, 138139 aspirin-sensitive asthma, 87 blood pressure control, 138
AIMS65 score, 153 ankylosing spondylitis, 74 aspirin therapy, 122, 178, 179, 199 blood pressure monitoring, 184
air bronchograms, 91, 91f antibiotic therapy asthma, 86, 87, 90 blood serology, 25
airflow obstruction, diffuse intrathoracic, for acute chest syndrome, 39 atelectasis, 9192, 92f boceprevir, 151
90, 90f for bacterial endocarditis, 180181 atorvastatin, 179 BODE index, 76, 87
albuterol, 21 for COPD exacerbation, 93 atrial fibrillation, 178, 184, 195 bone disease, 214
alcoholic hepatitis, 157 infectious endocarditis prophylaxis, 175 atrial myxoma, 183184 bone loss, 115
aldosterone, 118 for ruptured brain abscess, 217218 atrial septal defect, 173 bone scanning, 42
aldosterone-to-renin ratio (ARR), 118 spontaneous bacterial peritonitits (SBP) atrioventricular block, 178, 179180 Bordetella pertussis, 1415
alendronate, 114, 115 prophylaxis, 154, 158 atypical squamous cells of undetermined bowel ischemia, 150
alkalemia, acute metabolic, 9394 for Whipples disease, 151 significance (ASC-US), 213 bradycardia, sinus, 174175, 184
alkalosis, metabolic, 9394 anticholinergic overdose, 100 autoantibodies, 113114 brain abscess, 217218

223
brain edema, malignant, 198 chancroid, 24 contraceptives, oral, 116, 201 type 2, 113, 123, 124
brain imaging, 42, 199, 200, 213 chemotherapy, 4748 Coopers ligament pain, 213 diabetic amyotrophy, 70
BRCA 2 mutations, 38 chest pain, 181, 183, 184, 219 copper, serum, 201 dialysis, 96, 133, 137
breast cancer, 41, 217 chest scanning copper deficiency, 153, 201, 218 dialysis disequilibrium syndrome, 140
breast support, 213 with diffuse itching, 214 coronary angiography, 175, 184 diarrhea, 152
breathing index, 9596 ground-glass opacity on, 88, 88f coronary artery bypass grafting, 180, 182 dietary modifications
breathing trials, 9596 in Hodgkin lymphoma, 43 coronary artery disease, 179, 182 gluten-free free, 114115
bronchiectasis, 77f, 9091 in lung cancer, 42, 72 corticosteroids low-oxalate diet, 158
bronchiolitis obliterans, 93 in pulmonary disease, 14 for ANCA-associated vasculitis, 137 low-salt diet, 135
bronchiolitis obliterans organizing pneu- chest tubes, 218 for ARDS, 100 for prediabetes, 122
monia (BOOP), 88 chlorine-containing cleaning agents, 15 for idiopathic inflammatory myopathy, diffuse alveolar hemorrhage (DAH), 62f, 74
bronchitis, chronic, 221 chlorthalidone, 135136, 138 67 diffuse intrathoracic airflow obstruction,
bronchopulmonary aspergillosis, allergic, cholesterol, 213 inhaled, 9293 90, 90f
221 cholinesterase inhibitors, 196 for lupus nephritis, 68 diffuse large B-cell lymphoma, 2, 217
bronchoscopy, 6667, 74 chondrocalcinosis, 50f, 70 for paradoxical tuberculosis (IRIS), diffusion-weighted magnetic resonance
Burkitts lymphoma, 4546, 136 chorea, 201 1617 imaging, 199
bursitis chromosome translocation t(9;22), 46 for sepsis and septic shock, 100 digestive diseases, 141159
gouty, 71 chromosome translocation t(15;17), 4546 side effects of, 9293 digoxin, 150, 180
trochanteric, 69, 70 chronic bronchitis, 221 cortisol, 118 1,25-dihydroxy vitamin D (calcitriol), 114
bypass surgery chronic disease:anemia of, 218219, 220 free, 116, 120121 dilated cardiomyopathy, 180, 184
coronary artery, 180, 182 chronic hyponatremia, 135 salivary, 44, 115, 120121 distal extremity pain, 200
gastric, 153, 201 chronic inflammatory demyelinating poly- urinary, 44 diuresis, gentle, 221
neuropathy, 197 cortisol excess, 116 diuretics, 133, 217, 218
C chronic interstitial nephritis, 136 cortisol insufficiency, 118 DNA antibody, 68
C3 complement, 68 chronic itch, 214 cosyntropin stimulation test, 218 dog bites, 3, 15
C4 complement, 68 chronic kidney disease, 123124, 133, cough, severe, 23 dopamine, 94
calcific tendonitis, acute, 6869 137138, 218219 counseling, 70 dopamine agonists, 44, 120
calcitriol (1,25-dihydroxy vitamin D), 114 chronic lung disease, 1314 Cowdens syndrome, 155 dopamine blockers, 200
calcium channel antibodies, 200 chronic Lyme, 6566 C-reactive protein, 66 Doppler studies, 179
calcium-channel blockers, 100 chronic lymphocytic leukemia, 43 creatinine, 44, 120121 doxycycline, 3, 21, 24, 215
calcium oxalate stones, 139 chronic metabolic alkalosis, compensatory, critical care medicine, 76100 drug abuse, 219
calcium pyrophosphate dihydrate (CPPD), 9394 critical illness, 123 drug-induced parkinsonism, 200
50f chronic myelogenous leukemia, 46 critical illness myopathy, 200 drug interactions, 2526
calcium supplementation, 158, 213214 chronic obstructive pulmonary disease, 2, Crohns disease, small intestinal, 158 drug therapy. See medications; specific drugs
California score, 199 86, 90 Cronkhite-Canda syndrome, 155 duloxetine, 25
Campylobacter infection, 14 chronic obstructive pulmonary disease cryoglobulinemic vasculitis, 56f, 69 duodenal ulcers, 156
cancer, 174. See also oncology (COPD) exacerbations, 93 cryptogenic organizing pneumonia, 88 dyspepsia, functional, 156
candidiasis (thrush), 18 Churg-Strauss syndrome, 9091, 221 crystalloids, 1718, 96 dysphagia edside screening for, 198
Capnocytophaga canimorsus, 17 chylothorax, 88 Cushings syndrome, 44, 115, 117, 120121 dysphonia, 9293
captopril, 71, 134, 220 ciprofloxacin, 17 cyclophosphamide, 137 dyspnea, 173174, 177
carbon dioxide, end-tidal, 99100 cirrhosis, 69, 154, 218 cystadenoma, serous, 154 dystonic reactions, acute, 199
carcinoid syndrome, 157158 cisplatin, 41 cystic fibrosis, 116, 152
cardiac arrest, 97 clavulanate, 214215 cystic fibrosis transmembrane conductance E
cardiac masses, 183184 cleaning agents, chlorine-containing, 15 regulator (CFTR) gene, 116, 152 echocardiography, 174, 183, 219
cardiac resynchronization therapy, 177 clindamycin, 25 cystic neoplasms, 154 transesophageal, 180181
cardiac surgery, 180 clopidogrel, 175176, 178, 179 cystitis, 14, 214 transthoracic, 176, 218
cardiogenic shock, 98 Clostridium difficile infection, 151152 cytomegalovirus (CMV) colitis, 13 eculizumab, 137
cardiomyopathy coarctation of the aorta, 179 cytomegalovirus (CMV) infection, 18 Ehler-Danlos syndrome Type 3, 67
dilated, 180, 184 cocaine-induced myocardial infarction, 219 cytomegalovirus (CMV) polyradiculopa- electroencephalography, 199
hypertrophic, 180, 215216 coccidioidomycosis, pulmonary, 25 thy, 197 electrolytes, 219
cardiopulmonary resuscitation, 99100 cognitive-behavioral therapy, 179180 electrophoresis, 38, 66
cardiorenal syndrome, 140 cognitive impairment, mild, 196 D electrophysiology, 174175
cardiovascular disease, 122, 160184 colchicine, 178, 214 dapsone, 217 emergency neurosurgery, 217218
carotid angioplasty, 198 colistimethate (colistin), 19 decompressive hemicraniectomy, 198 emphysema, widespread (panlobular),
carotid artery dissection, 198 colitis deep venous thrombosis (DVT), 40, 47 8687
carpal tunnel syndrome, 70 cytomegalovirus (CMV), 13 dematomyositis, 67 encephalopathy, anoxic-ischemic, 97
carpometacarpal joint tenderness, 216 ischemic, 150 dengue, 13 endocarditis
catecholamine excess, 117 ulcerative, 93, 157 dental procedures, 175 bacterial, 17, 1920, 180181
catecholamines, 117 colon cancer, 4748, 216 dermatomyositis (DM), 53f, 67 infectious, 175
cauda equina syndrome, 72 colonoscopy, screening, 152153, 216 desmopressin (ddAVP), 135 endocrinology, 101124
cefepime, 18 comfort control, 98 dexamethasone suppression test, 44, 117, endoscopic retrograde cholangiopancrea-
ceftazidime, 23 compensatory chronic metabolic alkalosis, 120121 tography, 150, 154, 158159
ceftriaxone, 17, 2021, 158, 214, 217218 9394 5% dextrose in water (D5W), 135 endoscopy, 156, 158
celiac disease, 114115 computed tomography diabetes insipidus, central, 45, 121 endotracheal suctioning, 99
celiac sprue, 153154 of brain, 199 diabetes mellitus end-tidal carbon dioxide detection, 99100
cellulitis, 22 of lungs, 42, 42t blood pressure control in, 138 enteral nutrition, 157
central apnea, 8990 computed tomography angiography, 198, complications of, 113, 176 entrapment neuropathy, 70
central diabetes insipidus, 45 199 hypertensive management in, 135136, environmental contamination, 15
central nervous system infections, 218 congestive heart failure (CHF)associated 176 eosinophilia, peripheral, 221
cervical cancer screening, 214 liver disease, 155 insulin therapy for, 123 eosinophilic esophagitis, 152, 216
cervical myelopathy, 69 constipation-predominant irritable bowel management of, 113, 114, 123, 124 eosinophilic granulomatosis with polyangi-
cervical spine X-ray imaging, 69 syndrome, 159 mature-onset (MODY), 113114 itis, 215, 221
CFTR gene, 116, 152 continuous positive airway pressure with NSTEMI, 179 eosinophilic lichen planus, 152
CHADS2 score, 178 (CPAP), 138 prediabetes, 113, 122 epidural metastasis, 200
CHADS2-VASc score, 178 continuous renal replacement therapy soft-tissue infection in, 23 epidural spinal cord compression, 48
Chagas disease, 156 (CRRT), 96 type 1, 113114 epilepsy, juvenile myoclonic, 200

22 4 I n de x
eplerenone, 177178 Gitelman syndrome, 139 hepatic hydrothorax, 218 hypertrophic cardiomyopathy, 180,
Epstein-Barr virus infection, 18 glipizide, 123124 hepatitis, 16, 151, 155, 157 215216
erythema migrans, 153 Global Initiative for Chronic Obstructive hepatitis A, 1314 hyperuricemia, 72
erythrocyte sedimentation rate, 66 Lung Disease (GOLD) guidelines, 86 hepatitis B, 14, 16, 156157 hypocalcemia, 138
erythropoiesis stimulating agents (ESAs), globus, 151 hepatitis C, 16, 69, 151, 155, 221 hypoglycemia, 114
46, 133 glomerulonephritis, 221222 hepatitis C antibody, 69 hypogonadism, 115116
erythropoietin, 218219 glomerulosclerosis, focal segmental, 136 hepatitis Cassociated renal disease, 136 hyponatremia, 134, 135, 219
erythropoietin deficiency, 3839 glucocorticoids, 118 hepatitis C virus (HCV) antibody test, 150 hypoparathyroidism, 114
esophageal biopsy, 154 glucose control, 113, 123 hepatopathy, glycogenic, 156 hypophosphatemia, 115
esophagitis, eosinophilic, 152 glucose tolerance test, 200 hepatorenal syndrome, 139 hyposplenism, 154
ethylene glycol intoxication, 98 glutamic acid decarboxylase (GAD) hereditary hemochromatosis, 153, 155 hypothalamic amenorrhea, 116, 213214
exercise, 122, 181 antibodies, 113 hereditary neuropathy with predisposition hypothermia, therapeutic, 97
exercise treadmill test, 183 gluten-free diet, 114115 to pressure palsies (HNPP), 196197 hypothyroidism, 44, 116117, 119, 120
glycogenic hepatopathy, 156 hereditary spherocytosis, 39 hypoventilation, 8990
F goiter, multinodular, 119 herpes simplex virus (HSV) infection, hypoxemia, 88, 99
Fabry disease, 139140 gonococcal arthritis, 220 2122, 152
familial Alzheimer dementia, early-onset, gonorrhea, 21, 214 herpes zoster oticus, 24 I
200201 gout, 61f, 64, 7374, 214 highly active antiretroviral therapy ibuprofen, 173, 178
familial Mediterranean fever (FMF), 6768 gouty arthritis, acute, 68 (HAART), 134 idiopathic inflammatory myopathy (IIM),
fasting hyperglycemia, 113 gouty bursitis, prepatellar, 71 hilar and mediastinal lymphadenopathy, 67
fatty liver disease, nonalcoholic, 216 granulomatosis, eosinophilic, 215, 221 81f, 93 idiopathic membranous nephropathy,
female athlete triad, 116 granulomatous arteritis, 73 hip osteoarthritis, 64, 70 136137
female infertility, 213 Graves disease, 119, 120 HIV-associated nephropathy (HIVAN), idiopathic pulmonary fibrosis, 8889
fentanyl, 97 ground-glass opacity, 88, 88f, 91, 91f 134 immotile cilia syndrome, 9091
ferritin and Hb electrophoresis, 38 Guillain-Barr syndrome, 14, 197 hoarse voice, 9293 immune reconstitution inflammatory
fever Hodgkins lymphoma, 43, 136 syndrome (IRIS), 1617
familial Mediterranean (FMF), 6768 H home blood pressure diary, 184 immune suppression, 6667
in ICU, 97 Haemophilus ducreyi, 24 homocysteine levels, 220 immunoglobulin Anephropathy, 135,
neutropenia with, 18 hair loss, 213 homocystinuria, 220 221222
in returning travelers, 13 hands honeycombing, 77f, 91, 91f immunoglobulin E, 221
rheumatic, 221 chondrocalcinosis of, 50f, 70 Horner syndrome, 198 immunoglobulin G, 9091
of unknown origin, 64 inflammatory arthritis of, 216 hospitalization, 198199 immunosuppressive therapy, 7374
fibroblast growth factor-23 (FGF-23), 115 headache hospitalized patients, 113, 123 implantable cardiac defibrillators (ICDs),
fibromuscular dysplasia (FMD), 139 acute, with oculosympathetic paresis, 198 hot flashes, 41 177, 180
fibromyalgia syndrome, 70 migraine with aura, 201 human immunodeficiency virus (HIV) incidentaloma, adrenal, 218
fibrosis, nephrogenic systemic (NSF), 75 migraine without aura, 195, 200 cholangiopathy, 150 indomethacin, 154, 220
Fifths disease, 66 new-onset, 213 human immunodeficiency virus (HIV) infection mimickers, 22
FIP1L1-PDGFR alpha translocation test, short-lasting unilateral neuralgiform infection, 18, 2425, 155 infections. See also specific infections
221 headache with conjunctival injection human immunodeficiency virus (HIV) central nervous system, 218
Fleischner Society guidelines, 42, 42t and tearing (SUNCT), 201 testing, 214 fungal, 18
flexible bronchoscopy, 74 thunderclap, 198 human papillomavirus (HPV) testing, pulmonary, 6667
flow-volume curves, 90, 90f health care-associated pneumonia (HCAP), 213, 214 sexually transmitted (STIs), 18
fluid restriction, 218 18 human papillomavirus (HPV) vaccination, skin/soft tissue, 15, 23
fluid resuscitation, 96 heart disease 1314 after solid organ transplantation, 13
fluticasone, 19, 21, 9293, 216 cardiovascular disease, 122, 160184 hungry bone syndrome, 114 urinary tract, 22, 41
fluvastatin, 179 rheumatic, 221 Huntingtons disease, 201 infectious disease, 126
focal segmental glomerulosclerosis, 136 valvular, 175 hydrochlorothiazide, 133 infectious endocarditis, 175
follicular lymphoma, 46 heart failure hydrocortisone, 118, 136 infertility, female, 213
fomepizole, 98 congestive (CHF), 155 hydrothorax, hepatic, 218 inflammatory arthritis, 216
foot drop, 69 with depressed ejection fraction, hydroxychloroquine, 65, 72 inflammatory bowel disease, 151152
fosfomycin, 14 177178 5-hydroxyindoleacetic acid (5-HIAA), inflammatory myopathy, idiopathic (IIM),
fosphenytoin, 99 with preserved ejection fraction, 177 157158 67
fractures, 115 refractory ascites on, 156 1,25-hydroxy-vitamin D supplements, influenza vaccination, 1314
Francisella tularensis, 26 systolic, 182183 138 inhaled corticosteroids, 9293
fungal infections, 18 heart masses, 183184 hyperaldosteronism, 118 insulin resistance, 122123
fungal sinusitis, 17 heart murmurs, 175, 177, 183, 215216 hypercalcemia, 114, 136, 217 insulin therapy, 100, 113, 122123
furosemide, 133, 217 heart sounds, third, 177 hypercapnia, 90, 9394 intensive care unit, 97, 113
Helicobacter pylori infection, 214 hypercortisolism, 44, 115, 117, 120121 intermittent hemodialysis (IMD), 96
G hematocrit, 116 hyperglycemia, 113 internal carotid artery dissection, 197
gadolinium, 75 hematology, 2748 hyperkalemia, 9495, 133 internal carotid artery stenosis, 198
gallbladder cancer, 152 hemicrania, proximal, 201 hyperleukocytosis, 4647 internal medicine, general, 202222
gallstones, 158159 hemicraniectomy, decompressive, 198 hypermobility syndrome (HMS), 67 International Prognostic Scoring System
gangrene, Streptococcal, 26 hemochromatosis, 51f, 65, 153, 155 hyperparathyroidism (IPSS), 46
gastric bypass surgery, 153, 201 hemodialysis, 96, 133 primary (PHPT), 102, 114 interstitial lung disease (ILD), 67
gastric cancer, 38 hemoglobin, 38 secondary, 114115, 138 interstitial nephritis, 135, 136
gastroesophageal reflux disease, 154 hemolytic anemia, 39, 71, 220 hyperpigmentation, 222 interstitial pneumonia, nonspecific, 88, 88f
gastrointestinal bleeding, occult, 158 hemolytic uremic syndrome (HUS), 137 hyperprolactinemia, 4344, 120 interstitial pneumonitis, usual (UIP), 88
gastrointestinal illness, 14 hemorrhage hypertension, 118, 125140, 184, 198 intoxication
gastrointestinal stromal tumors (GISTs), diffuse alveolar (DAH), 62f, 74 in diabetic patients, 135136, 176 ethylene glycol, 98
150151 intracerebral, 197 intracranial, 195 lead poisoning, 218, 220
general internal medicine, 202222 subhyaloid, 199 permissive, 137 intracerebral hemorrhage, 197
genetic testing EFV, 6768 heparin, 40, 175, 197 pulmonary, 66 intracranial hypertension, 195, 218
genital herpes, 2122 heparin-induced thrombocytopenia resistant, 181182 intraductal papillary mucinous neoplasms
genotype, 151 (HIT), 40 secondary, 179 (IPMNs), 38, 154
gentle diuresis, 221 heparin-induced thrombocytopenia with hypertensive emergency, 133 intrathoracic airflow obstruction, diffuse,
giant cell arteritis, 64, 69 thrombosis (HITT), 96 hypertensive urgency, 138 90, 90f

I n d e x 2 25
iodine, radioactive, 120 lumbar spine imaging, 72 middle cerebral artery aneurysm, 198 O
IRIS. See immune reconstitution lumbar spine stenosis, 73 middle cerebral artery infarction, 195 obesity, 87, 122123
inflammatory syndrome lung cancer, 42, 43 migraine headache, 195, 200, 201 obesity-hypoventilation syndrome (OHS),
iron deficiency anemia, 220 lung disease mild cognitive impairment, 196 8990
irritable bowel syndrome, 114115, 150, chronic, 1314 minimal change disease, 136 obliterans organizing pneumonia, bronchi-
159 interstitial (ILD), 67 mitral stenosis, 173174, 221 olitis (BOOP), 88
ischemic colitis, 150 lung infection, 6667 mitral valve prolapse, 175, 176 obstructive cardiomyopathy, hypertrophic,
ischemic stroke, 137, 195, 198, 199 lung nodules mixed connective tissue disease (MCTD), 180
isopropyl alcohol toxicity, 140 small, 42, 42t, 91, 91f 68 obstructive lung disease, 98
isosorbide mononitrate, 180 surgical resection of, 92 mixing studies, 40 obstructive sleep apnea (OSA), 89, 138
isotonic saline, 135 tree-in-bud, 91, 91f Mobitz Iatrioventricular block, 179180 occupational therapy, 200
itching, 214 lupus anticoagulant, 47 mojitos, 222 octreotide, 158
lupus nephritis, 68, 137 mono-like illness, 18 older adults, 64
J Lyme disease, 3, 15, 6566, 153, 215 mononeuritis multiplex, 196197 omalizumab, 221
joint aspiration, 74 lymphadenopathy, hilar and mediastinal, montelukast (Singulair), 87 oncology, 2748
joint hypermobility syndrome (HMS), 67 81f, 93 morning stiffness, 72, 216 bladder cancer, 41
juvenile myoclonic epilepsy, 200 lymphangioleiomyomatosis (LAM), 8788 motor vehicle accidents, 89 breast cancer, 41, 217
juvenile polyposis syndrome, 155 lymphangitis, 222 MRSA (methicillin-resistant Staphylococcus Burkitts lymphoma, 4546, 136
lymph node biopsy, excisional, 43 aureus), 3, 15, 216 cervical cancer, 214
K lymphocytic leukemia, chronic, 43 mucinous cystic neoplasms, 154 chronic lymphocytic leukemia, 43
karyotyping, 115116 lymphoma mucormycosis, 17 colon cancer, 4748, 216
kidney stones, 139 Burkitts, 4546, 136 multiple myeloma, 66, 134, 217 diffuse large B-cell lymphoma, 2, 217
kidney transplantation, 137 diffuse large B-cell, 2, 217 multiple sclerosis, 200, 201 follicular lymphoma, 46
knee arthritis, septic, 74 follicular, 46 multiple-system atrophy, 201 gallbladder cancer, 152
knee arthroplasty, 6970 Hodgkin, 43 murmurs, 175, 177, 183, 215216 gastric cancer, 38
knee replacement, total, 174 parotid, 7475 mutations, 38, 200201 Hodgkins lymphoma, 43, 136
Lynch syndrome, 38 MUTYH-associated polyposis (MAP), leukemia, 43, 4546, 4647
L 154155 lung cancer, 42, 43
labetalol, 133 M myasthenia gravis, 151, 197 pancreatic adenocarcinoma, 38
lactic acidosis, 140 macroadenoma, pituitary, 44, 120 myasthenic crisis, 200 parotid lymphoma, 7475
Lambert Eaton myasthenic syndrome, 200 magnetic resonance angiography, 178179, myelitis, transverse, 195196 prostate cancer, 41
laminectomy, 73, 183 198, 200 myelodysplastic syndrome (MDS), 46 renal cell carcinoma, 40
Lane-Hamilton disease, 9091 magnetic resonance imaging myelogenous leukemia, chronic, 46 small-cell lung cancer, 42
Lane-Hamilton syndrome, 154 of brain, 200, 213 myelolipoma, adrenal, 117 transitional cell cancer, 40
Langerhans cell histiocytosis, 88, 217 diffusion-weighted (DW-MRI), 199 myelopathy, transverse, 195196 urothelial cancer, 41
lateral femoral cutaneous nerve entrapment, of lumbar spine, 72 myocardial infarction, 177178, 182183 opiate medication contracts, 65
70 magnetic resonance venography, 197 cocaine-induced, 219 oral contraceptives, 116, 201
lavage, 6667 malabsorption, 201 non-ST segment elevation (NSTEMI), organizing pneumonia, 88
lead poisoning, 218, 220 malignant brain edema, 198 175176, 179 orthodeoxia, 8990
left anterior descending artery stenosis, 180, mammary souffle murmurs, 177 right ventricular, 173 orthopedics, 6970
182, 184 Marfans syndrome, 220 ST segment (STEMI), 173, 176177 osmolal gap, 138139
left bundle branch block, 177, 178 marijuana use, 221 myoclonic epilepsy, juvenile, 200 osmotic demyelination syndrome, 135
left circumflex artery disease, 180 mastalgia, cyclic, 213 myopathy, critical illness, 200 osteoarthritis, 64, 6970, 216
left hemispheric stroke, 197 mature-onset diabetes of the young myxoma, atrial, 183184 osteomalacia, 115
left internal carotid artery dissection, 197 (MODY), 113114 osteomyelitis, vertebral, 17
left lateral femoral cutaneous nerve McMurrays test, 71 N osteopenia, 115
entrapment, 70 mechanical ventilation, 9596, 98, 99 nasojejunal tube feeding, 157 osteoporosis, 114
left ventricular outflow tract obstruction, mediastinal lymphadenopathy, 81f, 93 Neisseria gonorrhea, 214 osteoporotic fractures, 115
180 medication contracts, 65 neoplasms, 38, 154 overdose
leukapheresis, 4647 medications. See also specific medications nephritis acetaminophen (paracetamol), 95
leukemia, 43, 4546, 4647 over-the-counter, 181182 interstitial, 135, 136 beta-blocker, 100
leukoencephalopathy, progressive that inhibit catecholamine reuptake, 117 lupus, 68, 137 calcium-channel blocker, 100
multifocal, 215 Mediterranean fever, familial (FMF), tubulointerstitial, 140 over-the-counter medication use history,
leukostasis, 4647 6768 nephrogenic systemic fibrosis (NSF), 75 181182
levetiracetam, 201 MEFV genetic testing, 6768 nephrolithiasis, 158 oxycodone, 65
levofloxacin, 18, 21, 71 melena, 153 nephrology, 125140 oxygen therapy, 39
levothyroxine, 44, 116117, 118119, 120 membranous nephropathy, idiopathic, nephrotic syndrome, 136
lichen planus, eosinophilic, 152 136137 neuroleptic malignant syndrome, 97 P
lidocaine, 69 Menetriers disease, 155 neurology, 185201 pacemakers, 177, 178, 184
Li-Farumeni syndrome, 38 meningitis, bacterial, 2021 neuromuscular blockers, 9495 packed red cell transfusion, 97
lifestyle interventions, 122 men who have sex with men (MSM), 1314 neuropsychological evaluation, 196 Pagets disease, 115, 214
lime juice, 222 meralgia paresthetica, 70 neuroretinitis, 16 pain
linezolid, 25 mesalamine, 150 neurosurgery, 217218 back, 72
lisinopril, 138, 176, 182183 metabolic alkalemia, acute, 9394 neutropenia, 18 chest, 181, 183, 184, 219
lithium nephrotoxicity, 138 metabolic alkalosis, chronic, 9394 niacin deficiency, 153 Coopers ligament, 213
liver disorders, 141159, 216 metastasis, 41, 200, 217 nitrofurantoin, 14, 214 distal extremity, 200
Lofgrens syndrome, 72 metastatic solid tumors, 118 nonalcoholic fatty liver disease, 216 pamidronate, 115
loop diuretics, 133, 217 metformin, 113, 123124 nonalcoholic steatohepatitis, 155, 216 pancreas
lorazepam, 99, 138139 methemoglobinemia, 217 nongonococcal urethritis (NGU), 24 disorders of, 141159
lower extremity weakness, 14 methicillin-resistant Staphylococcus aureus nonspecific interstitial pneumonia, 88, 88f intraductal papillary mucinous neoplasm
low-oxalate diet, 158 (MRSA), 3, 15, 216 norepinephrine-serotoninreuptake inhibi- (IPMN) of, 38
low-salt diet, 135 methylprednisolone, 68, 195196, 201 tors, 117 pancreatic adenocarcinoma, 38
lumbago, 65 metoprolol, 174, 183 norovirus, 153 pancreatitis, 154, 157, 158159
lumbar disc herniation, 72 microangiopathic hemolytic anemia NovoSeven, 47 panlobular (widespread) emphysema,
lumbar laminectomy, 183 (MAHA), 71 nuclear stress testing, 181 8687
lumbar puncture, 195, 198, 218 midazolam, 98 nutrition, enteral, 157 Papanicolaou (Pap) smears, 213, 214

22 6 I n de x
papillary mucinous neoplasms, intraductal dyspnea in, 173174 renal cell carcinoma, 40 sleep-related hypercapnia, 90
(IPMNs), 38, 154 hepatitis B during, 156157 renal crisis, scleroderma (SRC), 71, 134, sleep routines, 70
paracetamol (acetaminophen) p oisoning, mitral stenosis in, 221 220 small-cell lung cancer, 42
95 normal physical findings, 177 renal disease, hepatitis Cassociated, 136 small fiber peripheral neuropathy, 200
paralysis, Todd, 201 syphilis infection during, 21 renal failure, 68 small intestinal bacterial overgrowth
parapneumonic effusions, 87 Type IvWD during, 47 renal grafts, 7374 (SIBO), 158
parkinsonism, drug-induced, 200 premature ventricular contractions (PVCs), renal transplantation, 137 small intestinal Crohns disease, 158
parotid lymphoma, 7475 182 renal tubular acidosis (RTA), 134 smoking cessation, 20
paroxetine, 41 prepatellar bursitis, gouty, 71 renin, 118 sodium restriction, 135, 218
partial thromboplastin time, activated presenilin-1 mutation, 200201 respiratory acidosis, acute-on-chronic, 95 solid organ transplantation, 13
(aPTT), 40 primary biliary cirrhosis, 150 respiratory disease, aspirin-exacerbated, 87 solid tumors, metastatic, 118
Parvovirus B19, 39 proctitis, ulcerative, 152153 respiratory distress. See acute respiratory somatostatin analogs, 4445, 121
patellofemoral syndrome, 68 progesterone, 213 distress syndrome spherocytosis, hereditary, 39
pazopanib, 40 progressive multifocal leukoencephalopa- resting hypoxemia, 88 spinal cord compression, 48, 200
pellagra, 153 thy, 215 reticulocytopenia, 39 spinal cord lesions, 195196
penicillin, 22 prolactinoma, 41, 44, 120 retrognathia, 89 spine surgery, 73
penicillin desensitization, 21 promyelocytic leukemia, acute, 4546 Revised Clinical Risk Index (RCRI), 174 spirometry, 86
percutaneous coronary intervention (PCI), propofol, 98 Revised IPSS (IPSS-R), 46 spironolactone, 116
176177, 184 propylene glycol toxicity, 138139 rhabdomyolysis, 135 splenectomy, 39
percutaneous renal artery angioplasty, 135 prostate cancer, 41 rheumatic fever, 221 spondylitis, ankylosing, 74
pericarditis, 153154, 178 prostate-specific antigen (PSA), 116 rheumatic heart disease, 221 spontaneous bacterial peritonitits (SBP),
peripheral arterial disease (PAD), 181 proteinuria, 176 rheumatoid arthritis, 64, 6667, 69, 75 154, 158
peripheral blood flow cytometry, 43 pseudoachalasia, 156 rheumatology, 4975 spontaneous breathing trials, 9596
peripheral eosinophilia, 221 pseudogout arthritis, 64, 70 rhinosinusitis, acute, 21, 214215 sputum Gram stain, 93
peripheral neuropathy, small fiber, 200 pseudo-osteo arthritis, 70 rifampin, 1516, 2526 squamous cells, atypical, of undetermined
peritonitis, spontaneous bacterial (SBP), pseudo-rheumatoid arthritis, 70 right coronary artery disease, 180 significance (ASC-US), 213
154 pseudotumor cerebri, 195 right coronary artery stenosis, 182 Staphylococcus aureus, 71, 180181
permissive hypercapnia, 9394 psoriasis, 67 right-to-left shunts, 218 methicillin-resistant, 3, 15, 216
permissive hypertension, 137 psoriatic arthritis, 67 right ventricular myocardial infarction, 173 STAT electrolyte panel, 219
pertussis, 219220 psychiatric counseling, 70 risedronate, 115 statin therapy, 213
petechiae, 56f, 69 pulmonary coccidioidomycosis, 25 ritonavir, 19 status epilepticus, 99, 199
Peutz-Jeghers syndrome, 154155 pulmonary effusions, parapneumonic, 87 rituximab, 14 steatohepatitis, nonalcoholic, 155, 216
pH, pleural fluid, 87 pulmonary embolism, 40, 47 rivaroxaban, 40 stenting, 198
pheochromocytoma, 117 pulmonary fibrosis, idiopathic, 8889 rotator cuff tears, 69 steroids, 137
Philadelphia chromosome t(9;22) pulmonary hypertension, 66 Stills disease, adult-onset (AOSD), 7273
(q34;q11.2), 46 pulmonary infection(s), 6667 S stool testing, 151152
phospholipase A2 receptor (PLA2R), pulmonary medicine, 76100 sacroiliitis, 157 Streptococcal gangrene, 26
136137 chronic lung disease, 1314 saline, isotonic, 135, 136 Streptococcus, gram-positive beta-hemolytic,
physical therapy, 65, 69, 70, 200 interstitial lung disease (ILD), 67 salivary cortisol, 44, 115, 120121 26
physostigmine, 100 lung infection, 6667 salmeterol, 86 Streptococcus infection, 22, 221222
phytophotodermatitis, 222 pulmonary nodules sarcoidosis, 81f, 93, 114, 217 stress ulcer prophylaxis, 9697
Pickwickian syndrome, 8990 small, 42, 42t, 91, 91f sausage digits, 220 stroke, 137, 195, 197, 198199
pituitary lesions, incidental, 45, 121122 surgical resection of, 92 Scl-70 antibody, 156 stromal tumors, gastrointestinal (GISTs),
pituitary macroadenoma, 44, 120 tree-in-bud nodules, 91, 91f scleroderma renal crisis (SRC), 71, 134, 220 150151
pivmecillinam, 14 pulmonary-renal syndrome, 73 scurvy, 153 strongyloides infection, 221
plantar fasciitis, 71 pulmonary sarcoidosis, 81f, 93 seizures, 199, 201 ST segment elevation, 173
plasma exchange therapy, 200, 215, 219 pulseless ventricular tachycardia, 97 selective serotonin-reuptake inhibitors, 117 ST segment myocardial infarction, 173
plasmapheresis, 40, 137, 197 pulse oximetry, 99 sepsis, 1718, 100 subglottic secretions, 13
pleural effusions, 218 pyrazinamide, 1516 septic arthritis, 74 subhyaloid hemorrhage, 199
pleural fluid, 87 pyrin protein, 6768 septic shock, 96, 100 succinylcholine, 9495
pneumococcal 13-valent conjugate serotonin syndrome, 25, 97 suctioning
(PCV13) vaccine, 1314 Q serous cystadenoma, 154 endotracheal, 99
pneumococcal vaccination, 1314 quadriceps tendon rupture, 71 sertraline, 97 of subglottic secretions, 13
Pneumocystis jiroveci pneumonia, 24 quinine, 25 serum electrophoresis (SPEP), 66 sunitinib, 40
pneumonia sex hormone-binding globulin, 116 superior vena cava syndrome, 48
acute, 87 R sexual activity, 180 supportive care, 157
health care-associated, 18 radioactive iodine, 120 sexually transmitted infections (STIs), 18 supraventricular tachycardia, 174175, 176
nonspecific interstitial, 88, 88f radiography, 42 shock surgery
organizing, 88 Ramsay Hunt syndrome type II, 24 cardiogenic, 98 aortic valve replacement, 181
Pneumocystis, 24 rapid shallow breathing index (RSBI), septic, 96, 100 cardiac, 180
ventilator-associated, 13, 9697 9596 treatment of, 94 for cardiac mass, 183184
pneumonitis, usual interstitial (UIP), 88 rasburicase, 136 short-lasting unilateral neuralgiform head- coronary artery bypass, 180, 182
polyangiitis, 215, 221 rash, dermatomyositis, 53f, 67 ache with conjunctival injection and elective noncardiac, 178
polyarthritis, acute viral, 66 Raynauds phenomenon, 7071 tearing (SUNCT), 201 gastric bypass, 153, 201
polychondritis, relapsing, 89 recombinant tissue plasminogen activator shrinking lung syndrome, 89 for lung cancer, 43
polycystic ovary syndrome (PCOS), 116 (rtPA), 197 shunts, right-to-left, 218 lung nodule resection, 92
polymyalgia rheumatica (PMR), 72 red blood cell transfusion, 39 sickle cell disease, 39 neurosurgery, 217218
polyposis, MUTYH-associated (MAP), red cell aplasia, transient (TRCA), 39 Singulair (montelukast), 87 perioperative evaluation for, 178
154155 red cell exchange transfusion, 25 sinus arrhythmia, 174175 postoperative central diabetes insipidus,
positive end-expiratory pressure (PEEP), 98 red flag features, 72 sinus bradycardia, 174175, 184 121
positron emission tomography, 42 regadenoson, 181 sinusitis, 17, 21 preoperative cardiac risk stratification,
postenteric reactive arthritis, 151 Reiters syndrome, 151 sitagliptin, 124 174, 183
prediabetes, 113, 122 relapsing polychondritis, 89 Sjgrens syndrome, 73, 7475, 150 resumption of sexual activity after, 180
prednisolone, 137 renal artery angioplasty, 135 skeletal muscle biopsy, 67 for ruptured brain abscess, 217218
prednisone, 24, 69, 151, 157 renal artery stenosis, 135, 179 skin/soft tissue infections, 15, 23 spine, 73
pregnancy, 4344, 120 renal biopsy, 134135 sleep apnea, obstructive (OSA), 89, 138 thyroid, 119

I n de x 227
Surviving Sepsis Campaign, 1718 tobacco use, 152 ulcers ventricular fibrillation, 97
swallowing, 198 Todd paralysis, 201 duodenal, 156 ventricular tachycardia, pulseless, 97
syncope, 178 tolvaptan, 137138 HSV-associated, 152 Venturi effect, 180
syphilis infection, 21 tophaceous gout, 61f, 7374 stress, 9697 verpamil, 100
systemic fibrosis, nephrogenic (NSF), 75 total knee arthroplasty, 6970 ultrafiltration, 140 Vibrio vulnificus infection, 153
systemic lupus erythematosus, 62f, 68, 74, total knee replacement, 174 ultrasound, abdominal, 182, 216217 viral arthritis, 66
134135 toxicity upper endoscopy, 156 viral hepatitis, 16
ethylene glycol intoxication, 98 upper gastrointestinal bleeding, 158 vitamin Adeficiency, 153, 156
T isopropyl alcohol, 140 upper motor neuron deficits, 218 vitamin B6 toxicity, 218
tachycardia lead, 218, 220 urea breath test, 214 vitamin B12 deficiency, 39, 156, 158, 201
pulseless ventricular, 97 Toxoplasma, 2223, 23f uremia vitamin C deficiency, 153
supraventricular, 174175, 176 transesophageal echocardiography, hemolytic uremic syndrome (HUS), 137 vitamin D deficiency, 156
tacrolimus toxicity, 136 180181 thrombotic thrombocytopenic vitamin D supplements, 114, 138,
tamoxifen, 4142 transfusions, 97 purpura-hemolytic uremic syndrome 213214
Tdap vaccination, 1314 transient ischemic attack (TIA), 198199 (TTP-HUS), 215, 215f vitamin E, 216
telaprevir, 151 transient red cell aplasia (TRCA), 39 urethritis, nongonococcal (NGU), 24 vitamin E deficiency, 156, 216
temporal artery biopsy, 64, 69, 73 transitional cell cancer, 40 urinary tract infections, 22, 41 vitamin K deficiency, 156
tendonitis, calcific, 6869 transjugular intrahepatic portosystemic urine electrophoresis (UPEP), 66 von Willebrand disease, 40, 47
tendon rupture, 71 shunt (TIPS), 156, 218 urine free cortisol, 116, 120121 von Willebrand factor, 40, 47
tenecteplase, 176177 transplantation urothelial cancer, 41 voriconazole, 18
tenofovir, 157 kidney, 137 usual interstitial pneumonitis (UIP), 88
tessalon perles, 21 solid organ, 13 uveitis, 140 W
testosterone, 116 transthoracic echocardiography, 176, 218 warfarin, 2526, 178, 195
tetanus vaccination, 1314 transurehral resection of bladder tumor V Wegeners syndrome, 9091
thalassemia, 39, 220 (TURBT), 41 vaccinations Wegners granulomatosis, 22
thiamine deficiency, 153 transverse myelitis, 195196 hepatitis A, 1314 weight loss, 70, 113, 122
Thompson test, 71 transverse myelopathy, 195196 hepatitis B, 157 Wenckebach block, 179180
thoracentesis, 218 travelers, 1, 13 influenza, 13 Wernicke-Korsakoff syndrome, 153
thoracic aorta, 178179 tree-in-bud nodules, 91, 91f live-virus, 72 wet beriberi, 153
thoracic duct obstruction, 88 triamcinolone, 213 pneumococcal, 1314 Whipples disease, 151
thrombocytopenia, heparin-induced, 40, 96 tricyclic antidepressants, 117 Tdap, 1314 Wilsons disease, 155
thromboembolism, venous (VTE), 214 trigeminal autonomic cephalalgia (TACs), tetanus, 1314 wireless capsule endoscopy, 158
thrombolytic therapy, 137 201 yellow fever, 7172 Wolff-Parkinson-White syndrome, 173,
thrombosis trigeminal neuralgia, 200 valacyclovir, 24 175
deep venous (DVT), 40, 47 trimethoprim-sulfamethoxazole valley fever, 25 wrist chondrocalcinosis, 50f, 70
heparin-induced thrombocytopenia with (TMP-SMX), 14, 24, 216 valvular heart disease, 175
(HITT), 96 trimethoprim-sulfamethoxazole vancomycin, 17, 18, 2021, 23, 217218 X
thrombotic thrombocytopenic purpura (TMP-SMX) or uncomplicated vascular access, 133 X-rays
(TTP), 40, 219 cystitis, 214 vasculitic neuropathy, 196197 cervical spine, 69
thrombotic thrombocytopenic trochanteric bursitis, 69, 70 vasculitis, 56f, 69, 137 chest, 14, 43, 214
purpura-hemolytic uremic syndrome tuberculosis, 14, 1516, 136 vasopressin receptor antagonists, 137138
(TTP-HUS), 215, 215f tuberculosis immune reconstitution inflam- vein sampling, adrenal, 134 Y
thrush (candidiasis), 18 matory syndrome (IRIS), 1617 venlafaxine, 117 yellow fever vaccine, 7172
thunderclap headache, 198 tubulointerstitial nephritis with uveitis, 140 venography, magnetic resonance, 197
thyroid hormone, 119120 tularemia, 26, 153 venous hums, 177 Z
thyroid nodules, 119 tumor lysis syndrome (TLS), 48, 136 venous thromboembolism (VTE), 40, 214 zafirlukast (Accolate), 87
thyroid scan, 119 tumors ventilation Zenkers diverticulum, 152
thyroid-stimulating hormone (TSH), gastrointestinal stromal (GISTs), mechanical, 9596, 98, 99 zileuton (Zyflo), 87
4344, 119, 120, 184 150151 noninvasive, 95 zinc, 201
thyroid surgery, 119 metastatic solid, 118 obesity-hypoventilation syndrome zoledronic acid, 115
thyroid uptake, 119 (OHS), 8990 Zyflo (zileuton), 87
thyrotoxicosis, 119 U ventilator-associated pneumonia (VAP),
tilt-table test, 67 ulcerative colitis, 93 13, 9697
tissue plasminogen activator (tPA), 197 ulcerative proctitis, 152153 ventilator support, 98

22 8 I n de x

Vous aimerez peut-être aussi